Nat Law Supplemental Quizzes

¡Supera tus tareas y exámenes ahora con Quizwiz!

Question 7 -Past Recollection Recorded- Intro After measuring skid marks and interviewing all eyewitnesses present at the scene of an automobile accident, an officer wrote out in longhand all the pertinent information on the standard police accident report form. The officer turned the report in to the department, where a police typist, according to the prescribed procedure, transcribed the officer's longhand record into a typed record which, after it was checked by the officer, was then placed on file in the police department. Two years later, the plaintiff sued the defendant for her injuries. The plaintiff called the officer to the stand to testify regarding the skid marks. The officer can remember nothing about the specific accident, as he has made at least 200 accident reports since the accident in question. After looking at the repot the officer can remember preparing the report but still cannot remember the accident scene. He wishes to read the information regarding the skid marks in the police report to the jury from the stand. The defense attorney objects. How should the court rule? A The officer may read from the repot because it is past recollection recorded. B The officer may read from the report, because it is a present sense impression. C The officer may not read from the repot because a police report does not qualify as a business record. D The report should be given to the jury to read, but it should not be read to them by the officer from the stand.

(A) Because a proper foundation has been laid, the officer may read from the report while on the witness stand. Past recollection recorded is an exception to the hearsay rule. When a witness, after consulting the writing while on the stand, still has insufficient recollection to testify, the writing itself may be introduced into evidence if a proper foundation is laid. The foundation must establish four elements: (i) the witness at one time must have had personal knowledge of the facts in the writing; (ii) the writing must have been made when the matter was fresh in the witness's mind; (iii) the writing must have been made by the witness or under his direction, or adopted by him; and (iv) the witness must be presently unable to remember the facts. [Fed R Evid 803(5)] The officer had personal knowledge of the skid marks. (Note that because of this personal knowledge requirement, any part of the report not relating to the officer's personal observation of the skid marks, such as statements of eyewitnesses, would not be admissible under this exception.) The officer also made the original writing when the facts were fresh in his mind, and he is presently unable to remember the facts. Thus, the record is admissible as a past recollection recorded. (B) is incorrect because a present sense impression is a statement describing or explaining an event made while perceiving the event or immediately thereafter. [Fed R Evid 803(1)] Generally, a present sense impression is an oral statement made to another rather than a written report. Here, the officer's statement was made in a report prepared sometime after he first perceived the skid marks and after he had interviewed all of the witnesses. Thus, the officer's report is not a present sense impression. (C) is incorrect. A police report may qualify as a business record under Federal Rule 803(6) because it was made in the course of a regularly conducted activity by any business, organization, occupation, or calling, even though it also qualifies as a public record or report under Federal Rule 803(8) or as past recollection recorded under Federal Rule 803(5). (D) is incorrect because the procedure is exactly the opposite. Under Federal Rule 803(5), a recorded recollection that qualifies under the exception may be read into evidence (because it is a substitute for the witness's testimony) but may not be received as an exhibit for the jury to read unless offered by an adverse party.

Question 15- Congressional powers - Intro To boost the slumping cruise ship trade, Congress passed a bill providing that six outmoded United States Navy vessels would be sold to a private cruise company for $1 each, on condition that the company refurbish the ships at its own expense and operate the ships as cruise ships for at least four years. The conditions for the refurbishing were highly specific, and it would cost the company at least $3 million to refit each ship as a modern cruise liner. The company agreed to the conditions, but a competitor cruise company filed suit in federal court to block the sale. Which of the following statements is most correct concerning disposition of the case? A The federal court should treat the sale as presumptively valid, because the Constitution expressly gives Congress the power to dispose of property of the United States. B The federal court should treat the sale as presumptively invalid, because the Constitution expressly denies Congress the right to deprive persons of property without due process of law. C The federal court should rule the statute constitutional only if the President or the Secretary of Defense has certified that the Navy ships are obsolete for defense purposes. D The federal court should rule the statute unconstitutional, because it denies other cruise ship operators the equal protection of the laws.

(A) The court should hold the sale presumptively valid pursuant to Congress's property power Article IV, Section 3 of the Constitution gives Congress the power to dispose of all property belonging to the federal government. There are no express limits placed on this power, and a disposal has never been invalidated on the ground that it places a competitor of the purchaser at a disadvantage. (B) is incorrect because the Due Process Clause would not prevent the sale here. The Due Process Clause prohibits the government from denying person's life, liberty, or property without due process of law. For the restrictions of the Clause to apply, a person must have a legitimate property interest in the property taken. "Property" includes more than personal belongings, but a mere expectation or desire for the benefit is not enough. One must have a legitimate claim or entitlement to the benefit before one may make a procedural due process challenge, and here the competitor has no claim to a right to be offered the ships that were sold to the first cruise company. (C) is incorrect because, as indicated above, there is no express limit on Congress's power to dispose of government property; nothing in the Constitution requires Congress to get the President's (or any other executive officers) permission to exercise the power. (D) is incorrect because there is no denial of equal protection here. While the Equal Protection Clause is not applicable to the federal government, equal protection guarantees are applicable through the Fifth Amendment Due Process Clause. Nevertheless, there is no equal protection violation here: Because no suspect class or fundamental right is involved, the action would be tested under the rational basis standard (discriminatory government action is valid as long as it is rationally related to any legitimate government interest) and would be upheld since the sale is rationally related to the government's interest in reviving the cruise ship industry. It does not matter that the government is not doing all that it can; a first step is permissible.

Question 26-Impeachment-Intermediate The plaintiff filed a personal injury action against a restaurant. The plaintiff alleges that she suffered food poisoning after consuming the restaurant's daily special—extra-strong coffee and chili over eggs. The restaurant defends by claiming that the special on the day in question was an exotic tea served with a yogurt and fruit blend and that the extra-strong coffee drink was not even on the menu that day. Counsel for the restaurant called a former waitress to testify as to whether the extra-strong coffee drink was on the menu that day. The former waitress, who lives with the plaintiff, testified that it was always on the menu. The restaurant's attorney then asked, "Didn't you in fact, at your deposition, state that the extra-strong coffee drink was not on the menu that day?" The Plaintiff's attorney objects. Is the question proper? A Proper, because the former waitress is hostile. B Proper, because it constitutes refreshing the witness's recollection. C Improper, because this is a leading question on direct examination. D Improper, because the restaurant cannot impeach its own witness.

(A) The question is proper. For the purpose of impeaching the credibility of a witness, a party may show that the witness has, on another occasion, made statements that are inconsistent with some material part of her present testimony. Under the Federal Rules, any party may impeach any witness (even its own witness), so (D) is incorrect. (C) is incorrect because leading questions are permitted on direct examination when a witness is hostile or identified with an adverse party. Thus, leading questions are permitted because the former waitress lives with the plaintiff. (B) is incorrect because the restaurant's attorney is not trying to refresh the former waitress's recollection of whether the drink was on the menu; rather, he is talking about a deposition and is trying to impeach her with an inconsistent statement.

Question 9 - Legislative Congressional Powers/Commerce Clause - Intro Congress enacted legislation prescribing the specifications for various types of containers that all suppliers of nonprescription drugs were required to use in marketing their products. From where does Congress's power to pass this legislation derive? A The Necessary and Proper Clause. B The Commerce Clause. C The General Welfare Clause. D The Tenth Amendment.

(B) Congress's power is derived from the Commerce Clause. The Supreme Court has construed the scope of the Commerce Clause very broadly, so that it covers the regulation of drug packaging, which has a substantial economic effect on interstate commerce. (A) is wrong because this type of regulation definitely falls under the Commerce Clause, and there is no need to imply a power (under the Necessary and Proper Clause) when an enumerated power Controls. Moreover, the Necessary and Proper Clause, by itself, does not grant Congress the power to do anything. (C) is wrong because the General Welfare Clause presents a limitation on Congress's spending and taxing powers, which are not relevant here. (D) is wrong because the Tenth Amendment is not a grant of power to Congress; rather, it reserves the powers not granted to Congress to the states.

Question 13 -Executive Power - Intro After the dictator of a Caribbean island country was deposed, the President of the United States extended official recognition to the country's new government. As ambassador to the newly recognized government, the President nominated an aging industrialist who was a close personal friend of the deposed dictator. Unable to muster enough votes to block approval of the new ambassador, the President's political opponents in the Senate caused a resolution to be passed requiring that all consular staff below the rank of ambassador be selected from a list of "approved" candidates who have been certified as acceptable by the new country's government. The President refused to consider any of the Senate's list of approved candidates. Which of the following is the President's strongest constitutional basis for refusing to obey the Senate resolution? A The Senate could have effectuated its policies by a less intrusive method, such as refusing to appropriate funds to staff the new embassy if the President's selections were inappropriate. B The President has the authority to nominate and appoint the diplomatic representatives of the United States. C The President has exclusive authority, as commander in chief, to protect American interests abroad. D The President's control over the foreign policy of the United States may not be limited by other branches of government.

(B) Article Il, Section 2 provides that the President shall nominate, and with the advice and consent of the Senate shall appoint, ambassadors and other officers of the United States. The section also provides that Congress may vest the appointment of inferior officers in the President alone, in the courts of law, or in the heads of departments. Under separation of powers principles, however, Congress may not vest in itself any broader appointment powers than what is provided for by the Constitution. Where Congress has not vested the appointment power in courts of law or the heads of departments, it is not permitted to restrict the candidates that the President may nominate for appointment. Thus, the Senate's attempt here to exert some control over the President's choice of lower-level diplomatic representatives is an unconstitutional violation of the separation of powers. (A) is incorrect for two reasons: First, if the Senate's action had been constitutionally permissible, there would be no requirement of using a "less intrusive method" to effectuate such action. The President would simply be required to seek the approval of the Senate. Second, if the Senate refused to appropriate funds to staff the embassy because it deemed the President's selections inappropriate, this would simply be another way of exerting control over the President's selection of lower-level diplomatic personnel, and would be as unconstitutional as the resolution passed by the Senate. (C) is incorrect because the President's status as commander in chief is not at issue here. The President does have rather extensive military powers as commander in chief of the armed forces. However, the appointment of a consular staff involves the President's power in foreign relations, not his power as commander in chief. In addition, (C) incorrectly states that the President has exclusive authority to protect American interests abroad. Although the President has broad authority to protect American interests abroad, Congress also has some authority in this field. For example, Congress has the power to declare war, to raise and support armies, and to give its advice and consent in the making of treaties. Similarly, (D) incorrectly asserts a limitless presidential control over foreign policy. While the President's authority in foreign policy is quite broad, it has some limits; e.g., the requirement of senatorial advice and consent in the making of treaties

Question 19- Substantive Due Process - Advanced A state bans the use of disposable diapers to reduce the volume of nonbiodegradable material in its landfills. The ban was a boon for diaper services within the state, but many parents of young children were displeased with the use of conventional diapers. With support from retail establishments that lost business from the disposable diaper ban, a grass roots coalition formed to fight the ban funded a study showing that the trucks and cleaning supplies used by diaper services within the state harmed the environment more than disposable diapers. The coalition and retailers then filed suit seeking to have the ban on disposable diapers declared unconstitutional. If the court strikes down the statute, on which of the following constitutional provisions would its decision most likely be based? A The Equal Protection Clause of the Fourteenth Amendment. B The Due Process Clause. C The Impairment of Contracts Clause. D The Privileges or Immunities Clause of the Fourteenth Amendment.

(B) Of the choices presented, the only likely basis to strike down the statute under the Due Process Clause as a violation of substantive due process. Substantive due process tests the reasonableness of a statute; it prohibits arbitrary governmental action under substantive due process, when the government action limits a fundamental right, the government must prove that the action is necessary to promote a compelling interest. If a fundamental right is not involved, the challenging party must prove that the act is not rationally related to any legitimate government interest. The retail sale of diapers is not a fundamental right, and so a challenger must prove that there is no rational basis for the statute. Almost any law can be justified under the rational basis standard. The law need not be the best law for accomplishing the government's goal. Thus, even if it is true that the disposable diaper ban causes more pollution than it prevents, because the ban is rationally related to reducing the volume of trash in landfills, the challenge is unlikely to succeed. Nevertheless, none of the other choices states a viable ground for invalidating the statute, and so (B) is the best choice. (A) is wrong because equal protection applies where a statute or governmental action treats similar people in a dissimilar manner (i.e., classifies people), and here there is no classification-- under the statute no one can sell disposable diapers for use within the state. Thus, an equal protection argument is not applicable. (C) is wrong because the Impairment of Contracts Clause prohibits only the substantial impairment of existing contracts (and there are exceptions even where there is substantial impairment), and nothing in the facts indicates that forbidding the retail sale of disposable diapers would substantially impair any existing contract. (D) is wrong because the privileges and immunities covered by the Fourteenth Amendment are those attributes peculiar to United States citizenship (e.g., the right to petition Congress for redress or the right to vote for federal officers). The statute here does not affect such rights.

Question 3 -Photographs - Intro The owner of a small business was injured in a traffic accident. A month after the accident, the owner asked an employee to take a photograph of the intersection where the accident occurred. The employee took the photograph and gave it to the owner, who in turn gave it to his lawyer. The lawyer wishes to introduce the photograph into evidence at trial of the owner's lawsuit against the defendant. The lawyer plans to have the employee testify that he took the photograph. The lawyer also plans to call a witness who lives in the neighborhood of the accident scene and arrived at the intersection shortly after the accident occurred. The witness is willing to testify that the scene in the photograph is in fact the intersection where the accident happened. Whose testimony is necessary to introduce the photograph into evidence? A The employee's testimony is necessary and the witness's is unnecessary. B The witness's testimony is necessary and the employee's is unnecessary. C The testimonies of both the employee and the witness are necessary. D The picture is inadmissible.

(B) Only the witness's testimony is necessary to introduce the photograph. To be admissible, a photograph must be identified by a witness as a portrayal of certain facts relevant to the issue, and verified by the witness as a correct representation of those facts. It is sufficient if the witness who identifies the photograph is familiar with the scene or object depicted It is not necessary to call the photographer to authenticate the photograph. Here, the actual physical appearance of the intersection is most likely relevant to the manner in which the accident occurred. As a resident of the neighborhood in which the accident took place, and as someone who was at the scene of the accident shortly after its occurrence, the witness is sufficiently familiar with the scene to testify that the photograph is an accurate representation of the accident scene. Such identification by the witness is needed for the photograph to be admissible. (A) incorrectly categorizes the employee's testimony as necessary. Generally, a photographer's testimony is not necessary to authenticate a photo. In this case, it is particularly unhelpful because the employee is not familiar with the scene as it was when the accident occurred. Also, the testimony of the witness is necessary as a verification by one who is familiar with the scene. (C) is incorrect because, as stated above, the testimony of the employee, the photographer, is not necessary. (D) is incorrect because the photograph is admissible if properly identified by the witness.

Question 11- Character evidence to establish identity - Intro The complainant was robbed by a man wielding an unusual knife with a pearl-studded handle. The defendant was arrested and charged with armed robbery of the complainant. At trial, the prosecution calls a witness to testify that, three days after the robbery of the complainant, she was robbed by the defendant with a knife that had a pearl-studded handle. Should the court rule that the witness's testimony is admissible? A Yes, as showing habit. B Yes, as establishing an identifying circumstance. C No, because it is improper character evidence. D No, because its probative value is substantially outweighed by the danger of unfair prejudice.

(B) The court should admit the evidence for purposes of establishing identity. Other crimes and wrongs are generally not admissible to prove that a person acted in conformity with his bad character. However, they are sometimes admissible for other purposes, such as to establish the identity of the accused. Other crimes are admissible on identity when they are committed in a very unique way that shows what amounts to a "signature" of the perpetrator. Theoretically, even signature crimes can be excluded if the judge determines that the probative value is substantially outweighed by prejudice. However, a crime qualifying as a signature crime is highly probative and would rarely be excluded under that theory. (C) and (D) are not wrong but, given the highly unique weapon, the court should hold that the evidence is admissible. (A) is wrong. One other crime could not establish habit.

Question 9-State of Mind- Intro A wife is on trial for the murder of her husband. She is accused of pushing him from the window of their 13th floor apartment; she claims he committed suicide. The wife called an operator for a suicide-prevention clinic to testify that the deceased husband had called the clinic on more than one occasion, each time telling the operator that he wanted to "end it all." Is the testimony admissible? A Yes, because the statements were made in "contemplation" of death. B Yes, because it tends to show that the husband intended to commit suicide. C No, because it violates the psychiatrist-patient privilege. D No, because no phone calls were made to the clinic by the husband on the day he died.

(B) The court should rule that the testimony is admissible under the state of mind exception to the hearsay rule, a declaration of intent to do something in the future is admissible as circumstantial evidence tending to show that the intent was carried out. Here, the husband's statements to the operator tend to show that the husband intended to commit suicide, so they are admissible to prove that he did so. (A) is incorrect because a "dying declaration" must concern the cause or circumstances of what the declarant believed to be his "impending" death. Although the husband made threats to end his life, there is no indication he believed his death was impending, and he did not discuss the cause or circumstances of his impending death. (C) is incorrect because the operator is not a psychiatrist, and there is no evidence that the husband assumed her to be one. (D) wrong because the state of mind need not be as of the time of the incident to be relevant.

Question 14- Public funds for Private schools- Intermediate As part of legislation enacted for the stated purpose of improving science skills of schoolchildren, Congress appropriated funds to permit public school teachers who had been certified by state school districts as science lab instructors to provide supplemental science instruction to any students in either public or private schools who did not have access to science lab resources. To help ensure content neutrality, the statute required the instructors coming to the private schools to use portable science labs supplied by the public school districts, which contained the equipment and experiments that the instructors used for the same purpose in the public schools. A citizens' group filed suit in federal district court to challenge the constitutionality of funding the science teachers for private schools, alleging that most of the private schools covered by the statute were religiously affiliated schools. No members of the group have any children in either public schools or private schools affected by the statute. How is the court likely to rule? A Dismiss the case on the pleadings, because the citizens' group does not have a sufficient stake in the controversy to have standing to challenge Congress's expenditure, which was authorized under its power to spend for the general welfare. B Decide the case on the merits in favor of the government, because the legislation defines the context in which instruction can be provided in private schools so as to avoid excessive government entanglement with religion. C Decide the case on the merits in favor of the citizens' group, because the appropriation's primary effect advances religion in violation of the Establishment Clause of the First Amendment. D Decide the case on the merits in favor of the citizens' group, because the court will presume that any instruction provided on the premises of a religiously affiliated school will be influenced by religion.

(B) The court will probably decide in favor of the government on the merits. Programs of aid to religiously affiliated grade schools and high schools are subject to the same three-part test as are other laws under the Establishment Clause: The program must (i) have a secular purpose, (ii) have a primary effect that neither advances nor inhibits religion, and (iii) not produce excessive government entanglement with religion. With respect to the first prong of the test, most of the time such programs (including this one) will have a secular purpose-to aid education. With respect to the second prong, the program may be deemed to have a primary effect that advances religion if it results in governmental indoctrination of religion or defines its recipients by reference to religion. Here, the statute establishes a religiously neutral program that funds a supplemental service for the schools, and offers the instruction to all disadvantaged students regardless of whether they choose to attend public or private schools. [See Agostini v. Felton (1997)--government program providing remedial education services to all disadvantaged children at their schools, including children at parochial schools, held not to violate the Establishment Clause]. Thus, (C) is incorrect (D) is incorrect because the courts will not presume that the instruction provided by this program will be influenced by religion. Furthermore, with respect to the "excessive entanglement" prong of the test, there is no indication that the program requires detailed monitoring of the government employees to prevent them from incorporating religion in their instruction--the equipment and experiments that they use in the private schools are the same as they use in the public schools. (A) is incorrect because the citizens' group has standing to challenge the expenditure on behalf of its members, who have a right to sue based on their status as federal taxpayers. The one recognized exception to the rule that people do not have standing as taxpayers to challenge the way tax dollars are spent by the federal government is if the expenditure was enacted under Congress's taxing and spending power and allegedly exceeds the specific limitation on that power found in the Establishment Clause. That exception applies here because the citizens' group is alleging that the federal appropriation is an unconstitutional attempt to provide government funds to religiously affiliated schools.

Question 6- Burden of Proof- Intro A defendant, on trial for robbing the victim of some jewelry, relied on the defense that he was only trying to recover property that the alleged victim had previously stolen from him. The trial court instructed the jury that the prosecution must prove guilt beyond a reasonable doubt, and that if the jury should find that the defendant had established by a preponderance of the evidence that he was only trying to recover his property, they should find him not guilty. After he was convicted of robbery, the defendant asserts that the instruction to the jury was error. Is it likely that his conviction will be reversed? A. Yes, because the defendant need only convince the jury of any defense to a reasonable certainty, not by a preponderance of the evidence. B Yes, because the instruction put a burden on the defendant that denied him due process of law. C No, because the defendant's burden to show that he was trying to recover his property was not one of ultimate persuasion, but only to produce evidence to rebut the legitimate presumption that the robbery was conducted with the intent to permanently deprive the victim of the jewelry. D No, because the instruction was an accurate statement of the law.

(B) The defendant's conviction should be reversed. Robbery requires an intent to permanently deprive the victim of her property. An intent to recover property that the defendant believes is his would not be a sufficient intent. The prosecution must prove every element of the crime beyond a reasonable doubt, and putting the burden of persuasion to show an innocent intent on the defendant would deprive him of due process of law, since it would relieve the prosecution of its burden to show the required intent for robbery. Thus, the instruction was improper and (D) is wrong. (A) and (C) are wrong. The defendant does not carry any burden of proof with respect to an element of the crime. The burden is on the prosecution to prove each element beyond a reasonable doubt.

Question 19-Prior Conviction- Intermediate A defendant was charged with arson (a felony) of an antique shop. Only one corner of the shop was damaged before the fire was extinguished under a plea agreement, the defendant pled guilty and received a suspended sentence. Because the owner of the shop had not yet insured a recently acquired 400-year-old refectory table that was destroyed by the fire, he sued the defendant for damages. At trial, the owner offers the properly authenticated record of the defendant's conviction for arson. Should the record be admitted into evidence? A Yes, as proof of the defendant's character in order to infer liability. B Yes, as proof that the defendant set the fire. C No, because the conviction was not the result of a trial. D No, because it is hearsay not within any exception.

(B) The record of the defendant's conviction should be admitted to prove that the defendant set the fire. The record of the conviction is hearsay; i.e., it is a statement, other than one made by the declarant while testifying at the trial or hearing, offered to prove the truth of the matter asserted under the Federal Rules, however, such judgments fall within the hearsay exception for records of felony convictions. Under the Federal Rules, judgments of felony convictions are admissible in both criminal and civil actions to prove any fact essential to the judgment, whether the judgment arose after trial or upon a plea of guilty. [Fed R Evid 803(22)] For purposes of this Rule, a felony is any crime punishable by death or imprisonment in excess of one year. Arson is a felony. Consequently, a properly authenticated copy of the defendant's conviction of this crime is admissible to prove the fact that the fire that destroyed the table was set by the defendant, a fact essential to the judgment of conviction. Note that the actual plea of guilty is also admissible as a statement of a party-opponent (commonly called an admission). This type of judicial admission is not conclusive, and the defendant may explain the circumstances of the plea. The plea, being an admission, is nonhearsay under the Federal Rules. (A) is incorrect because, in a civil case, evidence of character to prove the conduct of a person in the litigated event is generally not admissible. Circumstantial use of prior behavior patterns for the purpose of inferring that, at the time and place in question, a person probably acted in accord with such patterns creates a danger of prejudice and distraction from the main issues. Therefore, the record of the conviction cannot be used to infer liability by showing the defendant's character. (C) is incorrect because, as noted above, a judgment of a felony conviction is admissible under Federal Rule 803(22) regardless of whether the conviction resulted from a trial or a guilty plea. (D) is incorrect because, as discussed above, the judgment is within the exception to the hearsay rule for records of felony convictions.

Question 3-Excited Utterance - Intro In the prosecution of the defendant for bank robbery, it is established that as the robber came out of the bank, he was seen entering a car by a group of people, including a witness and his friend. The witness is prepared to testify that as the car drove off, someone yelled, "Get that number,' whereupon the friend screamed, "I've got it. The number is 07771!" How should the court rule on the admissibility of the witness's testimony? A Admissible only if the friend fails or refuses to testify to such facts, because the friend's testimony would be the best evidence thereof. B Admissible hearsay. C Inadmissible hearsay. D Inadmissible, because there is no proper foundation or identification of the hearsay declarant.

(B) The friend's statement is admissible under the excited utterance exception to the hearsay rule Although hearsay is generally inadmissible, certain kinds of hearsay are deemed to be reliable enough to be admitted. Among these exceptions is one for excited utterances under this exception, a declaration made during or soon after a startling event is admissible. The declaration must be made under the stress of excitement produced by the startling event (i.e., before the declarant has time to consciously reflect on the occurrence). Also, the declaration must concern the immediate facts of the startling occurrence. [Fed R Evid 803(2)] Here, the friend's statement (to be testified to by the witness) is an out-of-court declaration, offered to prove the truth of the matter asserted (i.e., that the license plate number of the getaway car was 07771). Thus, the friend's statement is hearsay. However, this statement was made during the course of a bank robbery, an event startling enough to produce nervous excitement in the friend. The statement was made under the stress of the excitement produced by the robbery, and concerned the immediate facts of the robbery (i.e., it referred to the car in which the robber was making his escape). Therefore, the statement qualifies as an excited utterance, rendering the witness's testimony admissible. (A) is wrong because it represents an incorrect application of the best evidence rule. The best evidence rule applies only when a party is trying to prove the terms of a writing. Here, there is no writing involved; thus, the best evidence rule is inapplicable. (C) is wrong in characterizing the friend's statement as inadmissible. Although the statement is hearsay, as explained above, it is admissible as an excited utterance. (D) is wrong because no foundation is required for the witness to testify to the excited statement he heard. Also, note that the hearsay declarant (the friend) contrary to the language of (D), identified as the person who made the statement.

Question 23- Hearsay Silence as Admission - Intermediate A plaintiff sued a defendant for negligence when the tractor that he was driving at a construction site collided with the plaintiff's car. The plaintiff alleged that she was driving in a proper lane when the tractor collided with her car. The plaintiff's counsel called the responding police officer to testify that the defendant's employee made a statement to the police officer, in the defendant's presence, that the defendant "accidentally went too far into traffic," and the defendant did not say anything. Should the trial judge rule that this evidence is admissible? A Yes, because the employee's statement is a statement against interest. B Yes, because silence may be deemed an admission. C No, because the employee's statement was hearsay, and the defendant's silence is also hearsay. D No, because the employee has not yet testified to his statement.

(B) The judge should rule the evidence admissible as an admission. Admissions-statements by or attributable to a party and offered against that party-are not considered hearsay under the Federal Rules. If a party fails to respond to accusatory statements where a reasonable person would have spoken up, his silence may be considered an implied admission. Although it is arguable that a person who may be liable for negligence would reasonably remain silent when in the presence of a police officer, this is the best answer. (D) is wrong because there need be no foundation evidence. (C) is wrong; the silence is deemed the adoption of the employee's statement. Thus, both the employee's statement and the defendant's silence would be admissible. (A) is wrong because the statement against interest exception does not apply here. To fall within the exception, a statement must be against the declarant's own interest when made-here, the employee's statement was against the defendant's interest, not his own. Furthermore, the exception requires the unavailability of the declarant (i.e., the employee), and there is no indication that the employee is unavailable.

Question 22 - Expert witness Authoritative text substantive evidence - Intermediate An expert witness, who has an advanced degree in engineering, is testifying at a jury trial to the possible causes for the failure of an enclosed pedestrian bridge in a shopping mall. Which of the following data, if relied on by the expert witness in forming his opinion, is admissible on direct examination as substantive evidence? A Statements told to him by witnesses to the collapse, as long as such statements are reasonably relied on by experts in his field. B Statements that he reads from a text on structural engineering that he has testified is authoritative. C Testimony regarding the repair of the bridge one week after the collapse, indicating that it is now equipped with special safety features that were not installed prior to the collapse. D Statements made to him by a former maintenance employee of the shopping mall indicating that the management knew the bridge needed repairs but delayed doing so because the bust holiday season was approaching.

(B) The jury may consider statements from an authoritative text as substantive evidence. Rule 803(18) provides that an expert may base an opinion on facts supplied to him outside the courtroom. One such source is authoritative texts and treatises. Statements from a treatise established as reliable (which may be done by the expert's own testimony) may be introduced on direct examination of the expert and read into the record as substantive evidence under an exception to the hearsay rule. [Fed R Evid 803(18)] Hence, the jury may consider the testimony in (B) as substantive evidence. (A) is incorrect because the evidence may not be considered by the jury as substantive evidence. Rule 703 provides that where an expert bases his opinion on facts made known to him outside the courtroom, the facts need not be of a type admissible in evidence as long as the facts are of a kind reasonably relied on by experts in the particular field. However, the expert will only be permitted to disclose such facts if the court determines that their probative value in assisting the jury to evaluate the expert's opinion substantially outweighs their prejudicial effect. The jury would only be permitted to consider them as the basis for the expert's opinion; the jury could not consider the facts as substantive evidence unless they were independently admissible. Here, the statements made to the expert by witnesses to the collapse are a proper basis of his opinion because they are of a kind reasonably relied on by experts in his field. However, they do not appear to be admissible under any exception to the hearsay rule and therefore could not be considered by the jury as substantive evidence. (C) is incorrect because it is too broad. Generally, evidence of repairs or other precautionary measures made after an incident is inadmissible if used to prove negligence, culpable conduct, product or design defects, or a need for a warning or instruction. Such evidence may, however, be admitted to prove ownership or control, to rebut any claim that precautionary measures were not feasible prior to the incident, or to prove destruction of evidence. Here, admissibility of the expert's testimony regarding the subsequent repair of the bridge will be contingent on the purpose of the testimony, and (C) is incorrect because it does not indicate any purpose. (D) is incorrect. Generally, a statement made by an agent concerning a matter within the scope of his agency is admissible against the principal as a vicarious admission if it was made during the existence of the employment relationship. In this case, the statement was made by a former employee of the mall, and therefore it was not made during the scope of the employment relationship. Because it does not fall within any of the hearsay exceptions, the statement is inadmissible.

Question 7 -Proper Impeachment-Bias-Intro A truck driver is suing a car driver for injuries he suffered when their vehicles collided at an intersection controlled by stoplights. The truck driver called a witness to the accident to testify that he saw the driver of the car drive through a red light. On cross-examination, the car driver's attorney asks the witness, "Isn't it true that the car driver's ex-wife is paying $500 for your testimony today?' The truck driver's attorney objects. Should the objection be overruled? A Yes, because the question gives the witness an opportunity to explain or deny the allegation. B Yes, because the question is a proper form of impeachment. C No, because the question addresses a collateral issue. D No, because it is a leading question.

(B) The objection should be overruled. It proper to impeach a witness by showing that the witness has a possible bias. Evidence that the witness is being paid to testify would be proper impeachment through bias. (A) states the requirement for introduction of a prior inconsistent statement; this is obviously inapplicable here. The credibility of a witness is not collateral, and so (C) is incorrect. Leading questions are proper on cross-examination; thus (D) is incorrect.

Question 8- Party/Opponent- Intro In a college student's civil suit for personal injuries against her stepfather, arising from acts of sexual abuse allegedly committed by him against her when she was a minor, the student calls as a witness a police officer who will testify that, 11 years ago, the stepfather confessed to the witness that he had committed the acts complained of by the student. Should the trial court admit the police officer's testimony over the stepfather's objection? A Yes, because past instances of misconduct may be used to impeach a witness. B Yes, because the stepfather's confession is an admission of a party-opponent. C No, because the best evidence of a conviction is the judgment of the court that convicted him. D No, because the sexual assault that is the subject of the evidence is more than 10 years old.

(B) The officer's testimony should be admitted because an admission by a party-opponent. Generally, evidence is admissible if it is relevant, e.g., if it has a tendency to prove any fact of consequence to the action. The testimony here is relevant because it makes it more likely that the defendant committed the acts complained of. It is also competent because its admission does not violate any exclusionary rule of evidence. Evidence law generally prohibits admission of hearsay--out-of-court statements offered to prove the truth of the matter asserted--but the rule does not apply to admissions by party-opponents, which are treated as nonhearsay under the Federal Rules. An admission is a statement made by a party and offered against that party, and here the stepfather's statement is being offered against him by the student. Thus, the testimony is admissible nonhearsay and (B) is correct. (A) may be a true statement--past instances of misconduct may be used to impeach a witness, at least if they are probative as to whether he is worthy of belief --but it is an incorrect answer because it focuses on impeachment, and the defendant is not being impeached here. (C) is incorrect because the best evidence rule is not applicable here. The rule requires that when the contents of a writing are sought to be proved, the writing itself should be admitted if it is available. Here, however, the witness is not trying to prove that the defendant has been convicted, but rather that he made the admission. (D) is incorrect for reasons similar to why (C) is incorrect. It true under the Federal Rules that convictions generally are not admissible for impeachment of a witness if more than 10 years old, but the officer is not trying to admit the defendant's conviction here (and is not trying to impeach the defendant, who may not even have testified). Rather, the student is seeking to admit the defendant's confession. The confession is an admission by a party-opponent and is admissible as substantive evidence.

Question 4 - Bias- Intro A defendant is charged with having been one of tm men who robbed a bar and its patrons at gunpoint at 5:30 pm on December 16. The defendant calls a witness in his defense who testifies that she drove to the defendant's home at 10 am on December 16 and picked up the defendant and his wife, then took them to a birthday party that lasted until 7 pm. The prosecutor asks on cross-examination, "What is your relationship to the defendant's wife?" Defense counsel objects. How should the court respond? A Overrule the objection, because the Question attacks the witness's truth and veracity. B Overrule the objection, because the Question is directed at discovering possible bias in the witness. C Sustain the objection, because the Question seeks to elicit irrelevant information. D Sustain the objection, because the answer to the Question would create prejudice that would outweigh its probative value.

(B) The prosecutors Question is aimed at discovering bias, which tends to show that the witness has a motive to lie. Impeachment involves the casting of an adverse reflection on the veracity of a witness, and it may take several forms. Evidence that a witness is biased tends to show that she has a motive to lie, and is thus a well-recognized method of impeachment. Inferences of bias may be shown by evidence of family or other relationship. Here, the prosecutor is attempting to show that, due to a family relationship or friendship with the defendant's wife, the witness may be biased and would thus have a motive to lie on behalf of the defendant. Therefore, the prosecutor is engaging in an accepted method of impeachment. (A) correctly states that the prosecutors Question attacks the witness's truth and veracity. However, not all methods of attacking a witness's truth and veracity are admissible. (B) is a better answer than (A) because it identifies the specific method of impeachment that the cross-examiner is using. (C) is incorrect because evidence that tends to prove or disprove the credibility of a witness is relevant. The information sought to be elicited by the prosecutor's Question will reflect on a possible motive that the witness may have to lie. Therefore, such information is relevant. (D) incorrectly states that the answer to the prosecutor's Question would create prejudice that would outweigh its probative value. Under Federal Rule 403, evidence may be excluded if its probative value is substantially outweighed by the danger of unfair prejudice, confusion of the issues, or misleading the jury. While any material evidence introduced by a party will probably be prejudicial to the adverse party's case, it is only unfair prejudice (i.e., suggesting a decision on an improper basis) that may be excluded under this rule. Here, the answer to the prosecutor's question would clarify the matter of whether the witness had a motive to lie. This would not be unfairly prejudicial because it would tend to make the witness's testimony as to the defendant's whereabouts at the time of the alleged crime more or less believable, which is the proper basis on which the trier of fact should accept or reject her testimony.

Question 2 - Custodial Interrogation - Introductory After being arrested on suspicion of murder, a suspect was taken to the police station and informed of his constitutional rights as required by the Miranda decision. He immediately requested that a lawyer be provided because he had no money to hire one. The arresting officer said that he would get the suspect's lawyer after he was booked, and the officer proceeded to book him. During the booking search, the suspect said to the arresting officer, "l only killed the bastard because he made a pass at me." If the suspect attempts to prevent introduction of the statement made by him to the officer during booking, will he most likely succeed? A No, because booking is not a critical stage of criminal proceedings requiring the assistance of counsel. B No, because the statement was not the result of a custodial interrogation. C Yes, because the statement was the product of illegal police conduct. D Yes, because the request for an attorney should have been honored immediately.

(B) The suspect will not be successful in preventing the introduction of his statement because it was not the result of a custodial interrogation. If, after being given his Miranda warnings, the suspect invokes his right to counsel, all interrogation must stop until counsel is present. "Interrogation" includes any words or actions by the police that the police should know are likely to produce an incriminating response. The suspect may volunteer information to the police at any time. (A) is not the best answer. While it is true that booking is not a critical stage for purposes of the right to counsel, that is not the reason the statement will be allowed. The statement would be excluded if it had been the result of interrogation during the booking. <(C) is incorrect; there is no illegal police conduct set forth in the facts. (D) is an incorrect statement of law.

Question 10- Lay Witness - Intro A homeowner was helping his neighbor clean out her garage. As the homeowner and the neighbor were moving some boards out of the garage, the neighbor ran into the overhead door's hinge, causing it to fall and hit the homeowner on his head, killing him. The homeowner 's wife sued the neighbor for the wrongful death of her husband, seeking damages of $100,000. The wife, who was present when the accident occurred, is called to testify that at the time of the accident, the homeowner was carrying the boards on his shoulder, and he therefore was unable to see that the neighbor had hit the door hinges. Is the wife competent as a witness? A Yes, if she is testifying as the personal representative of her husband's estate. B Yes, in spite of the fact she is the plaintiff. C No, because she is unqualified to give opinion evidence. D No, because she cannot testify for her husband in a civil case.

(B) The wife is competent to testify Persons interested in a lawsuit are not disqualified as witnesses. (A) is wrong because the wife need not be the personal representative of her husband's estate to testify. (C) is incorrect because she is testifying as to what she saw, not as to her opinion. In any event, a lay witness may testify as to an opinion if she is a percipient witness. (D) is wrong because the common law spousal incapacity has been abolished.

Question 22- Present Sense Impression - Intermediate A pedestrian was lawfully crossing the street when he was struck and seriously injured by a car that had run a red light. At the time, the driver was on his way home from a local tavern. The pedestrian brings suit against the tavern for his injuries, claiming that the driver was permitted to drink too much liquor at the tavern before leaving. The pedestrian calls a witness to the stand. The witness testifies that she and a friend had visited the tavern on the night in question. The witness seeks to testify that she remarked to her friend, "Look at that guy. He's so drunk he can't even stand up." Is the witness's testimony concerning her remark to her friend admissible? A Yes, as a prior consistent statement. B Yes, as a present sense impression. C Yes, as relevant nonhearsay testimony. D No, as hearsay not within any exception.

(B) The witness's remark is admissible under the present sense impression exception to the hearsay rule under this exception, a statement is admissible if the declarant comments on some event or condition while observing the event or condition or immediately thereafter. [Fed R Evid 803(1)] Here, the witness made the statement concerning the driver's intoxication while she was observing the driver, so the exception applies. (A) is wrong. A prior consistent statement can be used to rebut evidence that the trial testimony is a lie or an exaggeration because of some motive to falsify that has arisen since the event, or that it is a misstatement of the facts because of, e.g., sensory deficiencies. The facts do not indicate such a claim was made. The statement is hearsay because the witness is relating her own out-of-court statement that is now being offered for its truth (to prove that the driver was visibly intoxicated while at the tavern); therefore, (C) is wrong. (D) is wrong because the statement comes within the present sense impression exception to the hearsay rule.

Question 20 - Separation of Powers- Rulemaking Authority/Commission - Intermediate Congress enacted legislation intended to protect children from unsafe car seats. The act established a commission to supervise the manufacturing and sale of car seats, and empowered the commission to promulgate car seat safety regulations. The commission members were also required to investigate safe and sound methods of installing child car seats. The commission's chairperson was designated as an undersecretary of Health and Safety; the President appointed two commissioners from child safety groups; and the three major car seat manufacturers chose one commissioner each, who were then appointed by Congress to the commission. For its violation of the commission's rules with regard to car seat manufacturing, a car seat manufacturer was fined $5,000, to be paid immediately without a trial on the merits. The manufacturer files suit in the federal court to enjoin the commission's enforcement of this rule. Which of these is the manufacturer's best argument in support of its contention that the rule was illegal? A Regulations concerning criminal conduct cannot be made by agency rules, but must be made by federal statute. B The appointment of the commissioners was illegal; therefore, the rules promulgated by the Commission are invalid. C Because the fine was potentially $5,000 for violation of the rule, the manufacturer had a right to a trial by jury. D The presumptive fine violated the manufacturer's rights of equal protection as guaranteed by the Fourteenth Amendment.

(B) is correct. The appointment of the commissioners was illegal. The Appointment Clause of the Constitution permits Congress to vest appointments of inferior officers only in the President, the courts, or the heads of departments. Enforcement is an executive act; therefore, Congress cannot appoint its own members to the commission to exercise enforcement powers. A duly appointed commission does have the power to make rules and regulations governing the subject matter for which it is appointed. Those rules are not "criminal" statutes in this case. Thus, (A) is wrong. (C) is wrong because Congress may establish new public rights and actions that may be adjudicated by agencies, without juries. (D) is wrong because there is no actionable discrimination.

Question 5- Speech Permits, Void Ordinance - Intro A city ordinance provided that anyone who wanted to speak in a public park must have a permit to do so issued by the city. The ordinance granted the mayor the power to issue or deny such permits based on the mayor's judgment of whether the speech would be "in the public interest." The mayor has never denied a permit to anybody desiring to speak on a political topic. A city resident who was unhappy with the city government went to a public park in the city square. There, the resident made a 10-minute speech accusing the mayor and the city council of gross incompetence and urging voters to "throw the rascals out" at the next election. The city resident had not applied for a permit. After the resident completed his oration, the police arrested him and charged him with violating the permit ordinance. Would a conviction of the resident be constitutional? A Yes, because the resident did not have a permit to speak, and a municipality has the right to regulate the time, place, and manner of speech. B Yes, because the mayor would have issued the permit, because the resident's speech was on a political topic. C No, because the ordinance is void on its face. D No, if the resident could prove that the mayor would not have issued him a permit to speak.

(C) A conviction of the resident would not be constitutional because the ordinance is void on its face. Although a municipality can place reasonable time, place, and manner restrictions on certain aspects of speech, it may not adopt a regulation that gives officials broad discretion over speech issues. If a statute gives licensing officials unbridled discretion, it is void on its face, and speakers need not even apply for a permit. They may exercise their First Amendment rights even if they could have been denied a permit under a valid law, and they may not be punished for violating the licensing statute. Here, the law allows the mayor to grant or deny permits based on his assessment of public interest. This is too much discretion to be valid. Therefore, the ordinance is void. (A) is wrong because, as explained above, the ordinance was void on its face and thus need not be obeyed. Therefore, the resident did not need to apply for the permit. Also, although it is true that the municipality has a right to reasonable restrictions, this ordinance is not reasonable because it gives too much discretion to the mayor. (B) is wrong because even if the mayor has not abused his discretion, the ordinance is void on its face and thus need not be obeyed. (D) is wrong because, as stated above, the resident's case does not depend on whether the mayor would grant or deny the permit.

Question 11- Intro To combat the problem of obesity among the nation's youth, a state adopted a statute providing funds to train a corps of nutritionists who could go to schools and run six-week seminars designed to encourage healthy eating and exercise habits. The administration of a private school that has a stated policy of admitting only white students has requested that a state nutrition corps member be sent to the school to run a six-week seminar. Which of the following is the best constitutional argument supporting the state's rejection of the request? A The Constitution prohibits private discrimination. B The state may not aid private schools. C Segregation would be furthered by the provision of the seminar. D No legitimate educational function is served by giving nutrition seminars at private schools.

(C) If the state provides a nutritional corps member to a segregated school, it would represent a significant involvement in activities of a school that practices segregation and would thus violate the Equal Protection Clause. Thus, answer (C) is correct because segregation would be furthered by the provision of a nutritional corps seminar. (A) is incorrect because it is too broad. The Constitution prohibits slavery, and the Supreme Court has interpreted the enabling clause of the Thirteenth Amendment to give Congress the power to outlaw anything that it sees as an incident or badge of slavery. However, here we are dealing with state legislation, and nothing in the facts indicates that Congress has found the discrimination here to be a badge of slavery. (B) also is a gross overstatement. Government may aid private schools as long as such aid does not run afoul of the Constitution (e.g., does not support ethnic, racial, or religious discrimination). (D) is incorrect because that is not the test the courts would apply here.

Question 14- Executive privilege - Intro After the release of various news stories about the President's possible violation of political campaign funding laws, a federal grand jury investigation and an investigation by a special Senate subcommittee were initiated. The Senate subcommittee subpoenaed documents and records from several top officers of the executive branch. Learning of the subpoenas, the President ordered all executive officials to refuse to turn over materials, claiming "executive privilege." Which of the following statements is most accurate? A The subpoena violates the constitutional principle of separation of powers. B The President's executive privilege is absolute, except in cases of impeachment. C The presidential papers are presumptively privileged, but the privilege must yield to a demonstrated specific need for evidence in a pending legislative proceeding. D The President's executive privilege applies to proceedings by Congress, but not to proceedings by the courts

(C) Executive privilege is an inherent privilege necessary to protect the confidentiality of presidential communications under this privilege, presidential documents and conversations are presumptively privileged, but this privilege must yield to a demonstrated need for such materials as evidence in a criminal case in which they are relevant and otherwise admissible. [United States v. Nixon (1974)]. Although the Supreme Court has not expressly decided that the privilege must also yield to a demonstrated need for evidence in a pending legislative proceeding, such an extension of Nixon is likely, and none of the other alternatives is at all accurate. (A) is incorrect because it is too broad. In Nixon, supra, the Court decided that an evidentiary subpoena to the President in a criminal case does not violate the separation of powers principle. By extension a subpoena issued by a Senate subcommittee, pursuant to the well-established implied power of Congress to investigate, would not be deemed to violate separation of powers. (B) is also incorrect because it is too broad. As stated above, although a presumptive privilege applies to presidential documents and conversations, that privilege must yield to a demonstrated need in criminal cases. Thus, executive privilege is not absolute. (D) is incorrect because executive privilege does apply to proceedings by the courts; in fact, the privilege overridden only on a specific showing of need for specific information.

Question 18- Candidate Name on Ballot - Advanced To keep election costs manageable, a state law required a candidate in a general election to collect on a petition supporting her candidacy the signatures of at least 4% of the voters eligible to vote for the office for which the candidate was running. Studies revealed that, prior to adoption of the law, a candidate who did not have at least 4% of the voters supporting her initially had never won an election. An independent candidate for governor who had limited campaign resources was supported in the polls by 30% of the voters, but had been able to obtain only 3.5% of the eligible voters' signatures by the filing deadline. The board of elections refused to put her name on the ballot. If the candidate sues to have her name placed on the ballot, claiming that the state's petition requirements are unconstitutional, which of the following statements is most accurate? A She will have to show that the petition requirement is not rationally related to a legitimate state purpose. B She will have to show that the petition requirement is not necessary to achieve a compelling state interest. C The state will have to show that the petition requirement is narrowly tailored to achieve a compelling state interest. D The state will have to show that the petition requirement is rationally related to a legitimate state interest.

(C) Most likely, the state will have to show that its petition requirement is narrowly tailored to achieve a compelling state interest. The Supreme Court uses a balancing test in determining whether a regulation of the electoral process is valid: if the restriction on First Amendment activities is severe, the regulation will be upheld only if it is narrowly tailored to achieve a compelling interest. Requiring a candidate to obtain 4% of voters' signatures probably qualifies as a severe First Amendment restriction. Thus, the state must show that the regulation is narrowly tailored to achieve a compelling interest (e.g., running an honest and efficient election system). Note that the Court has approved a requirement that candidates obtain 1% of the voters' signatures before being placed on the ballot [See Munro v. Socialist Workers Party (1986)]. In any case, the Court probably would not apply the rational basis standard under these facts, so (A) and (D) are incorrect. (B) is incorrect because when the Court applies heightened scrutiny, requires the government to bear the burden of persuasion, not the person challenging the government's action.

Question 20- Probative Testimony- Intermediate A businessperson filed a defamation suit against a newspaper for printing a column that referred to the businessperson as "a nasty miser" and 'worse than Ebenezer Scrooge," and accused him of "never performing a real act of charity in his life." During the presentation of the businessperson's case, he wanted to put an agent of the Internal Revenue Service on the stand. The agent is prepared to testify that the businessperson, on his own initiative, reimbursed the IRS for an erroneous overpayment of a tax refund. Counsel representing the newspaper objects. How is the court likely to rule on the agent's testimony? A Admissible, because the businessperson's character is at issue in the case. B Admissible, because the businessperson has a right to defend his good character. C Inadmissible, because the agent's testimony, in and of itself, is not probative of any material issue in the case. D Inadmissible, because specific instances of conduct are not admissible to prove character.

(C) The agent's testimony is inadmissible because it is not probative of any material issue in the case. Relevant evidence tends to make the existence of any fact that is of consequence to the determination of an action more probable than it would be without the evidence. [Fed R Evid 401] While evidence tending to prove the businessperson's charitable nature, which is a material issue in this case, would be relevant, the evidence here tends to prove only the businessperson's honesty, which is not at issue here. Therefore, it is not relevant and should not be admitted. (A) is incorrect because even though the businessperson's character has been called into question in this case, only evidence that is probative of the particular character trait in issue may be admitted. When proof a person's character, as a matter of substantive law, is an essential element of a claim or defense in a civil action, is said that character is "directly in issue." In such a case, evidence of specific acts may be used to prove character. [Fed R Evid 405(b)] Because this is a defamation case, the businessperson's character as to generosity is directly in issue, and specific acts may be used to prove his generosity. However, as discussed above, the agent's testimony is not probative of the businessperson's generosity. (B) is similarly incorrect. The businessperson has a right to prove his good character, but only with regard to the particular character trait that has been defamed. (D) is incorrect because specific acts may be used to prove character when character is directly in issue, as discussed above. The businessperson's generosity is directly in issue.in this case and specific acts of his may be used to prove his generosity. Therefore, the agent's testimony is not inadmissible on these grounds.

Question 7- Intro Hoping to help control a growing problem with violent all-male teenage gangs who frequented a youth center, a city council passed an ordinance forbidding any male between the ages of 13 and 19 to enter the center unless accompanied by a female. An 18-year-old boy who was new to the city went to the youth center to try to meet new friends. He was refused admission because he was not escorted by a female. Angered, the boy filed suit in federal district court seeking admission to the center and asking the court to strike down the ordinance as unconstitutional. Which of the following standards should the court use in determining the validity of the ordinance? A The boy must show that the ordinance is not rationally related to a legitimate state interest. B The boy must show that the ordinance is not substantially related to an important government interest. C The city must show that the ordinance is substantially related to an important government Interest. D The city must show that the ordinance is necessary to achieve a compelling interest.

(C) The city must show the ordinance is substantially related to an important government interest. Classifications based on gender are quasi-suspect and are therefore subject to intermediate scrutiny. Ordinances and statutes based on gender will violate the Equal Protection Clause unless the government shows by exceedingly persuasive justification that the ordinance is substantially related to an important government objective. (A) is incorrect because it puts the burden on the boy and states the wrong level of scrutiny (rational basis test). (B) is incorrect because while it states the correct test, it puts the burden on the boy. (D) is incorrect because while it places the burden on the correct party (i.e., city), it states the strict scrutiny test rather than the intermediate scrutiny test.

Question 20-Witnesses called by the court-intermediate A trial judge presiding over a lawsuit plans to call a witness to the stand and question her. May the judge do so? A. Yes, but only if the witness has already been called and examined by one of the parties. B Yes, but only if the witness is not testifying as an expert. C Yes, but the parties are entitled to cross-examine the witness. D No, because only parties may call and examine witnesses.

(C) The judge may call and examine the witness, but the parties are entitled to cross-examine the witness. A court is entitled to examine any witness called by any party, and may also call a witness on its own or at a party's request. Therefore, (D) is wrong. (A) is wrong because the court may call a witness on its own, even if the witness has not been called by any of the parties. (B) is wrong because the court may call any witness, even an expert. Although the court has wide discretion to call and examine witnesses, each party is entitled to cross-examine the court's witness. A party may also object to the court's examining or calling a witness either at that time or at the next opportunity when the jury is not present (to spare counsel the potential embarrassment of objecting to the judge's questions in front of the jury).

Question 8-Ordinance - Intro A city passed an ordinance prohibiting persons on a public sidewalk within 100 feet of a health care facility from approaching those seeking access to the facility for purposes of protest, education, or counseling. The day after the city's new ordinance became effective, a person advocating against abortion went to an abortion clinic within the city, stood 25 feet from its entrance, stopped the first woman whom she saw about to enter the clinic, and gave her a leaflet espousing the leafletter's religious views and discouraging abortion. The leafletter was promptly arrested for violating the city's ordinance. At trial, the leafletter challenges the ordinance on First Amendment grounds. How should the court rule on the leafletter's First Amendment defense? A In favor of the leafletter, because the ordinance violates the Free Exercise Clause. B In favor of the city, because the ordinance is reasonable as to time, place, and manner. C In favor of the leafletter, because the ordinance infringes on the freedom of speech. D In favor of the city, because patients' and visitors' freedom of association right to be left alone is being infringed upon by the leafletter.

(C) The court should rule in favor of the leafletter, because the ordinance is not a valid time, place, and manner restriction on the exercise of First Amendment rights. The First Amendment protects the freedoms of speech and assembly; however, the protection is not absolute. The government is allowed to adopt regulations concerning the time, place, and manner of the exercise of speech and assembly in public forums and designated public forums to facilitate order and to protect other important government interests. To be valid, such a law must: (i) be content-neutral; (ii) be narrowly tailored to serve an important government interest; and (iii) leave open alternative channels of communication. The ordinance here serves an important government interest--unimpeded access to health care facilities. It is content-neutral because it prohibits all protesting, educating, and counseling around health care facilities and not just such conduct concerning a particular message. However, it is not narrowly tailored because it restricts more speech than necessary to ensure access to health care facilities. In McCullen v Coakley (2014), the Supreme Court held that a "buffer zone" law prohibiting leafletters from approaching patients within 35 feet of any abortion clinic in the state was too broad a speech restriction when the state could try other measures, such as targeted injunctions. The Court had upheld a state buffer-zone law similar to the one in this fact pattern in Hill v Colorado (2000). However, the Court evaluates the constitutionality of such laws on a very case-specific basis, and in McCullen the Court indicated that it is more likely to uphold such measures as narrowly tailored if the measures attempt to remedy a specific problem at a particular facility or facilities, rather than impose a blanket speech restriction statewide. (A) is incorrect because the Free Exercise Clause does not protect the leafletters conduct here. The Free Exercise Clause provides that the government may not prohibit the free exercise of religion. However, the Clause does not provide an absolute right, and the Supreme Court has held that the government may regulate general conduct without violating the Constitution, even if the regulation happens to interfere with a person's or group's religious practices. The Clause merely prohibits the regulation of conduct because it is religious. Because the statute here a broad regulation and is not specifically aimed at religious conduct, it does not violate the Constitution. (B) is incorrect for the reasons explained above. Reasonable time, place, and manner restrictions on speech in public forums are allowed; however the buffer-zone ordinance here is not narrowly tailored and therefore fails the second prong of time, place, and manner. (D) is incorrect because, as a general rule, the Constitution does not protect people from the acts of other people, but rather from the acts of the government. Thus, the rationale of (D) does not make sense - a law will not be held "constitutional" because it prevents persons from violating others' constitutional rights.

Question 15- Nonhearsay- Intermediate A defendant is charged with the burglary of a warehouse. At the request of the police investigating the burglary, the night watchman at the warehouse who had seen the thief leaving the premises wrote out a description of the thief, who bore a strong likeness to the defendant. However, the night watchman died of a heart attack before the defendant was arrested and brought to trial. The prosecution attempts to offer the description written out by the night watchman into evidence. Is the description admissible? A Yes, as a past recollection recorded. B Yes, as an identification of a person the night watchman knew committed the crime in question. C No, because it is hearsay not within an exception. D No, because it is an opinion of a non-expert.

(C) The description is inadmissible as hearsay not within an exception. (B) is wrong under the Federal Rules of Evidence, prior identification can be admissible. The description is in the nature of prior testimony or prior identification. To be admissible, however, the declarant must testify at the trial and be subject to cross-examination. Since the night watchman died, this requirement cannot be satisfied. On these facts, there are no exceptions to the hearsay rule that would make the description admissible. (A) is wrong because before a document can be admitted as a past recollection recorded, the person whose statement appears in the document must be on the witness stand. (D) would have no bearing on the admissibility of the description.

Question 8 - Expert Witness -Intro In a personal injury action, a doctor sat in court and listened to all of the evidence regarding the plaintiff's injuries. The defense subsequently calls that doctor to testify as to his opinion about whether the plaintiff's injuries will prevent the plaintiff from ever working again. May the doctor testify to this? A No, because he has not examined the plaintiff. B No, because whether the plaintiff will work again is an issue for the jury. C Yes, because an expert's opinion can be based on facts made known to him at the trial. D Yes, because the doctor is not testifying as an expert because he has not examined the plaintiff.

(C) The doctor may testify. An expert's opinion may be based on one or more of the following sources of information: (i) facts that he knows from his own observation, (ii) facts presented in evidence at the trial and submitted to the expert, or (iii) facts not in evidence that were supplied to the expert out of court and which are of a type reasonably relied upon by experts in the particular field in forming opinions on the subject. Here, the doctor's testimony will be based on facts presented in evidence at the trial. Thus, (C) is correct. (A) is incorrect because personal examination by the doctor is not required in order to provide expert testimony as to the plaintiff's injuries. (B) is incorrect because whether the plaintiff's injuries will prevent her from ever working again is an issue that requires specialized knowledge to assist the jury in its determination; it is not an issue left for the jury to ultimately decide. (D) is incorrect because the doctor's qualification as an expert is based on his specialized knowledge, skill, experience, etc. Personal examination of the plaintiff is not a prerequisite to testifying as an expert.

Question 25-Separation of Powers, Treaty- Advanced The United States entered into a treaty with a foreign nation under the treaty, the foreign nation agreed to give up its quest for a nuclear arsenal and the United States agreed to allow the foreign nation to export automobiles to the United States. After the treaty was signed, evidence was discovered indicating that the foreign nation might not be living up to its end of the treaty. Before an investigation could be completed, an enraged Congress enacted a statute that specifically terminated permission for the foreign nation to export automobiles to the United States. Is the federal legislation effective? A No, because the agreement with the foreign nation was a treaty rather than an executive agreement. B No, because it interferes with the executive power over foreign affairs. C Yes, because it works to repeal the substance of the treaty. D Yes, but only if signed by the President (rather than being passed over his veto) and ratified by two-thirds of the Senate.

(C) The federal statute is effective because it would work to repeal the treaty. Although a treaty has supremacy over conflicting state law, a treaty is only on a supremacy parity with an act of Congress Any conflict between an act of Congress and a treaty is resolved by order of adoption; i.e., the last in time prevails. Here, the congressional legislation conflicts with the treaty by specifically terminating the permission to freely export automobiles from the foreign nation that was conferred by the treaty. Because this statute was enacted after the treaty, the terms of the statute prevail, thus repealing the substance of the treaty. (A) is incorrect because the legislation has the same effect in this case regardless of whether the agreement was a treaty or an executive agreement; as discussed above, the legislation will prevail over the treaty. (B) is incorrect because there is no unconstitutional interference with the executive's powers here. Both the President and Congress have some authority over foreign relations (e.g., the Commerce Clause allows Congress to regulate foreign commerce), and Congress has a right to exercise its power here. As stated above, the last expression of the sovereign (here, the statute) controls. (D) states the requirements for adoption of a treaty (i.e., the treaty power is granted to the President by and with the advice and consent of the Senate, providing two-thirds of the Senators present concur). Legislation that has the effect of repealing a treaty need not be signed by the President (i.e., may be passed over the President's veto), nor must such legislation be ratified by two-thirds of the Senate (unless an attempt is being made to override a presidential veto, in which case a two-thirds vote of each house is required). Therefore, (D) is incorrect.

Question 36- Impeachment- Advanced A plaintiff and a defendant were involved in a tm-car collision. The defendant was indicted for drunken driving, a crime that carries a maximum sentence of two years' imprisonment. A witness to the collision testified before the grand jury. The defendant pled guilty to the charge of drunken driving and was fined $500. After the criminal charge was disposed of, the plaintiff sued the defendant for negligence and sought personal injury damages. In the negligence action against the defendant, the witness testified for the plaintiff that the defendant was on the wrong side of the highway at the time of the collision. On cross-examination, the defendant seeks to question the witness about his sworn grand jury statement that the defendant was driving normally at the time of the accident. Upon proper objection, should the court rule that the witness's statement before the grand jury is admissible? A Yes, for impeachment only. B Yes, as substantive evidence only. C Yes, for impeachment and as substantive evidence. D No, because it is hearsay not within any exception.

(C) The grand jury statement is admissible both as impeachment evidence and as substantive evidence, A prior inconsistent statement made under penalty of perjury at a prior trial or proceeding, or in a deposition, is admissible nonhearsay and thus may be used as substantive evidence as well as for impeachment. The credibility of a witness may be impeached by showing that the witness has, on another occasion, made statements that are inconsistent with some material part of his present testimony. Because it is made by the declarant other than while testifying at the trial or hearing, a prior inconsistent statement will usually constitute hearsay if offered to prove the truth of the matter asserted therein. Under such circumstances, the statement would be admissible only to impeach the witness. However, where the statement was made under penalty of perjury at a prior proceeding, including a grand jury proceeding, it is admissible nonhearsay (i.e., it may be considered as substantive proof of the facts stated) [Fed R Evid 801 (d)(l )(A)]. The witness's sworn statement before the grand jury that the defendant was driving normally at the time of the accident is inconsistent with his later in-court testimony that the defendant was on the wrong side of the highway at the time of the collision. Thus, this statement can be inquired into by the defendant to cast doubt on the witness's credibility. Because the statement was made at a prior proceeding, and was made under oath, it is nonhearsay, and is also admissible as substantive proof that the defendant was in fact driving normally at the time of the accident. (C) is the only answer that reflects the fact that the grand jury statement may be used both for impeachment and for substantive purposes. (A) reflects the view of prior law, which was that prior inconsistent statements were limited to impeachment regardless of the circumstances under which they were made. As noted above, Federal Rule 801 (d)(l )(A) deems such statements made under penalty of perjury at a prior trial or other proceeding to be nonhearsay, and as such, to be admissible as substantive evidence. (B) is incorrect because it precludes use of the witness's grand jury testimony for impeachment purposes. A prior inconsistent statement may always be used to impeach the credibility of a witness. (D) is incorrect for two reasons. First, even if deemed to be hearsay, a prior inconsistent statement would be admissible to impeach the witness. Second, because the prior inconsistent statement of the witness was made under oath at a grand jury proceeding, it is admissible nonhearsay.

Question 33-Expert Learned Treatise Impeachment - Intermediate While working on a construction project, a plaintiff was injured when a heavy object struck his knee. Although the plaintiff was fully compensated for his injuries at the time of the incident, he now seeks disability payments from the construction company because he has developed arthritis in the same knee. The construction company claims that the arthritis has nothing to do with the plaintiff's on-the-job injury and refuses to pay him disability money. The plaintiff sues. A doctor takes the stand to testify for the plaintiff. He is qualified as an expert witness and during direct examination states that in his opinion the blow to the plaintiff's knee caused his arthritis. On cross-examination, the construction company's attorney produces a treatise on arthritis and asks the doctor if the treatise is considered to be authoritative. The doctor responds that the treatise is a standard authority in the field, but that he did not rely on it in forming his professional opinion regarding the plaintiff's condition. The attorney then seeks to introduce into evidence a statement in the treatise that "the idea that arthritis can be caused by a single traumatic event is purely folklore, although it is widely believed by the ignorant who have no scientific basis for their beliefs." The plaintiff's attorney objects. How should the court rule on the admissibility of the statement from the treatise? A Admissible, but only for the purpose of impeaching the doctor's testimony. B Admissible, but only as substantive evidence. C Admissible, both as substantive evidence and for purposes of impeaching the doctor. D Inadmissible.

(C) The statement from the treatise is admissible to impeach and as substantive evidence under the Federal Rules, learned treatises can be used either for impeachment or as substantive evidence. One way the credibility of an expert witness may be attacked is by cross-examining him as to his general knowledge of the field in which he is claiming to be an expert. This can be done by cross-examining the expert on statements contained in any scientific publication that is established as reliable authority. Reliability of a publication may be established by: (i) the direct testimony or cross-examination admission of the expert, (ii) the testimony of another expert, or (iii) judicial notice. The Federal Rules recognize an exception to the hearsay rule for learned treatises and admit them as substantive evidence if: (i) the expert is on the stand and it is called to his attention, and (ii) it is established as reliable authority (see above). The doctor has admitted on cross-examination that the treatise is authoritative in the field. Thus, the attorney may use the statement in the treatise to attack the doctor's general knowledge of the field of arthritis by showing that the doctor's opinion that the blow to the plaintiff's knee caused his arthritis is considered to be ignorant and unfounded in the text of the treatise. As noted above, such an attack on the doctor's general knowledge of the field is a proper means of impeaching his credibility. In addition, pursuant to the Federal Rules, the statement may be read into the record as substantive evidence (i.e., as a means of proving that the plaintiff's arthritis could not have been caused by a single traumatic event, such as the blow to his knee). The statement may be used as substantive evidence because it has been brought to the attention of the doctor during cross-examination and he established it as a reliable authority, and it will be read into evidence while he is on the stand (A), which reflects the traditional view, is incorrect because the Federal Rules permit the use of the statement in the treatise as substantive evidence. (B) is incorrect because precludes use of the statement for impeachment purposes. (D) is incorrect because it would not allow introduction of the statement for either impeachment or substantive evidentiary purposes, and thus it an incorrect statement of the law.

Question 15 - Free Exercise Clause - Advanced In an effort to standardize laws pertaining to the solicitation of business by mail, Congress adopted a statute establishing certain requirements that must be met before an organization can solicit business through the mails. A national religious organization that solicited charitable contributions by mail determined that the federal statute would substantially interfere with the successful accomplishment of the organization's religious objectives. The organization files suit seeking a declaratory judgment that the federal law may not be applied to its solicitation activities. Which of the following, as a matter of constitutional law, best describes the burden that must be sustained? A The federal government must demonstrate that a rational legislature could believe that this law helps to achieve a legitimate national interest when applied to both religious and secular solicitation activities. B The federal government must demonstrate that the application of this statute to the solicitation activities of this organization is necessary to vindicate a compelling governmental interest. C The organization must demonstrate a specific congressional purpose to inhibit the accomplishment of the organization's religious objectives. D The organization must demonstrate that no reasonable legislator could think that the application of the statute to this organization would be helpful in accomplishing a legitimate governmental objective.

(C) The organization must prove that the legislation was motivated by a desire to inhibit the accomplishment of the organization's religious objectives. The Free Exercise Clause cannot be used to challenge a law of general applicability unless it can be shown that the law was motivated by a desire to interfere with religion. [Employment Division v. Smith (1990)] Thus, states may regulate conduct even if the regulation happens to interfere with a person's religious practices, as long as that was not the purpose of the legislation. (A) is incorrect because the burden of proof is on the organization, as the party challenging the law, to show that it was motivated by improper considerations. Even if the rational basis test were applied, the burden of proof would be on the organization rather than the government. (B) is incorrect because it states the test for upholding a law that was designed to suppress conduct solely because it was religiously motivated. Such a law would be justified only if the government could show that it was necessary to further a compelling interest. (D) is incorrect because the rational basis test is not applied to a Free Exercise Clause challenge to a law regulating general conduct. As discussed above, the challenger must show that the legislation was motivated by a desire to interfere with religion.

Question 10-Habit- Intro A plaintiff sued a defendant for personal injuries, claiming that while the plaintiff was driving through an intersection at the posted speed limit, the defendant failed to stop at a stop sign and struck her car. At trial, the plaintiff calls the defendant's friend to testify to the fact that the defendant never stops at the stop sign at the accident intersection and invariably 'runs" every stop sign. Should the defendant's objection to the testimony be sustained? A Yes, because it is not the best evidence. B Yes, because character evidence is inadmissible in a civil case. C No, because it is evidence of habit. D No, because it is self-serving.

(C) The testimony is admissible as evidence of habit. Habit describes a person's regular response to a repeated specific situation. Evidence of a person's habit may be admitted to prove that on a particular occasion the person acted in accordance with that habit. [Fed R Evid 406] According to the testimony, the defendant regularly fails to obey the stop sign at the intersection at which the collision occurred, and in fact, he regularly disregards any stop sign. This regular response to a specific circumstance constitutes a habit. Consequently, the testimony, which is evidence of this habit, is admissible, and so the defendant's objection should be overruled. (A) is incorrect because the best evidence rule is inapplicable to this question. Under the best evidence rule, where the terms of a writing are material, the original writing must be produced in proving the terms of the writing. Here, there is no writing material to the case; thus, the best evidence rule does not come into play. (B) is incorrect because the offered testimony is not character evidence. Character describes one's disposition with respect to traits or general patterns of behavior. If the testimony were that the defendant is generally a careless driver, it would be inadmissible character evidence. The testimony, however, describes a repeated response by the defendant to repeated specific circumstances, which is admissible habit evidence. (D) is incorrect because the fact that an objection is self-serving does not form a basis for overruling (or sustaining) the objection. In a sense, all objections are self-serving to the party making them, just as the evidence to which an objection is made self-serving to the party offering it.

Question 16 - Citizenship for Public Job-Alien- Intermediate A man from a foreign country obtained a doctorate in political science from a state university and applied to teach there. The man was denied employment at the university under a state law requiring all teachers within the state to be United States citizens. Is the state's citizenship requirement constitutional as it applies to the man? A Yes, because states have the right to set minimal standards for state employees under the Tenth Amendment. B Yes, because a university political science teacher would exert a great deal of influence over the attitudes of students toward government, the political process, and citizenship. C No, because the citizenship requirement is not rationally related to a legitimate state interest. D No, because the citizenship requirement is not necessary to achieve a compelling state interest.

(D) A state generally may not discriminate against aliens absent a compelling state interest, and no compelling interest is served by prohibiting aliens from teaching at a state university. (A) is incorrect. The Tenth Amendment reserves to the states power not granted to the federal government. The Constitution vests the power to regulate aliens in Congress, and thus the states do not have power to control aliens under the Tenth Amendment. (B) is incorrect because it states the standard that the Supreme Court has applied to primary and secondary school teachers. The Supreme Court has upheld state statutes prohibiting aliens from teaching primary or secondary school on the rationale that teachers at the elementary and high school level have a great deal of influence over the attitudes of young students toward government, he political process, and citizenship. It is doubtful that the Court would extend this rationale to university teachers. (C) is incorrect because it states the wrong standard. If state discrimination against aliens relates to participation of aliens in the functioning of state government, the rational basis test applies. Merely teaching political science at a state university is not equivalent to participating in the political process.

Question 7- Public Policy Relevance/Post-accident repairs- Intro A plaintiff brought a personal injury action against a defendant, the owner of a small fishing resort for injuries he suffered when a dockside chair he was sitting on collapsed. At trial, the plaintiff testified that he had reported to the defendant the previous day that one of the chairs had a loose leg, whereupon the defendant tightened the screws holding the leg to the chair body, but that the next day the repaired leg of the chair collapsed while the plaintiff was fishing from it, injuring him. The plaintiff now wishes to offer evidence showing that the defendant had attached a new chair leg after the accident. Should the defendant's objection to that evidence be sustained? A No, because it tends to prove the defendant's negligence. B No, because it is relevant to the defendant's state of mind. C Yes, because it constitutes assertive conduct. D Yes, for public policy reasons.

(D) Evidence that the defendant had attached a new chair leg after the accident is inadmissible because, for public policy reasons, evidence of repairs or other precautionary measures made after an injury is inadmissible to prove negligence or culpable conduct [Fed R Evid 407] The purpose of this rule is to encourage people to make such repairs. Here, the plaintiff is offering the evidence to prove the defendant's negligence in the original repair of the chair, by showing the need to attach a new leg. Thus, this evidence is inadmissible, and the objection should be sustained. (A) is wrong because, as discussed above, the evidence may not be used to show the defendant's negligence. Thus, the tendency of the evidence to prove negligence would constitute a reason for sustaining the objection, rather than overruling it. (B) is wrong because the defendant's state of mind is not at issue. In addition, the proffered evidence does not really tend to prove anything relative to her state of mind. (C) is wrong for two reasons: (i) Even if the act of replacing the chair leg constituted assertive conduct, it would not be hearsay. If the defendant's conduct was a statement, it would be an admission of a party-opponent, and thus it would be nonhearsay. (ii) The act of replacing the leg is not assertive conduct constituting a statement under the hearsay rule. Assertive conduct is conduct intended by the actor to be a substitute for words. The defendant was not trying to communicate anything by replacing the leg.

Question 17 -Rational Basis Standard- Advanced A state that is subject to severe winters generally allows the use of studded tires between October 1 and March 31. However, the legislation allows counties to opt out and prohibit the use of studded tires year round, because studded tires tend to tear up pavement more than nonstudded tires, thus necessitating more frequent road repairs. No other state in the region allows use of studded snow tires at all. The state law contains one exception: it excludes "doctors" from any county ban on the use of snow tires because they might have to cross county lines in emergencies. After the passage of the legislation, only one county in the state invoked its right to ban the use of studded snow tires. A lawyer who lives in the state was angered that the legislature had given special privileges to doctors but not to lawyers. One January day, with studded tires on his car, he drove from his home county, which allowed use of studded tires, into the county that banned them. A sheriff's officer noticed the lawyer's studded tires and cited him. After being convicted and fined, the lawyer appealed. What is the lawyer's best argument forgetting the ban invalidated? A The statute interferes with his fundamental right to practice his profession in violation of the Privileges and Immunities Clause of Article IV. B The statute violates his right to travel. C The statute violates the Commerce Clause by placing an unreasonable restraint on interstate Commerce. D The ban on studded snow tires is not rationally related to a legitimate state interest because it will likely result in an increased loss of life.

(D) The best argument for getting the law invalidated is that it is not rationally related to a legitimate state interest. The lawyer would argue that the statute violates equal protection because it singles out one class of citizens for special treatment. Because neither a fundamental right nor a suspect nor quasi-suspect class is involved here, the case would be decided under the rational basis standard. For a law to be held invalid under the rational basis standard, the plaintiff must show that the law is not rationally related to a legitimate state interest. Toward this end, the lawyer might argue that the law will really cost more money than it will save, perhaps because the resulting number of injuries due to the absence of studded tires will more than offset the money saved in road repair. (This argument will likely fail, however, because courts give legislatures broad discretion in making such determinations, and the statute does appear to be rational. Nevertheless, this is the lawyer's best argument.) (A) is incorrect because the Privileges and Immunities Clause of Article IV applies only to discrimination by a state against nonresidents, and here the lawyer is a resident of the state that enacted the legislation. (B) is incorrect because nothing in the facts indicates that the right to travel is involved--at least not the constitutionally protected right to travel. The right to travel involves interstate travel, and here, the legislation concerns only an intrastate travel issue. (C) is incorrect because there is no unreasonable restraint on interstate commerce. If Congress has not allowed or prohibited state regulation in the area, a nondiscriminatory state regulation will be upheld only if its burden on commerce does not outweigh a legitimate local interest. Here, because no other state allows studded snow tires, the ban does not discriminate against out-of- state vehicles and does not burden commerce. (If other states allowed these snow tires, there might be a viable Commerce Clause issue.)

Question 14- Equal Protection -Golf Membership - Intermediate A city has many parks, as well as a country club with a golf course. While the city's parks are accessible to all without a fee, the city charges a $1,000 application fee and $100 per month dues to belong to the country club. A resident of the city wishes to join the country club but cannot afford the application fee or monthly dues. If the resident brings suit against the city on the ground that the fee and dues discriminate against the poor in violation of the Equal Protection Clause, who likely will prevail? A The resident, because a person cannot be deprived of a public right or benefit on the basis of inability to pay. B The resident, because the poor qualify as a protected class. C The city, because only de jure discrimination against the poor has been held to violate the Equal Protection Clause. D The city, because the membership privilege is not an important enough deprivation.

(D) The city will prevail because the membership privilege is not a significant enough deprivation to implicate the Equal Protection Clause. Only the denial of particularly important rights (such as a marriage license) to those unable to pay for them has been held to violate equal protection. Therefore, (A) is wrong. (C) is wrong because a number of de facto discriminations against the poor have been held to violate equal protection. (B) is an incorrect statement of the law. The Supreme Court has never held that wealth alone is a suspect classification. Only when the lack of wealth prevents a person from exercising a fundamental constitutional right will equal protection issues be raised.

Question 11 - Congressional Powers- Intro Congress passed a statute that established a five-member commission to investigate issues relating to gun control and to make recommendations to Congress for new gun control laws. All five members of the commission were appointed by Congress. An entity that has organizational standing brings a suit to enjoin the commission from investigating and recommending new gun control laws to Congress. What should the court do? A Forbid the commission to take any action. B Order that all members of the commission be appointed by the President by and with the advice and consent of the Senate. C Allow the commission to investigate but not make recommendations regarding gun control. D Allow the commission to continue investigating and making recommendations to Congress regarding gun control.

(D) The court should allow the commission to continue investigating. Congress may establish legislative and investigative commissions, so long as all the members of the commission are appointed by Congress. In these facts, the commission is not exercising any enforcement powers but rather is merely investigating and making recommendations, and all of its members are appointed by Congress; therefore, for the limited purposes indicated, the Constitution is not violated. Thus, (B) is incorrect because this commission does not have any enforcement powers. Therefore, the President need not appoint any members of the commission. Furthermore, the advice and consent of the Senate is not required here. (C) is incorrect because Congressional commissions can both investigate and make recommendations to Congress. (A) is incorrect because forbidding the commission to take any action too strong a remedy and not constitutionally required

Question 2- Zoning & First Amendment - Intro A city amended its ordinance to require that "adult theaters" could not be located either within 100 feet of each other or within 500 feet of any residential area. This zoning requirement was passed to protect the residential character of neighborhoods from destruction. A company owns two adult theaters in the city. One is about 1,000 feet from another adult theater. The second theater is adjacent to a residential area, in violation of the zoning ordinance. The company has filed an action to have the zoning requirement declared unconstitutional. Should the court hold that the zoning ordinance is valid? A No, because it violates the First Amendment. B No, because it is a form of spot zoning. C Yes, in that it covers the entire city. D Yes, because the protections afforded by the First Amendment are subject to zoning and licensing requirements.

(D) The court should hold that the zoning ordinance is valid under the First Amendment. In Young v. American Mini Theaters (1986), the Court held that the mere fact that the commercial exploitation of certain material was protected by the First Amendment does not prevent the city from zoning or imposing other licensing requirements, as long as the businesses are not totally banned. The Court has upheld zoning ordinances such as the one here as reasonable regulations of the secondary effects of the speech involved. [See City of Renton v Playtime Theaters (1986)]

Question 2-Intro A state law grants each county great autonomy in setting the health standards governing the preparation, packaging, transportation, and sale of food items. A county council recently enacted an ordinance, valid under the constitution and statutes of the state, prohibiting the packaging and sale of any food item in any non-biodegradable material; the ordinance defines non- biodegradable and specifically lists as prohibited all forms of plastics, cellophane, or similar materials. The ordinance specifically exempts from its terms sales of food to public institutions such as hospitals, jails, and schools. A retail food seller in the county files an appropriate court action attacking the county ordinance on the grounds that it violates the Equal Protection Clause of the Fourteenth Amendment. How should the court rule? A For the food seller, because the state's interests could be effectuated by alternative methods less intrusive upon the food seller's constitutional rights. B For the food seller, because no compelling state interest is served by the challenged ordinance. C For the county, because the state may regulate in this area as Congress has not entered the field. D For the county, because the ordinance is rationally related to a legitimate state interest - the health and safety of its citizens.

(D) The court should rule for the county because the ordinance is valid. Where no suspect or quasi-suspect classification or fundamental rights are involved, the equal protection analysis uses the rational basis test: if the statute is rationally related to a legitimate state interest, it is valid. Because there appears to be a rational basis for the challenged legislation - protecting the environment and the citizenry from the effects of potentially permanent artificial materials - the law is valid. (A) is incorrect because it assumes that the state must use the least restrictive alternative, but that is not required except when strict scrutiny is used. There is no basis for such review under the facts of this question because there is neither a suspect classification nor a fundamental right. (B) is wrong for the same reason: under equal protection analysis, a "compelling purpose" is required only when strict scrutiny is used. Challenges to economic regulations are subjected only to a rational basis test. (C) is incorrect because whether Congress has entered the field is irrelevant to an equal protection analysis. The call of the question specifically asks for the determination of the equal protection challenge; federal preemption issues are irrelevant here.

Question 5 - Burden of Proof -Intro A defendant is charged with larceny. His principal defense was that he had no intent to permanently deprive the victim of her property. The judge instructs the jury that the State had to prove beyond a reasonable doubt that the defendant was guilty of larceny and that the evidence tended to show that the defendant had taken some belonging to the victim; but if they believed that the defendant had proven by a fair preponderance of the evidence that he did not intend to keep the but to return them, they should find him not guilty. The defendant was convicted of larceny. He appeals the conviction, contending that the judge erred in his instructions to the jury. Is the defendant's conviction likely to be reversed? A No, because the jury has the power to ignore the defendant's testimony if they do not believe him. B No, because the defendant had failed to rebut the State's evidence tending to show that he intended to keep the jewels. C Yes, because the judge cannot comment at all on the evidence. D Yes, because the instructions put some of the burden of proof on the defendant.

(D) The defendant's conviction will likely be reversed because the jury instructions put some of the burden of proof on the defendant. The prosecutor is required by the Due Process Clause to prove each and every element of a crime beyond a reasonable doubt. One of the elements of larceny is an intent to permanently deprive a victim of his or her property, and the instructions in this case put the burden of proof on the defendant to show that there was no such intent. This relieves the prosecution of its burden because it implies that there is a presumption that the defendant intended to permanently deprive her of her jewels. (A) is wrong. The jury does have the power to ignore the defendant's testimony, but the conviction will be reversed because of the unconstitutional instruction on burden of proof. (B) is wrong. The defendant is not required to rebut the state's evidence; the state must prove each element beyond a reasonable doubt. (C) is not the best answer. While it is true that in some jurisdictions the judge cannot comment on the evidence in a criminal case, that prohibition would not be violated by an appropriate instruction on burden of proof.

Question 12- Lay Witness-Intro In a tort case involving personal injury, a hospital orderly is called to the stand. There is some dispute as to whether the plaintiff ever lost consciousness. The plaintiff's attorney wishes to have the orderly, who was working in the hospital emergency room when the plaintiff was brought in, testify that the plaintiff was unconscious at the time she entered the emergency room. Would such testimony be admissible over the defendant's objection? A No, because the orderly is not an expert witness. B No, because it impermissibly intrudes upon the province of the jury. C Yes, because it is the best evidence. D Yes, because it is proper opinion testimony by a lay witness.

(D) The orderly's testimony should be admitted because it is proper opinion testimony by a lay witness. Where an event is likely to be perceived as a whole impression, rather than as more specific components, opinions by lay witnesses are generally admitted. Lay opinion testimony is admissible when: (i) it is rationally based on the perception of the witness; (ii) it is helpful to a clear understanding of his testimony or to the determination of a fact in issue; and (iii) it is not based on scientific, technical, or other specialized knowledge. [Fed R Evid 701] One matter about which a lay witness may testify is the general appearance or condition of a person. In contrast, expert opinion testimony is called for when the subject matter is such that technical or other specialized knowledge will assist the jury in understanding the evidence or determining a fact in issue. Here, the orderly is not being asked to describe specific injuries that may have been incurred by the plaintiff; that would more appropriately be left to the specialized knowledge of an expert. Rather, he is being asked to testify as to the plaintiff's general condition (i.e., whether she was conscious or unconscious). Having been on duty in the emergency room when the plaintiff was brought in, the orderly had the opportunity to observe the plaintiff. Thus, he is in a position to offer an opinion as to the plaintiff's unconscious condition based on his own perception. It is easier for the orderly to express his testimony this way than to go into detail about specific manifestations of the plaintiff's condition. Also, this opinion aids in the determination of a disputed factual issue (i.e., whether the plaintiff ever lost consciousness). Therefore, the orderly's testimony is admissible as proper lay opinion testimony. (A) is wrong because, as noted above, this testimony does not relate to a matter the understanding of which requires resort to specialized knowledge. Status as an expert is not necessary to be able to state whether a person was conscious. Therefore, the orderly's status as a nonexpert constitutes no basis for the exclusion of his testimony. (B) is wrong because, if anything, the testimony will assist the jury, rather than intrude upon its province. Generally, the jury is to make fact determinations. The orderly's opinion testimony is helpful to the determination of a disputed fact. The jury relies on such testimony to enable it to reach a conclusion as to whether the plaintiff was unconscious. (C) is wrong because the best evidence rule does not apply to these facts. In proving the terms of a writing, where its terms are material, the best evidence rule requires that the original writing be produced. This question does not involve a writing of any type. The orderly would simply be testifying to what he personally observed. Thus, the best evidence rule does not come into play.

Question 5- Case or Controversy - Intro A state law provides that all persons who have been residents of the state for more than three years shall be entitled to free tuition at the state's main university. It further provides that persons who have resided in the state for three years or less shall pay the nonresident tuition rate, which is significantly higher. A student at the state's university who had been a state resident for less than three years filed a class action in federal court on behalf of himself and other similarly situated university students, seeking a declaration that the state statute is unconstitutional. When the case came to trial, the student had been a resident of the state for more than three years and was no longer required to pay tuition. By that time, a number of amicus curiae briefs had been filed in the case, some supporting and some opposing the student's position. Nevertheless, the state moved to dismiss the case as moot. Should the state's motion to dismiss be granted? A Yes, because the student is now a three-year resident. B Yes, because the student lacks standing. C No, because amicus curiae briefs have been filed. D No, because there is a live controversy.

(D) There is a live controversy and the case is not moot. A federal court will not hear a case unless there is a real, live controversy at all stages of the proceeding, not merely when the case is filed. Because the student is no longer required to pay nonresident tuition, there is arguably no controversy and the case may seem moot. However, a class action is not moot, and the class representative may continue to pursue it--even if the representative's own controversy has become moot--because the claims of others in the class are still viable. Here, the student filed his suit as a class action for university students with less than three years' residency; undoubtedly some of those students will still have a real controversy at this time. Thus, the case not moot. (A) is wrong although it states a true fact (A) implies that the case should be dismissed because the student's claim is moot. As explained above, this is a class action and other members of the class have a viable case. Thus, even though the named student's case by itself would be moot, he may continue the case as a representative of the class action. (B) is wrong because standing (the requirement that a plaintiff have a concrete stake in the outcome of the case) is determined at the beginning of a lawsuit. At the beginning of this case, the student had standing because he had suffered an injury (i.e., had to pay nonresident tuition), caused by the government, that was remediable by the court. Thus, he had a concrete stake in the outcome of the case and had standing. (C) is wrong because it is irrelevant; the fact that amicus curiae briefs have been filed has no effect on mootness. A moot case will not be heard simply because amicus briefs have been filed.

Question 28- Hearsay-advanced A plaintiff sued a defendant for injuries arising out of a collision between vehicles driven by the parties. The plaintiff alleged that the defendant ran a red light when he struck the plaintiffs vehicle in an intersection. The plaintiff wishes to call a witness to the stand who was near the intersection at the time of the accident. The witness is prepared to testify that the defendant offered to pay the witness $500 to testify falsely in the defendant's favor. Is the testimony admissible? A Yes, as substantive evidence of the weakness of the defendant's case. B Yes, for the limited purpose of impeachment by specific bad conduct. C No, because it is irrelevant to the case. D No, because although relevant, such evidence is misleading and prejudicial.

(A) Testimony regarding the defendant's attempt to bribe the witness is admissible as substantive evidence against the defendant. Under the Federal Rules, a statement made by a party and offered against that party (commonly called an admission) is not hearsay. Various kinds of conduct, including attempts to bribe witnesses, may be held to manifest an awareness of liability or guilt. Because the defendant's liability is the issue (i.e., a relevant fact) in the case, his attempt to bribe the witness is admissible as a statement of a party-opponent. (B) is wrong because, as discussed above, the offer is also admissible as a statement of a party, and thus is not limited to impeachment. Moreover, the witness's testimony would not be admissible as evidence of a specific instance of misconduct to impeach the defendant. A specific act of misconduct can be elicited only on cross-examination; extrinsic evidence is not permitted. The facts do not even indicate that the defendant has testified. (C) is wrong because the offer to bribe the witness is relevant. Evidence is relevant if it tends to make the existence of a fact of consequence to the action more probable or less probable than it would be without the evidence. Evidence that the defendant tried to bribe a witness to testify falsely makes it more probable that the accident was his fault; thus, it is relevant. (D) is wrong because the fact that relevant evidence is misleading or prejudicial is not a sufficient reason, by itself, to exclude the evidence. Under Federal Rule 403, the judge has discretion to exclude otherwise admissible, relevant evidence if its probative value is substantially outweighed by the danger of unfair prejudice, confusion of the issues, misleading the jury, or by considerations of undue delay, waste of time, or needless presentation of cumulative evidence. Here, the choice states only that the evidence is misleading and prejudicial; it does not state that the probative value of the evidence is substantially outweighed by these facts. Furthermore, bribing a witness is highly probative of guilt, so it is unlikely to be misleading. Although the evidence is prejudicial, it is not at all clear that the prejudice would be unfair since there are few reasons other than guilt for bribing a witness.

Question 1-Judicial Review-Supreme Court Jurisdiction - Intro An ordinance of a city prohibits leafleting on the grounds of any hospital or on the sidewalks within five feet of the hospital entrances during visiting hours. A member of a religious group advocating prayer to restore the sick to good health is arrested for violating the ordinance, is fined $100, and is convicted. She appeals her conviction, claiming that her constitutional rights were violated. The case was heard by the state supreme court, which ruled that while the ordinance was permissible under the United States Constitution, it was unconstitutional under the state constitution because the fine money was designated to go to the city's only hospital, which was privately owned, rather than to the city. The city seeks to bring the case before the United States Supreme Court. Should the United States Supreme Court grant certiorari? A No, because the case was decided on independent state grounds. B No, because the case is moot. C No, because this is a political question. D Yes, the Supreme Court should hear the case on its merits, because it involves an important federal question.

(A) The Court should refuse to grant certiorari because the case was decided on independent state grounds. The Supreme Court will not hear a case from a state court, even though it has jurisdiction over the parties and the subject matter, if there are adequate and independent state grounds to support the decision. Here, the state court held the law invalid on state, rather than federal, constitutional grounds. Therefore, the Supreme Court should refuse to grant certiorari. (B) is incorrect because mootness goes to whether there is a real, live controversy at this stage of the proceeding. The case here is not moot because the city wants to challenge the ruling that the law was unconstitutional under the state constitution. If the city wins, it may enforce the ordinance. (C) is incorrect because no political question is involved here. A political question, which is nonjusticiable, is one involving an issue that the Constitution has committed to another branch of the government or that is inherently incapable of judicial resolution and enforcement. Whether a law is constitutional--a determination of law--is certainly capable of judicial resolution and is properly within the judiciary branch's powers. Thus, a political question is not involved. (D) is incorrect because, as explained above, the basis for the decision was a state, not federal, violation. Therefore, there is no federal question.

Question 17- Best Evidence Rule- Intermediate A defendant was arrested and charged with the murder of a member of the local athletic club. The victim was found dead near the locker room inside the club. The defendant allegedly entered the club, killed the victim, and left before anyone discovered the body. At trial, the prosecution called the doorman of the club who testified that, although he could not identify the defendant by sight, he remembered admitting a member the day of the murder who showed a membership card bearing the name of Arnold Kramer. The prosecution then sought to admit the testimony of a police officer that a membership card to the athletic club bearing the name of Arnold Kramer was found on the defendant's person at the time of his arrest. What is the defendant's best argument in seeking to exclude the police officer's testimony? A It violates the best evidence rule. B It violates the hearsay rule. C It is about a purely collateral matter. D It violates the defendant's privilege against self- incrimination.

(A) The best argument to exclude the police officer's testimony about the membership card is that violates the best evidence rule. The best evidence rule can be stated as follows: In proving the terms of a writing, where the terms are material, the original writing must be produced. Secondary evidence of the writing is permitted only after it has been shown that the original is unavailable for some reason other than the serious misconduct of the proponent. Generally, the rule applies to two classes of situations: (i) where the writing is a legally operative or dispositive instrument; or (ii) where the knowledge of a witness concerning a fact results from having read it in the document. Here, it likely that the police officer knew the details of the membership card found on the defendant because he read the card. Therefore, the best evidence rule applies, and secondary evidence of the contents of the card is not admissible unless some good reason is shown. (B) is incorrect because the police officer's testimony about the name on the card would not violate the hearsay rule. The evidence is not being offered to prove the truth of the matter asserted on the card. It is being offered to show that the defendant possessed the card, and is circumstantial evidence that the defendant committed the crime. (C) is incorrect because the existence of the membership card on the defendant's person is not collateral to the issues involved. (D) is incorrect because the privilege against self-incrimination prevents a witness from being compelled to testify against himself, which is not the case here.

Question 4 -Present Bodily Condition- Intro The plaintiff was riding down a department store escalator when the escalator malfunctioned, causing the plaintiff to fall forward to the floor and injure himself. His companion, who has no medical training, tried to make the plaintiff comfortable. While waiting for the ambulance, the plaintiff said, "My left arm is numb and have chest pains I'm afraid I'm having a heart attack." Upon being taken to the hospital, it was determined that the plaintiff had not suffered a heart attack, but had sustained severe damage to his chest and to the nerves affecting feeling in his left arm. The plaintiff sued the department store for his injuries. At trial, the plaintiff called his companion to testify to the statement made by the plaintiff. Is the companion's testimony as to the statement admissible? A Yes, to prove the extent of the plaintiff's injuries at the time. B No, because the plaintiff did not suffer a heart attack. C No, because the companion is not a doctor. D No, because it is hearsay not within any exception.

(A) The companion's testimony is admissible. Declarations of present bodily condition are admissible as an exception to the hearsay rule, even if not made to a physician. Such statements relate to symptoms, such as pain. Here, the plaintiff's statement was one of present bodily condition and is admissible to prove the extent of his injuries, which is an issue in the case. It follows that (D) is incorrect. (C) is incorrect because, as noted above, a declaration of present bodily condition is admissible even if not made to a physician. (B) is incorrect, because the plaintiffs statement is admissible as a declaration of present pain, not as proof that he had a heart attack.

Question 4- Harmless Error - Intro A defendant was charged with robbing and murdering a store clerk. A security camera had recorded the incident, clearly showing that the defendant shot the store clerk at point blank range, and that the defendant dropped the handgun immediately after shooting the store clerk. The gun, with the defendant's fingerprints on it, was recovered. Two eyewitnesses also were able to conclusively identify the defendant as the robber/murderer. After the defendant had been validly arrested, the police conducted a warrantless search of the defendant's home, and several items from the store were found. The items from the store were introduced at trial over the defendant's objection. The defendant was convicted after a nonjury trial. On appeal, the appellate court ruled that the warrantless search was unconstitutional under the circumstances. Must the appellate court also overturn the defendant's conviction? A No, because the error in admitting the evidence was harmless. B No, because the tainted evidence would not have affected a judge's decision as much as that of a jury. C Yes, because the police should have obtained a warrant before searching his home. D Yes, because a violation of a constitutional right is never harmless.

(A) The court need not overturn the conviction. A conviction will not necessarily be overturned merely because improperly obtained evidence was admitted at trial. Rather, the harmless error test applies, so a conviction can be upheld if the conviction would have resulted despite the admission of improperly obtained evidence. On appeal, the government bears the burden of showing beyond a reasonable doubt that the admission was harmless. In the instant case, there was plenty of other evidence--a security camera recording, eyewitnesses, and physical evidence recovered from the crime scene--that ties the defendant to the crime, Hence, the burden of proving harmless error beyond a reasonable doubt would be met under these facts. Thus, (A) is correct, and (C) is incorrect. (B) is incorrect because it implies that some lesser standard applies when the judge makes findings of fact rather than a jury. The standard remains the same--the government must show, beyond a reasonable doubt, that the error was harmless. (D) is an overbroad and incorrect statement of the law

Question 21- Hearsay Prior Testimony- Intermediate A woman was found strangled in the front seat of her car, and her friend was charged with murder. At trial, the woman's sister testified that she had seen the friend riding in the front seat of the woman's car with the woman one hour before she was found dead. The friend testified in his defense that, two years ago, the woman had informed her sister and the friend that she had named the friend beneficiary of a $100,000 insurance policy issued on her life, and the sister became very angry. The friend was acquitted of the murder. The friend asked the insurance company to pay him the proceeds from the insurance policy on which he was named beneficiary. When the insurance company refused, the friend sued to force payment. The insurance company defended on the grounds that the friend had killed the woman. Because the sister had died by the time of the trial on the insurance policy, the insurance company offers into evidence a duly authenticated transcript of the sister's testimony from the murder trial. Should the court admit the transcript? A Yes, because it is a record of the sister's former testimony. B Yes, because it is a past recollection recorded. C No, because on the same evidence, the friend was acquitted of murdering the woman. D No, because the motive to cross-examine in the former trial was not the same as in this trial.

(A) The court should admit the transcript because is a record of the sister's former testimony. The prior testimony of a now-unavailable witness given at a trial, hearing, or in a deposition is admissible in a subsequent trial as an exception to the hearsay rule if the testimony was given under oath and the party against whom the evidence is now offered (or, in civil cases, the party's predecessor in interest) was a party in the former action, and had an opportunity and similar motive to develop the testimony either on direct or cross-examination. [Fed R Evid 804(b)(1)] Here, the sister is dead and her testimony is being offered against the friend, who was the defendant in the prior action and had an opportunity to cross-examine the sister's testimony. Thus, the transcript falls within the former testimony exception. (B) is wrong because the past recollection recorded exception to the hearsay rule applies only when the witness whose statement was recorded testifies at trial, and here the sister is dead. (C) is wrong because the disposition at the previous trial has no bearing on the admissibility of prior testimony. (D) is wrong because the motive to cross-examine was identical at each trial--to discredit the sister's testimony.

Question 9-Jury Instructions-Advanced The criminal statutes of the state define manslaughter and murder as they were defined at common law. As to insanity, the state has the following provision: Under the defense of insanity a defendant may be entitled to acquittal if, because of mental illness, the defendant was unable to control his or her actions or to conform his or her conduct to the law. The defendant was put on trial in the state for the murder of his wife and her co-worker. The evidence at trial established that the defendant's wife was having an affair with the co-worker, and that the defendant learned of it and killed the pair. The defendant did not take the stand in his own defense. In his closing statement to the jury, the defendant's attorney made a statement, "Ladies and gentlemen, you must consider that there are some things that would provoke any one of us to kill, and there are things that make one unable to control one's actions." The defendant's attorney requested that the judge give the jury instructions on manslaughter and on insanity, and the judge agreed to do so. The judge also issued the following instructions: INSTRUCTION #6: In order to mitigate an intentional killing to voluntary manslaughter, the burden of proof is on the defendant to establish that adequate provocation existed. INSTRUCTION #8: Insanity is an affirmative defense and the burden of proof is on the defendant to establish that such insanity existed at the time of the killing. The jury found the defendant guilty of murder, and he appealed. He asserts that the jury instructions violated his rights under the federal Constitution. How should the appeals court rule? A Reverse the defendant's conviction, because Instruction #6 was improper. B Reverse the defendant's conviction because Instruction #8 was improper. C Reverse the defendant's conviction because both Instructions #6 and #8 were improper. D Uphold the defendant's conviction because neither Instruction #6 nor Instruction #8 was improper.

(A) The court should reverse the defendant's conviction because Instruction #6 requires the defendant to disprove one of the elements of murder. Due process requires in criminal cases that the state prove guilt beyond a reasonable doubt. The prosecution has the burden of proving all of the elements of the crime charged. Thus, if malice aforethought is an element of murder and voluntary manslaughter is distinguished from murder by the existence of adequate provocation, the defendant cannot be required to prove that he committed the homicide in the heat of passion (i.e., with adequate provocation). Such a requirement would impose on the defendant the burden of disproving the element of malice aforethought, because "heat of passion" negates malice. Although the defendant can be given the burden of going forward with some evidence on the provocation issue, once he has done so, the prosecution bears the burden of proving that the killing was not done in the heat of passion. In the case at issue, Instruction #6 requires a defendant to prove that he committed the intentional killing under adequate provocation. At common law, and consequently in the state, malice aforethought is an element of murder. Therefore, this instruction in effect requires the defendant to disprove the element of malice aforethought, thereby relieving the state of its burden of proving all elements of the crime. As discussed above, such an instruction cannot pass constitutional muster. On the other hand, for an affirmative defense such as insanity, it is permissible to impose the burden of proof on the defendant. Thus, Instruction #8 does not affect the state's obligation to prove all elements of the crime, and is permissible under the general principles mentioned above. Thus, (B) and (C) incorrectly state that this instruction is improper. (D) is incorrect because it states that Instruction #6 is proper. As explained above, this is not an accurate statement of the law.

Question 21- Responsiveness of witness testimony- Intermediate In a suit between the drivers of two cars involved in a collision, the plaintiff's attorney calls a witness to the stand. On direct examination, the witness testifies that while walking down the street, she heard a horn sounding, looked up, and saw two cars enter an intersection and collide. On cross-examination, the defense attorney goes over the same ground with the witness. He asked her whether there was any broken glass on the pavement, to which she responds, "Yes, lots of it," and before the defense attorney can ask his next question, the witness blurts out, "They had to be going over 50!" The defense attorney moves to strike the statement. How should the court rule? A Strike it, as unresponsive to any question asked. B Strike it, because the witness had no way of knowing how fast the cars were traveling. C Not to strike, because the defense attorney "opened the door" to anything the witness might say about the accident. D Not to strike, because the statement accuses both drivers of going over 50, and is not prejudicial to only one side.

(A) The court should strike the statement as unresponsive to any question asked. An unresponsive answer by a witness is subject to a motion to strike by examining counsel, but not by opposing counsel. Thus, examining counsel can adopt an unresponsive answer if it is not objectionable on some other ground. Here, the defense attorney asked the witness a question that was very specific and called for a specific answer (i.e., whether there was broken glass on the pavement at the time and place of the accident). Thus, the witness should only have stated whether there was any glass. Her volunteered information regarding the speed of the cars bore no connection to the question posed and was totally unresponsive to that question (or to any other question asked). Therefore, the defense attorney, as examining counsel, is entitled to move to strike the statement, and this motion should be granted. (B) is incorrect for two reasons: (i) The witness's comment should be stricken as unresponsive regardless of whether she could have known how fast the cars were traveling. Even if she was highly experienced in estimating vehicle speeds, her comment was still not in response to any question. (ii) It is not true that the witness could not offer an opinion on the speed of the cars (if she were asked a question on this). A lay witness is permitted to estimate in miles per hour the speed of a moving vehicle if it is shown that she has some experience in observing the rate of speed of moving objects. Thus, if the witness can establish such experience, her statement would be admitted into evidence (providing it was made in response to a question posed by examining counsel). (C) is incorrect because it misstates the concept of "opening the door." One who introduces evidence on a particular subject thereby asserts its relevance and cannot complain, except on grounds other than relevance, if her adversary thereafter offers evidence on the same subject. This is what is meant by "opening the door." The defense attorney is not complaining of evidence being offered by opposing counsel. Rather, the motion to strike is directed at a totally unsolicited comment from a witness. (D) is incorrect because the prejudicial nature (or lack thereof) of an unresponsive answer does not form the basis for a motion to strike. It is true that, in most cases, an attorney would move to strike only if the witness has made a statement harmful to his case. However, as noted above, the option of moving to strike rests entirely with the examining attorney, and he may move to strike based solely on the unresponsive character of the statement, without showing any prejudice

Question 6-Issue Preclusion/Collateral Estoppel-Intermediate A woman sued her ex-husband for installments due under their divorce property settlement agreement. The ex-husband defended the suit on the ground that the ex-wife was in breach of the agreement. However, the court granted judgment for the ex-wife. Now, the ex-wife is suing her ex-husband for other installments due under the agreement. The ex-husband raises the defense that the agreement was void and illegal because of fraud perpetrated by his ex-wife at the time the agreement was signed. The ex-wife moves to strike her ex-husband's defense, claiming issue preclusion. Should the court grant the motion? A No, because the issue was never litigated. B No, because judgment on one installment does not bar subsequent actions on other installments. C Yes, because the husband should have raised the defense in the first action. D Yes, because the issue of fraud is never waived.

(A) The court should not grant the motion. Generally, issues actually litigated between the parties are binding on them in subsequent actions concerning the same claim. If the second suit involves a different claim, the first judgment may be invoked as to all matters actually litigated and determined in the first action, provided that the findings were essential to the first judgment. Here, the fraud issue was not actually litigated in the first action, and thus it can be raised with respect to the later installments. (B) is incorrect. The general principle recited in this answer is true for claim preclusion (unless there is an acceleration clause). However, even if claim preclusion does not apply, if the defendant raises the same issues as in the first case, he may be prevented by issue preclusion (collateral estoppel) from re-litigating those issues in that subsequent case. (C) is incorrect. Claim preclusion (res judicata) would bar claims arising out of the same transaction or occurrence. Here, claim preclusion does not apply because a defense, not a claim, is involved, and the second installment is a separate transaction or occurrence. (D) is incorrect because there is no such rule.

Question 19 - Character Evidence - Intermediate The defendant is charged with the battery of a bouncer at a local tavern. At the trial, the prosecutor introduces evidence that, while the bouncer was attempting to question the defendant about her intoxicated demeanor, the defendant committed a battery on the bouncer. The defendant attempts to defend against the charge on the basis of self-defense, insisting that the bouncer used excessive force in stopping her from entering the tavern. The defendant attempts to introduce into evidence an authenticated copy of the tavern records that show that three patrons had written complaints against the bouncer within the past six months for the use of excessive force. The prosecutor objects on the grounds that the records are inadmissible character evidence. Should the court sustain the objection? A Yes, because the character of a victim can be established only by reputation or opinion evidence. B Yes, because there is no evidence that the incidents involving the three patrons were based on the same facts as the defendant's claim. C No, because the records were authenticated. D No, because the character trait of a victim may be established by opinion evidence, reputation evidence, or specific acts of misconduct.

(A) The court should sustain the objection because the records are evidence of specific bad acts. The Federal Rules permit a defendant to introduce evidence of a bad character trait of the alleged victim if it is relevant to the charge or the defense, but limit it to reputation and opinion evidence. Evidence of specific acts of the person in question that demonstrates that person's character is permitted only in a few instances, such as if the acts are relevant to some issue other than disposition to commit the crime charged. Here, no issue is raised by this evidence other than the bouncer's propensity to use excessive force. (A) is therefore correct and (D) is wrong. (B) is wrong because the facts do not have to be identical if evidence of bad acts were otherwise admissible. (C) is wrong; documentary evidence, even if fully authenticated and relevant, may be excluded if it violates a rule of competency, such as the rule for character evidence. Here, the objection should be sustained because the document improper evidence of a specific bad act.

Question 27- Hearsay- Advanced A plaintiff sues her employer for sexual harassment. During the trial, the plaintiff attempts to introduce into evidence company records that include written complaints from other employees alleging that they too were sexually harassed by the employer. The defense objects to the admission of the records on the basis of hearsay. Should the objection be sustained? A Yes, because the records are hearsay not within any recognized exception. B No, because the records qualify under the business records exception. C No, because the records qualify as a statement against interest. D No, because the records are not hearsay.

(A) The court should sustain the objection because the records are hearsay not within any recognized exception. Hearsay is a statement, other than one made by the declarant while testifying, offered into evidence to prove the truth of the matter asserted. Here, the records are being offered to prove that the employer sexually harassed other employees, to support the plaintiff's contention that the employer sexually harassed her. Because the statements are offered to prove the truth of the matter asserted, they are hearsay, and because there is no recognized exception that would allow the records to be admitted, they must be excluded. Therefore, (D) is incorrect. (B) is incorrect. The business records exception applies to records or writings made in the course of a regularly conducted business activity by one who was under a duty to do so. Here, because the employees were not under a business duty to file their claims, the business records exception does not apply to their statements. (C) does not agree with the facts. A statement against interest is a hearsay exception allowed when a declarant is unavailable. Here, there is no showing of unavailability, and also the employees said nothing against their interests.

Question 1- Right to Jury Trial - Intermediate A manufacturer sold to a consumer an expensive laser printer that never worked properly. Therefore, the consumer never fully paid for the printer. The manufacturer sued for specific performance of the contract of sale of the printer. The consumer filed a counterclaim for a breach of warranty, asking for $85,000 in damages. The consumer demanded a jury trial but the manufacturer objected. Assuming that the demand for a jury trial was timely made, how will the court rule on the availability of a jury trial? A For the consumer, because the underlying dispute is legal in nature. B For the consumer, because a defendant may always request a jury trial. C For the manufacturer, because it filed suit first. D For the manufacturer, because an action for specific performance is equitable in nature.

(A) The court will grant a jury trial because the underlying dispute is legal in nature. The court will look to the basic substance of the case to see if a jury trial is appropriate. Although the manufacturer's suit is equitable in nature, the consumer's counterclaim for breach of warranty is an action at law, in which a jury trial is available on demand. Thus, (D) is incorrect. (B) is an incorrect statement of law, and (C) is incorrect because filing suit first would not guarantee a jury (or nonjury) trial.

Question 16 -Obscenity - Advanced Based on recommendations of a state commission studying the effect of pornographic films on violent criminal activity, a state adopted legislation banning films intended for commercial distribution that appealed as a whole to the prurient interest in sex of the average person in the community, portrayed sex in a patently offensive way to citizens of the state, and which a reasonable person in the United States would find had no serious literary, artistic, political, or scientific value. In ruling on a constitutional challenge to the legislation from a film distributor in the state who was convicted of distributing films in violation of the legislation, will the federal court likely find the legislation to be constitutional? A Yes, because it uses a national "reasonable person" standard for determining the social value of the work. B Yes, because it uses a statewide standard rather than a community standard for determining whether the material is patently offensive. C No, because it uses a statewide standard rather than a national standard for determining whether the material is patently offensive. D No, unless the court finds that the legislation is necessary to advance the state's compelling interest in reducing violent criminal activity.

(A) The court will likely find the legislation to be a constitutional regulation of obscenity. Obscenity, which is not protected speech under the First Amendment, is defined by the Supreme Court as a description or depiction of sexual conduct that, taken as a whole, by the average person, applying contemporary community standards, appeals to the prurient interest in sex, portrays sex in a patently offensive way, and-using a national reasonable person standard- does not have serious literary, artistic, political, or scientific value. Thus, the legislation here is constitutional because it uses a reasonable person standard, rather than a community standard, for determining the value of the work. (B) is incorrect because while a statewide standard for determining whether the material is patently offensive is permissible, it is not mandatory. A state may use a "community standard" for making this determination. (C) is incorrect because, again, a statewide standard for determining whether the material is patently offensive is permissible. Only the "social value" element of the obscenity test requires a national standard. (D) is incorrect because the legislation is valid regardless of whether it is necessary to achieve the state's compelling interest in reducing violent crime. Speech that falls within the definition of obscenity is unprotected speech; the government does not need a specific compelling interest to ban it.

Question 1-State of Mind Hearsay Exception- Intro A daughter brought suit against an insurer for failure to pay benefits on a policy held by her famous father, who had been missing for years. The insurer had refused to pay because there had been no determination that the father was deceased. At trial, the daughter testifies that she has not heard from her father since prior to New Year's Day seven years ago. On that date, a chartered boat left Florida and has never returned. The ship was last seen near midday on that day sailing into an area known as the "graveyard of ships." No trace of the boat has ever been found. The daughter establishes that the charter operator filed a passenger list with the coast guard, and that one of the names on the list is "Madd Hatter." The daughter wishes to testify that on the day before the disappearance, her father told her that he planned to propose a television pilot fora comedy starring himself as "Madd Hatter." If the insurer's attorney objects, is the testimony as to the fathers alleged statement admissible? A Yes, as circumstantial evidence that the father was on the boat. B Yes, to create a rebuttable presumption that the father was on the boat. C Yes, to create a conclusive presumption that the father was on the boat. D No, the testimony is not admissible.

(A) The daughter's testimony is admissible as circumstantial evidence that her father was on the boat. The statement is hearsay, but is admissible under the state of mind exception to the hearsay rule. Hearsay is an out-of-court statement offered to prove the truth of the matter asserted upon objection, hearsay must be excluded unless it falls within an exception to the rule. Here, the father's out-of-court statement is being offered for its truth—i.e., to prove that he planned to propose a television show starring himself as Madd Hatter. The statement is, therefore, hearsay. Declarations of a declarant's then-existing state of mind, however, are admissible if made under circumstances of apparent sincerity. This exception includes declarations of intent offered to show subsequent acts of the declarant. In this case, the fathers statement to his daughter was made under circumstances of apparent sincerity and so may be admitted as circumstantial evidence that he used the name Madd Hatter and was aboard the boat that vanished. (B) is wrong because this evidence would not create a rebuttable presumption that the father was on the boat. A presumption is a rule (established by statute or case law) that requires that a particular inference be drawn from a particular set of facts. There is no indication in the facts, nor is it likely, that such a rule exists in this jurisdiction with respect to the subject matter of the daughter's testimony. Note that the daughter may be offering this testimony as part of her attempt to establish the rebuttable presumption that her father is dead because he has not been heard from in seven years. However, that presumption is distinct from a presumption regarding his presence on the boat. At best, the daughter's testimony allows the jury to find that the father was on the boat; it does not require them to do so. (C) is wrong. A conclusive presumption is a rule of substantive law rather than a true presumption because it cannot be rebutted. There is no conclusive presumption applicable here. (D) is wrong because, as discussed above, the testimony admissible as circumstantial evidence that the father was on the boat.

Question 6 -Entire Document, Fairness - Intro The plaintiff brought suit against the defendant for breach of a contract involving the sale of 100 imported coffee tables, alleging that the defendant improperly rejected a shipment of tables. The defendant defends by introducing evidence that the shipment delivered to his place of business was only 68 coffee tables, which he rejected. The plaintiff introduced part of a letter he sent to the defendant the day after the contract was signed stating that import issues may briefly delay the plaintiffs acquisition of the entire quantity of tables. The defendant then offers evidence that another part of the letter stated, "There is no obligation to receive any coffee tables unless they are delivered in one lot of 100 tables." If the plaintiff objects to this evidence, how will the court likely rule? A The defendant is entitled to introduce evidence of any part of the transaction necessary to make it understood. B By not objecting to the letter, the defendant has waived any right to introduce any other part of it. C The defendant's evidence is inadmissible under the parol evidence rule. D The defendants evidence is inadmissible as hearsay

(A) The defendant may introduce the evidence. Where a party has introduced part of a writing or recorded statement, the adverse party may require the introduction of any other part that in fairness ought to be considered at the same time. The party who introduced the original part of the writing cannot object to the introduction of the additional evidence on grounds of lack of competency, hearsay, etc. (B) is an incorrect statement of the law. (C) is incorrect because the parol evidence rule applies only to prior or contemporaneous agreements, and here the letter was subsequent to formation of the contract. (D) incorrect because the letter is not hearsay; it a legally operative fact (i.e., offered for its legal effect)

Question 6-Deposition - Intro In a personal injury case involving a tm-car collision, the plaintiff wishes to introduce a sworn deposition taken from a witness who died two weeks before the case came to trial. In the deposition, the witness stated that she saw the defendant run a red light at the time of the collision with the plaintiff's car. Both the plaintiffs and the defendant's attorneys were present at the deposition. The defendant objects in the appropriate manner to the introduction of the witness's statement. How should the court rule on the admissibility of the deposition? A Admissible, because the defendant had an opportunity to cross-examine the witness at the time the deposition was taken. B Admissible, as a dying declaration. C Inadmissible, because the statement was not made while the witness was testifying in court. D Inadmissible, because the defendant has no opportunity to cross-examine the witness at trial.

(A) The deposition testimony of the now unavailable witness is admissible under the former testimony exception to the hearsay rule. The witness's statement is hearsay because it is a statement, other than one made by the declarant while testifying at the trial or hearing, offered in evidence to prove the truth of the matter asserted. Unless such a statement falls within a recognized exception to the hearsay rule, it must be excluded upon appropriate objection to its admission. [Fed R Evid 802] Pursuant to the former testimony exception to the hearsay rule, the testimony of a now unavailable witness given at another hearing or in a deposition taken in accordance with the law is admissible in a subsequent trial as long as there is a sufficient similarity of parties and issues so that the opportunity to develop testimony or cross-examine at the prior hearing or deposition was meaningful. [Fed R Evid. 804(b)(1).] Here, the plaintiff is offering the deposition testimony of the witness to prove the truth of the matter asserted therein; i.e., that the defendant ran a red light at the time of the accident. Thus, the testimony is hearsay. The witness is unavailable because she is dead. Also, because the deposition was taken in connection with the same case that is currently the subject of the trial, there is an exact identity of parties and issues between the deposition and the trial. Thus, the defendant had an opportunity and a motive to develop the testimony of the witness at the time of the deposition by cross-examination. As a result, the elements of the former testimony exception are satisfied, and the deposition testimony of the witness is admissible in the trial. (B) is incorrect because the testimony of the witness does not constitute a dying declaration. In a civil action or a homicide prosecution, a statement made by a now unavailable declarant while believing her death was imminent, that concerns the cause or circumstances of what she believed to be her impending death, is admissible. [Fed R Evid 804(b)(2)] There is no indication that the witness's statements contained in the deposition were made at a time when she believed her death was imminent, or that such statements concerned the cause or circumstances of what she believed to be her impending death. Therefore, the requirements of a dying declaration are not met. (C) is incorrect because a statement need not be made in court to qualify under the former testimony exception to the hearsay rule. Deposition testimony is within the exception if the deposition is taken in compliance with the law and the party against whom it is offered (or his predecessor in interest) had an opportunity and similar motive to develop the testimony. (D) is incorrect because it is not necessary for the defendant to have the opportunity to cross-examine the witness at the trial. It is only necessary that the defendant have had the opportunity to develop the witness's testimony at the prior proceeding; i.e., at the deposition. Having been afforded this opportunity, the defendant cannot now obtain the exclusion of the proffered testimony on the ground that the witness cannot be cross-examined at trial.

Question 10 - Self-incrimination/Grand Jury - Intro Hijackers of a truck carrying dog food were arrested and indicted by a grand jury. During the grand jury's investigation the district attorney's office subpoenaed the truck driver as a witness. To her relief, she was not asked questions about a series of thefts of shipments of dog chew-toys that she was involved in, but believes that the subject might be brought up at trial by the defense lawyers. She is afraid that she likely will be fired if she invokes the Fifth Amendment privilege against self-incrimination on the witness stand, so she wants to avoid testifying at all at the trial, even if she is subpoenaed by the defense. As her lawyer, what advice should you give her about complying with the subpoena? A She cannot avoid testifying, because she can plead the Fifth Amendment on the stand. B She cannot avoid testifying, and she must answer all questions truthfully, even if her answers will incriminate her, because she is not on trial. C She can avoid testifying, because she is not a party to the action. D She can avoid testifying, because the potential that she will incriminate herself is greater than the interest the defense has in calling her as a witness.

(A) The driver must comply with a subpoena. The Fifth Amendment does not permit a person to avoid being sworn in as a witness or being asked questions. Instead, the person must listen to the questions and specifically invoke the privilege rather than answer the questions. Therefore, the truck driver must comply with any subpoena issued by the defense. (B) is incorrect because, while the truck driver must comply with a subpoena, she may listen to questions on the stand and specifically invoke the Fifth Amendment rather than answer the questions; she does not need to be the defendant to invoke the privilege against self-incrimination. (C) is incorrect. Only the criminal defendant has a right not to take the witness stand at trial. Because the driver was a witness to the offense and may have relevant evidence, she may be required to testify by either party. (D) is incorrect because, as explained above, the driver's interest in protecting herself from self-incrimination is protected by the Fifth Amendment right to refuse to answer a potentially incriminating question, not by refusing to testify altogether.

Question 10- Statement Attributable to a Party-Intro A large delivery truck collided with a car. At the time of the accident, the driver of the truck said to the car driver, "The accident was my fault; I wasn't paying any attention. Don't worry, my company will make it right." The delivery company had not authorized the truck driver to make that statement. The subsequent investigation of the accident by the delivery company revealed that the truck driver had been drinking on the day of the accident. He was fired. The car driver brings an appropriate action against the delivery company for damages resulting from the accident. The truck driver has disappeared. The car driver now seeks to testify as to what the truck driver said at the time of the accident. Is the evidence admissible? A Yes, as a statement attributable to the defendant. B Yes, as an excited utterance. C No, because the truck driver is no longer employed by the delivery company. D No, because the delivery company had not authorized the truck driver to speak on its behalf.

(A) The evidence is admissible. This question raises the issue of whether an employee's out-of-court statement will be attributed to the employer, and thus considered a statement attributable to a party (i.e., a vicarious admission). The statement can be attributed to the employer, provided: (i) it was made while the person was employed by the employer (not before or after the period of employment); and (ii) the statement related to the employment. Thus (A) is correct. The truck driver's statement was made while he was employed by the delivery company, and it related to his employment since it pertained to an accident that occurred when he was driving a company truck, presumably in the course of employment. Thus, (C) is incorrect. That the truck driver no longer employed by the delivery company does not bear on the admissibility of his statement; what counts is that he was employed at the time he made the statement. (D) is also incorrect That the delivery company did not authorize the truck driver to make the statement does not preclude it from being admissible. Although one basis for attributing an employee's statement to the employer is the employer's authorization for the employee to speak on its behalf about the matter, that is not the only one. (B) is incorrect because there is little reason to conclude that the truck drivers statement was an "excited utterance" within the meaning of that hearsay exception. The question does not indicate that the truck driver spoke in an excited manner or that he was agitated. That the truck driver spoke shortly after the accident would not alone be sufficient to make his statement an "excited utterance."

Question 13- Expert use of photos and reports - Intro A plaintiff was injured when a portion of a spiral stairway in a shopping mall collapsed. The plaintiff filed suit against both the owners of the mall and the designers of the staircase. At the trial of the case, the plaintiff wishes to call a highly qualified civil engineer to the stand to testify as an expert witness. The expert is prepared to testify that the spiral staircase was improperly designed, and the design defect caused a portion of the staircase to collapse under the plaintiff. The expert's proposed testimony is based in part upon a series of photographs taken by a structural engineer hired by the expert immediately after the expert was engaged by the plaintiff's attorney, and the accompanying report by the structural engineer. It is customary for civil engineers to form professional opinions based on these reports. The photographs are of the collapsed stairway and of an identical stairway located in another pan of the mall. Neither the photographs nor the report has been admitted into evidence, but the expert is willing to disclose to the jury the facts on which he relied in forming his opinion. Should the expert's testimony be ruled admissible? A Yes, because other civil engineers ordinarily reasonably rely on structural engineers' reports in forming professional opinions. B Yes, because the expert will disclose to the jury the facts on which he based his opinion. C No, because the expert based his opinion in part on the structural engineer's opinion. D No, because the photographs and report were commissioned solely for the purpose of preparing for litigation and were not admitted into evidence.

(A) The expert's testimony is admissible because other civil engineers ordinarily reasonably rely on structural engineers' reports in forming professional opinions. Expert testimony is admissible if the subject matter is one where scientific, technical, or other specialized knowledge would assist the jury in understanding the evidence or determining a fact in issue. [Fed R Evid 702] The proper design of a spiral staircase would not be a matter of common knowledge and the testimony of an expert would be of assistance in determining whether the design was faulty. The expert's opinion may be based on facts not in evidence that were supplied to the expert out of court, and which facts are of a type reasonably relied upon by experts in the particular field in forming opinions on the subject. [Fed R Evid 703] The expert may therefore give an opinion based on the photographs and the structural engineers report such photographs and reports are of a type reasonably relied upon by civil engineers in forming opinions on structural design. Federal Rule 703 allows the expert's testimony even though the photographs and report are not in evidence. (B) is incorrect because Federal Rule 703 does not require an expert to disclose the facts on which he relied in forming his opinion. In fact, the proponent of the expert opinion must not disclose those facts to the jury (because they may be of a type not admissible in evidence) unless the court determines that their probative value in assisting the jury to evaluate the expert's opinion substantially outweighs their prejudicial effect. (C) is incorrect because the Federal Rules allow the expert to base his opinion on opinions in the structural engineers report as long as such opinions in reports would be reasonably relied upon by civil engineers in forming opinions. An expert traditionally is not permitted to rely on the opinions of others as a predicate for his own opinion. However, Federal Rule 703 significantly expands the traditional rule and permits an expert to base his opinion on the opinion of others if they are of the type reasonably relied upon by experts in the field, as discussed above. Therefore, if civil engineers ordinarily reasonably rely on such opinions in reports from structural engineers, the Federal Rules allow the expert to base his opinion on opinions in the report. (D) is incorrect because the fact that the report and photographs were commissioned solely for the purpose of litigation and were not admitted into evidence is irrelevant when an expert is basing an opinion on facts reasonably relied upon by experts in that field. When determining whether a report comes under the business records exception to the hearsay rule, the courts sometimes look at whether the report was prepared for litigation in determining if it was a record maintained in conjunction with a business activity. [Palmer v. Hoffman (1943)] Here, whether the report is hearsay is not at issue because an expert may base his opinion on facts reasonably relied upon by experts in that particular field even if those facts are inadmissible hearsay.

Question 14-Declarant's state of mind- Intro A defendant was charged with the January 12 armed robbery of a grocery store in Texas. His defense is that he was not in Texas on the date of the armed robbery and thus he could not have committed the crime. To show that he was not in the area on January 12, the defendant wishes to introduce into evidence a letter he wrote to his sister stating, I will see you in Vermont on January 12." The prosecution objects. Is the letter admissible? A Yes, as evidence of the defendant's intent to go to Vermont on the date in question. B Yes, as a present sense impression. C No, because the statement in the letter is irrelevant. D No, because it is hearsay not within any recognized exception to the hearsay rule.

(A) The letter is admissible as evidence of the defendant's intent to go to Vermont on January 12. Hearsay is an out- of-court statement offered to prove the truth of the matter asserted upon objection, hearsay must be excluded unless it falls within a recognized exception to the rule. The letter is an out-of-court statement, and it is being offered to prove its truth; i.e., that the defendant intended to be in Vermont on January 12. The letter, therefore, is hearsay. However, there is an exception for declarations of state of mind, including declarations of intent offered to show subsequent acts of the declarant; i.e., a statement of intent to do something in the future is admitted as circumstantial evidence that the intent was carried out. This principle was established in the famous Hillmon case, which held that state of mind is admissible to show that the declarant acted in conformity with his expressed declaration. The defendant's letter was a statement of intent to go to Vermont on January 12 and is admissible as circumstantial evidence that he did so. (B) is wrong because it states the wrong hearsay exception. A present sense impression is a comment made concurrently with the perception of an event concerning the event perceived. Clearly this exception does not apply to these facts because the defendant was not perceiving an event and describing it in his letter; rather he was expressing his state of mind at the time he was writing. (C) is wrong because the letter is relevant. Evidence is relevant if it tends to make the existence of a material fact more probable or less probable than it would be without the evidence. The defendant's whereabouts on the day of the crime are certainly a fact of consequence to the determination of this action. The letter makes the fact that he was in Vermont on January 12 more probable than it would be without the letter; thus, the letter is relevant. (D) is wrong because the letter falls within the state of mind exception to the hearsay rule (see above).

Question 30-Impeachment - Prior Conviction remote in time- Intermediate At the defendant's trial for rape, he calls a witness who testifies that she was on her patio barbecuing some hamburgers at the time of the charged rape and saw the assailant run from the victim's apartment. She further testifies that the person who ran from the victim's apartment was not the defendant. On cross-examination by the prosecutor, to which of the following questions would a defense objection most likely be sustained? A "Weren't you convicted of perjury 1 1 years ago?" B "Weren't you under the influence of heroin at the time you were barbecuing those hamburgers?' C "Haven't you and the defendant known each other since grammar school?' D "Didn't you embezzle funds from your most recent employer?'

(A) The objection to the perjury question is most likely to be sustained Federal Rule 609 permits the prosecution to inquire into prior convictions of crimes requiring proof or admission of dishonesty or false statement unless over 10 years have passed since the date of conviction or date of release from confinement (whichever is later). While the facts do not indicate the latter date (or even whether a confinement occurred), (A) remains the best of the four choices. The conviction in (A) is more than 10 years old, so it probably would be subject to objection as being too remote. (B) relates to the witness's ability to perceive and would be a legitimate question on cross-examination. (C) shows a possible bias on the part of the witness, which is an acceptable method of impeachment. (D) relates to a prior bad act that shows dishonesty. Such acts may be asked about on cross-examination of the witness.

Question 3- Prior Identification - Intro As a woman was walking home on a dark cloudy night, a man came upon her from behind and stole her purse. A suspect was arrested nearby shortly thereafter. After the suspect had been booked, the police took his photograph. They then showed his photograph, along with the photographs of four people who had the same general features as the arrested man, to the woman. The woman identified the suspect as the culprit. At trial, after the woman had identified the suspect as the person who stole her purse, the suspect's attorney objects to the prosecution's introduction into evidence of the photographic identification. What is the most likely result? A The objection will be overruled. B The objection will be sustained, because the suspect's right to counsel was violated at the showing of the photograph to the woman. C The objection will be sustained, because the woman did not have a good opportunity to observe the culprit. D The objection will be sustained, because a photographic identification must be considered a critical stage of the proceeding.

(A) The objection will likely be overruled under the Federal Rules of Evidence, after a witness has testified, statements of prior identification are admissible to prove their truth unless the defendant can show that the circumstances of the identification were unnecessarily suggestive and likely to result in irreparable misidentification. There is no evidence of that in the facts. (B) is wrong because there is no right to counsel at a photo display. (C) is wrong because it goes to the weight to be given the evidence rather than its admissibility. (D) wrong because the photograph identification would not be considered a critical stage.

Question 2 -Credibility -Intro A plaintiff sued a defendant in a contract dispute. The plaintiff calls a witness to testify as to his personal knowledge of the agreement. The plaintiff now wants a second witness to testify as to her knowledge of the first witness's honesty. The defendant objects and the court sustains the objection. Why is the testimony of the second witness inadmissible? A The first witness's credibility has not been questioned. B It would be inadmissible under the hearsay rule. C The second witness may not testify as to an opinion. D Character evidence is generally inadmissible in civil cases.

(A) The plaintiff may not call the second witness to testify about the first witness's honesty because his credibility has not been questioned. Generally, a party may not bolster or accredit the testimony of her witness until the witness has been impeached. [Fed R Evid 608(a)] Here, the defendant has not tried to cast any adverse reflection on the first witness's truthfulness (i.e., he has not been impeached). Thus, the second witness's testimony as to her knowledge of the first witness's honesty will not be allowed. (B) is incorrect because these facts do not present a hearsay problem. Hearsay is an out-of-court statement offered in evidence to prove the truth of the matter asserted. [Fed R Evid 801 (c)] The second witness's testimony will relate only to her knowledge of the first witness's honesty; the second witness will not be testifying as to a statement made by an out-of-court declarant. Consequently, her testimony does not involve hearsay. (C) is incorrect because the credibility of a witness may be supported (if impeached) or attacked by opinion or reputation evidence regarding the witness's truthfulness. [Fed R Evid 608(a)] Thus, if the first witness's honesty had been attacked by the defendant, the second witness could testify as to her opinion of the first witness's character for truthfulness or as to his reputation for honesty. (D) is incorrect because, although evidence of character of a person in a litigated event is generally inadmissible in civil cases (except where character itself is an essential element of a claim or defense in the case), opinion or reputation testimony about a witness's character for truthfulness is generally admissible for purposes in both civil and criminal cases.

Question 24 -Recording - Intermediate A defendant was arrested for driving a car while intoxicated. The police made a recording of the defendant as he attempted to walk a straight line and touch his nose at the time of the arrest. The prosecution seeks to introduce this recording at the defendants. Is the recording admissible? A Yes, because it is more probative than prejudicial. B Yes, as an admission by a party-opponent. C No, because it violates the defendant's privilege against self-incrimination. D No, because a specific instance of misconduct cannot be proved by extrinsic evidence.

(A) The recording is admissible as relevant and material evidence not excluded by any rule. Furthermore, (A) is the correct pick by process of elimination. (B) is incorrect. For the purposes of the hearsay rule, a "statement" is an oral or written assertion or assertive conduct. An admission by a party-opponent is a "statement" made by a party and offered against that party, and is treated as nonhearsay under the Federal Rules. Here, the recording is not an admission because it is not a "statement" by the defendant. Rather, it is nonassertive conduct offered to show the defendant's mental and physical condition. Thus, the hearsay rule and its exclusions and exceptions are not in issue. (C) is incorrect because the privilege against self-incrimination applies only to testimonial evidence. (D) is incorrect because it recites a rule concerning impeachment of witnesses with evidence of prior acts of misconduct involving untruthfulness. This rule is inapplicable to these facts. Almost all charged crimes (i.e., instances of misconduct) are proven by extrinsic evidence. Note that because this recording would be considered documentary evidence, a proper foundation will have to be laid before it can be admitted.

Question 15 - 13th Amendment - Race- Intermediate Congress enacted a statute that purported to ban all discrimination against African-Americans in any commercial transaction taking place within the United States. Would the statute most likely be held constitutional? A Yes, under Thirteenth Amendment provisions barring badges or incidents of slavery. B Yes, because the federal government has an important interest in furthering the equal protection provisions of the Fourteenth Amendment. C No, because Congress's powers under the Commerce Clause do not extend so far as the statute would require. D No, because commercial transactions are not among the privileges or immunities of national citizenship.

(A) The statute is constitutional as a legitimate exercise of congressional enforcement powers under the Enabling Clause of the Thirteenth Amendment. The Thirteenth Amendment prohibits slavery. The Enabling Clause of the amendment has been held to confer on Congress the authority to proscribe almost any private racially discriminatory action that can be characterized as a badge or incident of slavery. Because the statute at issue bans all discrimination against African-Americans in commercial transactions, it necessarily reaches private conduct. Such congressional action is constitutionally permissible pursuant to the Thirteenth Amendment. (B) is incorrect application of the Fourteenth Amendment has been limited to cases involving state action [See United States v. Morrison (2000)]. The statute here reaches private action, and so the Thirteenth Amendment is the correct source for the law, since that amendment addresses private action. (C) is incorrect because, even if Congress's power over interstate commerce would not reach every commercial transaction, the statute would be enforceable under the Thirteenth Amendment, as discussed above. (D) is incorrect because it is irrelevant. While it is true that the commercial transactions here are not among the privileges or immunities of citizenship (which include rights such as the right to petition Congress for redress and the right to interstate travel), the law can be based on the Commerce Clause or the Thirteenth Amendment, and thus constitutional.

Question 3 -Time, place & manner- Intro A state statute prohibits speechmaking and loud public gatherings within 250 feet of the state's legislative chamber when the legislature is in session, but permits silent picketing at any time, as long as the picketing does not interfere with pedestrians or traffic. The nearest place to the legislative chamber where speeches could be made during a session is a large public park directly opposite the chamber. During a controversial debate on a proposed bill to ban abortions, a man in the park began voicing his support of the ban. As the man spoke, a crowd of about 250 gathered. When fervor built, the man urged the crowd to cross the street with him to the steps of the legislative chamber to make their voices heard within the legislature. When the chanting crowd reached the front of the chamber, the state police dispersed the crowd and arrested the man, who was subsequently charged with violating the statute. If the man challenges the constitutionality of the statute under which he was charged, will a court most likely find the statute constitutional? A Yes, on its face and as applied to the man. B Yes, on its face, but not as applied to the man. C No, on its face, because a state's citizens have a right to take their complaints to their state Legislature. D No, on its face, because it permits silent picketing while prohibiting other picketing.

(A) The statute is likely to be held constitutional on its face and as applied. The First Amendment protects the freedom of speech. Generally, content-based restrictions on speech will be found to violate the First Amendment. However, government may place reasonable restrictions on the time, place, and manner of speech. A speech regulation of a public forum will be found reasonable only if it is content neutral, it is narrowly tailored to serve an important government interest, and it leaves open alternative channels of communication. The regulation here passes muster. The area within 250 feet of the legislative chamber probably is a public forum. However, the regulation seems reasonable. It prohibits all loud public gatherings near the chamber when the legislature is in session to vote or debate. Thus, the regulation does not differentiate speech based on content. Moreover, the ban extends only to loud gatherings close to the chamber while the legislature is working. It appears to have the important purpose of enabling legislators to hear one another, and it seems to be narrowly tailored to that purpose. Moreover, the regulation leaves other channels of communication open (such as silent picketing) The application of the statute to the man does not make it unconstitutional as applied; there is no reason why he could not have spoken as noisily as he wanted to in the park. Thus, (B) is incorrect. (C) is incorrect because there are many methods to direct complaints to a legislator without yelling through his window. (D) is incorrect because silent picketing can be considered to be reasonable at the same time that loud picketing is prohibited.

Question 9 -Establishment Clause- Intro A state passed a statute that provided for direct reimbursement from public funds to nonpublic schools, including religious schools, of the cost of performing various testing services required of all schools by state law. The three state-prepared tests involved consisted of a student evaluation test, a comprehensive achievement test, and scholarship and college qualification tests. The law also provided for payment to the nonpublic schools for the grading of the tests, which are graded Objectively. On review by the Supreme Court, what is the likely result? A The statute is constitutional because objective grading standards are used and reimbursement covers only secular services. B The statute is constitutional, if the state interest is great enough to justify the burden on religion, and no alternative means are available. C The statute is unconstitutional because direct payment to nonpublic schools is a violation of the Establishment Clause of the First Amendment. D The statute is unconstitutional because the statewide administration of the test for nonpublic as public schools entangles the state with religious schools.

(A) The statute should be held constitutional Governmental action that does not contain a sect preference will pass muster under the Establishment Clause if it has a secular purpose, its primary effect neither advances nor inhibits religion, and it does not require excessive government entanglement with religion. Here, the nature of the aid provided by the statute has a secular purpose, is of legitimate interest to the state, and does not present any risk of being used to aid the transmission of religious views. (B) misstates the test; this is a test that was used for the Free Exercise Clause, and this is an Establishment Clause case. (C) is incorrect because not all aid to religious schools violates the Establishment Clause. The Court has upheld aid in the form of textbooks, transportation to and from school, and assistance for the administration of standardized tests. (D) has no basis in the facts given.

Question 23 - Article III Jurisdiction-Advanced Which of the following suits would not fall within the United States Supreme Courts original jurisdiction under Article III, Section 2? A A suit seeking to assert the interest of state citizens in retaining diplomatic relations with a foreign nation. B A suit seeking to protect a state's timber from allegedly illegal cutting by residents of another state. C A suit seeking to enjoin enforcement of an allegedly unconstitutional executive order that will greatly limit the state's authority to make policy decisions regarding admission to state universities. D A suit by the United States government seeking to enjoin state construction of a bridge over a navigable water way.

(A) The suit to assert state citizens' rights is not within the Supreme Court's original jurisdiction under Article III, Section 2, the United States Supreme Court has original jurisdiction in all cases affecting ambassadors, other public ministers, and consuls, and in which a state is a party. In (A), the state is not really seeking to advance or protect any interest of its own. Rather, the state is attempting to act in parens patriae (i.e., to act as a representative of its citizens, thereby asserting their interests). Thus, the state is not an actual party in this case in the sense that the Supreme Court has traditionally required to justify exercise of original jurisdiction. (B) would be a proper case for institution under the Supreme Court's original jurisdiction because it involves an attempt by a state to protect its own economic interest rather than to assert the interests of its citizens in a representative capacity. Similarly, (C) sets forth a situation in which a state is attempting to defend its asserted right to render decisions affecting admissions policies relative to its own state universities. Thus, in (C) the state is an actual party to the case. Finally, (D) describes an attempt by the federal government to prevent state construction of a bridge (presumably pursuant to the admiralty power). Clearly, this case involves an alleged grievance that will be directly committed by a state. Therefore, the state an actual party.

Question 5 - Impeachment Prior Inconsistent statement -Intro The defendant was on trial for driving while intoxicated and injuring a pedestrian. The pedestrian claimed that the defendant was driving the car; however, the defendant's spouse testified at trial that she had been driving the car at the time of the accident and had not consumed any alcohol that evening. In response, the prosecution calls a friend of the defendant's spouse to testify that the spouse told the friend that she and the defendant had changed seats in the car after the incident and that she remained in the driver's seat until the police arrived. Is the testimony admissible? A Yes, for impeachment purposes only. B Yes, to show bias. C No, because a witness who is available to testify can be impeached only through cross-examination. D No, because the witness must be given an opportunity to explain or deny the statement.

(A) The testimony is admissible to impeach the testimony of the defendant's spouse. The credibility of a witness can be impeached by showing that the witness has made prior statements that are inconsistent with some material part of her present testimony. Here, the friend is prepared to testify that the defendant's spouse stated that she had not been driving the car at the time of the incident, a fact that would be material to the allegations in the lawsuit and inconsistent with the spouse's trial testimony. Thus, the friend's testimony is admissible for impeachment purposes. (B) is incorrect. Extrinsic evidence of bias is not admissible unless a foundation has been laid--the witness must first be questioned about the facts that show bias on cross-examination. If the witness, on cross-examination, admits to the facts claimed to show bias, then the trial judge may allow the extrinsic evidence to be introduced. Here, nothing suggests that the spouse was cross-examined as to the facts tending to show bias; thus, this is an incorrect ground for admitting the testimony. (C) is incorrect because a witness ordinarily may be impeached by either cross-examination or extrinsic evidence. Here, the prosecution is seeking to impeach the defendant's spouse by extrinsic evidence of her prior inconsistent statement, and the evidence will be admissible for that purpose. (D) is incorrect because the opportunity to explain or deny the statement can be given subsequent to the friend's testimony; it does not need to be given prior to her testimony.

Question 2- Present State of Mind Declaration- Intro A mechanic was tried for the aggravated assault of a nurse. The mechanic called a witness to the witness stand. The witness was to testify that the night before the alleged crime, the mechanic stated to the witness that he was going to visit his mother in a distant city some 1,000 miles away. Is the testimony admissible? A Yes, because it is a declaration of intent to do a future act. B Yes, because of the verbal acts exception. C No, because it is not relevant. D No, because it is hearsay not within any exception.

(A) The testimony would be hearsay, but it would be admissible under the exception of a declaration of an intent to do a future act, sometimes referred to as a declaration of present state of mind. Under this exception, a declaration of intent to do something in the future is admitted as circumstantial evidence tending to show that the intent was carried out.[Fed R Evid 803(3)] The trip provides an alibi defense and is therefore relevant; so (C) is incorrect. (B) is wrong because this is not a verbal act; verbal acts are out-of-court statements that when spoken have legal significance, e.g. words of contract or defamation. (D) is incorrect because, while the statement constitutes hearsay, does fall under an exception: the declaration of intent to do a future act.

Question 10-Intro A state passed a statute requiring all employers in the state to provide a medical insurance plan for full-time employees. A state trade association to which many state employers belong brought suit in federal court, asking the court to strike down the statute as unconstitutional. Assume that the state statute is not preempted by any federal law. Which of the following best reflects the burden of persuasion in this case? A The burden is on the trade association to prove that the statute is not rationally related to a legitimate state interest. B The burden is on the trade association to prove that the statute is not necessary to achieve a compelling state interest. C The burden is on the state to prove that the statute is rationally related to a legitimate state interest. D The burden is on the state to prove that the statute is necessary to achieve a compelling state interest.

(A) The trade association will have to prove that the statute is not rationally related to a legitimate state interest. Whether the statute is treated as a due process challenge (because it affects all employees), or as an equal protection problem (because employers are singled out for special treatment), the same standard will apply. Because there is no fundamental right involved and employers are neither a suspect nor quasi-suspect class, the rational basis standard will apply. Under the rational basis standard, the party challenging the government action has the burden of proving that the action is not rationally related to any legitimate state interest - a very difficult burden to meet. Thus, the burden will be on the trade association and (A) is correct. (B) is incorrect because it states the wrong standard. (C) and(D) are incorrect because they place the burden on the wrong party.

Question 13- Prior Testimony- Intro A decedent was prosecuted for criminal violations of a hazardous waste disposal act and convicted, in part on the testimony of a witness. After the decedent's death, the plaintiff on whose property the decedent dumped the hazardous waste brought suit to recover the cleanup costs against the executor of the decedent's estate. Because the witness is currently incarcerated in another state, and beyond the reach of the courts subpoena powers, the plaintiff seeks to introduce the transcript of the witness's testimony from the decedent's criminal trial in the present action. The executor objects to its admission. How should the court rule on the admissibility of the transcript? A Admissible, because the witness was subject to cross-examination in the previous action. B Admissible, because the transcript of the criminal trial is a public record. C Inadmissible, because the plaintiff has not shown that the witness is truly "unavailable." D Inadmissible, because the executor was not a party to the previous action.

(A) The transcript is admissible under the former testimony exception to the hearsay rule under this rule, the testimony of a now unavailable witness given under oath at another hearing is admissible in a subsequent trial as long as there is a sufficient similarity of parties and issues so that the opportunity to cross-examine at the prior hearing was meaningful. In a civil proceeding, the parties do not need to be identical, but the party in the original action must be a predecessor in interest to the party against whom the testimony is being offered (e.g., a privity relationship), so that a similar motive existed to develop or cross-examine the declarant's testimony. Here, the decedent was a predecessor in interest of the executor of his estate and had adequate opportunity and motive to cross-examine the witness's testimony against him, which was given under oath at a criminal proceeding. Hence, the testimony is admissible under the former testimony exception. (B) is incorrect because the transcript of the witness's testimony, to the extent that it is being offered to prove the truth of her assertions, does not fall under the public records exception to the hearsay rule. While the Federal Rules also allow a judgment of a felony conviction to be used to prove any fact essential to the judgment, the transcript of a witness is not admissible for that purpose. (C) is incorrect because in civil cases, the unavailability requirement for the former testimony exception is satisfied if the declarant is beyond the reach of the court's subpoena and the statement's proponent unable to procure her attendance or testimony by process or other reasonable means. (D) is incorrect because, as discussed above, the similarity of issues and parties is sufficient to permit the testimony to be admitted against the executor even though he was not a party to the original action

Question 2- Article III Judges -Intro Congress created a tribunal by statute, which was empowered to review claims made by clients of a federal agency and make recommendations to the agency regarding their merits. A lawyer was properly appointed to the tribunal. Six years later, the tribunal was abolished by repeal of the authorizing legislation, and the lawyer was offered an administrative position in the transportation department at a lower salary. The lawyer brought an action against the federal government on the grounds that Congress may not remove a federal judge from office during good behavior, nor decrease his salary during continuance in office. Counsel for the government made a motion to dismiss. How should the trial court rule? A For the government, because the lawyer was not an Article III judge and is not entitled to life tenure. B For the government, because the lawyer lacked standing to raise the question. C For the lawyer, because he has established a property right to his position on the tribunal. D For the lawyer, because of the independence of the federal judiciary constitutionally guaranteed by Article III.

(A) The trial court should rule for the government. Only Article III judges enjoy the constitutional grant of life tenure, and the lawyer is not an Article III judge. The tribunal is an administrative agency with limited jurisdiction, and while the lawyer exercises some adjudicatory powers, all persons with some adjudicatory functions are not judges within the meaning of Article III. (A) is thus correct and (D) is wrong. (C) is also wrong, because the Constitution does not recognize property rights in employment. The lawyer could show some kind of wrongful discharge only if he could claim that he was deprived of something "in the nature of a property right." (B) is incorrect because the lawyer clearly has standing; he alleges a specific injury to himself and brings the claim in his own right, alleging the violation of a constitutional right.

Question 27 - Spousal Privilege & Confidential Marital Communications - Intermediate During the course of their marriage, a husband told his wife that he stole a famous painting from a federal museum. Six months after the admission, the couple divorced. Shortly after the divorce, the husband was killed in an automobile accident. Later, the wife read in the paper that a man had been charged with the theft of the painting her husband admitted to stealing and was about to be tried in federal district court. She told her friend that the man was probably innocent because the husband told her that he had stolen the painting himself. The friend told several other people what the wife had told her, and eventually the story got back to the defense attorney. The attorney now wants the wife to testify in court to the husband's statement. Can the wife be compelled to testify? A Yes, but only because the husband is dead and cannot invoke his privilege. B Yes, because there is no privilege when the defendant is not a spouse. C No, because the couple was still married at the time of the disclosure. D No, because her testimony is not essential to prevent a fraud on the court.

(A) The wife can be compelled to testify because her husband is dead and cannot invoke the privilege. There are two separate privileges related to marriage. There is spousal immunity, under which: (i) a married person whose spouse is the defendant in a criminal case may not be called as a witness by the prosecution, and (ii) a married person may not be compelled to testify against her spouse in any criminal proceeding. In federal court, this privilege belongs to the witness-spouse so that she may not be compelled to testify, but neither may she be foreclosed from testifying. This privilege terminates upon divorce. There is also a privilege for confidential marital communications, under which either spouse, whether or not a party, has a privilege to refuse to disclose, and to prevent another from disclosing, a confidential communication made between the spouses while they were married. Both spouses jointly hold this privilege. Divorce does not terminate this privilege retroactively. Because the communication must be made in reliance upon the intimacy of the marital relationship, if the communication is made in the known presence of a stranger, it is not privileged. Similarly, if one spouse voluntarily reveals the contents of the communication to a stranger, that spouse waives the protection of the privilege as to herself (i.e., she cannot use the privilege to refuse to disclose, or to prevent another from disclosing, the communication), but the other spouse (i.e., the one who did not reveal the communication) retains this privilege. Here, the spousal immunity between the husband and the wife terminated upon their divorce Thus, the only consideration is the applicability of the privilege for confidential marital communications. The husband's statement to the wife came during their marriage and was made in reliance upon the intimacy of their relationship (marital communications are presumed to be confidential). Thus, the statement was covered by the privilege for confidential marital communications. Their subsequent divorce did not terminate this privilege. However, when the wife revealed to her friend what her husband had told her concerning the theft of the painting, the wife lost her privilege to refuse to disclose the matter. If the husband were alive, he would retain the privilege despite the wife's disclosure and could prevent her from testifying to his statement concerning the theft of the painting. Because the husband is dead, he cannot invoke his privilege. Because the wife has waived her privilege and the husband is unable to foreclose her testimony, she can be compelled to testify. (B) is incorrect because the privilege for confidential marital communications applies to the disclosure of matters communicated during and in reliance on the intimacy of the marital relationship regardless of whether one of the spouses is a defendant in a criminal case. Even spousal immunity is deemed to preclude the compelled testimony of one spouse against the other in any criminal proceeding, regardless of whether the other spouse is a defendant. The difference when a spouse is a criminal defendant is that the other spouse may not even be compelled to take the stand. (C) is incorrect because it does not take into account the fact that the wife waived her privilege when she communicated her husband's admission to her friend, as explained above. (D) incorrectly concludes that the wife cannot be compelled to testify. Due to her knowing and voluntary revelation of the husband's statement to her friend, the wife has waived her privilege and may be compelled to testify. Note that, if the privilege were still applicable (i.e., if the wife had not waived it), she could not be compelled to testify as to the contents of the privileged communication simply on the ground that such testimony would be essential to prevent a fraud on the court.

Question 4- Voice Identification - Intro In a criminal trial, the prosecutor called a witness to the stand to authenticate the voice in a tape recording as the defendant's. The only other time the witness had heard the defendant's voice was after his arrest. Assuming a proper foundation has been laid, may the witness properly authenticate the defendant's voice? A Yes, because the witness is now familiar with the defendant's voice. B Yes, because the prosecutor can qualify the witness as an expert on the defendant's voice. C No, because the witness's testimony would be inadmissible hearsay. D No, because the witness did not hear the defendants voice until after he was arrested.

(A) The witness may properly authenticate the defendant's voice because she is now familiar with his voice. Any person familiar with an alleged speaker's voice may authenticate a recording of the voice by giving an opinion as to its identity. Thus, because the witness is now familiar with the defendant's voice, she may give her opinion as to whether it is his voice on the tape. (B) is incorrect because the witness does not need to be qualified as an expert--lay opinion testimony is sufficient to identify a voice (assuming the lay witness is familiar with that voice). Expert testimony is appropriate only when the subject matter is one where scientific, technical, or other specialized knowledge would assist the trier of fact in understanding the evidence or determining a fact in issue. [Fed R Evid 702] Here, identification of the defendant's voice does not require such specialized knowledge; rather, all that required familiarity with his voice. (C) is incorrect because the witness's testimony would not be hearsay at all. Hearsay is a statement, other than one made by the declarant while testifying at the trial or hearing, offered in evidence to prove the truth of the matter asserted. [Fed R Evid 801 (c)] The witness is not going to be testifying to an out-of-court statement. Rather, she will testify that, being familiar with the voice of the defendant by virtue of having heard that voice before, she can now identify the voice on the tape as being that of the defendant. Because the witness will not be testifying as to any particular statement made by the defendant, there is no hearsay problem. (D) is incorrect because, as long as the witness is familiar with the voice of the defendant, it makes no difference that she acquired such familiarity only after he was arrested. Thus, the witness may properly authenticate the voice.

Question 22- Separation of Powers - Advanced Congress adopted legislation prohibiting the federal courts from ordering busing as a remedy for past racial discrimination in a public school district. Which of the following is the strongest argument that the federal legislation is unconstitutional? A The courts, not Congress, have the primary responsibility for defining the minimum requirements of the Equal Protection Clause of the Fourteenth Amendment. B The Privileges or Immunities Clause of the Fourteenth Amendment prohibits Congress from limiting the forms of relief afforded by federal courts. C Congress cannot limit the authority of federal courts to hear and decide cases properly presented for decision. D The legislation unduly burdens interstate commerce.

(A)The strongest argument against the legislation's constitutionality is that Congress cannot usurp the courts' authority to define the scope of the Equal Protection Clause. If the Equal Protection Clause requires a remedy for past racial discrimination, then Congress could not constitutionally interfere with the fashioning of a judicial remedy to achieve constitutionally required conduct. (D) is wrong because Congress may burden interstate commerce, because it has very broad power on that subject. (C) is wrong because Congress may limit the jurisdiction of federal courts. (B) is wrong because the Privileges or Immunities Clause protects individual rights against infringement by state government, it does not limit the powers of Congress vis-a-vis the federal courts.

Question 7- Tax- Intermediate During extensive hearings, a state legislature determined that double tractor-trailer rigs - trucks consisting of a tractor (the motorized portion) towing two large, connected trailers - caused the roadway to deteriorate faster than other freight vehicles and autos because of their weight. Traffic safety experts also produced evidence showing that double tractor- trailer vehicles were involved in more accidents than other freight vehicles, primarily due to "jackknifing,' where the rear trailer loses traction and swerves violently, causing the entire vehicle to be upended. Consequently, the legislature passed a statute requiring the owners and users of double tractor-trailer vehicles to pay a user's fee, in addition to normal vehicle licenses, of 10 cents per mile traveled over state highways and an annual registration fee of $5,000. The owner of 30 tractors in a neighboring state that almost exclusively pull double trailer rigs through the state imposing the fees determined that about 30% of the total mileage of all of the owner's vehicles is accumulated in that state, and that there is no easy way to avoid traveling through that state to get to the delivery destinations in other states. The mileage fees and registration fees for 30 trucks in a year would be about 60% of the owner's gross annual income. The owner brought suit in federal district court seeking a judicial declaration that the fees imposed by the state statute are unconstitutional. At trial, attorneys for the state produced evidence of highway destruction and safety hazards from the double tractor-trailer rigs as found by legislative committee hearings. The owner proved the relevant facts about his operations and the cost the statute would impose. If the court finds the tax unconstitutional, what is the most likely reason? A It seeks to regulate by taxation what it could not do directly, regulate interstate commerce. B The state's interests in preserving its highways and in promoting traffic safety are outweighed by the interference with interstate transportation of goods. C It violates the owner's right to the equal protection of the laws. D A use tax on companies engaged in interstate commerce violates the Commerce Clause.

(B) If the court should find the statute unconstitutional, it will be because the state's safety concerns are outweighed by the burden on interstate commerce. Where a state enacts a law that does not discriminate against interstate commerce, it may still be invalid if the benefits from the law are outweighed by the burdens the law places on interstate commerce. The determination is made on a case-by-case basis, depending on the facts. Here, there is no discrimination against interstate commerce, and the state has shown that tandem trailer trucks are more dangerous than other vehicles and they cause more road damage. However, the owner has shown that the state law makes expensive to operate his interstate trucks in that state. The court could easily find that the added expense on interstate carriers is not outweighed by the benefits to the state (Note that the Supreme Court has twice addressed this issue in similar cases [Raymond Motor Transportation, Inc v. Rice (1978); Kassel v. Consolidated Freightways Corp. (1981)] with a similar result). (A) is incorrect because it assumes that the states cannot regulate interstate commerce in general or trucks in particular. As indicated above, safety aspects of interstate commerce can be regulated by the states as long as the regulation does not discriminate against or unduly burden interstate commerce. If a state has the power to regulate an aspect of interstate commerce, it has the power to tax it as well, and a tax such as the one here could be valid (see (D), below). (C) is incorrect because the Equal Protection Clause is not violated here under the Clause, a law that regulates economic interests where no suspect class or fundamental right is involved will be upheld as long as the classification is rationally related to any legitimate governmental interest. Preventing accidents and saving money on road repair are legitimate government interests; so the law here is valid. (D) is incorrect because states can impose use taxes on interstate businesses as long as the activity taxed has a substantial nexus to the taxing state; the tax is fairly apportioned; the tax does not discriminate against interstate commerce; and the tax is fairly related to the services provided by the state. The tax here appears to meet these conditions.

Question 15- Larceny, Embezzlement, False Pretense- Intermediate An art restorer, after attending art school for a number of years, secured a job restoring the paintings for an art museum. After several years on the job, the artist discovered that he could imitate the artwork of nearly any artist. He decided that he could make some extra money copying the artwork of up-and-coming artists, while staying away from more well-known artists to reduce his chance of getting caught. An art collector searching for a painting by a new artist saw the restorer at an art fair selling various paintings, one of which appeared to be by the new artist. The restorer was selling the painting for $100. The collector thought that the price was very low and that the painting should probably sell for around $500, but she bought the painting anyway, giving the restorer $100 after the restorer confirmed that the painting was an original from the new artist. After taking it to an art appraiser for insurance purposes, she discovered that the painting was a forgery. However, she also discovered that the painting's frame was about $125 With which theft offense may the art restorer be charged? A Larceny by trick B False pretenses C Embezzlement D No theft offense

(B) The art restorer has committed false pretenses because his misrepresentation concerning the authenticity of the painting induced the collector to convey title to the $100. The offense of false pretenses consists of obtaining title to the property of another by an intentional (or knowing) false statement of past or existing fact, with intent to defraud the other. The art restorer falsely represented to the collector that the painting he sold her was an original from a new artist, intending that the collector would rely on such misrepresentation by paying money for the painting. The collector, acting in reliance on this misrepresentation, conveyed to the art restorer title to the $100. Thus, all of the elements of false pretenses are present in the art restorer's dealings with the collector. Because the art restorer has committed false pretenses, which is a theft offense, (D) is incorrect. Monetary loss on the part of the victim is not an element of the crime. Thus, although the lack of damages might prevent the collector from suing in civil court, the fact that the collector suffered no monetary loss from the crime is irrelevant for criminal law purposes. (A) is incorrect because the art restorer obtained title to the $100 rather than mere possession. If a victim consents to someone's taking possession of property, but such consent is induced by a misrepresentation, the consent is not valid. The resulting offense is larceny by trick. False pretenses differs from larceny by trick in what is obtained. If the victim intends to convey only possession of the property, the offense is larceny by trick. However, if the victim intends to convey title, the offense is false pretenses Here, the collector intended to convey title to the $100, acting in reliance on the art restorer's false representation that the painting was an original. Because the art restorer obtained title, the offense of which he can be convicted is false pretenses rather than larceny by trick. (C) is incorrect because embezzlement consists of the fraudulent conversion of property of another by a person in lawful possession of that property. In embezzlement, misappropriation of the property occurs while the defendant has lawful possession of it. Here, the art restorer did not convert the $100 while he was in lawful possession of it; rather, he obtained title to the money by means of a misrepresentation. Because the art restorer did not have lawful possession of the money, he has not committed embezzlement.

Question 9- Homicide/Self-Defense - Intermediate Late one evening, a cook at a diner coming off his shift was grabbed in the parking lot by a large man wearing a ski mask. The man threatened to kill the cook and demanded his wallet. The man then pulled a knife from his pocket and lunged at the cook. The cook, having taken several self-defense courses, was able to fend off the man's attack. After being struck by the cook several times, the man dropped the knife and fell to the ground. The cook, angry at the assault, took the knife and stabbed the man, killing him instantly. Should the cook be convicted of murder? A No, because he was acting in self-defense. B No, but he may be convicted of manslaughter. C Yes, because the killing was committed during the course of a felony. D Yes, because the killing was not committed while acting in self-defense.

(B) The cook may be convicted of manslaughter. At common law, murder was the killing of another human being with malice aforethought. Malice is: (i) the intent to kill; (ii) the intent to inflict great bodily injury; (iii) a reckless indifference to an unjustifiably high risk to human life; or (iv) the intent to commit a felony. A killing committed in self-defense, however, is not murder. A person may use deadly force in self-defense if: (i) he is without fault; (ii) he is confronted with unlawful force; and (iii) he reasonably believes that he is threatened with imminent death or great bodily harm if he does not respond with deadly force. Additionally, a killing that ordinarily would be murder is reduced to voluntary manslaughter when the killing is committed under the stress of adequate provocation (e.g., being subjected to a serious battery, being confronted with deadly force, or discovering one's spouse in bed with another person). In the instant case, although the cook would have been justified in using deadly force to repel the initial attack, the facts indicate that the cook had successfully repelled the attack, and it appears that the cook no longer was facing a threat of imminent death. Thus, the right to use deadly force had passed. This makes (A) an incorrect answer choice. As a result, at first glance, it would appear that the cook committed murder, in that stabbing another in the heart with a knife indicates, at a minimum, that the defendant had the intent to inflict great bodily harm. However, it appears that the cook was still acting under adequate provocation, as he had just been subjected to a serious battery and had been confronted with deadly force. Thus, the killing that ordinarily would be murder is reduced to voluntary manslaughter, making (B) the correct answer choice. (C) is incorrect. While a killing committed during the course of an independent felony is felony murder, the rule does not apply to the homicide felony itself, such as manslaughter. Because the cook was committing the felony by killing the man, he cannot be guilty of murder based on a felony murder theory. (D) is an incorrect answer because it fails to take into account the reduction to voluntary manslaughter, as explained above

Question 6- Intro A state adopted legislation making it a crime to be the biological parent of more than tm children. The stated purpose of the statute is to preserve the state's natural resources and improve the quality of life for the state's resident. A married couple has just had their third child. They have been arrested and convicted under the statute. Which of the following is the strongest argument for voiding the convictions of the couple? A The statute is an invalid exercise of the state's police power because there is no rational basis for concluding that the challenged statute would further the government's stated interests. B The statute places an unconstitutional burden on the fundamental privacy interests of married persons. C The statute places too much discretion in state officials to determine who will be permitted to bear children. D The statute denies married persons equal protection of law.

(B) The couple's strongest argument is that the law burdens their fundamental right of privacy. The right of privacy is not specifically mentioned in the Constitution, but the Supreme Court has recognized the right on a number of occasions. The right is strong when it involves acts of procreation by married couples. When the government restricts a fundamental right such as the right here, the government must show a compelling interest in the regulation and that the regulation is necessary to achieve that purpose. This is a difficult standard to meet, and thus the statute will probably be struck down. Here, the stated purposes for the law (improving the quality of life and preserving natural resources) may be found by the Court to be compelling, but the limitation on the number of children that couples may have certainly is not necessary for these purposes. Thus, this is the couple's best argument. (A) is not a good argument for two reasons: (i) because a fundamental right is involved, the rational basis standard (suggested by this choice) is improper, as strict scrutiny is the appropriate standard and (ii) the statute here in fact is rationally related to its stated purpose because this standard is a very easy one to meet. (C) is not as good an argument as (B) because the facts do not indicate that there is any discretion in state officials under the statute. The statute is violated when anyone is the biological parent of more than two children. (D) is incorrect because the Equal Protection Clause merely prohibits treating classes of persons differently without adequate justification and here, the statute applies equally to all persons; it does not classify.

Question 4 - Attempted Murder - Intro A coal company developed a new extraction process that enabled it to produce coal at marketable cost from 'worn out" mines that it owned, and began operating the mines using the new process. As a cost-cutting measure that increased profits, the corporate officers knowingly permitted the new process to be utilized in such a way that federally and state mandated methane gas detection measures were not complied with, and a consequent methane explosion severely injured several miners. In addition to being prosecuted for felony violations of the methane detection statutes, the corporation and its officers were prosecuted for attempted murder. What should be the outcome of this prosecution? A Not guilty, because the corporate officers had not proceeded sufficiently beyond the planning stage for their actions to constitute an attempt. B Not guilty, because the corporate officers did not possess the requisite intent to constitute the crime of attempted murder. C Guilty, because the corporate officers' actions created a situation so dangerous to human life or safety that their mental state would be considered an abandoned and malignant heart. D Guilty, because the violation of the federal and state methane detection statutes is a felony.

(B) The corporate officers cannot be guilty of attempted murder because criminal attempt requires a specific intent to commit a particular completed crime. A criminal attempt is an act that, although done with the intention of committing a crime, falls short of completing the crime. A defendant must have the intent to perform an act and obtain a result that, if achieved, would constitute a crime Hence, attempt is a specific intent crime. Guilt of attempted murder requires that a defendant have the specific intent to commit the crime of murder. In contrast to the malice aforethought required for murder, which is satisfied not only by intent to kill but also by awareness of an unjustifiably high risk to human life, intent to inflict great bodily injury, or intent to commit a felony, attempted murder is satisfied only by intent to kill. Here, the corporate officers, in permitting the mining to proceed without compliance with methane detection measures, subjected the miners to an unjustifiably high risk to their lives. This "abandoned and malignant heart" demonstrated by the officers would suffice to convict the officers of murder if the miners had been killed. However, there were no deaths; the crime charged is attempted murder. Consequently, there must be a showing that the officers specifically intended to cause the deaths of the miners. Because the facts do not indicate such intent, the officers are not guilty of attempted murder. (C) is incorrect because, as explained above, the mental state of an abandoned and malignant heart will not satisfy the intent required for attempted murder. (D) is incorrect because, even though a felony was committed, the intent to commit a felony (although sufficient to constitute malice aforethought for a murder charge) will not satisfy the intent to kill that is necessary for a conviction of attempted murder. (A) is incorrect because the officers' actions had gone well beyond the planning stage; allowing the mining to proceed without compliance with methane detection measures satisfies the overt act requirement for attempt. Had the other requirements for attempt been satisfied, their actions would be sufficient to support a conviction.

Question 25 - Relevant Circumstantial evidence - Intermediate A defendant is on trial for the murder of the victim, who was found beaten to death in his home. Evidence already presented has shown that the victim was killed when no one was at home except for the victim and his dog. The prosecution wishes to call a neighbor to the stand who is prepared to testify that she went to the victim's home the day after his murder and that when the defendant came by, the dog ran to a corner, where he cringed and whimpered. The neighbor is also prepared to testify that the dog is normally a very friendly dog, usually greeting visitors to the house, including the defendant, by approaching them with his tail wagging. The defense objects to the neighbor's proposed testimony. How should the court rule on the neighbor's testimony regarding the dog's behavior? A Admissible, because the dog could be brought into court for a demonstration of his reaction to the defendant. B Admissible, as circumstantial evidence against the defendant. C Inadmissible, because the dog may have been reacting as he did for reasons other than those implied by the neighbor's testimony. D Inadmissible, because even though the testimony has probative value, such value is outweighed by its prejudicial nature.

(B) The court should admit the neighbor's testimony because it is relevant circumstantial evidence. The Federal Rules of Evidence define relevant evidence as evidence having any tendency to prove or disprove a fact that is of consequence to the action. [Fed R Evid 401] Generally, all relevant evidence is admissible unless it is barred by a specific exclusionary rule or by the general balancing test of Rule 403, which permits exclusion of relevant evidence if its probative value is substantially outweighed by the danger of unfair prejudice, confusion of the issues, etc. The neighbor's testimony is relevant because the dog's behavior when the defendant came by tends to prove circumstantially (i.e., indirectly) the prosecution's contention that the defendant beat the victim to death (in the dog's presence). The neighbor is competent to testify as to the dog's behavior toward the defendant both before and after the murder, and no other competency rule warrants excluding the testimony; hence, it should be admitted. (A) is incorrect because the availability of other evidence that might demonstrate the dog's reaction more clearly does not preclude the neighbor's testimony on that issue. As long as she is competent to testify regarding the dog's behavior, the dog's availability is irrelevant. (C) is incorrect because it is up to the trier of fact to evaluate the inference for which the circumstantial evidence is being offered. The defense may attack the neighbor's testimony on cross-examination by suggesting other reasons for the dog's reaction, but it cannot exclude the neighbor's testimony on this basis. (D) is incorrect because the balancing test of Rule 403 provides only that a court may exclude relevant evidence if its probative value is substantially outweighed by the danger of unfair prejudice. While all evidence is prejudicial to the opposing party, "unfair' prejudice refers to suggesting a decision on an emotional or otherwise improper basis. There is nothing in the neighbor's testimony to justify excluding it on unfair prejudice grounds.

Question 8 -Character - Intro During the defendant's trial for embezzlement, the defense calls a witness to testify as to the defendant's reputation for honesty and veracity. The prosecution objects. Should the court allow the testimony? A Yes, because the prosecution put the defendant's character for truthfulness in issue by filing charges against him. B Yes, because it is admissible character evidence. C No, because a party cannot bolster the testimony of his witness until he has been impeached. D No, because the evidence is inadmissible hearsay

(B) The court should allow the testimony. A criminal defendant may introduce character testimony about his good reputation for a pertinent trait to show that he is innocent of the charged crime. Here, the defendant is offering the testimony of a witness as to his reputation for honesty and veracity, which is a trait directly pertinent to his embezzlement charge. Therefore, the testimony should be allowed, and the prosecution will have the opportunity to rebut the testimony on cross-examination. (A) is incorrect because only the defendant can put his character in issue, and the filing of criminal charges does not have the effect of putting the defendant's character in issue. (C) is incorrect because there is no impeachment issue here and the defense is not attempting to bolster the testimony of its witness (which is generally done after the credibility of the witness is attacked on cross-examination), but rather to offer an opinion as to the defendant's character. (D) is incorrect because, to the extent that the witness's testimony is hearsay, it falls within the exception to the hearsay rule for reputation evidence of a person's character.

Question 9- State Power to Tax - Advanced A state with a number of automobile manufacturing facilities within its borders and a high unemployment rate because of declining sales of automobiles, especially ones built domestically, enacted a statute calling for a $l00-per-car tax on all foreign-built automobiles sold within the state. The tax revenues were to be placed into a state fund to be used to retrain the state's unemployed automobile workers. A major automobile importer and dealership owner brings suit in federal district court seeking to halt the enforcement of the statute on constitutional grounds. Should the court find the statute to be constitutional? A Yes, because it is a proper exercise of a state's rights under the Import-Export Clause. B Yes, if consented to by Congress. C No, because the statute violates the Privileges and Immunities Clause of Article IV. D No, unless the state can show that the statute is necessary to promote a compelling state interest.

(B) The court should find the statute constitutional as long as Congress has consented to the tax. Article l, Section 10, Clause 2 provides: "No state shall, without the Consent of the Congress, lay any Imposts or Duties on Imports or Exports, except what may be absolutely necessary for executing its inspection Laws." Hence, the Import-Export Clause prohibits the states from imposing any tax on imported goods as such or on commercial activity connected with imported goods as such (i.e., taxes discriminating against imports), except with congressional consent. Thus, <(B) is correct and (A) is incorrect. (C) is wrong because the Privileges and Immunities Clause does not apply to corporations or aliens. (D) is wrong because without congressional consent, the statute is unconstitutional despite the compelling nature of the state's interest. Constitutional Law Quiz Three 19 Supplemental Practice Questions with Answer Explanations

Question 3 - Specific Acts of misconduct-Intro During the course of his trial for assault, the defendant placed his neighbor on the stand. The neighbor testified that the defendant had a good reputation in their community for being a peaceable man. On cross-examination, the prosecutor asked the neighbor if she filed a false income tax return last year. The neighbor has in fact been convicted of filing a false tax return; however, the defense immediately objected. Should the court permit the question? A Yes, because the neighbor has been convicted of filing a false tax return. B Yes, because the question is relevant to the truthfulness and credibility of the witness. C No, because specific instances of conduct are inadmissible. D No, because the question does not go to a relevant character trait.

(B) The court should require the neighbor to answer the question because relevant to her truthfulness and credibility as a witness under the Federal Rules, a witness may be impeached on cross-examination with her prior specific acts of misconduct that are probative of truthfulness. Specific "bad acts" that show the witness unworthy of belief (i.e., acts of deceit or lying) are probative of truthfulness. Filing a false income tax return reflects on the neighbor's veracity and, thus, her credibility. Therefore, the neighbor should be required to respond. (A) is incorrect because inquiry into bad acts to impeach a witness's credibility is permitted even if the witness was never convicted of a crime relating to the act. Even though the neighbor could have been impeached by evidence of her conviction for the crime, here she is just being asked about her conduct rather than the criminal conviction. (C) is incorrect because Federal Rule 608 permits inquiry about specific acts of misconduct, within the discretion of the court, if they are probative of truthfulness. An important limitation is that extrinsic evidence is not admissible to prove such acts; here, however, the impeachment was properly limited to inquiry on cross-examination. (D) incorrect because the question relates to truthfulness, and a witness's credibility is always relevant.

Question 3 -Usury rate- Intro To rein in states passing restrictive usury laws, Congress enacted legislation establishing a uniform usury rate. The legislation, in essence, establishes that the usury rate will be determined by the Federal Reserve Board on the date of the loan. The law is intended to be of great assistance to the private home market as well as the commercial market. One state has a usury law that permits the charging of interest in excess of that which Congress has provided. Mortgage lenders in that state brought an action in federal court seeking to void the federal law with respect to loans in the state. The lenders presented evidence that loans made in the state during the last two years for the purpose of housing all were at rates exceeding those permissible under the provision of the federal law. The lenders allege that enforcing the federal law would bring the housing industry to a stop. The federal judge should: A Rule that he has no jurisdiction to hear the case. B Rule that the state law is invalid as a result of the Supremacy Clause. C Rule that granting temporary relief by enjoining the federal act pending a decision on the merits would be proper. D Impanel a three-judge court because of the seriousness of the issue.

(B) The court should rule that the Act of Congress is valid under Congress's commerce powers, and that, under the Supremacy Clause of the Constitution, the state law is invalid. While states and the federal government may pass legislation on the same subject matter, the Supremacy Clause provides that federal law is supreme, and any conflicting state law is rendered void. It is possible that the court would enjoin the application of the federal law pending a trial on its merits, but highly unlikely. Thus, (C) is wrong (A) is wrong; the court has jurisdiction because a federal law is involved. (D) is wrong; no such procedure would apply here.

Question 15 - Doctor/Patient Privilege A defendant is charged with federal mail fraud based upon misrepresentations in his application for medical insurance coverage and for claims on policies that were subsequently issued. The application and claim forms containing the misrepresentations were mailed to the insurance company through the United States Postal Service. Specifically, the defendant failed to state his previous illnesses on both the application and claim form. The defendant denies the charges of having misrepresented his medical history. The prosecution wishes to offer as proof of his fraud the testimony of the defendant's physician, who will testify as to the defendant's history of previous illnesses. The defendant's attorney objects to the physician's testimony. The jurisdiction's physician-patient privilege applies to both civil and criminal proceedings. How should the court rule on the admissibility of the physician's testimony? A Admissible, because the physician is qualified to testify as an expert witness. B Admissible, because the physician-patient privilege does not apply. C Inadmissible, because it violates the physician-patient privilege. D Inadmissible, because the defendant's medical records are the best evidence.

(B) The court should rule that the physician's testimony is admissible because the physician-patient privilege does not apply. Generally, the physician-patient privilege provides that a physician cannot be compelled to disclose information obtained from a patient while treating the patient in a professional capacity if that information is related to the treatment. The privilege does not, however, apply in federal cases where state law does not supply the rule of privilege (i.e., federal question cases). Here, the defendant is before the federal court on charges arising under a federal statute. Thus, the physician-patient privilege does not apply and the physician's testimony is admissible. (A) is incorrect because it implies that the testimony of an expert witness supersedes the physician-patient privilege. The privilege applies regardless of whether the physician is testifying as an expert witness. (C) is incorrect because, although the jurisdiction applies the physician-patient privilege to criminal cases, state privilege law does not apply in this case as explained above. (D) is incorrect because the best evidence rule does not apply in this situation. The best evidence rule applies when a party is seeking to prove the terms of a writing. In this case, the prosecution not seeking to prove the terms of any writing (e.g., the medical records), but is rather offering the oral testimony of the physician to establish that the defendant did in fact have a medical history that he fraudulently concealed in his application and claim form.

Question 18 - Criminal Battery- Intermediate The defendant was walking down a street when he realized that a long-lost friend was walking in the opposite direction. The defendant stalled waving his arms in a desperate attempt to get his friend's attention. The defendant did not notice an old woman, who was walking past him, and negligently struck her with his elbow with sufficient force to cause her to stumble to the pavement. The fall did not, however, cause her any major injury. If the defendant is prosecuted for criminal battery, what is the most likely verdict? A Not guilty, because his act did not cause serious bodily injury. B Not guilty, because he did not have the mental state required for criminal battery. C Guilty, because he failed to exercise due care in flailing his arms about near a public sidewalk. D Guilty, because he caused an offensive touching.

(B) The defendant will probably be found not guilty. Battery is an unlawful application of force to the person of another resulting in either bodily injury or an offensive touching. The mental state necessary for criminal battery is general intent, which is satisfied by intentional, reckless, or criminally negligent conduct. Clearly, the defendant did not intend to have bodily contact with the old woman, and in light of the circumstances it will be unlikely that his conduct will be considered such an extreme deviation from ordinary behavior as to establish criminal negligence. (A) is wrong; a serious injury is not required for battery. (C) is wrong because criminal negligence requires a greater deviation from ordinary behavior than simply a "failure to exercise due care," which is the tort standard. (D) is not as accurate as B. It does not address the critical issue in the case—the mental state of the defendant.

Question 32- Impeachment- Intermediate A defendant is on trial for manslaughter after he hit a victim in a bar, causing the victim to fall and hit his head on the marble bar top. The defendant claims that he hit the victim in self-defense after the victim lunged at him with a knife. During the prosecution's case, a witness testifies that she heard the victim's friend shout at the defendant, "You just killed a helpless man!" A defense witness is called to testify that he was there and does not remember hearing the victim's friend say anything. Should the defense witness's testimony be admitted? A No, it is irrelevant to any issue in the case. B Yes, it is proper impeachment of the prosecution's witness. C No, it is improper impeachment of the prosecution's witness because it relates to a collateral matter. D No, it is improper impeachment because it does not positively controvert the prosecution witness's testimony, as the defense witness merely says he does not remember.

(B) The defense witness's testimony should be admitted as proper impeachment of the prosecution's witness. Impeachment is the casting of an adverse reflection on the veracity of a witness. A witness may be impeached by either cross-examination or extrinsic evidence, such as by putting other witnesses on the stand who contradict the witness's testimony. Here, the defense is using the testimony of its witness to impeach the prosecution witness's testimony as to what the victim's friend said. This is proper. (A) is wrong because a witness's credibility is always relevant. Furthermore, the defense witness's testimony relates to a crucial issue in the case; i.e., whether the victim was armed with a knife or was "helpless." Thus, the testimony is relevant. (C) is wrong because this is not a collateral matter. Impeachment on a collateral matter is prohibited, but a collateral matter is one that arises when a witness makes a statement not directly relevant to the issues in the case. The prosecution witness's statement about the victim being "helpless" is directly relevant to the issue of the defendant's self-defense claim and, thus, is a proper subject of impeachment. (D) is wrong because impeachment evidence need not positively controvert the prior testimony; it need only tend to discredit the credibility of the prior witness.

Question 13- Specific acts of conduct - Intro A plaintiff sued his neighbor over a 10-foot-high stockade fence that the neighbor was building adjacent to the plaintiff's backyard. The local zoning ordinance permitted a fence of this height unless it was a "spite fence," defined as a fence erected solely for the purpose of interfering with neighboring landowners' use and enjoyment of their property. The plaintiff alleged that the neighbor was building the fence to block sunlight to the garden that the plaintiff had planted. The neighbor denied that she was building the fence for that purpose. The plaintiff wishes to introduce evidence that the neighbor had sprayed herbicide towards the garden previously. Should the judge permit the plaintiff's testimony? A Yes, because the neighbors character is at issue in the case. B Yes, because it pertains to the neighbor's motivation in building the fence. C No, because the plaintiff's testimony is evidence of specific conduct, which is not admissible in this case because the neighbor's character is not in issue. D No, because character evidence generally is not admissible in a civil case.

(B) The judge should permit the plaintiffs testimony because evidence of specific acts of misconduct is admissible to show motive under Federal Rule 404(b), evidence of other acts may be admissible in a criminal or civil case if they are relevant to some issue other than character, such as motive. Here, whether the neighbor was motivated by an improper purpose in building the fence is the key issue in the lawsuit by the plaintiff. The neighbor's prior misconduct in spraying herbicide toward the plaintiff's garden is circumstantial evidence that her hostility toward the garden motivated her to build the fence. (A) is wrong because even though the neighbor's motivation and intent are at issue in the case, her character is not. In the absence of character being directly in issue in the case, evidence of character to prove the conduct of a person in the litigated event is not admissible. (C) and (D) are wrong even though they correctly state general rules: evidence of specific acts of misconduct is generally inadmissible, and character evidence is generally inadmissible in a civil case. However, when the specific acts are being offered for a purpose other than to show bad character or conduct in conformity to character, they are admissible in both criminal and civil cases.

Question 22 - Offers to Compromise - Intermediate The plaintiff is suing the defendant for personal injuries suffered when she was struck by the defendant's car. The plaintiff testified regarding the extent of her injuries. The defendant seeks to testify that when she visited the plaintiff in the hospital, the plaintiff said that if the defendant paid all of her medical bills and gave her $25,000, she would not institute legal proceedings. The plaintiff's attorney objects to the offered testimony. How should the judge rule on the admissibility of the testimony? A Inadmissible, because it is hearsay. B. Inadmissible, because it was an offer of compromise. C Admissible, because it is a statement against interest and, thus, an exception to the hearsay rule. D Admissible, because it is relevant to show that the plaintiff tried to "blackmail" the defendant, and does not really have a claim.

(B) The judge should rule the testimony inadmissible. Evidence of compromises or offers to compromise is inadmissible to prove or disprove the validity or amount of a disputed claim. (A) is incorrect; the evidence is not hearsay because it is a statement made by and offered against a party. (C) is incorrect because the statement against interest exception requires that the witness be unavailable. Here, the plaintiff is available to testify. <(D) may be relevant, but the facts show that it was merely an offer to compromise a disputed claim, and therefore public policy encouraging settlements precludes the introduction of such evidence. Thus, (B) is correct.

Question 13- Due Process/Business Owner License - Intermediate A state statute provided for the suspension or revocation of a retail business license where there have been "repeated violations" of specified regulatory statutes by any one individual or business. A store owner had already been fined three times under the statute. Several days later, the store owner received a notice from the regulatory agency that was created by the statute stating that his business license will be suspended for 21 days for violation of the statute. If the store owner files suit to set aside the suspension, will the owner most likely prevail? A Yes, because the statute fails to set forth the exact number of violations that a business would have to commit before its business license is revoked. B Yes, because the store owner has not been given a hearing before the agency. C No, because the agency could by its rules construe the term "repeated violations" in such a manner as to comport with due process requirements. D No, because the agency is merely a regulatory agency, and therefore the store owner has no right to a hearing.

(B) The most obvious problem in this situation is that the store owner has not been afforded the right to a hearing either before or after his license was suspended. A business license is a valid property right, and procedural due process under the Fourteenth Amendment requires notice and an opportunity to be heard before the government may deprive a person of property. Thus, the store owner should be granted a hearing before his license can be suspended. Thus, (C) is wrong, because regardless of how the agency construes violations of the statute, revoking a business license still involves interference with a property right, and a hearing is thus required before this can be done. The agency obviously had enforcement powers; therefore, (D) is wrong. (A) is an arguable complaint by the store owner, but, because there appears to be a regulatory agency that can make rules regarding the enforcement of the statute, the agency can construe "repeated violations" so as to comport with due process requirements.

Question 9 -Character Reputation-Intro At the defendant's trial for arson, a witness for the defense is asked whether the defendant has a reputation for honesty and veracity. The prosecution objects. How should the court rule on the objection? A Sustained, because the evidence is hearsay. B Sustained, because the evidence is irrelevant. C Overruled, because the prosecution put the defendant's character at issue by filing charges against him. D Overruled, because the reputation evidence is admissible to establish a character trait.

(B) The objection should be sustained because the testimony of the witness irrelevant. Generally, a defendant may introduce evidence of his good character to show his innocence of the alleged crime. However, the evidence must be relevant; i.e., it must have some tendency to make the existence of any fact that is of consequence to the determination of an action more probable or less probable than it would be without the evidence. Here, the defendant is charged with arson and there is nothing to indicate that the defendant's honesty is at issue. Therefore, the witness's testimony is inadmissible because it is irrelevant. (A) is incorrect. Hearsay is an out-of-court statement offered to prove the truth of the matter asserted. However, reputation of a person's character in the community is an exception to the hearsay rule. Therefore, the witness's statements regarding the defendant's reputation fall under that exception to the hearsay rule. (C) is incorrect because only the defendant can put his character at issue; the prosecution cannot initiate evidence of the defendant's character Furthermore, filing criminal charges against a person does not put his character at issue. (D) is incorrect because reputation evidence is admissible to establish a character trait only when the trait sought to be established is pertinent to the crime charged. Here, the defendant's reputation for honesty and veracity is not pertinent to the crime of arson.

Question 2 - Photo demonstrative purposes - Intro In a trial for bank robbery, a teller has identified the defendant as the robber Defense counsel offers into evidence a still frame from a video taken by the bank security camera the day after the robbery to show that a column obstructed that teller's view of the defendant. Is such evidence admissible? A Yes, upon testimony by the camera operator that the still frame was developed from film that was taken from that camera the day after the robbery. B Yes, upon testimony by a bank employee that the photo accurately portrays the scene of the crime. C No, not admissible into evidence but usable by a witness for explanatory purposes. D No, if a still frame can be obtained from a video taken at the time of the robbery.

(B) The photo should be admitted into evidence upon testimony that it is an accurate representation of the location depicted. To be admissible, real or demonstrative evidence must not only be relevant but must also be authenticated, i.e., identified as being what the proponent claims it to be. For a photograph that is used as demonstrative evidence, authentication is by testimony that the photo is a faithful reproduction of the object or scene depicted. Here, testimony by a bank employee that the still frame from the video accurately portrays the setting where the robbery took place is sufficient for admissibility. (A) is incorrect because the frame from the video is not being offered as original evidence that played an actual role in the robbery itself, such as a gun used by the robber, which would require the "chain of custody" type of authentication in (A). Here, the still frame is only being used for demonstrative purposes; hence, authentication focuses on whether it is an accurate representation rather than how it was handled. (C) is incorrect. Charts and diagrams that are used solely to help explain a witness's testimony may be permitted at trial but not admitted into evidence where they are not offered as representations of a real object or scene but only as aids to testimony. Here, however, the photo is being offered as a faithful representation of the scene of the crime and should therefore be admissible into evidence. (D) is incorrect because it is a misapplication of the best evidence rule. The best evidence or original document rule, which is made applicable to photographs by the Federal Rules, generally requires that in proving the terms of a writing the original writing must be produced where the terms are material. The terms are material and the rule applies only when (i) the document is a legally operative or dispositive instrument, or (ii) the witness's knowledge results from having seen the fact in the document. Neither situation arises in this case. The location of the columns in the bank and the circumstances of the robbery are facts that exist independently of the document (the videotape on the day of the robbery), and thus may be proved by other evidence

Question 34- Impeachment- Intermediate A defendant is charged with theft of a car. On cross-examination of the defendant, the prosecution asked her whether she had been convicted of fraudulent business practices six months earlier. Is this question proper? A Yes, to show that the defendant has a bad character. B Yes, to show that the defendant is inclined to lie. C No, because the probative value of the evidence is outweighed by the danger of unfair prejudice. D No, because the conviction has insufficient similarity to the crime charged.

(B) The question is proper to show that the defendant is inclined to lie. A witness may be impeached with evidence of a prior conviction for (i) any felony or (ii) any crime involving dishonesty or false statement. Because the question involves a crime of dishonesty, and the conviction is less than 10 years old, the question is proper on cross-examination because it goes to the defendant's credibility. (A) is wrong because evidence is not admissible to show general bad character or propensity to commit crimes. (C) is wrong because this crime has plenty of probative value on the issue of credibility and because most courts hold that impeachment with a crime requiring proof or admission of an act of dishonesty or false statement cannot be excluded as too prejudicial. (D) is wrong because similarity to the crime charged is not required by the Federal Rules of Evidence.

Question 8- 10th Amendment - Intermediate A state passed a law providing that legislative staff members in the state legislature were to receive an hourly wage that was less than the federal minimum wage. A legislative staff member filed suit in federal court challenging this state law. Which of the following is the strongest argument in support of the staff member's constitutional challenge? A Congress may extend federal minimum wage requirements to legislative staff members by virtue of the Necessary and Proper Clause. B The minimum wage law applies to private sector employees as well as state employees. C Congress may extend federal minimum wage requirements to state legislative staff members as a means of guaranteeing a republican form of government. D Congress has plenary power to extend federal minimum wage requirements under the General Welfare Clause.

(B) The strongest argument is that the minimum wage law applies to both private sector and state employees. Generally, the Tenth Amendment provides that powers not delegated to the federal government are reserved to the states. Under the Tenth Amendment, the court will usually uphold federal legislation that applies both to the private sector and to state governments, but will generally not uphold attempts by Congress to regulate the states as states. Here, the federal minimum wage law applies to all private and government employers. Thus, it will be upheld. (C) is therefore incorrect because extending minimum wage laws to legislative staffers does not interfere with the Constitutional guarantee of a republican form of government. (A) is incorrect because the Necessary and Proper Clause alone does not furnish authorization for federal laws. Congress must be acting in the exercise of one of its express powers, and (A) only deals with the Necessary and Proper Clause. (D) is incorrect because the General Welfare Clause is a congressional taxing and spending authorization and is not related to minimum wage requirements.

Question 12 - Congressional Tax & Spending Power -Intro Congress passed a law imposing a 50% excise tax on each pack of cigarettes manufactured for sale in the United States. An amendment was successfully added to the original bill requiring that all proceeds from the tax be used for antismoking educational programs in the audio, video, and print media and elsewhere. The amendment also provided for the establishment of federal stop-smoking clinics funded through the excise tax. The various tobacco companies were required to pay the tax directly to the federal government. A tobacco company filed suit in the appropriate federal court contending that the tax should be struck down as unconstitutional. Is the court likely to find the tax constitutional? A Yes, because the tax is severable from its purpose. B Yes, because the tax is a proper exercise of Congress's taxing and spending powers. C No, because it does not provide equal time for the tobacco companies to present their side of the smoking controversy. D No, because it abridges the First Amendment rights of tobacco manufacturers by forcing them to pay for messages with which they may not agree.

(B) The tax is constitutional because it represents a proper exercise of the power of Congress to tax and spend for the general welfare. Pursuant to the Constitution, Congress may tax and spend to provide for the general welfare. A congressional tax measure will be upheld if it bears some reasonable relationship to revenue production or if Congress has the power to regulate the taxed activity. Congress may spend for any public purpose, not merely the accomplishment of other enumerated powers. The tax at issue here bears a reasonable relationship to revenue production. Also, the purchase and sale of cigarettes in the United States is subject to congressional regulation, as an activity having a substantial economic effect on interstate commerce. Thus, the tax itself is valid. The amendment to the original tax bill is also valid, as a reflection of a congressional determination to use the proceeds of the tax for the promotion and implementation of an antismoking program, presumably in furtherance of public health. This is a public purpose for which Congress can spend pursuant to the General Welfare Clause. (A) is incorrect because it implies that the purpose of the tax is not constitutional. The purpose of the tax is to raise revenue to fund a federal antismoking program. As explained above, this is a proper exercise of the congressional taxing and spending power. Therefore, the constitutionality of the tax does not hinge on the severability of its purpose. (C) is incorrect because there no "fairness doctrine" under the Constitution; i.e., when Congress determines a course of action pursuant to its power to tax and spend for the general welfare, it need not provide equal time for opponents of the action to express their views. (D) is incorrect because the tax in no way abridges the First Amendment rights of the manufacturers. While the freedom of speech is very broad and includes the freedom to not speak and to refrain from endorsing views with which one does not agree, it does not invalidate the tax here because the tax does not force the tobacco companies to endorse the stop-smoking clinics or the government's antismoking stance. Neither does the tax forbid or control the tobacco manufacturers from endorsing a pro-smoking message. The manufacturers' First Amendment rights are simply not burdened here.

Question 12 -Character - Intro A defendant is on trial for aggravated assault. The defendant claims that he was merely a bystander and that another person was the actual aggressor. The defendant calls a witness to testify that the defendant has a reputation for being a "good and loving person who would never hurt anyone." Should the trial judge rule that the testimony is admissible? A Yes, because it is not hearsay. B Yes, because this is a criminal matter, and it tends to show the defendants relevant good character. C No, because the prosecution has not introduced evidence concerning the defendant's reputation. D No, because character evidence is not permissible to show that an accused has acted in conformity with the character

(B) The testimony is admissible because it tends to show the defendant's good character for a pertinent trait. In a criminal matter, the accused is permitted, at any time during the trial, to introduce evidence of his good character for the trait involved in the case to support the probability that he did not commit the crime alleged. Here, the crime in question involves violence. Therefore, evidence of the defendant's good character for nonviolence may be admitted. Thus, (C) and (D) are wrong as misstatements of the law. (A) is wrong because just the fact that evidence is not hearsay is no reason to admit it. Choice (B) is a better option because it addresses the rules for admissibility of character evidence, which is the issue raised by the Question.

Question 1 - Video Evidence - Intro The police set up an undercover "sting" operation in which they posed as fences of stolen property and bought and sold such property to anyone who came into their warehouse. A defendant is being prosecuted for receiving stolen property in connection with his arrest by the undercover police officers, and the prosecution attempts to introduce a video recording showing the defendant offering to sell a television set to one of the undercover police officers. Which of these is the most likely reason that this evidence will be held to be inadmissible? A It is hearsay not within any exception. B It violates the defendants privilege against self- incrimination. C A proper foundation was not established for its introduction into evidence. D Character for criminality may not be proven by specific instances of misconduct.

(C) A proper foundation was not established for the video's introduction into evidence. Recorded video evidence treated the same as photographic and recorded audio evidence. If it is properly authenticated, it is admissible, but it may not be properly admitted without the proper foundation first being established. (A) is wrong because the recording contains the defendant's own statements, which are now being offered against him. Thus, the defendant's statements would constitute admissions by a party-opponent, which are not considered hearsay under the Federal Rules. (B) is wrong because the privilege against self-incrimination applies only to testimonial evidence. (D) is incorrect. While character generally may not be proven by specific instances of conduct, the recording is not being offered to show the defendant's general character for criminality.

Question 12- Double Jeopardy - Intro A husband and wife were charged with stealing credit cards and charging expensive items on the misappropriated cards. An attorney was appointed by the court to represent the couple jointly. At the preliminary hearing, the judge found that the attorney would have no conflict representing both defendants in the joint trial. Halfway through the trial, however, a conflict arose between the defenses of the husband and wife. At the wife's request, the attorney moved that another attorney be appointed to represent the wife and that a mistrial be declared. The trial judge moved favorably on the attorney's motion. Another attorney was appointed to represent the wife, and as soon as the wife's trial began, her attorney moved to dismiss the case on the ground that jeopardy had attached during the wife's first trial and that she was being retried in violation of the United States Constitution. Should the judge grant the wife's attorney's motion? A Yes, because jeopardy attached when the jury began to hear evidence in the first trial. B Yes, because the judge incorrectly ruled that there would be no conflict of interest from the joint representation. C No, because the wife requested the mistrial. D No, because it is premature to move for a dismissal based on double jeopardy until the defendant is convicted.

(C) Although jeopardy attached in the wife's first trial, her retrial is not barred because she initiated the grant of the mistrial in her first trial. As a general rule, the right to be free of double jeopardy for the same offense bars a retrial for the same offense once jeopardy has attached in the first trial. However, one of the exceptions permitting retrial even if jeopardy has attached is when a mistrial is granted in the first trial at the request of the defendant on any ground not constituting an acquittal on the merits. Here, the wife requested the mistrial because a conflict arose between the defenses of her and her co-defendant in the joint trial, and the judge granted the mistrial solely to allow the wife to obtain another attorney. Thus, no acquittal on the merits occurred and the double jeopardy rule does not apply. (A) is incorrect for several reasons. Merely because jeopardy attaches does not mean that the double jeopardy rule will apply; retrial will be permitted under certain exceptions, one of which is applicable here. Furthermore, (A) is not a correct statement of law. Jeopardy attaches in a jury trial when the jury is impaneled and sworn in, even if it has not yet heard any evidence. (B) is incorrect because the judge's finding at the preliminary hearing stage appears to be an honest error rather than bad faith conduct. In the absence of bad faith conduct by the judge or prosecutor designed to force the defendant to seek a mistrial, the defendant's securing of a mistrial does not preclude a retrial. (D) is incorrect because the right to be free of double jeopardy creates a bar as soon as the defendant is retried for the same offense, rather than on her conviction.

Question 8- Intro A state statute provides that no alien may own a restaurant within the state and that it is unlawful for anyone to give, sell, or otherwise convey a restaurant to an alien. A citizen of Canada who legally resides in the state has entered into a contract to buy a restaurant located within the state from a restaurant owner. If the buyer and the seller join in a declaratory judgment action to test the state statute in a federal court which of the following is true? A The case may not be heard because the buyer does not have standing. B The burden of proof is on the buyer and seller to show that the statute is rationally related to a legitimate state interest. C The burden of proof is on the state to show that the statute is necessary to achieve a compelling state interest. D The burden of proof is on the state to show that the statute is substantially related to an important state purpose.

(C) Because the state statute is based on alienage, it is subject to strict scrutiny and thus the state has the burden of proof. State laws based on alienage are subject to the strict scrutiny test, except when the law concerns alien participation in the functioning of the state government (and possibly "illegal" alien adults), in which case the rational basis test is applied. Under the strict scrutiny test, the government bears the burden of showing that the law is necessary to a compelling state interest. The state law here does not concern alien participation in the functioning of state government (or illegal aliens), and thus the strict scrutiny test will be used. The state thus has the burden of proof. (A) is wrong because the buyer has standing. A person has standing if he can demonstrate a concrete stake in the outcome of the controversy; i.e., he has been or will be injured by the governmental action and a decision in his favor will remedy the situation. The buyer has standing because he has been injured (the statute directly impairs his right to own a restaurant and thus violates his right to equal protection). A declaratory judgment in his favor will remedy this situation; the buyer will be able to buy a restaurant if the law is declared unconstitutional. Thus, he has standing. (B) is wrong because, as explained above, the burden of proof is on the government and not on the buyer and seller. (B) is also wrong because it implies that the validity of the statute would be determined by the rational basis test. Because the statute here is based on alienage and does not pertain to participation in the functioning of state government, the strict scrutiny standard, rather than the rational basis test, will be applied. (D) wrong because, as discussed, the strict scrutiny standard applies.

Question 24- Judicial Review - Advanced In which of the following cases does Congress have the power to restrict the jurisdiction of the United States Supreme Court? A A case involving an ambassador. B A case involving a dispute between two states. C A case involving maritime jurisdiction. D A case involving a state and the federal government.

(C) Congress has the power to limit the Supreme Court's jurisdiction in maritime cases. The Supreme Court has original jurisdiction in all cases affecting ambassadors, other public ministers and consuls, and those in which a state shall be a party. Congress may neither restrict nor enlarge the Supreme Court's original jurisdiction. Thus, (A), (B), and (D) are incorrect. In all other cases, such as maritime disputes, the Supreme Court has only appellate jurisdiction, which Congress has extensive power to regulate and limit. Hence, (C) correct.

Question 14- Prior Bad Acts - Intro A woman was struck by a brick with her name scrawled on it that was thrown through her bedroom window. The victim believes that her ex-boyfriend, who is a gang member, threw the brick, because she had become active in anti-gang groups, but she did not actually see him throw it If the ex-boyfriend is arrested and put on trial for battery, which of the following items of the victim's proposed testimony is LEAST likely to be admitted? A The victim recently moved to a new apartment and only her ex-boyfriend and a few family members knew its location. B The victim had testified against a member of her ex-boyfriend's gang last month in a drug case. C On another occasion, the victim had seen her ex-boyfriend throw a rock through the window of a rival street gang member. D Immediately after the brick went through her window, the victim heard a voice she recognized as her ex-boyfriend's yell, "If you don't start minding your own business, you'll get a lot worse than this next time!"

(C) Evidence of the defendant's other crimes or misconduct is admissible only if relevant to some issue other than the defendant's character or propensity to commit the crime charged. Such acts would be admissible to show motive, intent, absence of mistake, identity, or a common plan or scheme. Of these, the only one possibly relevant to these facts is identity. Evidence that the accused committed prior criminal acts that are so distinctive as to operate as a "signature" may be introduced to prove that the accused committed the act in question. Merely throwing an object, such as a brick, through a window could not be considered so distinctive as to operate as a signature. Thus, this evidence would not show identity. The only possible reason for offering the evidence is to show the ex- boyfriend's propensity to commit the crime charged, in which case the testimony will be inadmissible. (A) is wrong because it is circumstantial evidence that the ex-boyfriend threw the brick. It is relevant because it tends to make more probable that he threw the brick than it would be without the evidence. (B) is wrong because it is relevant and goes to motive. It too makes it more probable that the ex-boyfriend threw the brick than it would be if the victim had not testified against a member of his gang. (D) is wrong because the victim's identification of the ex-boyfriend's voice places him at the scene and is thus relevant. It is more probable that he threw the brick than it would be in the absence of this testimony. The identification of a voice is properly authenticated by the opinion of a person familiar with the alleged speaker's voice. As his ex-girlfriend, the victim would be sufficiently familiar with the ex-boyfriend's voice to make a proper identification.

Question 19- Charitable Organization Solicitations - Advanced A city passed an ordinance prohibiting door-to-door solicitation of contributions by charitable organizations that did not use at least 75% of their receipts for "charitable purposes." The ordinance further provided that anyone wishing to solicit for purposes of charity must obtain a permit and present satisfactory proof that at least 75% of the proceeds of such solicitation will be used directly for the charitable purposes of the organization. An organization seeking to solicit support within the city applied for and was denied a permit because it did not meet the 75% requirement. As applied to the organization, is the ordinance constitutional? A Yes, because the 75% rule serves a legitimate state interest in preserving the integrity of charities. B Yes, because the right to solicit for a charity is balanced against the interest of the state in preventing fraud and crime. C No, because the ordinance violates the protections afforded by the First Amendment. D No, because the ordinance has as a purpose the protection from undue annoyance and the preservation of residential privacy.

(C) The Court should declare the ordinance unconstitutional, because charitable solicitations for funds in residential areas are within the protection of the First Amendment. In Village of Schaumburg v Citizens for a Better Environment (1980), the Supreme Court held unconstitutional a municipal ordinance that prohibited the door-to-door solicitation of contributions by charitable organizations that did not use at least 75% of their receipts for charitable purposes. After review, the Court stated that the precedent of earlier decisions established clearly that the charitable appeal for funds involves a variety of speech interests that are within the protection of the First Amendment. The Court concluded that the ordinance unduly intruded on the rights to free speech because the justifications for the restriction were not sufficiently compelling. (A) is wrong because the state's "legitimate interest" is not enough to justify violation of First Amendment rights. Similarly, (B) and (D) are wrong because the First Amendment rights outweigh the state's purposes and there are less intrusive ways of accomplishing the state's goals.

Question 17 - Privilege Against Self-Incrimination- Intermediate A plaintiff brought a civil action against a defendant for embezzlement of funds missing from a trust account, for which the defendant also is being investigated by the district attorney. At trial, the plaintiff calls the defendant as an adverse witness and asks him one question, "Is it not true that you embezzled funds from the trust?" The defendant refuses to answer, claiming a privilege against self-incrimination. How should the trial court rule on the defendant's claim of privilege? A Order him to disclose to the court in camera what happened to the missing funds so that the court can determine whether he reasonably fears prosecution for a crime. B Order him to answer because the privilege against self-incrimination does not apply in civil proceedings. C Sustain his claim of privilege, because no witness can be compelled to answer questions that may tend to incriminate. D Sustain his claim of privilege because he is a likely subject of criminal prosecution.

(C) The Fifth Amendment of the United States Constitution provides that a witness cannot be compelled to testify against himself, Pursuant to this privilege, a witness may refuse to answer any question if its answer might tend to incriminate him. Testimony is incriminating if it ties a witness to the commission of a crime or would furnish a lead to evidence tying the witness to a crime. The privilege against compelled self-incrimination can be claimed at any proceeding, whether civil or criminal, at which the witness's appearance and testimony are compelled. Here, the defendant's answer to a question as to whether he embezzled funds from the trust might tend to incriminate him by tying him to the commission of a crime (e.g., embezzlement). Thus, the defendant is privileged to refuse to answer the question posed by the plaintiff. (B) is incorrect because, as noted above, a witness may claim the privilege against self- incrimination in a civil proceeding. If the testimony might expose the witness to criminal liability, then the privilege applies even in a civil proceeding. (A) is incorrect because, where testimony might be incriminating, the witness may refuse to answer a question, regardless of whether the witness has a reasonable fear of prosecution. There is no basis for the judge predicating her ruling on a determination that the defendant has a reasonable fear of prosecution. Also, the privilege against self-incrimination allows the defendant to refuse to answer any questions at all relating to whether he embezzled trust funds. As a result, he cannot be compelled to disclose such matters, even in an in camera proceeding. Similarly, (D) is incorrect because the applicability of the privilege against self-incrimination does not depend on the likelihood of actual criminal prosecution of the witness. Testimony that might tend to incriminate the witness triggers the privilege, without regard for whether the witness is a likely target of criminal prosecution

Question 7 - taxpayer standing- Intro Congress adopted a law granting a state university $2.5 million to study the progenies of snowflakes. An outraged taxpayer filed suit to challenge the appropriation. Which of the following is the best argument against this challenge? A The legislation is a valid exercise of the federal taxing and spending powers. B The federal court lacks jurisdiction to enjoin such expenditures voted by Congress, under the doctrine of separation of powers. C The taxpayer lacks standing to challenge the expenditure. D There is no diversity in this suit because the spending appropriation being challenged is federal, and the plaintiff is a citizen of the United States

(C) The best argument is that the taxpayer lacks standing. Generally, taxpayers lack standing to challenge federal appropriations. There is an exception for expenditures violating the Establishment Clause, but that is not the case here. (A) also is a good answer, but it is not as good as (C). (C) stops the case at its inception. While (A) correctly states that the spending measure here is a valid exercise of the federal taxing and spending power, such an argument would be made later in the case. (B) is wrong because the doctrine of separation of powers does not prevent the courts from enjoining unconstitutional activities by Congress. (D) is wrong because the spending appropriation represents a federal question, so diversity not required.

Question 13- Religion, Free Exercise and Children - Intermediate A state statute makes it unlawful to willfully cause or permit the life or health of a child under the age of 18 to be endangered. The statute also makes it unlawful to willfully cause or permit a child to be placed in circumstances that endanger the child's life or health. A local prosecutor brought charges against parents who brought their child to their church's religious service in which venomous snakes were passed around. The parents were convicted and have brought an appeal on constitutional grounds. Which of the following best describes the likely result of the parents' appeal? A Their convictions will be upheld because the parents lack standing to challenge the statute on "free exercise of religion" grounds. B Their convictions will be upheld because the state's interest in regulating activities involving children necessarily outweighs any rights of members of a church under the Free Exercise Clause of the First Amendment. C Their convictions will be reversed if it can be shown that the statute is being applied only to interfere with religion. D Their convictions will be reversed because the freedom to engage in conduct connected with one's religion is absolutely protected under the First and Fourteenth Amendments.

(C) The convictions will be reversed if it can be shown that the statute is being applied only to interfere with religion. The Free Exercise Clause prohibits government from punishing religious belief. The Clause prevents government from punishing conduct merely because it is religious and from regulating conduct for the purpose of interfering with religion. However, the Clause does not prohibit government from regulating general conduct, even if the regulation happens to interfere with a person's ability to conform conduct to sincerely held religious beliefs. Thus, if it can be shown here that the statute is not really a regulation of general conduct but rather is being applied only to interfere with religion, the convictions will be reversed. (A) is incorrect because the parents would have standing. All that required is a concrete stake in the outcome of the litigation; having been prosecuted for violating the statute, the parents' stake is about as concrete as it can get. (B) is incorrect because it implies that the court will balance the interests involved in determining the validity of the application of the statute here under the former constitutional test, the Court would make such a determination, but since Employment Division v. Smith (1990), the Court has abandoned the balancing approach in favor of the approach discussed above. (D) is incorrect because, as stated above, a person's conduct can be regulated by a generally applicable conduct regulation; religiously motivated conduct has very narrow protection.

Question 17 - Intermediate -Judicial Review A state statute made it unlawful to sell soft drink beverages in any type of container except refundable glass containers. A violation of the statute constituted a misdemeanor, subjecting the violator to a fine of up to $1,000. An out-of-state corporation manufacturing and selling aluminum soft drink cans in the above state brought suit in federal district court contesting the constitutionality of the statute as an undue burden on interstate commerce under the Commerce Clause. What should the court do? A Deny a hearing, because the Eleventh Amendment forbids the exercise of judicial power by the federal government in a suit prosecuted against any state by a citizen of another state. B Deny the hearing, because only state courts may decide the constitutionality of state laws. C Grant a hearing, because a federal question is involved. D Grant a hearing, because jurisdiction exists on the basis of diversity.

(C) The court should grant a hearing because a federal question is involved. Federal district courts are courts of limited jurisdiction and only hear cases in which there is a federal question involved or those in which there is diversity of citizenship and the amount in controversy exceeds $75,000. In these facts, the company is alleging that the state statute violates the Commerce Clause by placing an undue burden on interstate commerce. Thus, a federal question based on the United States Constitution is involved here. (A) is wrong because the purpose of the Eleventh Amendment is to protect the sovereignty of a state by preventing lawsuits against that state being brought in a federal court; the appeal in this case deals only with the constitutionality of a state statute. (B) is wrong because federal courts may also decide whether state laws violate the United States Constitution. (D) is wrong under these facts; even if there is diversity of citizenship, there is no amount in controversy alleged in these facts.

Question 12 - Establishment Clause - Intro The federal government recently constructed a radio telescope in Puerto Rico. In addition to receiving radio signals, the telescope was set up to beam radio waves far out into space. As part of the bill providing for operational funding for the facility, Congress provided for a program to "inform any aliens who might be listening in outer space of the 'American Way of Religion.'" A $10 million appropriation was provided; any religious group whose membership exceeded 500 members in the United States was permitted to prepare a five-minute presentation, and the federal government would pay for the recording of the presentations and broadcast them into space using the transmitter in Puerto Rico. The President signed the bill and it became law. A religious group with a large following in Europe, but only 100 members in the United States, protested and filed suit. Will the court find the religious broadcasts to be constitutional? A Yes, because the power to regulate commerce with foreign nations and among the several states implies that commerce with outer space is included as well. B Yes, because the commerce power is not limited by First Amendment prohibitions. C No, because the requirement of a minimum number of members violates the First Amendment. D No, because the broadcasts are a waste of money and exceed the spending powers of Congress.

(C) The court will find that the requirement of a minimum number of members violates the Establishment Clause of the First Amendment. The Establishment Clause prohibits any law "respecting an establishment of religion." While usually a three-part test based on Lemon v. Kurtzmanis used to determine whether legislation creates improper government involvement with religion, the "compelling government interest" test is used if a law or government program discriminates among religions. Here, the law differentiates among different religious groups, allowing only those with larger memberships to record presentations. There is no compelling government interest for discriminating among the religious groups in this way; thus the legislation is unconstitutional. (A) is incorrect despite the fact that the federal power to regulate commerce could be interpreted as extending to commerce with outer space should the occasion arise. Nevertheless, the commerce power does not override independent constitutional restrictions (e.g., the Establishment Clause) on the conduct in question here. (B) is an incorrect statement of law. The federal commerce power cannot be used to abrogate freedom of speech or to discriminate in favor of religious groups. An exercise of the commerce power generally is subject to all limitations placed on government by the Constitution, including those of the First Amendment. (D) is incorrect. Regardless of merit, almost all expenditures made by Congress are permissible under its spending power. [U.S. Const. art I, §8] Rather than limit the power only to spending for accomplishment of other enumerated powers, this provision grants Congress broad power to spend for the "general welfare" (i.e., any public purpose). As long as the expenditure is not conditioned on requiring a recipient to forgo an individual constitutional right, it is within the spending power of Congress.

Question 18 - FRE 409-offer to pay medical expenses - Intermediate During the trial of her personal injury action against a chemical company, the plaintiff testifies in response to a question by her own counsel that, shortly after she and her family were forced to leave their home because of fumes from its plant, the president of the chemical company telephoned her motel room and said, "If you or any member of your family requires medical treatment, our company will pay all medical expenses in full. We will not have it said that our company's negligence resulted in the illness of a local family." The company's counsel makes a motion to strike all of the plaintiff's testimony, and the court does so. Was the courts action correct? A Yes, because the testimony relates to inadmissible hearsay. B Yes, because the statement was made in connection with an offer to pay medical expenses. C No, because the statement includes an opposing party's admission that it was negligent. D No, because the statement is a factual admission made in connection with an offer to compromise.

(C) The court's action was not correct Federal Rule 409 excludes offers to pay medical expenses, but not statements made in connection with such offers. Thus, (B) is wrong. (A) is wrong because the statement constitutes an admission by a party-opponent (i.e., statement made by or attributable to a party and offered against that party), and admissions are nonhearsay under the Federal Rules. The president of the company, obviously authorized to speak for that entity, has made an admission of negligence, and that admission is admissible against the company as a vicarious admission. (D) is wrong because there was no offer to compromise--the company merely said that it would pay medical expenses, without bargaining for anything in return. In addition, if it were an offer to compromise, a statement made in connection with the offer would not be admissible

Question 8 - Congressional Powers - Intro Congress enacted legislation that was intended to open up federal lands to private industry to explore for, and extract, oil and coal deposits. The act established the Federal Lands Exploitation Commission to supervise the exploration and extraction of fossil fuels from federal lands, and empowered the Commission to enter into contracts on behalf of the federal government with the private companies that wish to mine for coal and drill for oil. The Commission members were also required to investigate safe and sound methods of exploiting the oil and coal deposits without doing unnecessary harm to the environment and to make recommendations to Congress for new laws that would govern the exploitation of federal lands. Further, the commissioners were given the power to make the rules and regulations concerning the contracts with the companies and to appoint administrative law judges to conduct hearings regarding violation of the Commission's rules and disputes concerning the contracts. What is the most likely authority for the establishment of the Federal Lands Exploitation Commission? A The Commerce Clause. B The taxing and spending power. C Congress's authority to regulate federal land. D The war and defense power.

(C) The establishment of the commission is based on Congress's authority to regulate federal land Congress is specifically given the power to regulate federal lands in Article IV. Because there is no other authority that can regulate the use of federal lands but Congress, this is all the authority Congress would need to enact this type of statute. Thus, (B) and (D) are wrong (A) is wrong because although Congress has broad powers under the Commerce Clause, this is not a matter falling under that clause.

Question 19- Polygraph - Intermediate Three masked men robbed a convenience store, during the course of which a clerk was killed. An investigation led the police to believe the defendant was one of the robbers and they placed him under arrest. The defendant protested that he was innocent and volunteered to take a lie detector test. The test was conducted by a qualified polygraph expert. According to the expert's analysis of the test, the defendant lied about his participation in the armed robbery. At the defendant's jury trial for the armed robbery, the prosecution calls the expert to the stand to testify as to her analysis of the results of the polygraph test. The defense objects. If the objection is sustained, what is the likely reason? A The expert's testimony would violate the defendant's right against self-incrimination if he elects not to take the stand in his own defense. B The expert's testimony would violate the defendant's right against self-incrimination, regardless of whether he elects to take the stand in his own defense. C Polygraph evidence is considered to be unreliable and potentially confusing to jurors. D The Federal Rules specifically classify polygraph evidence as irrelevant.

(C) The expert's testimony should not be permitted under Federal Rule 403 because the probative value of polygraph evidence is substantially outweighed by the tendency of its results to mislead and confuse the jury. Federal Rule 702 permits opinion testimony by a qualified expert where the subject matter is one where scientific, technical, or other specialized knowledge would assist the trier of fact in understanding the evidence or determining a fact in issue. However, the methodology underlying the opinion must be reliable (i.e., the proponent must show that the opinion is based on sufficient facts or data, the opinion is the product of reliable principles and methods, and the expert has reliably applied the principles and methods to the facts of the case). Furthermore, Rule 403 gives a trial judge broad discretion to exclude relevant evidence if its probative value is substantially outweighed by the danger of unfair prejudice, confusion of the issues, or misleading the jury. Most jurisdictions have concluded that the probative value of a polygraph test is slight because of its significant rate of error. On the other side of the balancing test, the tendency of jurors to give too much weight to a polygraph test makes the danger of unfair prejudice high. Therefore, the test's unreliability and the risk of confusion from the test's results justify excluding the expert's testimony. (A) is incorrect because the defendant volunteered to take the polygraph test. The Fifth Amendment right against self- incrimination applies only when the defendant is compelled to make the statements. The fact that the defendant did not take the stand in his own defense would not change this result because once a voluntary statement is made, cannot be kept out of the trial on self-incrimination grounds. (B) is also incorrect because, as discussed above, the defendant volunteered to take the polygraph test. His right against self-incrimination is violated only if he is compelled to testify against himself. Just as a defendant's voluntary confession cannot be kept out of the trial on self- incrimination grounds, so the defendant's voluntary statements during the polygraph test cannot be barred on those grounds. (D) is incorrect because the Federal Rules do not specifically classify polygraph tests as irrelevant; in fact, polygraph evidence could be relevant under the Federal Rules Relevant evidence is evidence having any tendency to make the existence of any fact that is of consequence to the determination of an action more probable than it would be without the evidence. [Fed R Evid 401] Polygraph evidence is relevant under this test because it does tend to make the existence of a fact of consequence whether the person is telling the truth more probable. However, as discussed above, this relevant evidence can still be excluded. Under Federal Rule 403 if the court decides its probative value is substantially outweighed by the danger of unfair prejudice.

Question 1 -Number of Jurors - Introductory A defendant going to trial for the felony of aggravated battery filed a motion to have the court seat a jury with only five members. He stated that five was his lucky number and that he would be willing to sign any release to ensure a jury of five. Should the judge grant the defendant's motion to impanel a jury of five members? A Yes, the number of jurors is up to the defendant. B Yes, the number of jurors is typically six, but five is permitted as long as the verdict is unanimous. C No, the Constitution requires at least six jurors. D No, the number of jurors is up to the judge, and the defendant does not have any say in the matter.

(C) The judge should not grant the defendant's motion. The Constitution requires at least six jurors. There is no constitutional right to a jury of 12, but there must be at least six jurors to satisfy the right to jury trial under the Sixth and Fourteenth Amendments. [Ballew v Georgia (1978)] (A) is incorrect because, even were a defendant to waive his right to a jury of six, the judge could not constitutionally impanel such a jury. (B) is incorrect because, as stated above, at least six jurors are required by the Constitution. Note also that six-person juries must be unanimous, although 12-person juries need not be. [Burch v Louisiana (1979)] (D) is incorrect because the number of jurors is not up to the judge in the sense that a judge may not waive the six-juror minimum. In most jurisdictions, the number of jurors (above six) is established by statute or court rule, not by the judge.

Question 11 - Credibility-Intro A witness is called in a contract action between a plaintiff and a defendant. The witness takes his oath and testifies. During cross-examination, the defendant's attorney asked the witness this question: "Isn't it true that even though you took an oath to tell the truth so help you God, you are an atheist and don't even believe in God?" Upon the proper objection, will the judge require that the witness answer this question? A Yes, because the question is relevant to the witness's character for truthfulness. B Yes, because instead of taking the oath, the witness could have requested to testify by affirmation without any reference to God. C No, because evidence of the beliefs or opinions of a witness on matters of religion is not admissible to impair credibility. D No, because an attack on the competency of a witness must be made at the time the witness is sworn.

(C) The judge should not require that the witness answer the question because evidence of the religious beliefs of a witness is not admissible to challenge credibility. Lack of religious belief is no longer a basis for excluding a witness. Not only are a person's religious convictions irrelevant in determining the competence of a witness, Federal Rule 610 provides that a witness's religious beliefs or opinions are not admissible to show that the witness's credibility is thereby impaired or enhanced. Thus, (C) is correct and (A) is incorrect (B) is incorrect. While it is true that the witness could have requested a different type of oath, Rule 610 prohibits this type of question because it would have shown his lack of religious beliefs. (D) is incorrect because, as discussed above, lack of religious belief is no longer a basis for disqualification; thus, this would not constitute an attack on the witness's competency.

Question 18- Relevance - Intermediate The defendant is charged with the murder of his employer. At the trial, the prosecution offered the testimony of another employee who testified that he had accidentally picked up the phone when the defendant was talking to an unidentified man who said, "I hear your boss is going to confess about the money you and she embezzled from the company. What are you going to do about it?" If the defendant's attorney objects to the employee's testimony on the ground that it is irrelevant, what should the judge do? A Sustain the objection. B Sustain the objection, on the grounds that whatever probative value the testimony has is substantially outweighed by the danger of unfair prejudice, confusion of the issues, or misleading the jury. C Overrule the objection. D Overrule the objection, but instruct the jury not to consider the employee's testimony as evidence of the defendant's involvement with any embezzlement of funds.

(C) The judge should overrule the objection. Evidence is relevant if it tends to make the existence of any fact of consequence to the determination of the action more probable or less probable than it would be without the evidence. This evidence, which would help establish a motive, which in turn would help establish the guilt of the defendant, would clearly be relevant. Therefore, an objection based solely on the relevance of the evidence should be overruled. Therefore (A) is wrong. (B) is wrong. Although the court has discretion to exclude relevant evidence if its probative value is substantially outweighed by the danger of unfair prejudice, confusion of the issues, or misleading the jury, there is no indication that this is the case here. (D) is wrong. On the facts of this case, the involvement of the defendant in embezzlement would be relevant, and since the objection was based on grounds of relevance, the limiting instruction would not be required. The statement might be hearsay when used for that purpose, but it might qualify as an admission by silence on the part of the defendant. In any event, the objection was not on hearsay grounds.

Question 23- Lay Opinion Testimony - Intermediate In a real property dispute over a decedent's vacation home, the plaintiff offers a deed to show that the home had been transferred to her tm months before the decedent died. The defendant, the decedent's heir, disputes the plaintiff's claim and alleges that the decedent's signature on the deed was forged. The defendant testifies that he is familiar with the decedent's signature and the signature on the deed is not his. How should the judge rule on this testimony? A Inadmissible, because the defendant is not a handwriting expert. B Inadmissible, because the defendant has a stake in the outcome and his opinion is unreliable. C Admissible, because the defendant knows the decedent's signature. D Admissible, because he is disputing the genuineness of the document, not seeking to establish it.

(C) The judge should rule this testimony admissible because the defendant knows the decedent's signature. Lay opinion testimony is permissible and often essential to identify telephone voices and handwriting. Any lay witness who is familiar with the signature of a person may testify as to his opinion as to its genuineness. In such a case, a foundation must first be laid to show familiarity with the handwriting, as was done here by the defendant's testimony. Therefore, (A) is wrong. (B) is wrong because it goes to the weight of the testimony, not the admissibility. (D) is wrong because the lay witness may testify in support of, or against, the genuineness.

Question 27 - Photographs and Foundation - Intermediate A driver was driving north on a local road when his car went out of control, crossed the center line, and struck the vehicle of another driver who was driving south on the same road. Immediately after the accident, an off-duty officer came by and photographed the accident scene for the police report. In a suit between the drivers, the plaintiff seeks to introduce the photograph taken by the officer. The officer is present in court but has not been called as a witness. Is the photograph of the scene of the accident admissible? A Yes, because the photograph was taken by a police officer who took the photo for an official report. B Yes, because the officer is available to testify at trial. C No, because a proper foundation has not been laid. D No, because of the best evidence rule.

(C) The photograph is not admissible because a proper foundation has not been laid. To be admissible, a photograph must be identified by a witness as a portrayal of certain facts relevant to the issue, and verified by the witness as a correct representation of those facts. It is sufficient if the identifying witness is familiar with the scene or object that depicted. Here, the photograph taken by the officer must be verified by a witness who is familiar with the accident scene as an accurate representation of that scene. Absent such verification and identification (i.e., a proper foundation), the photograph is not admissible. (A) is incorrect because a photograph's admissibility does not require that the photographer be a police officer or that the photograph be taken for an official report. The identity of the photographer and the purpose for which the photograph was taken are irrelevant to the issue of admissibility of the photograph. (B) is incorrect because the photographer need not be available to testify at trial. To authenticate a photograph, any person familiar with the scene may authenticate the photograph. (D) is incorrect because the best evidence rule (also known as the original document rule) is inapplicable to these facts. The best evidence rule states that in proving the terms of a writing (including a photograph), where the terms are material, the original writing must be produced. Secondary evidence of the writing, such as oral testimony regarding the writing's contents, is permitted only after it has been shown that the original is unavailable for some reason other than the serious misconduct of the proponent. [Fed R Evid 1002] Here, the admissibility of the original photograph is in issue. A copy of the photograph is not being offered. Thus, no problem arises under the best evidence rule.

Question 28-Impeachment with prior conviction- Intermediate A plaintiff sued a defendant for serious personal injuries he incurred when the defendant allegedly drove through a red light and collided with the plaintiff's car. Calling the defendant as an adverse witness, the plaintiff asked her if she had been drinking before the accident. The defendant refused to answer, asserting her privilege against self-incrimination. The plaintiff then offers in evidence a certified copy of a court record indicating that, eight years previously, the defendant had been convicted of reckless driving while intoxicated that caused serious personal injury, a felony. How should the trial court rule on the admissibility of the court record? A Admit the record as relevant character evidence because the plaintiff suffered serious personal injuries. B Admit the record as impeachment evidence. C Exclude the record as irrelevant because as yet the defendant has given no testimony to be impeached. D Exclude the record because the conviction is too remote and does not necessarily reflect on the defendant's credibility as a witness in the present proceedings.

(C) The record of the conviction should be excluded because the defendant has given no testimony to be impeached. Impeachment involves the casting of an adverse reflection on the truthfulness of a witness. Although the defendant has been called as a witness, she has not given any testimony at this point. Consequently, the plaintiff is unable to introduce evidence that would otherwise constitute proper impeachment evidence. (A) is incorrect because evidence of character to prove the conduct of a person in the litigated event is generally not admissible in a civil case. Circumstantial use of prior behavior patterns for the purpose of inferring that, at the time and place in question, a person probably acted in accord with such patterns raises the danger of unfair prejudice and distraction from the main issues. Consequently, even if the prior conviction resulted from driving while intoxicated, the record of that conviction is not admissible to show that the defendant was intoxicated when she collided with the plaintiff. (B) is incorrect because, as has been noted, evidence cannot be used for impeachment purposes before there is anything to be impeached. (D) is incorrect for two reasons. First, it is unnecessary to address the issue of whether the conviction constitutes proper impeachment evidence, because impeachment is not even called for on these facts. Second, if properly offered to impeach testimony by the defendant, the conviction would not be considered too remote under the Federal Rules, a conviction is not too remote if fewer than 10 years have elapsed since the conviction or release from prison.

Question 3- Supreme Court Review of State Court Decisions - Intro A state legislature adopted a statute requiring that state school districts be funded equally on a per capita basis, because the previous method of funding school districts based on the amount of taxes paid by residents of the district resulted in schools in the state's wealthiest district receiving twice as much funding per pupil as did schools located in poorer districts. A resident of the wealthy district whose children attend public schools brought an action in state court to have the new statute declared invalid. He established at trial that the disparities in the previous funding system were not based in any way on racial or ethnic discrimination. Nevertheless, when the case reached the state supreme court, it ruled that, based on a provision in the state constitution similar to the Fourteenth Amendment Equal Protection Clause of the United States Constitution, all school districts in the state must be funded equally on a per capita basis. Subsequent to this decision, a taxpayer in a neighboring state sued in federal court, demanding equal per capita funding in his state's school districts. The taxpayer's case eventually reached the United States Supreme Court, which ruled that the Fourteenth Amendment does not compel equal funding, provided there is no probable racial discrimination in the funding. After that decision, the resident of the wealthy district who had lost his state court case filed a petition for a writ of certiorari to have the decision by his state supreme court overturned. Is the resident likely to prevail? A Yes, because the Supremacy Clause renders the state decision invalid. B Yes, because the issue is res judicata. C No, because the state decision turned on state law grounds. D No, because the resident lacks standing.

(C) The resident will not prevail because the state decision was based on state law grounds. The Supreme Court will hear a case from a state court only if it turned on federal grounds. If it finds adequate and independent state grounds for the decision, it will refuse jurisdiction. Here, the facts state that the state decision was based on a provision of the state constitution. The fact that the state provision is similar to the federal Equal Protection Clause is irrelevant to determining whether the decision here was based on state law, since the state court did not base its decision on interpretation of the federal provision, but rather interpreted the state provision. Therefore, the Supreme Court will refuse jurisdiction, and the resident will not prevail. (A) is incorrect because the Supremacy Clause renders state laws invalid under implied preemption rules only if they actually conflict with federal law, they prevent achievement of federal objectives, or Congress has preempted the field. Nothing in the facts indicates that Congress has preempted the issue of school funding, and the state decision is not in conflict with the Supreme Court decision here and does not prevent achievement of the federal objectives; the Supreme Court held that the federal Equal Protection Clause does not require equal funding, not that equal funding violates equal protection. Thus, the state court was free to grant its citizens more protection under the state equal protection provision than is granted under the federal provision. (B) incorrect because res judicata not a constitutional issue. Moreover, the Supreme Court did not decide whether equal funding was invalid, only that it was not required. (D) is incorrect because the resident has standing. To have standing, a party must have a concrete stake in the outcome of a controversy. This requires an injury in fact caused by the government action that can be remedied by a decision in the litigant's favor. The resident has an injury in fact since his children are being deprived of educational money under the new state statute. This injury could be remedied by a decision in his favor, because if the statute is held invalid, the state would go back to the unequal funding system

Question 17- Commercial Billboards - Advanced In order to raise revenue, a city erected billboards on the sides of all government buildings and planned to sell the space for commercial advertising. A city ordinance provided that any advertiser could rent the space, provided that the activity or product advertised was legal and had "nothing to do with religion or politics" because the city sought to "avoid controversy." The owner of a bookstore that specialized in books on all religions and "spiritual" subjects sought to lease a billboard on a city building to place an ad. In addition to selling books, the owner conducted daily reading and study groups in the store on all of the major religions. The proposed ad implored onlookers to come to the bookstore in order to find their own path to God and to attend study groups at the store held at 6 pm nightly. The ad was rejected by city officials. If the owner files an appropriate suit against the city in federal district court asserting violation of her First Amendment rights, is she likely to prevail? A Yes, because the city has made the sides of civic buildings public forums. B Yes, because restrictions on commercial speech must be narrowly tailored to serve a substantial government interest directly advanced by the restriction. C No, because the restriction is viewpoint neutral and reasonably related to a legitimate government purpose. D No, because the city must avoid excessive entanglement with religion.

(C) The restriction will probably be upheld because it is viewpoint neutral and reasonably related to a legitimate government purpose. The billboards are not traditional public forums; rather, they will be found to be "commercial ventures" by the city. In a similar setting, the Supreme Court held that cities could differentiate between broad categories of speech in accepting advertising on city-owned buses (i.e., the Court allowed a city to refuse political advertising and accept only commercial advertising) [see Lehman v. Shaker Heights (1974)], as long as the restriction was viewpoint neutral and reasonably related to a legitimate government interest. The city rule here is constitutional because it is viewpoint neutral (it distinguishes between broad categories of speech but does not distinguish based on content within a category) and it is reasonably related to the legitimate government interest of avoiding controversy. (A) is incorrect because the Supreme Court has held that allowing advertising on government- owned property does not make that property a public forum; rather, the property is a commercial forum. [See Lehman v. Shaker Heights, supra] (B) is incorrect because although it states the general rule for regulation of commercial speech, the more specific Supreme Court precedents regarding advertising on city-owned property used for a proprietary venture (discussed above) would apply here. (D) is incorrect because nothing in the city policy would cause an excessive entanglement with religion; rather, the policy seeks to avoid excessive entanglement by prohibiting religious ads on government property.

Question 26- Hearsay-Intermediate A plaintiff initiated a personal injury action against the defendant, alleging that the defendant struck her with his bicycle while she was rollerblading on a park trail. At trial, the plaintiff calls a witness who testifies that he was standing a short distance away and ran up to offer help. He further testifies that as the ambulance was leaving with the plaintiff, the defendant threatened to come after him unless he testified falsely that the plaintiff had suddenly swerved into the path of the defendant. Should this last statement have been admitted into evidence? A No, it is hearsay not within an exception. B No, it is not relevant to the issue of negligence. C Yes, it is relevant and not hearsay. D Yes, it is a declaration against interest by the defendant.

(C) The statement is admissible because it is relevant and not hearsay. A statement made by a party and offered against that party (commonly called an admission by a party-opponent) is considered nonhearsay under the Federal Rules. The defendant's attempt to threaten the witness would qualify as an admission because it is being offered against him at trial. Alternatively, the defendant's conduct could be classified as conduct not intended as an assertion, and thus it would not fall within the definition of hearsay. Either way, the defendant's conduct is not hearsay, so (A) is clearly wrong. (B) is wrong. The evidence helps establish that the defendant believed that he acted negligently. (D) is wrong. Because admissions are not hearsay, the evidence would not come in under an exception to the hearsay rule. Furthermore, the hearsay exception for declarations against interest applies only when the declarant is unavailable, which is not the case here.

Question 18- Party Opponent - Intermediate The defendant was at work when her husband called her and said, "The neighbor just tripped over those roots told you to take out. He's badly hurt and I'll bet he sues us for all we're worth. " The defendant then told her secretary, "The neighbor just got hurt because forgot to do my yard work." On returning home, however, the defendant discovered that the neighbor had actually tripped over roots from his own tree in his own yard. The neighbor disagreed and sued the defendant and her husband. At trial, the neighbor called the defendant's secretary to testify as to the defendant's statement to him. Will the secretary's testimony be admitted? A No, because the defendant had no firsthand information when she made her statement to the secretary. B No, because it is inadmissible lay opinion. C Yes, because it is not hearsay. D Yes, to impeach the defendants expected testimony as to the result of her own investigation.

(C) The testimony is not hearsay because the defendant's statement constitutes an admission by a party-opponent-i.e. a statement by a party (the defendant) being offered against her. An admission is not considered a hearsay statement under the Federal Rules. [Fed R Evid 801 (d)(2)] Thus, (C) appears to be correct--that the secretary's testimony about the defendant's statement will be admitted because it is not hearsay. (C) is not an ideal answer, though, because it is so incomplete. The fact that an item of evidence is nonhearsay does not automatically render it admissible. For example, if nonhearsay evidence is irrelevant, it would not be admissible. Thus, deciding whether (C) is the best answer requires a thorough assessment of the alternatives. <(D) can be quickly discarded as incorrect. Impeachment evidence is not admissible until after the witness to be impeached has testified. It would be improper to admit the secretary's testimony for the purpose of impeaching the defendant's expected testimony as to the result of her own investigation of the accident. (A) is tempting, but it is also incorrect. Normally, an out-of-court statement of a declarant, like the in-court testimony of a witness, is admissible only if it was made with personal knowledge. This requirement, however, does not apply to admissions. Thus, although the defendant's statement ("The neighbor just got hurt because I forgot to do my yard work") was made without personal knowledge, it will be admissible. As a party, the defendant will have ample opportunity to explain why she made the statement even though she lacked personal knowledge about the accident that injured the neighbor. The persuasiveness of her explanation will determine whether the jury takes her admission seriously. (B) is also incorrect. The common law requirement that lay witnesses were not permitted to give opinions, but only allowed to state facts, has been rejected in favor of a helpfulness standard. For example, Rule 701 allows lay witnesses to give testimony in opinion form as long as it is helpful to a clear understanding of the testimony or the determination of a fact in issue. Similarly, an admission is not rendered inadmissible merely because it is in opinion form. For example, a party's statement that "l was negligent" or "It was all my fault" will be admitted into evidence as an admission. Therefore, although (C) is not an ideal answer, it is basically correct, and (A), (B), and (D) are clearly incorrect.

Question 10 - Establishment Clause-Intro A state enacted legislation providing for the testing of all high school students in the performance of certain adult-world tasks. In public schools, the test was administered at the school by government employees. In private schools—both religious and secular—the test was administered by school personnel, and the schools were reimbursed for the costs of administering the tests and reporting the results to the state. An association with appropriate standing filed suit seeking to enjoin the reimbursement to the religious schools, claiming that the reimbursement violates the Establishment Clause of the First Amendment to the United States Constitution. Should the trial court grant the injunction? A Yes, because direct state payment to religious schools violates the Establishment Clause. B Yes, because the state may not require the religious schools to conduct tests that violate their religious principles. C No, because the reimbursement has a secular purpose that neither advances nor inhibits religion and that will not entangle the state in religion. D No, because the tests do not involve matters of religion.

(C) The trial court should not grant the injunction, because the reimbursement does not violate the Establishment Clause. In Committee for Public Education and Religious Liberty v. Regan (1980), the United States Supreme Court held that a similar statute passed by the state of New York did not violate the Establishment Clause because it had a secular legislative purpose. Its principal effect did not advance or inhibit religion, and it did not foster an excessive government entanglement with religion. Here, our facts show a state law that provides for secular testing and reimbursement for testing administration costs to religious schools. As such, the state law does not violate the Establishment Clause because it had a secular legislative purpose (namely, reimbursing schools for secular testing that was conducted in all schools - religious and secular alike), its principal effect did not advance or inhibit religion, and it did not foster an excessive government entanglement with religion. (A) is therefore incorrect; aid can be given to religious schools that pass this test. (B) is wrong. It misstates the facts given, which make no mention of a specific religious group. Rather, the suit was filed by an association challenging the reimbursement. (D) is irrelevant. The question is not whether the tests involve "matters of religion," but whether the reimbursement program impermissibly advances religion.

Question 25- Privilege Against Self-Incrimination - Intermediate A beneficiary sued a trustee of a trust for breach of fiduciary duty. During the beneficiary's case in chief, the beneficiary's counsel asks the trustee on direct examination whether the trustee embezzled $50,000 from the bank at which she was a trust officer. Must the trustee answer this question? A Yes, because by taking the stand she waived her privilege against self-incrimination. B Yes, because the Fifth Amendment has no application to a civil trial. C No, if she has not waived the privilege against self- incrimination. D No, because a witness may always assert the privilege against self-incrimination.

(C) The trustee does not have to answer the question unless she has waived the privilege against self-incrimination. A person may assert the privilege against self-incrimination in any proceeding, civil or criminal, in which testimony that could incriminate the person (i.e., expose her to criminal liability) is sought. Thus, the trustee may raise the privilege unless she has waived it. (A) is incorrect because the privilege here cannot be waived merely by taking the stand. While a criminal defendant may refuse to take the witness stand at all, in a civil action, a witness is required to take the stand if called, but taking the stand is not a waiver of the privilege. The privilege must be raised as to each objectionable question asked and will be waived only if the witness discloses incriminating information. (B) is incorrect because the Fifth Amendment applies to all trials at which the witness's appearance and testimony are compelled. (D) is incorrect because it is too broad; if a witness waives the privilege by testifying as to the subject, and then attempts to raise the privilege later, it is too late. She can no longer assert the privilege.

Question 37-Impeachment-Advanced The plaintiff sued a local restaurant, claiming that she injured her teeth, gums, and mouth when she bit into a hamburger that contained a large, jagged piece of glass. The plaintiff called to the stand a waiter for the restaurant, who testified that, when he heard the plaintiff scream, he looked in her direction and saw her remove a piece of glass from her bleeding mouth. On cross- examination, the defense asked the waiter, "Isn't it a fact that three months ago you were fired by the restaurant for serving drinks to your friends and not charging for them?" The waiter responded, "Yes, but wasn't trying to steal anything I just forgot to charge them." The defense then asked, "Isn't it a fact that last month you threw a rock through the plate glass window at the restaurant?" The waiter replied, "That's not true; was there but didn't throw the rock." The defense then offered the testimony of a witness who was prepared to testify that she saw the waiter throw the rock through the restaurant's window. Assuming that there have been no criminal charges filed as a result of the broken window, is the witness's testimony admissible? A No, because specific acts of misconduct that did not result in a conviction cannot be used to impeach a witness, either on cross-examination or through extrinsic evidence. B No, because specific acts of misconduct that did not result in a conviction cannot be established through extrinsic evidence. C Yes, as evidence of bias. D Yes, to establish that the waiter lied under oath.

(C) The witness's testimony is admissible to show bias. A witness can be impeached, either on cross-examination or by extrinsic evidence, with evidence that suggests a bias on the part of the witness, because it tends to show that the witness has a motive to lie. Evidence that the witness disliked the party he is testifying against would qualify as evidence of bias. The witness could testify that she saw the waiter throw the rock through the restaurant's window, because such evidence would help establish the waiters bias against the restaurant. (A) is incorrect for two reasons: Federal Rule 608 provides that, if offered to impeach, prior bad acts may not be proved through extrinsic evidence but may be inquired into during cross-examination. Furthermore, if the prior bad act also helps establish bias, the courts have held that extrinsic evidence also will be admissible. (B) is incorrect as well for this latter reason. (D) is too broad a statement. In a broad sense, the evidence is offered to impeach the credibility of the waiter and to suggest to the jury that he may be lying under oath. However, the defense does not need to establish that the waiter lied; the reason the evidence is relevant and does not constitute impeachment on a collateral matter is because it is offered to show bias, making (C) the better answer.

Question 26 -Relevance - Intermediate In its lead editorial in the Sunday edition, a suburban daily newspaper characterized a real estate developer as a "common thief." The developer promptly filed suit against the newspaper for defamation. During the course of the presentation of the plaintiff's case, he sought to put a witness on the stand who is prepared to testify that the plaintiff once saved the life of a fellow soldier in Vietnam. If the newspaper's lawyer objects, should the court rule that the testimony is admissible? A Yes, because the plaintiff has a right to introduce evidence of his good character. B Yes, because the plaintiffs character has been brought into question by the editorial. C No, because the witness's testimony is not probative of any material issue. D No, because specific instances of conduct are not admissible to prove character.

(C) The witness's testimony is inadmissible because it is not probative of a material issue (i.e., whether the plaintiff is a thief). Relevant evidence tends to prove or disprove a material fact in issue. Here, the testimony tends to prove that the plaintiff is brave and selfless, but it is not relevant as to the fact in issue, which is whether he is honest. (A) is incorrect because character evidence is admissible in a civil suit only when, as here, proof of a person's character, as a matter of substantive law, is an essential element of a claim or defense. However, even when character is in issue, the evidence must be relevant to the particular character trait in issue; here, it is not relevant to the issue of the plaintiff being a thief. (B) is incorrect for the same reason; to be admissible, the evidence must be relevant. (D) is incorrect because proof of specific instances of a person's conduct admissible when character is directly in issue. [Fed R Evid. 405(b)]

Question 24- Expert Testimony Qualifications - Intermediate At the trial of a lawsuit that arose out of a collision between the plaintiffs and the defendant's cars, the plaintiff's attorney calls an automobile mechanic as a witness, who testifies that he has 12 years' experience and was the only witness to the accident. The witness also testifies that he arrived at the scene immediately after the accident, which caused both cars to overturn, and saw the wheels on both cars still spinning. He testifies that the wheels of the defendant's car were spinning faster than the wheels of the plaintiff's car. The plaintiff's attorney asks the witness to testify as to what speed the respective cars were traveling at the time of the accident based upon his observations of the spinning wheels. The defendant's attorney objects. Should the testimony regarding the speed of the cars be admitted? A Yes, as the witness's personal opinion. B Yes, as a matter based upon personal observation. C No, because the witness has not been qualified as an expert in accident reconstruction. D No, because there is not another witness to corroborate the witness's presence at the accident scene.

(C) The witness's testimony regarding the speed of the cars should not be admitted because he has not been qualified as an expert in accident reconstruction. If the subject matter is such that scientific, technical, or other specialized knowledge is required to render an opinion, expert testimony is admissible and appropriate. In fact, in such an area, opinions by laypersons would not be permitted. To testify as an expert, a person must have special knowledge, skill, experience, training, or education sufficient to qualify him as an expert on the subject to which his testimony Relates. The expert must possess reasonable certainty or probability regarding his opinion. Here, the witness is being asked to testify as to the speed of the cars, not based on actually viewing the cars while in motion (in which case lay opinion is often accepted), but on his observation of the spinning wheels after the accident. Determination of the speed of vehicles based upon observation of the spinning wheels of such vehicles after a collision would certainly call for the application of technical or specialized knowledge, thus making the subject matter appropriate for expert testimony. To testify as an expert, the witness must be qualified by virtue of having special knowledge or experience regarding accident reconstruction, which encompasses rendering opinions on the speed of vehicles based on the spinning of their wheels. The witness's experience as an auto mechanic would not suffice to establish him as an expert in accident reconstruction. Since he is not qualified as an expert, his opinion testimony as to the speed of the cars based upon his observation of the spinning wheels will not be admitted. (A) is incorrect because, as has been explained, the witness's personal opinion is not admissible without proper qualification as an expert. (B) is incorrect because, if the witness is not qualified as an expert, the fact that his opinion is supported by a proper factual basis (i.e., personal observation) will not render that opinion admissible. (D) is incorrect because the presence of a witness at the scene of events to which his testimony relates need not be corroborated by another witness.

Question 4-Intro A woman and a man had lived together for several years but had never married because the man was already married. During the time the woman and the man lived together, they had accumulated property. One day, the man told the woman to move out. The state in which they lived recognized in general the validity of property agreements entered into by unmarried couples who lived together, but there was a statute that provided that such agreements were void as being against public policy when one of the parties living together was married to another. The woman brought suit against the man, alleging that they had an agreement to share equally all property accumulated by them during the time they lived together, and challenging the constitutionality of the statute. Will the woman prevail in her challenge to the statute? A Yes, if the state fails to prove that the statute is rationally related to a legitimate state purpose. B Yes, if the woman can prove that the statute is not necessary to effectuate a compelling state interest. C No, if the woman fails to prove that the statute is not rationally related to a legitimate government purpose. D No, if the state proves that the statute was necessary and proper.

(C) The woman will not prevail if she fails to prove the statute is not rationally related to a legitimate government purpose. Property rights are tested on a rational basis standard. Under the rational basis test, the burden of proof is on the plaintiff, not the state. Thus, the woman will not prevail if she fails to prove that the statute is not rationally related to a legitimate government purpose. Hence, (A) is wrong. (B) is wrong because it uses the wrong standard and because the burden of proof is on the plaintiff, not the state (D) is incorrect; the Necessary and Proper Clause has nothing to do with state statutes.

Question 5 - Intro In certain pans of a state, single-family residences had become so expensive that the vast majority of families could no longer afford to buy a home. To alleviate this problem, the legislature enacted statutes creating a housing agency. The agency, organized along the lines of a private corporation, was authorized to act as general contractor and build homes in counties where the average cost of a new home exceeded by 50% the national average cost of a new home, then to sell the homes at the cost of materials and labor to first-time homebuyers. In one medium- sized city in the state, the average cost of a new home exceeded the national average by 15%, while in a nearby large city, the average cost of a new home exceeded the average by 50%. The agency began building and selling homes in the large city, but did not operate in the medium city. About 35% of the population of the medium city is of Armenian ethnicity. A citizen of Armenian heritage brings a class action against the state, seeking to have the agency's failure to operate in the medium city declared a violation of the right to equal protection of the Armenian citizens of that city. What fact would be most helpful for the citizen in challenging the statute? A The state could have permitted the agency to build and sell homes in all areas of the state. B Armenian citizens experience difficulty in affording single-family residences. C The legislation setting up the agency was intended to discriminate against Armenian citizens. D The percentage of Armenian citizens is much higher in the medium city than in the large city. (C) The most helpful fact would be an intent to discriminate State action that is facially neutral will nevertheless be struck down if it can be shown that it was intended to discriminate on the basis of race or national origin and does in fact have that effect. The citizen must show that the legislature selected the specific cost guidelines with full knowledge of the population characteristics in the medium city and with the specific intent to disqualify that city from participation in the program because of its high concentration of Armenians. If he proves that intent, he will prevail, and (C) is accordingly his best argument. (A) would not be as helpful because the state could show a rational basis for not operating in the medium city--that housing costs were not high enough to warrant state intervention--which would suffice to justify different treatment not based upon purposeful discrimination. (B) is not a good answer because the difficulty experienced by Armenian citizens in obtaining housing may have nothing to do with state action and may be experienced by other groups as well. (D) is not the best answer because, while it is arguably some evidence of purposeful discrimination, it does not state it as clearly as (C).

(C) The woman will not prevail if she fails to prove the statute is not rationally related to a legitimate government purpose. Property rights are tested on a rational basis standard. Under the rational basis test, the burden of proof is on the plaintiff, not the state. Thus, the woman will not prevail if she fails to prove that the statute is not rationally related to a legitimate government purpose. Hence, (A) is wrong. (B) is wrong because it uses the wrong standard and because the burden of proof is on the plaintiff, not the state (D) is incorrect; the Necessary and Proper Clause has nothing to do with state statutes.

Question 11-Double Jeopardy -Intro A woman was arrested outside of a house shortly after she had broken in and stolen some jewelry. She was indicted for larceny and later for burglary. She was tried on the larceny indictment and convicted. Thereafter, she was brought to trial on the burglary indictment. Relying on the Double Jeopardy Clause of the Constitution, the woman moves to dismiss the indictment. Should the motion be granted? A Yes, because the Double Jeopardy Clause requires that all offenses arising out of the same transaction be adjudicated in the same trial. B Yes, because the Double Jeopardy Clause allows the imposition of separate sentences for separate offenses occurring during the same criminal episode only if the offenses are tried together. C No, because larceny and burglary are offenses that may constitutionally be tried and punished separately, even if they arise out of the same transaction, because each requires proof of a fact that the other does not. D No, because the only protection double jeopardy affords to a defendant charged with multiple counts is under the doctrine of collateral estoppel.

(C) The woman's motion should be denied because the Double Jeopardy Clause does not prohibit the second prosecution. The Double Jeopardy Clause of the Fifth Amendment provides criminal defendants with the right to be free of double jeopardy for the same offense. However, two crimes do not constitute the same offense if each crime requires proof of an additional element that the other crime does not require, even though some of the same facts may be necessary to prove both crimes. [Blockburger v. United States (1932)]. Here, larceny requires a taking and carrying away of the property of another, which burglary does not require, and burglary requires a breaking and entry, which larceny does not require. Hence, they are distinct offenses for purposes of the Double Jeopardy Clause. (A) and (B) are incorrect because the Supreme Court does not use a "same transaction" or "same episode" test suggested by these answer choices; instead, the Blockburger test is used regardless of whether the two offenses were tried together at a single trial or at separate trials. (D) is incorrect because while double jeopardy also protects against inconsistent factual determinations at a subsequent trial, it protects against multiple prosecutions as well, as long as the crime is the "same offense."

Question 19 - Congressional Powers - Intermediate Congress determined that there should be a uniform law for handgun registration throughout the United States, and enacted the Federal Firearms Act. Which of the following constitutional provisions could most easily be considered the basis of such enactment? A The Equal Protection Clause of the Fourteenth Amendment B The Second Amendment C The Commerce Clause D The Necessary and Proper Clause

(C) This statute deals with activities that have an effect on interstate commerce and, therefore, is within the commerce power. (B) is wrong because the Second Amendment is prohibitory; therefore, it could not be the basis of enactment, but could only prohibit some types of enactments. (A) is wrong because the Fourteenth Amendment Equal Protection Clause applies only to the states, not the federal government. (D) is wrong because the Necessary and Proper Clause only broadens congressional power authorized under some other provision; it does not itself create the power to act.

Question 5- Commerce Clause -Intermediate A new federal law prohibited the use of various pesticides in areas with a certain population density near navigable waters. A city located in the southeastern United States was plagued by a sharp increase in disease-carrying mosquitoes. The city's board of health recommended that all residential areas be sprayed with a pesticide proven to be highly effective against mosquitoes. Despite the fact that the federal law would prohibit use of that pesticide in these areas, the city council passed an ordinance adopting the board of health plan, relying on the opinions of several independent experts that the health benefits of reducing the mosquito population outweighed the risks of spraying. An environmentally minded citizen of the city brought an action in federal court challenging the ordinance. Assuming that the citizen has standing, is the court likely to find the ordinance valid? A Yes, because pursuant to the police power, cities have a compelling interest in laws designed to protect the health, safety, and welfare of their citizens. B Yes, because controlling health hazards is an integral governmental function. C No, because it is superseded by the power of Congress to adopt laws to protect the health, safety, and welfare of citizens. D No, because it conflicts with a federal law that Congress had the power to make under the Commerce Clause.

(D) Congress's power to regulate commerce has been construed broadly, so that it may regulate any activity, local or interstate, that either in itself or in combination with other activities has a substantial economic effect on interstate commerce. If Congress has determined that the use of chemical pesticides and their runoff into waterways (which are channels of interstate commerce) will have an overall detrimental impact on the environment, this determination will be sufficient in this case to satisfy the standards established by the Supreme Court. Therefore, the law probably is a valid exercise of the commerce power. Any state or local action that conflicts with a valid act of Congress is invalid under the Supremacy Clause. (A) is incorrect because while the police power (the power to adopt regulations for the health, safety, and welfare of citizens) belongs to the states, a police power regulation that conflicts with a federal law is invalid under the Supremacy Clause. (B) is incorrect because state and local government activities may be regulated by a general law that applies to both the public and private sectors, even if the regulation affects integral governmental functions, as long as there is a constitutional basis for the law. (C) is incorrect because Congress does not have a general "police power' to adopt laws on health and safety. The laws that Congress has passed banning activities that it has deemed harmful to public health have been based on its power to regulate interstate commerce

Question 15- FRE 412 - Intro At a victory party after a hard-fought election, the campaign director consumed several drinks. A campaign worker who had also been drinking took the director to her hotel room for a nightcap. They later had intercourse. The worker filed a complaint with the police, claiming that the director had intercourse with her against her will, and the director was charged with rape. Which of the following is most likely to be admitted in the director's defense? A The worker has a reputation in the community as being sexually promiscuous. B Since the incident occurred, the worker has had sexual intercourse with two other campaign workers. C Two years ago during the candidate's previous campaign, the worker maintained a sexual relationship with the former campaign director. D The director and the worker had had consensual sex on two prior occasions.

(D) Evidence of prior consensual sexual relations between the director and the worker is most likely admissible. Although Federal Rule 412 generally excludes evidence of an alleged victim's sexual behavior, evidence of specific instances of sexual conduct between the alleged victim and the accused may be admitted to show consent. Thus, if the director raises consent as a defense to the rape charge, evidence of his previous consensual sexual encounters with the worker is admissible. (A) is incorrect because it presents evidence that Federal Rule 412 specifically intends to exclude; i.e., evidence of the alleged victim's sexual behavior. The Federal Rules also contain an exception for specific instances of the alleged victim's sexual conduct tending to show that someone other than the accused was the source of semen, injury, or other physical evidence. (B) does not fit this exception because the worker had intercourse with two other workers after the incident with the director occurred. Likewise, (C) is not likely to fit this exception as the worker's relationship with the previous campaign director occurred two years ago, and is therefore probably not helpful in explaining the presence of physical evidence.

Question 1 -Rational Basis- Intro A state conducted a study in which it found that many parents were moving out of certain areas of the state because they could not find affordable day care. As a result, the state legislature enacted a statute that created an agency to operate subsidized day care centers throughout these areas. These day care centers would charge parents a nominal fee compared to private day care centers in the area. A private day care center has discovered that its business has fallen off about 30% since the agency started operating a day care center in its city. It brings an action seeking to enjoin the operation of the agency. What will be the probable outcome of this litigation? A The center will win, because operation of a state agency in competition with a private business violates the Due Process Clause of the Fourteenth Amendment. B The center will win, because permitting some citizens to do business with a state business while others do business with a private business violates the Equal Protection Clause of the Fourteenth Amendment. C The center will lose, because it lacks standing to challenge the legislation, having suffered no direct injury. D The center will lose, because the agency is a valid exercise of the state's powers.

(D) Providing day care centers for its citizens is clearly within the reserved police powers of the state. Because this is facially neutral economic and social legislation, the state simply needs a rational basis for its actions, and that is present. (A) and (B) are, accordingly, wrong. A state may undertake traditionally private business activities, and the analysis for both due process and equal protection purposes is rational basis review, under which the state will prevail. (C) is not a good answer because the center has suffered a direct injury (lost business) caused by the state government action. It has standing to challenge the legislation, but standing has nothing to do with the merits of the claim, which it will lose.

Question 16 -Judicial review - Intermediate A state statute was struck down by the supreme court of the state on the grounds that it was in conflict with the Supremacy Clause of the United States Constitution as well as the Equal Protection Clause of the state constitution. Does the United States Supreme Court have jurisdiction to hear an appeal of the state supreme court's decision? A Yes, because the Supreme Court has original jurisdiction of all cases in which a state is a party. B Yes, because the requirements for an appeal to the United States Supreme Court are met under the facts. C No, because the requirements of certiorari are not met under the facts. D No, because of the "adequate and independent state ground" theory.

(D) The Supreme Court probably would not review the state decision because of the "adequate and independent state ground" theory. The Supreme Court will hear a case from a state court only if the state court judgment turned on federal grounds. Here, the judgment is, in part, based on a violation of the Equal Protection Clause of the state constitution, which is an adequate and independent state ground on which the decision would rest even if the federal issue were resolved (assuming that the state court's disposition of the state constitutional issue did not depend on federal doctrines). (A) is wrong because this is a question of appellate jurisdiction, not original jurisdiction. (B) is wrong because this type of case would not meet the very narrow requirements of appeal. (C) is wrong because if it were not for the adequate and independent state ground, this type of challenge could be heard by the Court in its discretion.

Question 14- Attorney-Client Privilege- Intermediate A 12-year-old child was injured in an automobile accident. The child's father brought the child to see an attorney to bring suit against the defendant. During the paid consultation with the attorney, the seriousness of the child's injuries was discussed with candor. After the discussion, the attorney told the father and child that they would be better off with a lawyer who specialized in personal injury work. Eventually, another attorney was hired to bring the child's lawsuit against the defendant. Defense counsel has reason to believe that the child's injuries are not serious at all. She therefore subpoenas the first attorney for an oral deposition. During the course of the deposition she asks the attorney about his discussion with the child regarding the child's injuries. May the attorney invoke the attorney-client privilege? A No, because the child never hired the attorney as her counsel. B No, because the privilege is held by the client rather than the attorney. C Yes, because the child paid the attorney for consultation. D Yes, because the presence of a third party did not negate the privilege.

(D) The attorney may invoke the attorney-client privilege because the presence of a minor client's parent does not waive the privilege. A client has a privilege to refuse to disclose, and to prevent others from disclosing, confidential communications between herself and her lawyer. The attorney-client privilege requires that, at the time of the communication, the client be seeking the professional services of the attorney. Disclosures made before the lawyer has decided to accept or decline the case are covered if the other requirements of the privilege are met. A communication is confidential if it is not intended to be disclosed to third persons; thus, communications made in the known presence and hearing of a stranger are not privileged. However, statements made in front of third persons whose presence is reasonably necessary to the consultation (e.g., this client's parent) are still considered confidential. Here, the child was consulting with the attorney for the purpose of seeking the attorney's professional services. During this consultation, the child made disclosures concerning her injuries that were not intended to be disclosed to third persons. Thus, the child's communication would be deemed confidential. This confidentiality would not be lost by virtue of the fact that the communication was made in the presence of the father, whose presence was reasonably necessary given the child's age. Because the elements of the attorney-client privilege are thus satisfied, the attorney may invoke the privilege on behalf of the child to refuse to disclose his discussion with the child concerning the child's injuries. (A) is incorrect because the attorney-client privilege does not depend on an actual hiring of the attorney. The requisite relationship exists simply by virtue of the fact that, at the time of the communication, the child was seeking the attorney's professional services. (B) is incorrect because the person who was the attorney at the time of the communication can claim the privilege on behalf of the client. The attorney's authority to do this is presumed in the absence of any evidence to the contrary. (C) is incorrect because application of the privilege does not hinge on payment for services. The confidential communications of a client receiving a professional consultation free of charge are protected to the same extent as those made to a lawyer charging for his time.

Question 10 - Plenary Power - Intro A state legislature recently enacted a statute imposing a "land utilization tax" on those operating businesses within the state who pay no property taxes. The state seeks to apply the tax to the owner of a small restaurant and store that is located in a national park within the state. The owner operates the restaurant and store pursuant to a contract with the United States government and pays no property taxes. Which of the following is the owner's best constitutional argument against application of the tax to her? A The owner has been denied the privileges and immunities of national citizenship, protected by the Fourteenth Amendment, because the state tax impairs her fundamental right to conduct business on federal lands. B The tax is unconstitutional as applied to the owner, because it interferes with interstate tourism in violation of the federal commerce power. C The owner is being denied equal protection of law, because those operating businesses on federal lands in other states are not subject to the D The tax is unconstitutional as applied to the owner, because under the property power the federal government has plenary power to regulate federal lands.

(D) The best argument is that the tax interferes with the plenary power of Congress to regulate federal lands Article IV, Section 3 of the Constitution gives Congress the power to make all necessary rules and regulations concerning property belonging to the federal government. The owner would argue that this power, when combined with the Supremacy Clause (which makes federal law the supreme law of the land, superseding conflicting state law) would prevent any attempt by a state to tax persons on federal land, absent congressional consent to the tax. In fact, this argument will probably fail because the Court has indicated that while the states may not directly tax or regulate the federal government, they may indirectly do so by adopting taxes or regulations on persons dealing with the federal government, as long as the tax or regulation does not unduly burden the federal government. The tax here seems to fall into the permissible category; nevertheless, this is the owner's best argument. (A) is not a good argument because the Privileges or Immunities Clause of the Fourteenth Amendment does not protect the right to conduct business on federal lands. The Clause protects a narrow range of privileges of United States citizenship, such as the right to enter onto federal lands, the right to petition Congress for redress, and the right to vote for federal officers. The Clause does not protect the entire Bill of Rights against infringement, and the owners right here falls outside the scope of the Clause. (B) is incorrect because the commerce power would not preempt the tax here. Generally, a state may adopt taxes that will affect interstate commerce as long as the tax does not discriminate against interstate commerce (the tax here does not) and (i) there is a substantial nexus between the state and the taxpayer (here, the owner does business on land within the state); (ii) the tax is fairly apportioned (the tax here is fair since it merely equalizes the tax burden on users of land within the state); and (iii) there is a fair relationship between the tax and the benefits provided by the state (this requisite is met because the owner gets the protection of doing business in the state in exchange for the tax). (C) is incorrect because there is no equal protection violation here. The Equal Protection Clause merely prohibits the states from treating similar people in a dissimilar manner without a valid reason. When, as in this case, the law does not involve a suspect or quasi-suspect class or a fundamental right, the law will be upheld as long as there is a rational basis for the discrimination. Here, it is questionable whether the law discriminates, since it is applicable to all persons using land in the state. Assuming that the law does discriminate on the basis of the location of land, the discrimination is permissible because it is rationally related to a legitimate state purpose (a state may impose taxes to raise revenues but may not impose a tax on land or activities outside of the state)

Question 4 -Time, Place, Manner -Intro To increase tourism, a city began sponsoring laser light shows, which proved to be very popular. Several charitable organizations received permission from the council to sponsor a show and charge admission to raise money to help support their causes. One of them hired a famous laser light artist to give their show. When the artist arrived, he began setting up his lasers for the show. A city official soon stopped him, informing him that he could use only the city's lasers because the city feared that outsiders might use powerful lasers that could cause eye damage to viewers. The artist told the charitable organization that had hired him that the success of his art depends on the power of his lasers and that he could not produce desirable effects using the city's lasers. The charitable organization appealed to the city, but the city held fast to its rule requiring all laser light artists to use the city's lasers. If the charitable organization files an action against the city, how will the court most likely rule? A Find for the charitable organization, because an is protected by the First Amendment and the city rule interferes with the artist's freedom of expression. B Find for the charitable organization, because the city rule is not the least restrictive method for achieving the city's goals. C Find for the city, because the laser light show is not speech and therefore is not protected by the First Amendment. D Find for the city, because the rule is a reasonable time, place, and manner restriction.

(D) The city will prevail because its rule is a reasonable time, place, and manner restriction. Speech protected by the First Amendment includes not only verbal communication, but also conduct that is undertaken to communicate an idea. The laser light show, like other art, probably is protected speech. While the content of speech generally cannot be limited, the conduct associated with speech in public forums can be regulated by reasonable time, place, and manner restrictions. To be valid, the regulation must be content neutral, narrowly tailored, to serve an important government interest, and leave open alternative channels of communication. The city's rule meets these requirements: the types of images displayed are not controlled, just the means of showing them; the rule is narrowly tailored because does not regulate substantially more speech than is necessary to further an important government interest (here, preventing eye damage); and alternative channels of communication are available because the artist can use the city's equipment, albeit with less spectacular results. (A) is incorrect because while the artist's art is protected by the First Amendment, it may still be regulated by reasonable time, place, and manner regulations, as indicated above. (B) is incorrect because it states the wrong standard. To be valid, a time, place, and manner regulation need not be the least restrictive means for achieving the desired result, but rather only narrowly tailored to the result. [See Ward v. Rock Against Racism (1989)] (C) is incorrect because art, including performance art such as the laser light show, protected by the First Amendment. As discussed above, the First Amendment guarantee of freedom of speech protects more than merely spoken or written words; it includes conduct and other forms of expression undertaken to communicate an idea.

Question 10-Accomplice's Confession- Advanced The defendant and an accomplice were on trial together for burglary. Both had given confessions implicating themselves and their accomplice. At trial, the defendant maintained that his confession had been obtained through improper coercion by the police. For the purpose of countering the claim of coercion, the prosecution seeks to place the accomplice's confession into evidence. After objection by the defendant's counsel, the judge agrees to issue a limiting instruction to the jury that the confession is to be considered only with regard to the question of whether the defendant's confession was coerced. May the accomplice's confession be admitted under that condition? A No, because admission of the confession violates the defendant's right of confrontation. B No, unless the accomplice takes the stand and subjects himself to cross-examination regarding the confession. C Yes, as long as all portions of the confession referring to the defendant can be eliminated. D Yes, because the judge's instruction limits consideration of the confession only to the issue of coercion.

(D) The confession is admissible with the judge's limiting instruction. Where two persons are tried together and one has given a confession implicating the other, the general rule is that the Sixth Amendment right to confront adverse witnesses prohibits the use of such a statement. This problem arises because of the inability of the nonconfessing defendant to compel the confessing co-defendant to take the stand for cross-examination at their joint trial. As exceptions to the general rule, the statement may be admitted if: (i) all portions of the statement referring to the other defendant can be eliminated (so that there is no indication of that defendant's involvement); (ii) the confessing defendant takes the stand and subjects himself to cross-examination with respect to the truth or falsity of what the statement asserts; or (iii) the confession of the nontestifying co-defendant is being used to rebut the defendant's claim that his confession was obtained coercively, in which case the jury must be instructed as to the purpose of the admission. The accomplice's confession, which the prosecution seeks to introduce into evidence, implicates the defendant in the commission of the crimes charged. Consequently, introduction of this confession raises a problem based on the right of confrontation. However, given that the judge will issue the limiting instruction, the confession is admissible. (D) is therefore correct and (A) is incorrect. (B) and (C) are incorrect because neither of those conditions is necessary for the confession to be admitted as long as the judge issues a limiting instruction, as discussed above.

Question 6- Free Exercise - Intro Citing traffic concerns, a city passed an ordinance prohibiting residents from holding meetings in their homes with more than 15 persons unrelated to the resident. A resident of the city who belonged to a religious group with 50 members allowed the group to hold weekly services at his home. Between 30 and 40 people attended meetings at the resident's home each week. A neighbor of the resident who was upset by the number of cars parked in front of her home on the day of the services contacted the police. Police officers investigated and the resident admitted that he was holding services in his home in violation of the ordinance. When he refused the officers' requests to cease holding such services, they issued him a citation. At his trial on the citation, how should the court rule on the resident's claim that application of the ordinance to him violates his right to the free exercise of religion? A The ordinance is unconstitutional as applied to the resident because it does not include a religious exemption. B The ordinance is unconstitutional as applied to the resident because traffic concerns are not compelling interests that justify an interference with religion. C The statute is constitutional as applied to the resident because the constitution does not protect religious practices, only religious beliefs. D The statute is constitutional as applied to the resident because the burden on his religion was incidental.

(D) The court should rule that the statute is constitutional as applied to the resident because the burden on his religion was incidental. The statute here raises a free exercise issue. The Free Exercise Clause provides that government shall not interfere with the free exercise of religion. However, the Supreme Court currently interprets this prohibition narrowly. States are permitted to adopt regulations that incidentally interfere with religious practice or belief. The Clause only prevents the states from targeting religious practice. Because the ordinance here applies to all meetings (not just religious meetings), and nothing in the facts suggests that the ordinance was adopted to prevent the resident or anyone else from holding a religious meeting in his home, the ordinance is valid. (A) is incorrect because the Supreme Court has held that laws of general application need not include an exemption for religious conduct. (B) is incorrect because it is based on the former test for free exercise questions - formerly, government action that interfered with religious practice was said to be justified only if it was necessary to achieve a compelling interest. As discussed above, that is no longer the test. (C) is incorrect because it is not true. Religious practices are protected, but only to the extent that government cannot prohibit conduct just because it is religious [See, e.g., Church of the Babalu Aye, Inc. v Hialeah (1993)- city could not outlaw religious slaughter of chickens while allowing similar nonreligious slaughter].

Question 8 - Jury Instructions and Burden of Proof- Advanced A defendant is charged with larceny. His principal defense was that he had no intent to permanently deprive the victim of her property. The judge instructs the jury that the State had to prove beyond a reasonable doubt that the defendant was guilty of larceny and that the evidence tended to show that the defendant had taken some belonging to the victim; but if they believed that the defendant had proven by a fair preponderance of the evidence that he did not intend to keep the but to return them, they should find him not guilty. The defendant was convicted of larceny. He appeals the conviction, contending that the judge erred in his instructions to the jury. Is the defendant's conviction likely to be reversed? A No, because the jury has the power to ignore the defendant's testimony if they do not believe him. B No, because the defendant had failed to rebut the State's evidence tending to show that he intended to keep the jewels. C Yes, because the judge cannot comment at all on the evidence. D Yes, because the instructions put some of the burden of proof on the defendant.

(D) The defendant's conviction will likely be reversed because the jury instructions put some of the burden of proof on the defendant. The prosecutor is required by the Due Process Clause to prove each and every element of a crime beyond a reasonable doubt. One of the elements of larceny is an intent to permanently deprive a victim of his or her property, and the instructions in this case put the burden of proof on the defendant to show that there was no such intent. This relieves the prosecution of its burden because it implies that there is a presumption that the defendant intended to permanently deprive her of her jewels. (A) is wrong. The jury does have the power to ignore the defendant's testimony, but the conviction will be reversed because of the unconstitutional instruction on burden of proof. (B) is wrong. The defendant is not required to rebut the state's evidence; the state must prove each element beyond a reasonable doubt. (C) is not the best answer. While it is true that in some jurisdictions the judge cannot comment on the evidence in a criminal case, that prohibition would not be violated by an appropriate instruction on burden of proof.

Question 9- Intro A state's highway speed limits were 65 miles per hour in its flat land regions and 55 miles per hour in its mountainous regions. To reduce traffic fatalities and combat the fact that most of the vehicles on state highways were exceeding posted speed limits, the state legislature proposed banning the use of radar detectors. Citizens in the mountainous regions of the state, where most of the state's highway fatalities occurred, generally supported the ban, but citizens in the flat regions of the state opposed the ban, so the legislature adopted a law banning use of radar detectors on any road with a speed limit below 60 miles per hour. A driver whose car was equipped with a radar detector lived in the mountainous region of the state but frequently drove to the state's flat region. While on a mountain highway with a posted speed limit of 55 miles per hour, the driver was pulled over by a state trooper for speeding. While approaching the driver's car, the state trooper noticed that the driver's radar detector was turned on. The trooper ticketed the driver for both speeding and illegal use of a radar detector. The driver challenges his ticket for use of the radar detector, arguing that it is unfair to allow people in the flat lands to use radar detectors while prohibiting residents of the mountainous region from using them. Which of the following statements is correct regarding the burden of proof in such a case? A The state will have to prove that the ban serves a compelling state interest. B The state will have to prove that the ban is rationally related to a legitimate state interest. C The driver will have to prove that the ban does not serve a compelling interest. D The driver will have to prove that the ban is not rationally related to a legitimate state interest.

(D) The driver will have to prove that the ban is not rationally related to a legitimate state interest. Whether the law here is examined under the substantive provisions of the Due Process Clause or the Equal Protection Clause, the analysis is the same: If no fundamental right or suspect or quasi-suspect class is involved, the law will be assessed under the rational basis standard. Under that standard, government action will be upheld unless a challenger can prove that it is not rationally related to a legitimate government interest. Here, the right involved (using a radar detector) is not fundamental, and no suspect or quasi-suspect class is involved. Thus, the regulation will be assessed under the rational basis standard. (A) is incorrect because, as explained above, the compelling interest standard does not apply when neither a fundamental right nor a suspect classification is involved. (B) is incorrect because, although it states the correct standard, it placed the burden of proof on the wrong party. The challenger must prove that the law is irrational; the government need not prove that it is rational. (C) is incorrect because it states the wrong standard (compelling interest). Moreover, if the compelling interest test were applicable, the driver would not have to prove that the standard was met; the burden would be on the government.

Question 16 -Judicial Notice - Intermediate In a criminal battery case brought against the defendant, the prosecutor asked the court to take judicial notice of the fact that a car driven from Chicago to Detroit has to cross state lines. The defense attorney raised no objection, and the judge declared that she was taking judicial notice of the fact as requested by the prosecution. What is the effect of such judicial notice? A To raise an irrebuttable presumption. B To satisfy the prosecutors burden of persuasion on that issue. C To shift the burden of persuasion on that issue to the defendant. D That the judge should instruct the jury that it may, but is not required to, accept the noticed fact as conclusively proven.

(D) The effect of the judge's noticing that a car driven from Chicago to Detroit must cross state lines is that the judge will now instruct the jury that it may, but is not required to, accept that fact as conclusively proven. Under the Federal Rules, in a civil case, the court must instruct the jury to accept the judicially noticed fact as conclusive. [Fed R Evid 201 (g)] Because this question deals with a prosecution for criminal battery, the applicable rule is that the jury be instructed that the fact that has been judicially noticed may be accepted by it as conclusive, but that the jury is not required to do so. (A) would be correct if this were a civil case. In such an instance, the jury would be instructed to accept as conclusive the judicially noticed fact. This would have the effect of raising an irrebuttable presumption. (B) is incorrect because, in a criminal case, the prosecution has the burden of proving every element of the crime beyond a reasonable doubt. Only the jury can decide, after all of the evidence is in, whether the burden of persuasion is satisfied. (C) is incorrect because the burden of persuasion does not shift from party to party during the course of the trial. The burden of persuasion is never on a criminal defendant.

Question 6 -Prior Inconsistent Statement-Intro A woman was injured when the car she was driving was struck by a moving truck. The woman brings an action for personal injuries against the moving company. The complaint alleges that the driver was drunk at the time of the accident and that the moving company was negligent in hiring him and permitting him to drive knowing that he had a drinking problem and convictions for drunk driving. The driver is called as a witness by the moving company and is expected to testify that he was not drunk at the time of the accident. Instead, the driver states on direct examination that he had had several beers as he drove his truck that evening and was under the influence of drugs when his truck struck the woman's car. The counsel for the moving company wants to confront the driver with his deposition testimony that he was completely sober at the time of the accident. Will this evidence be permitted? A No, the statement is hearsay not within any recognized exception. B No, the moving company cannot impeach its own witness. C Yes, but it may be used only to impeach the driver. D Yes, it can be used to impeach the driver and as substantive evidence that the driver was sober.

(D) The evidence will be permitted to impeach and as substantive evidence. For the purpose of impeaching the credibility of a witness, a party may show that the witness has, on another occasion, made statements that are inconsistent with some material part of his present testimony. Furthermore, a prior inconsistent statement made while under oath and subject to penalty of perjury in a deposition or prior hearing is not hearsay under the Federal Rules. (A) is therefore incorrect. (B) is incorrect because the Federal Rules permit a party to impeach its own witness, even if not "surprised." (C) is incorrect because it is too narrow.

Question 11 -Lemon Test- Intro A state adopted a statute requiring all public school students to recite the Pledge of Allegiance. A parent of a public school student objected to the requirement because the Pledge includes the words, "under God." The parent brought suit in state court, seeking an injunction based on an alleged violation of the Establishment Clause of the First Amendment, as applied to the states through the Due Process Clause of the Fourteenth Amendment. In deciding the substance of the parent's claim, which of the following factors would be most relevant? A Whether the statute constitutes a law of general application. B Whether the student involved sincerely believes that reciting the Pledge is against the tenets of his faith. C Whether requiring students to recite the Pledge is narrowly tailored to promote a compelling interest. D Whether requiring students to recite the Pledge has a secular purpose, has a primary effect that neither advances nor inhibits religion, and does not produce excessive entanglement between government and religion.

(D) The factors that comprise the three-part test set forth in the Lemon case are most relevant. The Establishment Clause restricts government action respecting the establishment of religion. If government takes an action that tends toward the establishment of religion and the action does not prefer one religious sect over another, the determination of whether the action is valid generally is made under the Lemon test. The action will be upheld only if it has a secular purpose, its primary effect neither advances nor inhibits religion, and it does not produce excessive government entanglement with religion. (A) and (B) are incorrect because they each involve factors affecting the validity of government action under the Free Exercise Clause. Under the Free Exercise Clause, government action that incidentally burdens religious practices (i.e., laws of general application) are upheld despite the interference with religious practice. Also, under the Free Exercise Clause, courts may question the sincerity of a litigant's beliefs, but not the truth of his beliefs. (C) is incorrect because it gives the factors that are used to assess an Establishment Clause claim involving a sect preference, and there is no sect preference here.

Question 12- Inadmissible Hearsay - Intro The defendant was on trial for trying to sell a stolen antique ring to an antique dealer. The defendant claims that she had bought the ring from a street vendor earlier on the day of her arrest. The prosecution calls to the stand a woman who had given a party in her home which the defendant attended, and who had discovered later that evening that her antique ring was missing from her bedroom. She seeks to testify that her friend, who is now backpacking in another country, told her three days after the party that she had seen the defendant the previous evening in a restaurant wearing a ring that looked exactly like the woman's ring, The defense attorney objects. Should the objection be overruled? A Yes, because the friend's statement is a present sense impression. B Yes, because the friend's statement is relevant and the friend is unavailable, having gone backpacking in another country. C No, because this is circumstantial evidence within circumstantial evidence. D No, because the friend's statement is hearsay not within any exception.

(D) The friend's comment is inadmissible hearsay. It is hearsay because it is an out-of-court statement offered for its truth (that the defendant wore the woman's ring to a party), and it does not fall within a recognized exception to the hearsay rule. (A) is incorrect under the present sense impression exception to the hearsay rule, a declarant's statement about an event is admissible if made while perceiving the event or immediately thereafter. Here the friend was telling the woman about something she had seen the previous evening, so the timing requirement is not met. (B) is incorrect because, even if the statement is relevant, it is still hearsay, and the friend's unavailability does not bring it within an exception to the hearsay rule. (C) does not state a reason for excluding the evidence; circumstantial evidence is admissible

Question 7-Competency -Advanced A defendant charged with driving while intoxicated pleaded not guilty and insisted on a trial. Right before the trial began, he fired his attorney and decided to defend himself. At one point during opening arguments, the defendant began to act like a cat, meowing and chasing an imaginary squirrel out of the courtroom. If no one else raises the issue of the defendant's competency to stand trial, what is the responsibility of the trial judge here? A The trial judge has no responsibility, because the defendant decided to defend himself. B The trial judge has no responsibility, because she cannot decide whether the defendant is competent to stand trial. C The trial judge must raise the issue of competency, because the defendant is representing himself. D The trial judge must raise the issue of competency, because the Constitution obligates her to do.

(D) The judge must raise the issue of competency. If it appears to the judge that the defendant might be incompetent, the judge has a constitutional obligation to conduct further inquiry and determine whether in fact the defendant is incompetent. If the defendant is tried and convicted but it later appears that he was incompetent to stand trial, the judge's failure to raise the issue or to request a determination of competency does not constitute a waiver of the competency issue. [Pate v Robinson (1966)]. Therefore, if the trial judge observes the defendant acting in such a way that may indicate he is incompetent to stand trial (e.g., meowing, chasing imaginary squirrels), she should conduct further inquiry to determine the competency of the defendant. (A) is incorrect. As explained above, the trial judge has a duty to conduct an independent inquiry if there is evidence that the defendant may not be competent to stand trial. This would be true even if the defendant were represented by counsel. (B) is incorrect. As explained above, the judge can and should decide the issue of competency if there is evidence that the defendant might not be competent to stand trial. (C) is incorrect. If there is evidence that the defendant might not be competent to stand trial, the judge should conduct an independent inquiry into competency regardless of whether the defendant is representing himself or represented by an attorney.

Question 5-Intro The defendant is charged with the murder of his accountant. At trial, the prosecution calls as a witness the defendant's business partner, who testifies that he overheard a telephone conversation between the defendant and the accountant, in which the accountant threatened to report him to the IRS for cheating on his income tax return. The defendant's attorney objects to the business partner's testimony on the ground that it is hearsay. What should the judge do? A Sustain the objection. B Overrule the objection because the testimony, although hearsay, is being introduced to show the defendants state of mind, a hearsay exception. C Overrule the objection because the testimony, although hearsay, qualifies as a business record since the call was made during business hours on a business phone. D Overrule the objection because the evidence is offered to help prove the defendants motive.

(D) The judge should overrule the objection. Out-of-court statements offered for the purpose of showing the statement's effect on the listener or reader are not hearsay. In this case, the statement is being offered to establish that the defendant had a motive to kill the accountant; thus, it is not hearsay. Therefore, (B) is incorrect. The state of mind exception to the hearsay rule applies when the out-of-court statement helps establish the declarant's state of mind, not the listeners state of mind. (C) is incorrect. The business records exception to the hearsay rule applies to certain writings or records made as a memorandum or record of some act, transaction, occurrence, or event. A phone conversation is not a record or writing. (A) is incorrect because the statement is being offered to show motive, not for its substantive truth, and therefore does not qualify as hearsay.

Question 21- Separation of Powers- Advanced Which of the following clauses of the Constitution would most likely permit Congress to impose on the states a uniform child custody law? A The Commerce Clause. B The Police Power Clause. C The Privileges and Immunities Clause of Article IV. D The Taxing and Spending Power Clause.

(D) The most likely method the United States could use to impose a uniform child custody law on all the states through the taxing and spending power, making an allocation of funds available to each state that adopts the uniform law. No other selection would pass constitutional muster. The Commerce Clause has not been applied to child custody matters, and it is unlikely that Congress could use its commerce powers to regulate in this area under the Commerce Clause, Congress can regulate channels of interstate commerce, instrumentalities of interstate commerce, and economic or commercial activities that in aggregate have a substantial economic effect on interstate commerce. It is doubtful that regulations of noneconomic activities, such as child custody laws, can be grounded in the Commerce Clause. Thus, (A) is not the most likely basis for such a statute. (B) is wrong; there is no general federal police power. (C) is wrong because the Privileges and Immunities Clause of Article IV does not apply, since this is an action of the federal government

Question 11-Dying Declaration - Intro A victim was struck by a car in a hit-and-run accident. A police officer arrived half an hour after the accident. The victim was in shock and came in and out of consciousness. As the officer applied first aid, the victim muttered, "l know I'm going to die. Oh my, he ran the light!" The victim fell back into unconsciousness, but revived again and muttered, "Why didn't he stop?" The officer heard the comments clearly and made a note of them. Good police work by the officer and others led to the discovery of the driver of the car that struck the victim. The victim survived and filed a tort action against the driver. Before the case came to trial, the victim died of a heart attack. The causes of the heart attack were totally unrelated to the accident. The laws of the jurisdiction allow for survival of personal injury actions. Thus, the victim's estate is substituted for the victim as plaintiff. If the plaintiff's attorney seeks to have the officer testify to the victim's statements at the time of the accident, how will the court rule? A Inadmissible, because the victim did not die as a result of the accident. B Inadmissible, because this is a civil case and not a criminal matter. C Admissible, because the victim's statements were present sense impressions. D Admissible, because the statements were made at a time when the victim feared impending death

(D) The officer's testimony as to the victim's statements is admissible because the statements were made when the victim feared impending death and so they qualify under the dying declaration exception to the hearsay rule. Hearsay is a statement, other than one made by the declarant while testifying at the trial or hearing, offered in evidence to prove the truth of the matter asserted. [Fed R Evid 801 (c)] Upon appropriate objection, a hearsay statement to which no exception is applicable must be excluded under the dying declaration exception to the hearsay rule, a statement made by a now-unavailable declarant while believing her death was imminent that concerns the cause or circumstances of what she believed to be her impending death is admissible. [Fed R Evid 804(b)(2)] The declarant need not actually die as a result of the circumstances giving rise to her belief of imminent death. Here, testimony as to the victim's statements would be hearsay, because they are out-of-court declarations offered for the truth of the matter asserted; i.e., that the driver of the car that hit her ran a red light. However, these statements related to the circumstances of what the victim believed to be her impending death and the victim (who is now unavailable due to her death) made these statements under a fear of imminent death, as indicated by her condition and her statement "I know I'm going to die." Consequently, all of the elements of the dying declaration exception are present, and the officer's testimony as to the statements is admissible. (A) is incorrect because the declarant need not actually die as a result of the incident that gives rise to the statements. Indeed, the declarant need not die at all. All that is required is that the declarant be unavailable at the time the statements are offered. (B) is incorrect because it reflects the traditional view, which limited the admissibility of dying declarations to homicide cases, rather than the position of the Federal Rules, which allow such declarations in both civil cases and homicide prosecutions. (C) is incorrect because the statements do not qualify under the present sense impression exception to the hearsay rule statement that describes or explains an event or condition, and is made while or immediately after the declarant perceives the event or condition. Here, the victim's statements were made at least one half-hour after the accident. This time lapse between the accident and the statements means that such statements were not made either at the time the victim received a sense impression or immediately thereafter; thus, the present sense impression exception is inapplicable to these facts.

Question 1 - Supremacy Clause -Intro A city's airport board that oversees a large international airport in the United States has adopted a policy of reviewing current vendor licensees every three years. During the process, the board reviews customer comments, assesses the utility of the vendor's services, and reviews the profitability of the vendor (licensees pay rent based on a percentage of their gross profit). The licensee is entitled to present evidence on all of the issues. The owner of a franchised currency exchange on wheels had a license to operate the cart within the airport until last week, when his license came up for review. After an appropriate hearing, the board refused to renew the owner's license mainly because of an excessive number of customer complaints. Nevertheless, the owner continued operating his cart pursuant to a license granted to him under the Federal Borders Act that allowed him to operate his cart at all borders or their functional equivalent. A city police officer patrolling the airport asked to see the owner's license. The owner showed his federal license, but the officer issued the owner a citation because he did not have a city license. Which of the following is the owner's best constitutional defense? A The city deprived the owner of his license without due process of law. B The licensing scheme was arbitrary and capricious. C The licensing scheme substantially interferes with interstate and foreign commerce. D The license requirement here was superseded by the owner's federal license.

(D) The owner's best defense is that the license requirement was superseded by the owner's federal license. The Supremacy Clause makes federal law the supreme law of the land. This means that whenever a valid federal law conflicts with a state law, the state law is inapplicable and the federal law controls. The federal law granting the owner a license here would be valid pursuant to Congress's power to regulate interstate and foreign commerce. In addition to actually regulating such commerce, Congress can adopt any law necessary or proper to implement its power, and providing licenses to vendors to ensure that people can exchange money at the border and its functional equivalents would certainly be within the scope of the broad federal power. Because the city license requirement interferes with the federal licensing scheme, it cannot be enforced against the owner. < (A) is incorrect because the owner appears to have had all the process that was due. It may be that the owner had a property interest in his license under the Fourteenth Amendment Due Process Clause in that he could expect to keep the license as long as he performed well. He could not be deprived of his property interest in his license without due process of law, including a hearing. Here, he was given a hearing prior to revocation of his license at which he was allowed to present relevant evidence, and nothing in the facts indicates that the board was biased against the owner. Thus, due process was satisfied. (B) is incorrect because nothing in the facts indicates that the licensing scheme was arbitrary. An arbitrary licensing scheme would violate substantive due process, which requires at a minimum that laws be rational. Note also that this choice is something of a red herring - the "arbitrary and capricious" standard is the standard that courts use in reviewing determinations of fact by administrative agencies that are not "on the record." (C) is incorrect because nothing in the facts indicates that the city's licensing scheme interferes with foreign or interstate commerce - nothing in the scheme favors local economic interests over out-of-state or foreign interests, so there is no unlawful discrimination, and the scheme appears to place little if any burden on interstate or foreign commerce.

Question 14- Deadly Force/Self Defense- Intro A carjacker pulled a gun on a motorist at a stoplight, intending to force her out of her car so he could steal it. The motorist, however, was carrying a gun and shot the carjacker, killing him. The motorist was charged with violation of a concealed weapons statute, a felony in the jurisdiction, and with common law murder. Assuming that the motorist has committed a felony with regard to the gun, will she be found guilty of murder? A Yes, because one cannot use deadly force in defense of property. B Yes, because the defendant committed a felony by carrying the handgun. C No, because one may use deadly force in defense of property. D No, because she was defending herself against an imminent threat of deadly force.

(D) The motorist will not be guilty of murder because she acted in self-defense. At common law, murder is the unlawful killing of another human being with malice aforethought. Malice aforethought exists if the defendant: (i) intends to kill; (ii) intends to inflict great bodily injury; (iii) acts in reckless indifference to an unjustifiably high risk to human life; or (iv) intends to commit a felony. A person may use deadly force in self-defense if (i) she is without fault; (ii) she is confronted with unlawful force; and (iii) she is threatened with imminent death or great bodily injury. The majority rule is that there is no duty to retreat. Here, all three prongs for a valid self-defense claim are present. The defendant was without fault in creating the carjacking, and she was obviously confronted with unlawful force, in that the carjacker was pointing a gun at her. Such threatened force clearly is also "deadly" and "imminent." Thus, the defendant's killing was not "unlawful" as required by the definition for common law murder. (A) is incorrect because, although the majority rule is that a person may not use deadly force in defense of personal property, a person may use deadly force in self- defense. Here, the defendant was justified in using deadly force in self-defense, as explained above. (B) is also incorrect. The felony murder rule historically has been used to impute malice to the victim. As such, courts have imposed some limitations on the felony murder rule. Some courts will limit the felonies to "inherently dangerous" felonies, while others will require the felony to be dangerous as committed. Here, the felony of carrying a concealed handgun is not an inherently dangerous felony, in that the felony could be carried out without the resort to violence. Furthermore, there is no indication from the facts that the defendant used the concealed handgun in a dangerous manner. As a result, most courts probably would not convict the defendant of felony murder absent some unlawful misuse of the gun. This is especially true given that the defendant was acting in lawful self-defense. (C) is an incorrect statement of the law; deadly force generally may not be used in defense of property.

Question 5- Past Recollection Recorded admitting the writing- Intro A 54-year-old employee filed an employment age discrimination suit against a corporation, alleging that its personnel director had improperly terminated his employment. In defense, the corporation presents a written report summarizing a meeting between the personnel director and the employee that was prepared directly after the meeting and placed in the employee's personnel file, and that contains several damaging admissions by the employee. Which of the following is NOT among the foundational facts the corporation will have to establish if it wants to have the report admitted under the past recollection recorded exception to the hearsay rule? A That the report was written while the meeting was fresh in the memory of the personnel director. B That the report accurately records what was said by the employee. C That the report was written by the personnel director or adopted by her. D That the personnel director is not available and cannot be called as a witness at trial.

(D) The past recollection recorded exception to the hearsay rule is one of a class of hearsay exceptions that do not require unavailability of the declarant. If a witness has insufficient memory of an event to enable her to testify fully and accurately, even after consulting a writing given to her on the stand, the writing itself may be introduced into evidence if a proper foundation is laid for its admissibility. The foundation for receiving such a writing into evidence must include proof that: (i) the witness at one time had personal knowledge of the facts recited in the writing; (ii) the writing was made by or under the direction of the witness or has been adopted by her; (iii) the writing was timely made when the matter was fresh in the mind of the witness; (iv) the writing is accurate; and (v) the witness has insufficient recollection to testify fully and accurately. [Fed R Evid 803(5)] The corporation need not establish the unavailability of the personnel director. (A), (B), and (C) each correspond to one of the foundational requirements. (A) corresponds to requirement (iii), (B) corresponds to requirement (iv), and (C) corresponds to requirement (ii).Because (A), (B), and (C) reflect foundational facts that the corporation will have to establish to have the report admitted under the past recollection recorded exception, they are incorrect.

Question 7 -Press and Publication - Intro An author who was about to release a book on military censorship in warzones was by a newspaper Reporter. During the interview, the author spoke generally of military censorship in warzones, but would not give the reporter any specific instances of censorship that he intended to include in his book. During the interview, the author received a phone call and excused himself. While the author was out of the room, the reporter found a pre-publication copy of the author's book and quickly took photographs of several pages. The reporter later printed the pictures and published an article that included the photographed sections of the book verbatim. The author brought an appropriate action against the reporter for copyright infringement. The reporter defended on First Amendment grounds. Assume the court properly concludes that any fair use exception in the copyright laws is not applicable. Is the reporter's defense likely to be successful? A Yes, because the book included matters of public concern, and so the reporter had a right under the First Amendment to publish it. B Yes, because newspapers have an absolute right under the First Amendment to print whatever information they receive. C No, because the author has a Fifth Amendment property right in his book. D No, because the newspaper did not get the author's permission to print the pages from his book.

(D) The reporters First Amendment defense is not likely to succeed because the newspaper did not get his permission. The author had a property right in his manuscript that can be protected by the copyright laws regardless of the public importance of the content. The best way to answer this Question is to eliminate the wrong choices. (A) is wrong because there is no First Amendment exception to copyright laws. It does not matter that the author may have been a public figure or that his book discussed a matter of public concern; magazines have no right to publish copyrighted material without permission beyond the statutory fair use exception, not the case here. [Harper & Row Publishers v. Nation Enterprises (1985)] (B) is wrong because it is too broad; the press generally has no greater freedom to speak than does the public. (C) is wrong because the Fifth Amendment is not applicable here. The Fifth Amendment prohibits government from taking private property without due process or just compensation, and here a private party has acted. Therefore, (D) is correct; the author will prevail in an appropriate action for copyright infringement against the reporter because the reporter published portions of the authors work without permission.

Question 1 -Religious Beliefs- Intro Pursuant to an edict recently issued by the elders of their religion, a mother and father instructed their son who just turned age 14 to report to a community woodworking shop instead of school. A state law requires all children to attend school until the age of 16, and the woodshop does not qualify as a school under state law. Because the parents did not report their son's absence, a truant officer visited the family and warned them that parents who willfully refuse to comply with the mandatory attendance law are subject to a $500 fine and up to 30 days in jail for each day of noncompliance. The parents listened, but informed the officer that they could not comply with the state law because of their religious views. The following day, the 14-year-old again went to work in the community woodshop instead of to school. His parents were then arrested and charged with violating the state mandatory school attendance law. At the parents' criminal trial, which of the following may the court constitutionally consider in determining guilt or innocence? A Whether the tenets of the parents' religion are true. B Whether the parents' religion is a traditional, established one. C Whether the parents believe that the tenets of their religion are derived from a supreme being. D Whether the parents sincerely believe the tenets of their religion.

(D) The sincerity of the parents' religious beliefs is a factor that can be inquired into as a way of determining whether they can avail themselves of the protection of the Free Exercise Clause. The Free Exercise Clause of the First Amendment, applicable to the states through the Fourteenth Amendment, prohibits punishing people for their religious beliefs. When a person claims that he is being punished for his religious beliefs, the court may consider whether the person challenging the law sincerely holds those beliefs. Thus, the court may consider whether the parent' beliefs are sincerely held. (A) is incorrect because the First Amendment forbids a court from determining whether a person's religious beliefs are true. A court must respect a sincerely held religious belief, even if it appears to be illogical or incapable of proof. (B) is incorrect because the Free Exercise Clause protects all sincerely held religious beliefs, regardless of whether a specific religion is deemed to be "established" or "traditional." (C) is incorrect because religious beliefs need not be theistic to qualify for constitutional protection. An asserted religious belief must occupy a place in the believer's life parallel to that occupied by orthodox religious beliefs. Even an internally derived belief is entitled to protection.

Question 4- 10th Amendment - Intro To encourage economic development in areas of a state currently underdeveloped, state legislation created an agency to build retail buildings in those parts of the state and then to sell the stores to buyers at the cost of materials and labor. The federal government has enacted legislation in effect for the relevant period that regulates the manner in which building contractors conduct business. Is the state agency subject to these federal regulations? A No, because its activities take place entirely within the state. B No, because as an agency of state government, it is immune from federal regulation. C No, because the federal government is not empowered to enact legislation regulating state governments or their agencies. D Yes

(D) The state agency is subject to federal regulation. In theory, the Tenth Amendment prohibits federal regulation of state activities if the regulation would virtually eliminate the state's local functions. As a practical matter, the Court's holding in Garcia v. San Antonio Metropolitan Transit Authority (1985) gives Congress broad authority when it acts pursuant to its enumerated powers. Accordingly, a federal regulation that controls the states as well as private persons is valid. Thus, the state agency would have to follow the federal regulations which control due to the Supremacy Clause. Thus, (D) is correct, and (B) and (C) are wrong. (A) is wrong because even intrastate activities are subject to federal Commerce Clause regulation if they substantially affect interstate commerce.

Question 17 -Hearsay Vicarious Party Admission - Intermediate The plaintiff was injured when her car was hit head-on by a pickup truck belonging to a landscaping company. The plaintiff brought an action against the truck driver and the landscaping company, alleging negligence by the truck driver in driving too fast. Counsel for the plaintiff seeks to introduce the statement of a former employee of the landscaping company, who said to the truck driver just before the accident, "You had better slow down. You've been warned by the boss many times not to go this fast." Which of the following would cause the judge to rule the statement admissible? A The statement was made immediately after the accident, and was made under oath. B The former employee is unavailable to testify to the matters in the statement. C Plaintiff's attorney produces a record of three citations that the truck driver received in the last year for speeding, driving the same truck. D Plaintiff's attorney first proves as a preliminary question of fact that during the trip the former employee was an agent of the landscaping company and that the statement concerned the scope of his employment.

(D) The statement is admissible only if is determined that the former employee was an agent of the landscaping company and that his statement concerned a matter within the scope of his employment. Under Federal Rule 801 an out-of- court statement of an agent or an employee can be introduced as a statement attributable to the principal or employer if it concerns a matter within the scope of the agency or employment, made during the existence of the relationship. Given the establishment of that preliminary fact, the statement here constitutes a statement by a party-opponent (i.e., a vicarious admission) and is not hearsay. (A) and (C) state requirements for admissibility that have no basis in law. Admissibility of a vicarious admission is not conditioned on the availability of the declarant; (B) is therefore incorrect.

Question 29- Bolstering- Intermediate The plaintiff sued the defendant in a contract dispute. At trial, the plaintiff's sister testifies as a witness on behalf of the plaintiff, stating that the defendant agreed to sell a computer to the plaintiff for $250. To that the sister is telling the truth, plaintiff's counsel asks the sister on direct examination about a conversation she had with her mother, in which she told her mother that the defendant agreed to sell a computer to the plaintiff for $250. The defendant objects to the testimony. How should the court rule? A Admissible, because it is a prior consistent statement. B Admissible, because it is not hearsay. C Inadmissible, because leading questions cannot be asked on direct examination. D Inadmissible, because the sister has not been impeached.

(D) The testimony is inadmissible. A party cannot bolster the testimony of a witness until the witness has been impeached. Here, plaintiff's counsel is seeking to introduce the sister's prior statement, which is consistent with her in-court testimony, to prove she is telling the truth. The testimony is inadmissible for this purpose because the sister has not been impeached. Therefore, (D) is correct. (A) is incorrect. A prior consistent statement is admissible if offered to rebut a charge that a witness is lying because of some improper motive, or to rehabilitate the credibility of a witness who has been impeached on some other noncharacter ground. Here, the credibility of the sister has not been attacked; therefore, there no justification for bolstering her credibility. (B) is incorrect. While the statement would not be hearsay if it is not being offered to prove the truth of the matter asserted, it is nevertheless inadmissible because it is improperly offered to bolster the credibility of the witness. (C) is incorrect. There is no indication that plaintiff's counsel asked any leading questions. Furthermore, leading questions are sometimes permitted on direct examination (e.g., when the witness is hostile).

Question 18- Double Jeopardy-Impeachment/Criminal - Intermediate A federal district court judge was accused of misconduct in office and was impeached by the House of Representatives. At trial in the United States Senate, the judge was convicted and removed from office. Nevertheless, the President directed the Attorney General to institute criminal proceedings against the judge. After presentation to a federal grand jury, an indictment was issued against the judge and signed by the Attorney General. At the opening of his trial, the judge moved to have the indictment dismissed. How is the trial judge most likely to rule? A Dismiss, because the President had told the Attorney General to prosecute. B Dismiss, because the criminal proceeding violates the Fifth Amendment's proscription against double jeopardy. C Deny the dismissal, because the federal grand jury issued the indictment. D Deny the dismissal, because the judge has not been previously tried in a criminal proceeding.

(D) The trial judge will most likely deny the motion to dismiss the indictment. The Fifth Amendment right to be free of double jeopardy for the same offense applies to subsequent criminal actions, but not to civil actions or impeachment proceedings, which are distinct from criminal proceedings. Article l, Section 3 of the Constitution specifically states that a conviction by impeachment does not prevent the party convicted from being subject to indictment, trial, judgment, and punishment according to the law. Hence, (B) is incorrect. (A) is incorrect because there is nothing in the facts to show that the Attorney General was not acting within his prosecutorial discretion even if he was complying with the wishes of the President. (C) is incorrect because the fact that the grand jury issued the indictment is irrelevant. If double jeopardy did apply or if the Attorney General had abandoned his prosecutorial discretion in instituting criminal proceedings, the fact that the grand jury issued an indictment would not prevent the indictment from being dismissed.

Question 3 - Intro To provide jobs for its citizens, stimulate future tourism, and help the environment, a state legislature enacted a statute authorizing the state's department of parks and recreation to hire up to 5,000 persons to plant trees on land in the state that has been denuded of trees by overlogging. Among other things, the statute provides that resident aliens may be employed only if no United States citizens are available to fill the necessary positions. In a challenge to the constitutionality of that provision by a plaintiff with standing to raise the claim, which of the following constitutional provisions would be most helpful to the plaintiff? A The Privileges or Immunities Clause of the Fourteenth Amendment. B The reserved powers of the state under the Tenth Amendment. C The Equal Protection Clause of the Fourteenth Amendment. D The Fourteenth Amendment Due Process Clause.

(D) might provide a viable argument, as a statute that affects a fundamental right can be struck down under the Due Process Clause under the same strict scrutiny test as set out above under the Equal Protection Clause. However, it is a less direct argument than (C) because strict scrutiny applies under the Due Process Clause only if the challenger can show that a fundamental right is involved.(C) The Equal Protection Clause of the Fourteenth Amendment is the most helpful provision. State classifications based on alienage that do not involve alien participation in the self-government process are suspect under the Equal Protection Clause and are subject to strict judicial scrutiny. They will be upheld only if the government can show that the classification is necessary to achieve a compelling state interest. While the state's interests here in providing jobs, stimulating future tourism, and helping the environment may be compelling, it cannot be said that these goals can be achieved only by discriminating against resident aliens. Thus, the legislation would be found unconstitutional under the Equal Protection Clause. (A) is wrong because the Privileges or Immunities Clause of the Fourteenth Amendment protects the privileges and immunities of United States citizens, not aliens. (B) is wrong because, even if applicable, the Tenth Amendment could only help the state (by reserving to the state powers that are not delegated to the federal government); it does not carry any prohibitions.

Question 6- Standing- Intro A state's legislature passed a statute that required every used car sold in the state to be tested prior to sale to determine whether it was in compliance with a set of strict exhaust emission standards that were also included in the legislation used cars would have to be brought up to standard and pass the emissions test prior to sale. Certain persons in the state object to the legislation because one of its results will be to raise the average price of used cars in the state. Only cars to be sold for junk are exempt from the statute. Among the following, who would be most likely to have standing to raise a constitutional challenge to the legislation? A A state resident who was thinking about selling used cars in the state. B A state resident who was thinking about buying a used car in the state. C An out-of-state dealer of used cars who had a contract to sell cars to a large dealer in the state. D An out-of-state manufacturer who might be required to indemnify its dealers in the state for costs arising from the statute.

C is correct. An out-of-state dealer of used cars would be most likely to have standing. Standing requires a concrete stake in the outcome of a controversy which includes an injury in fact. Abstract injury is not enough; the plaintiff must show that he has sustained or is immediately in danger of sustaining a particularized and concrete injury as the result of the challenged official conduct. The injury or threat of injury must be real and immediate, not conjectural or hypothetical. Here, the potential plaintiffs in (A) and (B) are merely thinking about selling or buying used cars in the state. Thus, any injury or threat of injury to them is strictly hypothetical and abstract. Consequently, the potential plaintiffs in (A) and (B) are unlikely to have standing. Likewise, the out-of-state manufacturer in (D), who might be required to indemnify its dealers in the state, is at this point able to assert merely a hypothetical injury, not one that is real and immediate. On the other hand, the potential plaintiff in (C), who already has a contract to sell used cars to a state dealer, is immediately in danger of sustaining a direct economic injury as a result of the statute - i.e., costs associated with testing each car and bringing up to standard those cars found to be deficient

Question 4 - Standing- Intro A pool hall seeking to attract more afternoon business advertised a $2 per hour table rental discount between the hours of 2 pm and 6 pm. The advertisement caught the attention of students who attended a high school in the city where the pool hall was located. Soon the hall was full of students in the afternoon. To discourage city youth from frittering away their afternoons, the city council passed an ordinance imposing a $2 per hour rental tax on pool tables rented before 6 pm on weekdays. Which of the following people or entities would not have standing to bring a lawsuit to challenge the tax? A A civic watchdog and good government group. B The owner of the pool hall. C A citizen of the city who opposes every new tax and who has taken advantage of the discount on several occasions. D A 16-year-old boy who regularly plays pool at the hall after school.

(A) The civic watchdog group is not likely to have standing to challenge the tax. To have standing, a plaintiff must show that it has a concrete stake in the outcome of a controversy that arises out of an injury in fact. On a constitutional issue, the plaintiff must be able to show that it is or will be injured by the government action involved and that its injury can be remedied by a decision in its favor. The injury must be concrete and particularized and not conjectural or hypothetical. The civic watchdog group is not likely to have standing because it has no concrete stake in the outcome. It has not shown that it will be injured by the tax in any more than the general way in which everyone is injured by unlawful governmental conduct, and that is not a sufficient injury to sustain standing. (Note that an organization may have standing to represent its members if there is a sufficient injury to the members that is related to the organization's purpose, and the nature of the claim and the relief sought does not require participation of the members; but since this organization is not limited to pool hall owners or customers, its injury is not sufficient to allow standing). (B) is incorrect. The tax could have its intended consequence of discouraging high school students from shooting pool in the afternoon. If that happens, the owner's revenues would decrease. Thus, he has a concrete stake in the outcome and his injury could be prevented by removing the tax. Therefore, the owner has standing. (C) is incorrect. Anyone who plays pool regularly in the afternoon would be injured by the tax even though its purpose is to keep high school students from shooting pool. Note that the rule that there is no "citizen standing" is not applicable here - while the citizen generally opposes new taxes, this is not a case where he is attempting to challenge the tax as a mere citizen - he has patronized the pool hall in the afternoon and would be directly impacted by the tax. (D) is incorrect for much of the same reason that (C) is incorrect. As a regular patron of the hall, the student would be directly impacted by the tax and eliminating the tax would eliminate any harm caused by the tax. Therefore, the student has standing.

Question 9 - Expert Witness- Intro A contractor for a large multistory building used an excavation subcontractor to dig the excavation for the foundation, and a structural subcontractor to begin structural work on the foundation. Just after the foundation was completed, an employee of the structural subcontractor was killed when the walls of the excavation collapsed. The employee's survivors brought an appropriate action against all of the involved parties. At trial, the structural subcontractor calls a civil engineer licensed by the state to testify that he examined the geologists reports of the soil conditions surrounding the construction site, as well as a report by the investigator who examined the site of the collapse, and that it is his (the engineer's) opinion that the collapse was caused by the excavation subcontractor's failure to take into consideration the composition of the soil being excavated. Is the engineer's testimony admissible? A Yes, if civil engineers in his field rely on such materials as reports by geologists and others in reaching conclusions such as his. B Yes, if he was not professionally negligent in his analysis. C No, because his opinion relates to an ultimate issue that must be determined in the case. D No, because his opinion was based upon facts not personally within his knowledge.

(A) The engineer's testimony is admissible as relevant opinion testimony by an expert witness. The Federal Rules permit witnesses qualified as experts to testify in the form of an opinion if the subject matter is one where scientific, technical, or other specialized knowledge will assist the jury in understanding the evidence or determining a fact in issue. [Fed R Evid 702] Under Federal Rule 703, the expert may base his opinion on facts not known personally but supplied to him outside the courtroom (e.g,, reports of other experts). Such facts need not be admissible in evidence as long as the facts are of a kind reasonably relied on by experts in the particular field. Here, the engineer, who was licensed by the state, probably qualifies as an expert on the subject of his testimony and therefore can state his opinion as to the cause of the collapse of the excavation wall. As choice (A) states, he may base his opinion on the geologist's and the investigators reports, and the engineer's report if civil engineers in his field rely on this type of data in reaching conclusions such as his. Thus, choice (D) is incorrect. Choice (B) is incorrect because whether this analysis constitutes professional negligence is irrelevant to its admissibility; this fact can be brought out by cross- examination. A prudent analysis will still be inadmissible if it was based on materials that experts in his field did not reasonably rely on. Choice (C) is incorrect; Federal Rule 704(a) provides that otherwise admissible opinion testimony is not objectionable because it embraces the ultimate issue to be decided by the trier of fact.

Question 18- Candidate for Public Office - Advanced A woman decided to run for office as a park district trustee in the next general election. A city ordinance provided that all candidates running for the office of park district trustee must present to the city clerk within go days of the election petitions signed by at least 500 qualified voters, a $500 filing fee, and a personal financial statement that will be open to the public for inspection. The woman timely filed the petitions with sufficient signatures, but refused to file the financial statement or pay the filing fee. When the clerk refused to put her name on the ballot, the woman filed suit, claiming that the filing fee and personal financial statement requirements are unconstitutional. Which of the following statements best describes the likely outcome of the woman's suit? A The filing fee requirement will be held invalid if the woman is indigent, but the financial statement requirement will be upheld. B The filing fee requirement will be upheld, but the financial statement requirement will be held invalid because it impairs the woman's right of personal privacy. C Both requirements will be held invalid and the woman will be entitled to have her name placed on the election ballot. D Both requirements will be held invalid unless the requirements are shown to serve some legitimate state purpose.

(A) The filing fee is invalid if the woman is indigent, but the financial statement requirement is valid. A state may not impose on candidates a fee that renders it impossible for indigents to run for office. Even as applied to nonindigent candidates, an unreasonably high filing fee that is not tailored to promote a substantial or overriding state interest might be held invalid. However, even a reasonable and otherwise valid fee would have to be waived for an indigent candidate unable to pay the fee. If the woman is indigent, then a $500 filing fee would certainly preclude her running for office. Other types of restrictions on the ability of persons to be candidates must be examined to see the restrictions violate either the First Amendment right of political association or the Fourteenth Amendment Equal Protection Clause. A ballot access regulation must be a reasonable, nondiscriminatory means of promoting important state interests (such as running an honest, efficient election system). The financial statement requirement here is a reasonable means of promoting the important state interest of disclosing possible conflicts of interest that might compromise the integrity of elected officials. (It also might be a legitimate method for determining whether a candidate is truly indigent, thus justifying waiver of the filing fee.) This requirement is nondiscriminatory because it is applied to all candidates, rather than only some candidates. Any impairment of a candidate's right of privacy resulting from this requirement is slight in comparison to the important governmental interest served thereby. Thus, the financial statement requirement is valid. (B) is incorrect in stating the invalidity of the financial statement requirement. Also, (B) is incorrect because it cannot be said with any certainty that the filing fee requirement is valid. The fee requirement will certainly be invalid as applied to indigent candidates, and it might be so high and lacking in promotion of a substantial state interest that it will be invalid as applied to all candidates. (C) is incorrect because it states that the financial statement requirement is unconstitutional in addition to the filing fee requirement. As noted above, the requirement of a financial statement is valid. (D) is incorrect because the validity of restrictions on the ability of persons to be candidates is determined by a balancing test: A severe restriction such as a filing fee making it impossible for indigents to run for office would require a compelling state purpose to be valid

Question 12 - Equal Protection - Intermediate Congress has recently enacted legislation that makes it a federal crime for any person to interfere with any right conferred by the Equal Protection Clause of the Fourteenth Amendment. The statute may be applied constitutionally in which of the following situations? A A person who hates Asians bribes a federal official so that he fails to distribute free dairy products to otherwise eligible Asians. B A person who believes women are inferior to men persuades the dean of a private school licensed by the state to deny admission to otherwise qualified women because of their sex. C By threats of violence, a person coerces the coach of a public high school's basketball team to exclude white athletes from the team solely because of their race. D A person persuades the members of his church council to deny shelter and food to homosexual men who seek aid at the church-run downtown relief center. (C) The facts of (C) are the only ones in which the person has compelled a state official to deny equal protection of the law to some person. The Fourteenth Amendment prevents states from depriving any person of life, liberty, or property without due process of law and equal protection of the law. Because the Equal Protection Clause protects against state action only, the federal statute at issue prohibits only behavior that causes or induces a state official, agency, or instrumentality to deny the equal protection of the law to some person. In (C), the person coerced a state agent (the coach of a public high school basketball team) to exclude people from participation in an activity at a state institution solely on the basis of race. Because this is a state governmental act that classifies people based on a suspect trait (race) in the absence of a compelling state interest, violates equal protection. For bringing about this violation, the person is guilty of violating the federal statute.

(A) is incorrect because the person there is inducing a federal official to discriminate in the distribution of free dairy products. The Fourteenth Amendment, unlike the Fifteenth Amendment, does not curtail the actions of the federal government. Therefore, the person acting has not interfered with any right conferred by the Fourteenth Amendment Equal Protection Clause. (B) is incorrect because the person there has induced discriminatory action against women by the dean of a private school. The Fourteenth Amendment does not protect against the actions of private persons or institutions. The actions of a private entity may constitute state action if the state is significantly involved in the private entity. However, the mere granting of a license is not sufficient state involvement with a private entity so as to convert its action into state action. Thus, the fact that the private school in (B) is licensed by the state does not make the school's discriminatory treatment of women state action. (D) is incorrect because the person there has induced discriminatory action by his church, which is a private entity with no apparent significant state involvement. As explained above, the Fourteenth Amendment Equal Protection Clause does not protect against the actions of such an entity. Thus, the person has not interfered with a right conferred by the Equal Protection Clause.

Question 9- Self-Incrimination - Intro A bookie testified before a grand jury regarding allegedly illegal gambling activities. As a result, the bookie was indicted and a warrant was issued for the bookie's arrest, along with a search warrant for the bookie's home. The police went to the bookie's home, informed him of the charges against him, and placed him in handcuffs. The officers then conducted a search of the bookie's home and found a desk calendar, which had possibly incriminating information written on it relating to appointments. They seized the desk calendar and one of the officers asked the bookie what he had to say about their find. The bookie made an incriminating statement in response. Before trial, the prosecutor obtained an exemplar of the bookie's handwriting to compare it with the handwriting on the calendar. If introduced at trial, which of the following would most clearly violate the bookie's Fifth Amendment self-incrimination rights? A The grand jury testimony. B The bookie's response to the police officer. C The bookie's handwriting exemplar. D The bookie's desk calendar.

(B) The bookie's response to the police officer is the only evidence that was clearly taken in violation of the bookie' s Fifth Amendment self-incrimination rights. The Fifth Amendment privilege against compelled self-incrimination forms the basis for ruling upon the admissibility of a statement obtained while a defendant is in custody. A person in custody must, prior to interrogation (except for standard booking questions), be clearly informed that: he has the right to remain silent, anything he says can be used against him in court, he has the right to an attorney, and if he cannot afford an attorney, one will be appointed for him if he so desires. These Miranda warnings are a prerequisite to the admissibility of any statement made by the defendant during a custodial interrogation. Here, the bookie was in custody (handcuffed) and was questioned ("what do you have to say . . . ") and the facts do not indicate that Miranda warnings were given. Thus, there has been a direct violation of the Fifth Amendment privilege against compelled self-incrimination. (A) is wrong because use of a defendant's grand jury testimony at trial does not violate the Fifth Amendment. Pursuant to the Fifth Amendment, a criminal defendant may invoke the privilege against self-incrimination by refusing to answer grand jury questions on the grounds that it may incriminate him. If he testifies, he has waived his privilege. Here, the bookie testified at the grand jury proceeding and thus waived the privilege. (C) is wrong because the Fifth Amendment protects only testimonial or communicative evidence, not real or physical evidence. Thus, the state may compel a person to give a handwriting exemplar without violating the Fifth Amendment, even if the evidence may be incriminating. (D) is wrong because the Fifth Amendment protects against being compelled to communicate information, not against disclosure of communications made in the past. Thus, the police may search for and seize documents tending to incriminate a person. Here, the bookie was not compelled to give any testimony. Rather, the police, pursuant to a valid search warrant, seized the calendar with the appointments marked on it.

Question 25- Notice on Listener- Intermediate A plaintiff brought a lawsuit against a department store for injuries she suffered while riding an escalator that accelerated and then came to a sudden stop. At trial, a witness is prepared to testify for the plaintiff that he heard another customer tell the store manager the day before the plaintiff was injured that the escalator was speeding up and stopping without warning. The customer still lives in the area but she has not been called to testify. Should the court admit the witness's testimony? A Yes, because it is a statement against interest. B Yes, because it is relevant evidence that the department store was aware of the defect and did nothing to correct it. C No, because the customer is available to testify. D No, because it is hearsay not within any exception.

(B) The court should find the witness's testimony admissible as relevant evidence that the department store was aware of the defect. Hearsay is a statement, other than one made by the declarant while testifying at the trial or hearing, offered in evidence to prove the truth of the matter asserted. [Fed R Evid 801 (c)] A hearsay statement to which no exception to the hearsay rule is applicable must be excluded upon appropriate objection to its admission. [Fed R Evid 802] A statement that would be inadmissible hearsay may be admissible to show the effect of the statement on the listener or reader. For example, in a negligence case where knowledge of a danger is in issue, a third person's statement of warning is admissible to show notice or knowledge on the part of a listener. Here, the customers remark to the manager is an out-of-court statement. However, the statement can be offered to show that the department store had notice of a possible danger posed by the escalator. (Note that the remark would be inadmissible hearsay if offered to prove the escalator was speeding up and stopping.) (A) is incorrect because it states an exception to the hearsay rule, and the testimony offered is not hearsay (see above). Even if the testimony were offered to prove the escalator was malfunctioning, this exception would not apply because the declarant (the customer) made no statement against her interest. Also, she is available to testify, which takes her statement out of the exception. (C) is incorrect because the witness's testimony is admissible nonhearsay. In addition, the unavailability of a declarant is only significant with regard to certain hearsay exceptions that require unavailability (e.g., the statement against interest). If the witness's testimony is offered to show notice to the department store of a potential problem with the escalator, there is no hearsay problem. The absence of a hearsay problem precludes resort to a hearsay exception, and the availability of the customer is of no significance. (D) is incorrect because the customers statement is not hearsay because it is not offered to prove the truth of the matter asserted (i.e., that the escalator was malfunctioning), but to show notice of the defect.

Question 35- Impeachment- Advanced During the trial of a personal injury case, the plaintiff calls a witness to testify that he saw the defendant spill a slippery substance in the roadway. Following the testimony of the witness, the defendant calls the witness's neighbor, who testifies that the witness has a poor reputation for truthfulness in the community. The plaintiff's attorney then cross-examines the neighbor, asking her, in good faith, if she committed the crime of false pretenses last year. Last year, the neighbor had in fact been charged with and convicted of the crime of false pretenses. The defendant's attorney objects to this question. Should the objection be sustained? A No, because the neighbor was convicted of the crime of false pretenses. B No, because the plaintiff's attorney asked the question in good faith. C Yes, because an impeaching witness cannot be impeached on collateral matters. D Yes, because such an inquiry is not proper on cross-examination.

(B) The question by the plaintiff's attorney should be allowed because he was acting in good faith. A witness may be impeached by means of being interrogated upon cross-examination, in the discretion of the court, with respect to any act of misconduct that is probative of truthfulness (i.e., an act of deceit or lying). The cross-examiner must act in good faith with some reasonable basis for believing that the witness may have committed the bad act inquired about, but it is not required that the witness have been convicted of a crime. Here, the plaintiff's attorney is attempting to cast an adverse reflection on the truthfulness of the neighbor. The commission of the crime of false pretenses involves the making of a false representation and is therefore an act of misconduct that is probative of the actor's truthfulness. Thus, because the plaintiff's attorney inquired as to this matter in good faith, his question is a permissible method of impeachment, and the objection of the defendant's attorney should be overruled. (A) is incorrect because it implies that the objection could be sustained if the neighbor was not convicted of the crime. As noted above, such an inquiry can be conducted regardless of whether the witness was convicted. Therefore, the objection to the plaintiff's attorney's good faith inquiry would be overruled even if the neighbor was not convicted of false pretenses. (C) is incorrect. Although impeaching witnesses who testify to a witness's reputation for truth and veracity are often impeached by asking the "Have you heard" and "Do you know" questions, that is not the only method of impeachment available. Any witness who takes the stand puts her character for honesty and veracity in issue and may be impeached by evidence that might show her to be unworthy of belief Instances of misconduct may properly be inquired into only if they are probative of truthfulness. By taking the stand, the neighbor has put her character for honesty in issue. The crime of false pretenses is probative of truthfulness and is a proper subject for impeachment. (D) is incorrect because a specific act of misconduct offered to attack the witness's character for truthfulness can be elicited only on cross-examination of the witness. Extrinsic evidence is not permitted. Thus, (D) states the opposite of the correct rule.

Question 16 - Admissible Nonhearsay- Intermediate A plaintiff sued a defendant for damages suffered when a load of bricks fell off the defendant's truck directly in front of the plaintiff while she was driving on a highway. The plaintiff charged that the defendant was negligent in supplying his truck with a defective load chain clamp, which helped tie the load to the bed of the truck, and in failing to secure the load properly on the truck. The plaintiff calls a witness who testifies that he was formerly employed as a truck driver and is an acquaintance of the defendant. The witness further testifies that immediately prior to the accident he had coffee with the defendant at a cafe, and mentioned to the defendant that the tie chains holding the load of bricks looked kind of loose. Assuming proper objection by the defendant's attorney, how should the court rule on the admissibility of such testimony? A Admissible under an exception to the hearsay rule. B Admissible nonhearsay. C Inadmissible hearsay. D Inadmissible opinion evidence.

(B) The witness's testimony is admissible nonhearsay. The statement by the witness is not being offered to prove the truth of the matter asserted therein and thus is not hearsay. Hearsay is a statement made out of court by the declarant, offered in evidence to prove the truth of the matter asserted. [Fed R Evid 801 (c)] Although hearsay is inadmissible (unless an exception to the hearsay rule is applicable), a statement that would be inadmissible hearsay to prove the truth thereof may be admitted to show the statement's effect on the listener or reader. Thus, in a negligence case, where knowledge of a danger is at issue, a person's warning statement is admissible for the limited purpose of showing knowledge or notice on the part of a listener. Here, one of the theories of recovery underlying the plaintiff's lawsuit is that the defendant negligently failed to secure the load. Therefore, the plaintiff must show that the defendant either knew or should have known that the load was not properly secured. Consequently, the witness's statement that the chains looked loose is admissible to show that the defendant had notice of the possible danger. If this same out-of-court statement were offered to show that its contents were true (i.e., that the chains were in fact loose), then it would constitute hearsay, but because the statement is offered to show notice to the defendant of a possible danger, it is nonhearsay and (C) is incorrect. (A) is incorrect because the admissibility of the statement arises from its status as nonhearsay. If a statement is nonhearsay, then there is no need to refer to hearsay exceptions in determining the statement's admissibility. (D) is incorrect for two reasons: First, the statement is not being offered to show the witness's opinion that the chains were loose. Rather, the statement is offered to show that the defendant had notice of a possible danger involving the chains. Because the testimony simply relates this statement made to the defendant, such testimony cannot be characterized as opinion testimony. Second, (D) incorrectly implies that opinion evidence is inadmissible. Even opinions of lay witnesses are admissible when they are: (i) rationally based on the perception of the witness; and (ii) helpful to a clear understanding of the witness's testimony or to the determination of a fact in issue; and (iii) not based on scientific, technical, or other specialized knowledge. [Fed R Evid 701] Certainly, the witness would be permitted to testify that the chains looked loose at the time he observed them, because such an opinion would be based on personal observation and would be helpful to the determination of a fact in issue (i.e., whether the load was properly secured), and would not be based on technical knowledge.

Question 6 -Tax - Intermediate To reduce deer overpopulation in state forests, state Blue adopted a statute allowing anyone with a valid deer hunting license from any state to hunt deer within state Blue. The act also imposed a $0.25 per pound tax on each deer killed within the state. Funds from the tax were earmarked to support state forest land. State Red is adjacent to state Blue and also has an overabundance of deer. To encourage hunting, state Red does not impose a tax on deer taken from its forests. A hunter who is a resident of state Red and who is licensed to hunt there earns his living by supplying wild game to several high-end restaurants in state Red. While legally hunting deer within state Red, the hunter inadvertently crossed the state line and killed a deer in state Blue. Upon hearing the hunter's shot, a state Blue game warden arrived at the scene, approximated the weight of the kill, and handed the hunter a tax bill based on the approximation. The bill provided a method for challenging the approximated weight of the deer, but the hunter refused to pay any tax on his kill. He instead filed suit in federal court to enjoin collection of the state Blue tax on constitutional grounds. Which of the following results is most likely? A The hunter will prevail because the tax is invalid under the Commerce Clause. B The hunter will prevail because the tax is invalid under the Interstate Privileges and Immunities Clause of Article IV, Section 2. C State Blue will prevail because the tax is valid under the Commerce Clause. D State Blue will prevail because the tax is valid under the Import-Export Clause.

(C) State Blue will prevail because the tax is valid under the Commerce Clause. A tax is valid under the Commerce Clause if: (i) the tax does not discriminate against interstate commerce; (ii) there is a substantial nexus between the activity taxed and the taxing state; (iii) the tax is fairly apportioned; and (iv) the tax fairly relates to services or benefits provided by the state. The state of Blue tax is applicable equally to residents of Blue and nonresidents. Thus, there is no discrimination against interstate commerce. Because the taxed deer are taken from within the state, there is a substantial nexus between the activity taxed and the taxing state. There is fair apportionment if a tax is based on the extent of the taxable activity or property in the state. Here, the killing of a deer within the state of Blue obviously occurs entirely within the state. Thus, the state tax is fairly apportioned. Also, there is a fair relationship between the tax and any benefits provided by the taxing state, because the state is permitting those engaged in hunting to take deer from its forest lands, in return for a rather modest amount of $0.25 per pound. That revenue, in turn, is used to support state Blue forest land, which provides hunters with a place to hunt. Thus, the state tax meets all of the requirements for validity under the Commerce Clause. (A) incorrectly states that the tax is invalid under the Commerce Clause. (B) is incorrect because the Interstate Privileges and Immunities Clause prohibits discrimination by a state against nonresidents when such discrimination involves fundamental rights, such as those involving important commercial activities. Here, while the tax may affect the hunter's commercial activity (because he earns a living from hunting and selling meat), the tax treats residents and nonresidents equally. Thus, there is no constitutional violation under the Privileges and Immunities Clause. (D) is incorrect because the Import-Export Clause applies to the authority of a state to tax foreign commerce. This question here does not deal with imported or exported goods. Thus, the Import-Export Clause is inapplicable to these facts

Question 23- Relevance (Subsequent Remedial Measures in answer) - Intermediate A well-known actor sued a resort hotel for damages to his new limited edition sports car caused by the hotel's parking valet while the actor was a guest at the hotel. His lawsuit, based on theories of respondeat superior and negligent hiring, alleged that after he gave the valet the keys, the valet, who had been working for the hotel for nine months, took the car for a drive without permission and negligently drove it into a tree, causing extensive damage to the car. At trial, the actor's counsel offers evidence that six months before the accident, but three months after the valet was hired, the hotel instituted new hiring procedures for all potential employees, including parking valets. Included in the new rules was a requirement that all persons must pass a thorough background check before being hired. The valet had been required only to have a valid driver's license when he was hired. In fact, he had an extensive record of traffic offenses at the time he was hired. Is the evidence regarding the new employment requirement's admissible? A No, because it is irrelevant. B No, because it is evidence of remedial measures. C Yes, because it is evidence of the hotel's negligence. D Yes, because it is evidence that the valet was incompetent.

(C) The evidence is admissible because it tends to show that the hotel was not acting prudently when it hired the valet, an employee who damaged a guest's car; thus (C) is correct and (A) is incorrect. (B) is not a good answer because only subsequent remedial measures (i.e., those taken after the injury to the plaintiff occurred) may not be proven as evidence of negligence; here the change in hiring procedures took place before the car was damaged, and so would be allowed. (D) is not accurate--the evidence does not show that the valet was incompetent, but rather that the hotel did not investigate his competence when he was hired, an issue related to the actor's negligent hiring claim.

Question 21- Real Evidence Gun- Intermediate An officer was driving in her squad car when she spotted the defendant, whom the officer knew because she had arrested him for an armed robbery in the past. She followed the defendant for awhile and noted that he kept looking nervously over his shoulder at the squad car and that he was carrying a brown paper bag in his hand. Suddenly, the defendant darted into an alley. A few moments later, he emerged from the alley without the paper bag and began running. The officer put on her siren and pursued the defendant. He was quickly apprehended and searched by the officer. She then drove back to the alley to search it. About six feet from the entrance to the alley, the officer found a paper bag hat contained a handgun. She copied the gun's serial number before taking the gun back to the police station. The defendant was charged with illegal possession of a handgun and carrying a concealed weapon. At his trial, while the officer is testifying, the prosecution seeks to admit the gun that the officer found into evidence against the defendant. The defense attorney objects on the grounds that the gun lacks proper identification. Should the objection be sustained? A Yes, because the gun was not in the defendant's possession at the time of his arrest. B Yes, because there is insufficient proof that the gun belonged to the defendant. C No, because there is sufficient evidence that the gun belonged to the defendant. D No, because the objection should have been based on chain of custody.

(C) The gun should be admitted into evidence. The gun is a form of real evidence, in that the object in issue is presented for inspection by the trier of fact. To be admissible, the object must be authenticated (i.e., identified as being what the proponent claims it to be). One method of authentication is recognition testimony, in which a witness may authenticate the object by testifying that it is what the proponent claims it is. Here, the officer found the gun in a paper bag in the alley shortly after having seen the defendant run into the alley holding a paper bag and emerge from the alley without the bag. The officer could now be called to identify the gun being offered into evidence as the one she found in the alley. This should be particularly easy in this case because the officer noted the serial number of the gun when she found it. The fact that the gun was found in the alley is circumstantial evidence that the gun was carried by the defendant on the night of the arrest. The evidence here is sufficient to withstand an objection to its admissibility on the ground that the gun has not been properly identified. Thus, (B) is incorrect in concluding that there is insufficient proof that the gun belonged to the defendant. (A) is incorrect. Real evidence may be circumstantial; i.e., facts about the object are proved as a basis for an inference that other facts are true. The circumstances surrounding the discovery of the gun support the inference that it had been in the bag carried by the defendant. It is not necessary for the gun to have been in his possession at the time of his arrest in order to admit the gun into evidence against him. (D) is incorrect because chain of custody problems arise where the evidence is of a type that is likely to be confused or can be easily tampered with (e.g., evidence of a blood alcohol test) after it is in custody. In such a case, the proponent of the evidence must show that the object has been held in a substantially unbroken chain of possession. Here, there is no evidence of a break in the chain of custody after the gun was taken by the police. Furthermore, a gun is generally not a type of evidence that is susceptible to confusion or tampering. Also, if need be, the serial number can be compared with the officer's written copy of the serial number. Thus, there is no viable objection based on chain of custody.

Question 3 - Products Liability - Child, no comparative fault Introduction A six-year-old girl went with her father to look at Christmas decorations in the neighborhood One house had a variety of displays in the yard, including a large mechanical Santa Claus figure that waved its arm and moved its head. While the father was talking to the homeowner about his electricity bill, the girl climbed up on the Santa. It toppled over and she hit her head on the ground, suffering a serious injury. The girl, through her guardian ad litem, brought a products liability action based on strict liability against the manufacturer of the Santa figure in a jurisdiction that does not apply its comparative negligence rules to strict liability actions. Which of the following would provide the best defense for the manufacturer? A The girl is unrelated to the purchaser of the Santa figure. B The girl was contributorily negligent in climbing on the Santa figure. C The girl's misuse of the Santa figure was not reasonably foreseeable. D. The girl's father was negligent in his supervision of her.

(C) The manufacturers best defense is that the girl's climbing on the Santa figure constituted a misuse of the product that was not reasonably foreseeable, thus relieving the manufacturer of any potential strict liability. prima facie case in products liability based on strict liability in tort requires proof that: (i) the defendant is a commercial supplier (ii) that produced or sold a defective product (iii) that actually and proximately caused (iv) damage to the plaintiff's person or property. The plaintiff must show that the defendant sold or produced the product in a defective condition unreasonably dangerous to users. Some products are safe if used as intended, but may pose serious dangers if used in other ways. Thus, suppliers must anticipate reasonably foreseeable uses (even if they are misuses) of the product. Here, there is no indication that the Santa figure was defective and unreasonably dangerous for the purpose for which it was designed. The product was not designed for climbing and the manufacturer at least can make the argument that it was not reasonably foreseeable that a child would do so. Hence, the manufacturers best defense is that the product was not dangerously defective for reasonably foreseeable use. (A) is incorrect because it raises a privity defense. Privity is not required to apply the protection of strict liability. The strict duty is owed not only to purchasers but also to family, guests, friends, and employees of the purchaser, as well as foreseeable bystanders such as the girl here. Thus, the manufacturer cannot raise the lack of privity between itself and the girl as a defense. (B)is incorrect because ordinary contributory negligence is not a defense to a strict products liability action unless the jurisdiction applies its comparative negligence rules to these actions (which this jurisdiction does not). Only voluntarily and unreasonably encountering a known risk or misusing the product in an unforeseeable manner (as (C) states) would serve as a defense. (D) is incorrect because, even if the father were negligent in his supervision of his daughter, such ordinary negligence will not be deemed a superseding intervening force that would break the causal connection between any initial wrongful conduct by the manufacturer and the ultimate injury. Hence, this would not serve as a defense for the manufacturer.

Question 1 -Impeachment Bias-Intro A brother and a sister were arrested on the federal charge of tax evasion in connection with the family business. Prior to trial, the prosecutor told the sister that he believed he could get her sentence reduced to probation if she pleaded guilty to a lesser charge and agreed to testify against her brother; the sister reluctantly agreed. During the jury trial, the sister is called by the prosecution. On cross-examination, the defense attorney brings out the fact that the sister was arrested on the same charge. The attorney then asks her whether it is true that after her arrest, the prosecutor told her that if she testifies against her brother her sentence can be reduced to probation. The prosecutor objects. How should the court rule on the objection? A Sustained, because it is against public policy to reveal information about plea bargains to a jury. B Sustained, because it calls for hearsay. C Overruled, because the question goes to bias or interest. D Overruled, because the sister waived the attorney- client privilege by testifying.

(C) The prosecutor's objection should be overruled because the question goes to the witness's bias or interest. Evidence that a witness is biased or has an interest in the outcome of the suit tends to show that the witness has a motive to lie. A witness may always be impeached by evidence of interest or bias, either on cross-examination or, if a proper foundation is laid, by extrinsic evidence. In a criminal case, it is proper for the defense to ask a prosecution witness whether she has been promised immunity from punishment or a reduction of punishment for testifying. This shows a motive for the witness to curry favor with the state. Here, the defense attorney is trying to impeach the sister by showing that because she was offered an attractive sentence, she has a motive to curry favor with the prosecution. This is perfectly proper. Note that there is no need for a foundation because the attorney is eliciting this evidence on cross-examination, rather than attempting to introduce extrinsic evidence of the deal. (A) is wrong because it misapplies and misstates the rule with regard to plea bargains. Under Federal Rule 410, withdrawn guilty pleas, pleas of nolo contendere, offers to plead guilty, and evidence of statements made in negotiating such pleas are inadmissible against the defendant who made the plea or was a participant in the plea discussions. This rule does not apply in this case because it does not apply to accepted guilty pleas, and the sister is not the defendant. The rule applies only to offers and withdrawn pleas. After the plea is accepted, it is admissible. (B) is wrong for two reasons: (i) the statement by the prosecutor is not hearsay because it is not being offered for the truth of the matter asserted, but rather to show its effect on the hearer; and (ii) even if the statement were hearsay, it would not make the question improper because evidence that is substantively inadmissible may be admitted for impeachment purposes if relevant to show bias or interest. (D) is wrong because no attorney-client privilege arises with respect to communications between the prosecutor and the sister. The attorney-client privilege requires that the attorney-client relationship exist at the time of the communication. To be covered, the client must be seeking the professional services of the attorney at the time of the communication. The sister was not seeking the services of the prosecutor. The prosecutor could not in any way be considered to be her attorney; they are clearly adversaries. Furthermore, even if this were not the case, the client holds the privilege and may waive. Thus, the question would be proper, and it would be up to the client-witness to decide whether to waive the privilege.

Question 31- Impeachment- Intermediate While cross-examining a defendant on trial for robbery and assault with a deadly weapon, the prosecutor asks him whether he was convicted of fraud within the previous year. Is this question proper? A No, because fraud is not probative of a tendency to commit violence. B No, unless the proper foundation was laid. C Yes, because fraud is a form of stealing, and so it will tend to show that the defendant could commit robbery. D Yes, because it tends to show that the defendant would lie.

(D) The Question is proper. The defendant has taken the stand in his own defense, and therefore the prosecutor can attack his credibility as a witness. Under Federal Rule 609, evidence of conviction of a crime requiring proof of an act of dishonesty or false statement can always be used to attack a witness's character for truthfulness. (A) is incorrect because even if fraud were probative of the tendency to commit violence, evidence of other crimes is not admissible to prove that a person has a propensity to commit criminal acts. (C) is incorrect for the same reason. (B) is wrong because no foundation is needed to show a prior conviction for impeachment purposes.

Question 6- Burglary/Larceny -Intro A farmer was in the middle of plowing his field when his tractor broke down. While attempting to repair it, he discovered that he needed a special wrench. He knew that his neighbor used the same type of tractor and kept a large cache of tools in his basement. Not wanting to make the long drive into town to buy one wrench that he probably would not use much, the farmer went to his neighbor's house to borrow the wrench. However, no one was home so he decided to look in his neighbor's basement for the wrench, thinking that he would return it before the neighbor came back. To gain entry, the farmer opened an unlocked window and climbed through the opening to the basement. Once inside, the farmer found the tool and took it with him to work on the tractor. His neighbor returned soon after and contacted the police when he discovered that one of his tools was missing. The police determined that the farmer took the tool and he was charged with burglary. What is the farmer's best defense against that charge in a common law jurisdiction? A Nobody actually lived in the basement. B The farmer knew that the house was unoccupied and would not have entered without permission had the neighbor been home. C The farmer entered the house through an unlocked window. D The farmer intended only to keep the wrench for a couple of hours.

(D) The farmers best defense is that he intended only to keep the wrench for a couple of hours. Given that the farmer intended merely to borrow the tool, he lacked the intent to commit larceny, and thus would not be guilty of burglary. Common law burglary consists of: (i) a breaking; (ii) and entry; (iii) of the dwelling; (iv) of another; (v) at nighttime; (vi) with the intent of committing a felony therein. The farmer entered his neighbor's house intending to remove the tool. Thus, the facts indicate that the only felony he could have intended to commit at the time of entry would be Larceny. Larceny consists of: (i) a taking; (ii) and carrying away; (iii) of tangible personal property; (iv) of another; (v) by trespass; (vi) with intent to permanently (or for an unreasonable time) deprive the person of his interest in the property. At common law, if the defendant intended to return the property within a reasonable time, and at the time of the taking had a substantial ability to do so, such an unauthorized borrowing would not constitute larceny. Consequently, if the farmer intended to keep the tool only for the short time to fix his tractor, then he did not intend to permanently deprive his neighbor of his interest in the wrench. Because the farmer thus lacked the intent to commit a felony in his neighbor's home at the time he entered, the farmer would not be guilty of burglary. (A) is incorrect because, for purposes of the crime of burglary, a structure is deemed to be a dwelling simply if any part of it is used regularly for sleeping purposes. Thus, the fact that nobody lived in the basement is irrelevant. (B) is incorrect because the fact that the house was unoccupied is irrelevant to his culpability for burglary. The crime of burglary would have been complete if the farmer had broken and entered his neighbors home with the intent of committing a felony therein, regardless of whether the home was currently unoccupied. Consequently, the farmers knowledge that the house was unoccupied would provide him with no defense to a charge of burglary. (C) is incorrect because the breaking needed for burglary requires only minimal force to gain entry. Opening an unlocked window is a sufficient use of force to constitute a breaking.

Question 24-Grand Jury Testimony - Intermediate A husband and a wife were arrested by federal agents and charged with distributing obscene materials through the United States mails. When called before a grand jury, the wife refused to say anything, invoking her Fifth Amendment right to be protected from compelled self-incrimination. The husband was terrified of the grand jury and readily admitted under questioning that he sent obscene matter through the mail. He also incriminated his wife in the illegal activity. The thought of a trial and a prison term drove the husband over the edge, and he committed suicide two days before his trial was to begin. A month later, the wife was put on trial in federal district court. The federal prosecutor seeks to introduce a transcript of the husband's grand jury testimony into evidence against the wife. The defense attorney objects. How should the court rule on the admissibility of the grand jury transcript? A Admissible, as a vicarious admission. B Admissible, as former testimony. C Inadmissible, because the wife can invoke the testimonial privilege, even though her husband is now deceased. D Inadmissible, because the husband's testimony was not subject to cross-examination.

(D) The grand jury transcript is not admissible because the husband's testimony was not subject to cross-examination. The husband's testimony was hearsay because it was an out-of-court statement offered to prove the truth of the matter asserted. [Fed R Evid 801 (c)] If a statement is hearsay, and no exception to the rule is applicable, the evidence inadmissible. [Fed R Evid 802] Under the former testimony exception to the hearsay rule, the testimony of a now unavailable witness given at another hearing is admissible in a subsequent trial as long as there is a sufficient similarity of parties and issues so that the opportunity to develop testimony or cross-examine at the prior hearing was meaningful. [Fed R Evid 804(b)(1)] The party against whom the former testimony is offered must have had the opportunity to develop the testimony at the prior proceeding by direct, cross-, or redirect examination of the declarant. Thus, the grand jury testimony of an unavailable declarant is not admissible as former testimony against the accused at trial. This is because grand jury proceedings do not provide the opportunity for cross-examination. Therefore, because the husband's testimony was in front of the grand jury and was not subject to cross-examination, inadmissible as hearsay. (A) is incorrect because the husband's testimony cannot be considered a vicarious admission. A statement by an opposing party (commonly known as an admission) is not hearsay under the Federal Rules. [Fed R Evid 801 (d)(2)] An admission is a statement made by a party and offered against that party. An admission does not have to be the statement of the party against whom the statement is being offered at trial if it qualifies as a vicarious admission. For example, admissions of one conspirator, made to a third party in furtherance of a conspiracy to commit a crime, may be admissible against co-conspirators. Here, however, the husband's grand jury testimony was not made in furtherance of a conspiracy. Because he was not a party here, and his testimony does not otherwise qualify as a vicarious admission of the wife, it cannot be considered an admission of a party-opponent. (B) is incorrect because the husband's grand jury testimony was not subject to cross-examination. Federal Rule 804(b)(1) allows the former testimony of an unavailable witness to be admitted under circumstances where the opportunity to develop testimony or cross-examine at the prior hearing was meaningful. The husband was an unavailable declarant because he was unable to testify because of death. [Fed R Evid 804(a)(4)]. However, as discussed above, his grand jury testimony is not admissible as former testimony because grand jury proceedings do not provide the opportunity for cross-examination. (C) is incorrect because the testimonial privilege does not belong to the wife in federal court and because it may only be asserted while the marriage relationship exists. In federal courts, this privilege belongs to the witness-spouse. This means that one spouse may testify against the other in criminal cases, with or without the consent of the party-spouse. Thus, while the husband could not have been compelled to testify against his wife, he could not be foreclosed by her from testifying (except as to confidential communications).

Question 16- Clergy Privilege- Intermediate A merchant sued a company for breach of contract, alleging that the products she purchased failed to conform to contract specifications. Shortly before the trial was to begin, the merchant suffered a stroke that left her paralyzed and virtually unable to communicate. Her guardian was properly substituted as the plaintiff in the lawsuit. At trial, following presentation of the plaintiff's case, the company calls as a witness a priest to question him about a conversation he had with the merchant at a church fundraiser. In this conversation, the merchant told the priest in confidence that the products she received were actually quite functional, but that she had become aware of a lower price being offered by another vendor, and thus wanted to get out of her contract with the company. The plaintiff's attorney immediately objects on the basis of clergy-penitent privilege. How should the court rule on the objection? A Sustained, because the merchant's statement was made to the priest in confidence. B Sustained, because this is not a criminal case. C Overruled, because the privilege can be invoked only by the person who made the confidential statement. D Overruled, because the circumstances under which the merchant made the statement take it outside the scope of the privilege.

(D) The plaintiff's objection on the basis of the clergy-penitent privilege should be overruled. Pursuant to the clergy-penitent privilege, a person has a privilege to refuse to disclose, and to prevent others from disclosing, a confidential communication by that person to a member of the clergy in the clergy members capacity as a spiritual adviser. The operation of this privilege is very similar to that of the attorney-client privilege. Here, the merchant made the statement to the priest during a conversation at a social occasion. There is no indication that this was a communication made to the priest in his capacity as a spiritual adviser, as would be the case, for instance, with a statement made in the confessional or during a counseling session. Thus, the matters stated to the priest by the merchant do not come within the clergy-penitent privilege, and the priest cannot be prevented from disclosing the contents of the conversation on the basis of this privilege. (A) is incorrect because, although the merchant undoubtedly made the statement in confidence (i.e., intending and expecting that it would not be disclosed to third persons), as explained above, it was not made to the priest in his capacity as a spiritual adviser. Therefore, the clergy-penitent privilege is inapplicable. (B) is incorrect because it implies that the clergy-penitent privilege does not apply to civil cases. Actually, this privilege applies to both civil and criminal cases. (C) is incorrect because, where the privilege exists, it can be claimed by the person who made the confidential communication, her guardian or conservator, or her personal representative if she is deceased. Thus, if the privilege were applicable, the merchant's guardian (through her attorney) would be able to invoke the protection of the privilege to prevent the priest from disclosing the contents of the conversation with the merchant.

Question 20- Hearsay - Intermediate A test preparation course sued a rival program for interference with business relations. The rival program had taken the test preparation course's reply cards from various colleges and mailed them in, hoping to swamp the clerks with work and cut the course's profit margin. The rival program also induced a professor under contract with the course to break his contract and lecture with the rival program for a percentage of the profits. The course director caught on to what the rival program was up to after the professor suddenly left the course. The director ordered his clerks to log in, for a two-week period, any and all business reply requests from the rival program's city. Immediately after the tally record was complete, the test preparation course filed its suit. At trial, the course's attorney offers the tally record into evidence. The defense attorney objects. Should the court find the tally record to be admissible? A Yes, as past recollection recorded. B Yes, as a business record C No, because unless it is shown that the clerks are unavailable, their testimony is the best evidence. D No, because it is hearsay not within any exception.

(D) The tally record should not be admitted because it is hearsay that does not fall within a recognized exception. The Federal Rules define hearsay as a statement, other than one made by the declarant while testifying at the trial or hearing, offered in evidence to prove the truth of the matter asserted. [Fed R Evid 801 (c)] Any written document, such as the tally record, that is offered into evidence constitutes a "statement" for hearsay purposes. The tally record was prepared out of court by the clerks and is being offered to prove the truth of its assertion, i.e., that the rival program had sent many reply cards. Because the tally record fits the definition of hearsay, and no exception to the rule is applicable, as discussed below, it is inadmissible. [Fed R Evid 802] (A) is incorrect because the tally record is not being used to substitute for the forgotten memory of the clerks. Where a witness states that she has insufficient recollection of an event to enable her to testify fully and accurately, even after she has consulted a writing given to her on the stand, the writing itself may be introduced into evidence if the proper foundation is laid for its admissibility. [Fed R Evid 803(5)] This is the past recollection recorded exception to the hearsay rule. Here, the tally record is not being used to substitute for the insufficient recollection of any witness. It is being offered to stand on its own as evidence, not in connection with a witness on the stand. Therefore, the tally record does not qualify as a past recollection recorded exception to the hearsay rule. (B) is incorrect because the tally record was recorded in preparation for litigation. Any writing or record, whether in the form of an entry in a book or otherwise, made as a memorandum or record of any act or event, is admissible in evidence as proof of that act or event if made in the regular course of a business, as long as it was the regular course of such business to make it at the time. Because the records must have been maintained in conjunction with a business activity to qualify for this exception to the hearsay rule, courts generally exclude reports prepared primarily for litigation. The Federal Rules deal with this problem by giving the trial court discretion to exclude any business record if circumstances indicate the record lacked trustworthiness. [Fed R Evid 803(6)] Because the tally sheet was prepared only in anticipation of litigation instead of in conjunction with a business activity, it does not fall within the business record exception. (C) is an incorrect statement of law. The best evidence rule, also known as the "original document rule," may be stated as follows: In proving the terms of a writing, where the terms are material, the original writing must be produced. [Fed R Evid 1002] The rule does not mean that the "best" evidence must be used to prove a fact. In this case, the test preparation course is not trying to prove the terms of a document. The fact to be proved (i.e., that the rival program was flooding the office with business reply requests) exists independently of any writing. Thus, the best evidence rule does not apply.

Question 2 -Intro The director of a one-person field station of the United States Department of Agriculture ("USDA") in a small town was instructed by his superiors to sell surplus government cheese and butter to local low income residents at 10% of market value. All sales were conducted at the USDA warehouse next to the field station. Pursuant to state statutes allowing municipal governments to establish reasonable regulations governing the retail sale of foodstuffs, the town in which the field office is located requires any establishment for the retail sale of food to pass a health and sanitation inspection and meet other specified criteria for obtaining a city license. The director of the field office failed to obtain a license from the town and was prosecuted for the failure. Which of the following will provide the best defense for the director in this prosecution? A The ordinance under which the director is being prosecuted is invalid as an undue burden upon interstate commerce. B The ordinance under which the director is being prosecuted violates the Equal Protection Clause of the Fourteenth Amendment. C The ordinance under which the director is being prosecuted deprives him of property without due process of law. D The ordinance under which the director is being prosecuted violates the principles of intergovernmental immunity as applied to him.

(D) The town licensing requirement unconstitutionally impinges upon a duly authorized federal program. The United States Government, as well as its agencies and instrumentalities, is immune from state regulation that interferes with federal activities, functions, and programs. To the extent that state regulations substantially interfere with an authorized federal program, the state laws must yield. Here, the director, as an agent of the federal government, was carrying out a duly authorized program of the Department of Agriculture by conducting sales of surplus government food at a federally owned warehouse. To sustain the power of the town to prosecute the director for not having a retail food sale license would give the town overriding authority over the selection of personnel to administer a federal program, as well as over the means by which this program is to be implemented. Thus, the licensing requirement would substantially interfere with the proper functioning of this federal program by directly interfering with a federal employee in the carrying out of his orders. (A) is incorrect because the facts do not indicate that the licensing requirement is in any way a measurable burden upon interstate commerce, much less an undue burden. The licensing requirement appears to affect, almost exclusively, the peculiarly local concerns of health and sanitation. (B) is incorrect because an equal protection violation exists where a law unreasonably limits the liberty of some persons but not others, i.e., where a law treats similar persons in a dissimilar manner. The licensing regulation at issue here is apparently being applied in an even-handed fashion, and the director is not being treated differently from anyone else who does not have the required license. Thus, there is no equal protection problem. (C) is incorrect because the director is not being deprived of any property or interest to which he has a legitimate claim; e.g., he is not being deprived of employment to which he is entitled. Because there is no deprivation of property, there no due process issue.

Question 1 - Default Judgments-Intermediate A minivan driver from State A and a semi-truck driver from State B were involved in a serious collision on a highway in State. The minivan driver sued the truck driver in State A for negligence, and sewed the truck driver with the summons and complaint via first-class mail. Although the truck driver received the documents, he failed to respond to them or appear in court. The minivan driver eventually obtained a valid default judgment in state court. State A's requirements for service of process are the same as the requirements under the Federal Rule of Civil Procedure. If the minivan driver seeks to enforce the State A judgment against the truck driver in State B, will the driver be successful? A Yes, under the Constitution's Full Faith and Credit Clause, because a default judgment is considered "on the merits. B Yes, under the Constitution's Full Faith and Credit Clause, because the truck driver received the summons and complaint. C No, because claim preclusion precludes the minivan driver from asserting two actions that involve the same parties and the same transaction and occurrence. D No, because the truck driver may collaterally attack the default judgment for insufficient of process.

D) The minivan driver will not be successful. Default judgments which are constitutionally or procedurally defective are subject to a collateral attack and may not be enforced under the Constitution's Full Faith and Credit Clause. Generally, under the Federal Rules of Civil Procedure (Rule 41, service of process can be made by: (i) personal service, (ii) service left at the defendant's usual place of abode with one of suitable age and discretion residing therein, or (iii) service upon an authorized agent of the defendant. Alternatively, service may be made under state rules or by mail under the waiver of service provision of Rule 4(d). A collateral attack is the name used to describe a defendant's ability to challenge a default judgment where the defendant never appeared in the action at all. Default judgments which are either constitutionally or procedurally protected are subject to collateral attacks. Under Rule 4 of the Federal Rules of Civil Procedure, which is the same in State A, if no acknowledgment is made to a summons and complaint that was improperly served via first class mail, a defendant must be served according to the Federal Rules of Civil Procedure. If not, it is considered "procedurally defective" and is therefore subject to a collateral attack. Here, because service by first class mail is insufficient under the federal rules, the truck driver may collaterally attack the default judgment arguing insufficient service of process. (A) is wrong because, although a default judgment is usually considered a final judgment "on the merits," the default judgment in State A is not entitled to Full Faith and Credit because the minivan driver failed to properly serve the truck driver. (B) is wrong, because, despite actual receipt of documents, the truck driver was not properly served in accordance with the applicable law. Therefore, as stated above, he can collaterally attack the default judgment. (C) is wrong because, while the previous judgment in State A would prevent the truck driver from relitigating the claim in State B, the minivan driver is only seeking to enforce the prior judgment, not relitigate it. Therefore, claim preclusion does not apply.

Question 7- Intermediate- employee, child, special relationship A farmer employed a 16-year-old high school student for a summer agricultural labor job. One afternoon, a violent storm suddenly erupted as the farmer was driving a tractor up a hill in an open field with the student in the wagon behind. When loud claps of thunder erupted, the farmer stopped his tractor, jumped off without saying anything, and ran swiftly down the hill toward the low ground, which he knew would be safer. The student, who lived in a nearby city and had never seen an electrical storm in open country (except as a passenger inside an automobile), had never been told how to act safely during such a storm. Once the storm began, the student was struck by lightning and seriously injured as he stood at the crest of the hill watching the farmer run. Is the farmer liable to the student for the injuries caused by lightning? A Yes, because the student was an employee, acting within the scope of his employment. B Yes, because the student was a minor. C No, because the student was injured by an act of God. D No, because lightning is never foreseeable.

(A) As an employer, the farmer breached his duty of care owing to the student and therefore is liable for the student's injuries on a negligence theory. To establish a prima facie case for negligence, the following elements must be proved: (i) the existence of a duty on the part of defendant to conform to a specific standard of conduct for the protection of the plaintiff against an unreasonable risk of injury; (ii) breach of that duty by defendant; (iii) the breach of the duty by defendant was the actual and proximate cause of plaintiff's injury; and (iv) damage to the plaintiff's person or property. The first issue raised by these facts is whether the farmer owed a duty of care to his employee. As a general matter, no legal duty is imposed upon any person to affirmatively act for the benefit of others. However, the existence of a special relationship between the parties may create a duty. Modern cases extend the duty to employers when employees are injured in the course of employment. Thus, the farmer owed the student a duty to protect him against an unreasonable risk of injury while he was acting within the scope of his employment. The farmer breached this duty by not warning and instructing the student in how to act safely during an electrical storm. The breach of that duty was the cause in fact and proximate cause of the student's injuries. An act or omission to act is the cause in fact of an injury when the injury would not have occurred but for the act. The "but for' test applies where several acts combine to cause the injury, but none of the acts standing alone would have been sufficient. But for any of the acts, the injury would not have occurred. Thus, but for the farmer's failure to instruct the student on how to act during an electrical storm, the student would not have been injured. The farmer's failure to instruct is also the proximate cause of the student's injuries. The general rule of proximate cause is that defendant is liable for all harmful results that are the normal incidents of and within the increased risk caused by his acts. This is an indirect cause case because an independent intervening force (the lightning) came into motion after the farmers negligent conduct and combined with it to cause the student's injury. Independent intervening forces are foreseeable (and thus do not cut off defendant's liability) where defendant's negligence increased the risk that these forces would cause harm to the plaintiff. The farmer's negligent failure to instruct the student about the need to seek low ground during an electrical storm greatly increased the risk that the student would be struck by lightning when the storm came up. Because the lightning was foreseeable and brought about a foreseeable harmful result to the student, it was not a superseding force that would cut off the farmers liability for the student's injuries. (B) is incorrect because the student's minority does not create a duty toward him by the farmer. The duty of care arises out of the employer/employee relationship. (C) is incorrect because, as noted above, the act of God (the lightning) would not be a superseding intervening force since it was foreseeable. Here, the farmer was negligent in not seeking to minimize the chances of the student's being struck by lightning, when the farmer knew that such danger existed and owed the student such duty as a result of his relationship (employer/employee) with the student. (D) is similarly incorrect because lightning can be foreseeable and was foreseeable here. The rain and loud claps of thunder were a clear signal that lightning might occur, and the farmers failure to warn the student created a foreseeable risk that the lightning would strike him.

Question 7- Roadblock- Intro A woman was stopped at a police roadblock to check for drunk drivers. The police were stopping every third vehicle that came through the checkpoint, and the woman's car turned out to be a third vehicle. After failing a field sobriety test, the woman was arrested and charged with driving while intoxicated. Was the stop of the woman's car legal? A Yes, because the car was stopped at a fixed checkpoint to check for drunk drivers. B Yes, because temporarily stopping a car does not constitute a seizure of the automobile. C No, because not every car was being stopped. D No, because there was no probable cause to stop the vehicle.

(A) Because the stop was at a fixed checkpoint, it was a legal stop. The police may set up roadblocks to stop cars without individualized suspicion that the driver has violated some law, as long as they: (i) stop cars on the basis of some neutral, articulable standard (e g, every car or every third car); and (ii) are motivated by a particular problem related to automobiles and their mobility (e.g., drunk driving). Here, the police were entitled to stop the woman because the stop was made at a fixed checkpoint to check for drunk drivers and used a neutral standard as the basis for stopping the cars. (B) is incorrect because stopping a car constitutes a seizure for Fourth Amendment purposes. (C) is incorrect because every car need not be stopped at a fixed checkpoint, as long as the roadblock is based on some neutral, articulable standard. Here, the neutral standard was stopping every third car; thus, the roadblock would not be deemed improper for failure to stop every car. (D) is incorrect because, as discussed above, probable cause is not required under these circumstances.

Question 3 - Service of Process- Intermediate A state's civil procedure rules allow for "nail and mail" service (posting the summons at the defendant's dwelling and thereafter mailing him a copy by certified mail) when "regular" service (service by physical delivery or by leaving a copy of the complaint with someone of suitable age and discretion at the defendant's usual place of abode) cannot be accomplished with due diligence. A plaintiff brought suit against a defendant in the federal district court for that state. After the plaintiff's special process sewer made many attempts at serving the defendant, the plaintiff's attorney directed the process server to nail the complaint and summons to the defendant's front door, and the attorney mailed (by certified mail) a copy to the defendant in accordance with the state rule. Has the defendant been properly served? A Yes, if "nail and mail" service is reasonably calculated to give the defendant notice of the action. B Yes, because under the Erie doctrine the federal court must apply the state's service of process rules. C No, if the defendant only rented the dwelling at which service was posted. D No, because "nail and mail" service is not authorized by the Federal Rules of Civil Procedure.

(A) Federal Rule 4 provides that summons and complaint may be served on an individual other than an infant or incompetent pursuant to the law of the state in which the district court is located. Notwithstanding, the state provision must be constitutional; i.e., it must be reasonably calculated to give the defendant notice of the action. Thus, (A) is correct and (D) is incorrect. (B) is incorrect because the Erie doctrine provides that federal courts apply state substantive law but federal procedural law. The Federal Rules expressly authorize the use of state service of process methods, and the method of service of process would probably be considered procedural. Thus, the Erie doctrine is inapplicable. (C) is incorrect. The key to substituted service is whether the dwelling was the defendant's usual place of abode, not whether it was owned or rented.

Question 20 -Lack of Specific Intent - Intermediate An employee of the state government always received his state paycheck on the latest workday of the month. The employee was not a good money manager, and just barely managed to make it from paycheck to paycheck each month. On the second to the last workday of the month, the employee had $45 in his checking account, and, needing to buy a birthday gift for his sister, he wrote a check to a gift boutique for $100. He knew that he would be receiving his paycheck the next day, so he could deposit the paycheck before the check would be sent to the bank. However, unknown to the employee, the state legislature was having a budget impasse. Because the state constitution prohibited any deficit spending, state employees were not paid as usual. Without a paycheck to deposit, the check written to the gift boutique was returned for insufficient funds. The merchant complained to the police, who arrested the employee and charged him under a statute that prohibited "issuing a check knowing that it is drawn against insufficient funds, with intent to defraud the payee of the check." What should be the outcome of the employee's prosecution? A Not guilty, because the employee intended to deposit his paycheck the next day. B Not guilty, because it was reasonable for the employee to expect that he would receive his paycheck as usual. C Guilty, because the employee knew when he wrote the check that he did not have sufficient funds in his account to honor it. D Guilty, because reliance on a future source of income does not vitiate the employee's violation of the statute when he wrote the check

(A) Given that the employee intended to deposit his paycheck before the checks cleared, he lacked the intent to defraud required by the statute. The statute under which the employee is being prosecuted is a variation of the offense of false pretenses. As with false pretenses, the statute requires a specific intent, i.e., an intent to defraud. If the employee intended to deposit sufficient funds to honor the check before it reached his bank, then the employee did not intend to defraud the gift boutique. Thus, the employee lacked the specific intent that is a necessary element of the crime charged. (B) is incorrect because the employee's expectation that he would receive his paycheck as usual need not have been reasonable. Even if such an expectation were unreasonable, the employee would not be guilty if he did not intend to defraud the payees, as required by the statute. (C) is incorrect because it would result in a verdict of guilty without requiring intent to defraud. Knowledge that the check was drawn against insufficient funds is just one element of the statute. The intent to defraud is also required to convict under the applicable statute. <(D) also incorrectly assumes that the employee violated the statute merely by knowingly writing a check on insufficient funds. As explained above, the requisite intent to defraud is absent. Thus, there is no "violation" to be vitiated.

Question 2- Order of Trying Claims-Intermediate A restaurant owner properly sued a food supplier in federal district court for breach of contract and timely demanded a jury trial. The complaint asserted both legal and equitable claims. Which of the following statements correctly states the proper order for trying both claims? A All legal claims should be tried first by the jury. B All equitable claims should be tried first by the court. C Legal and equitable claims may not be tried together, so the order does not matter since there will be separate trials. D It is up to the federal district court judge's discretion which claim will be tried first.

(A) lf legal and equitable claims are joined in one action involving common fact issues, the legal claim should be tried first to the jury and then the equitable claim to the court (the jury's finding on fact issues will bind the court in the equitable claim). (B) is wrong, because it misstates the rule. As stated above, the legal claim will first be tried by the jury followed by the equitable claim. (C) is wrong because legal and equitable claims can be consolidated together when the actions have a common question of law and fact. (D) is wrong because the Supreme Court has held that if legal and equitable claims are joined in one action involving common fact issues, the legal claim should be tried first.

Question 25-Insanity M'Naghten Rule-Advanced A high school teacher shot and killed one of the students in his class on the spur of the moment. Psychiatric examinations indicated that the teacher believed that the student was trying to ridicule him in front of other students in the class and that he had to do something to stop him. The examinations also indicated that the teacher did not comprehend that killing was condemned by society when he shot his student. If the teacher pleads not guilty by reason of insanity in a jurisdiction that applies the "M'Naghten test," what would be his best argument? A He did not know that the act of shooting the student was wrong. B He lacked the substantial capacity to appreciate the criminality of his act. C He did not know the nature and quality of his act. D His act was the result of an irresistible impulse.

(A) If the jurisdiction uses the M'Naghten test, the teacher's best argument is that he did not know that his act was wrong. The M'Naghten test provides for a defendant's acquittal if he has a disease of the mind causing a defect of reason so that at the time of his actions he lacked the ability to know the wrongfulness of his actions or understand the nature and quality of his actions. (A) states one branch of this test and is consistent with the facts (which state that the teacher did not understand that the killing was wrongful), and so it is the teacher's best argument. (C) is wrong because it is contrary to the facts. Although (C) also states part of the M'Naghten test, the teacher's illness has not left him so irrational that he is unable to comprehend that his act would result in the student's death. He seemed to have known that he was killing his student; he just did not know that killing was wrong. (B) is wrong because although the teacher did lack the substantial capacity to appreciate the criminality of his act, this is not a criterion for insanity in a state that follows the M'Naghten test; rather, (B) states the Model Penal Code standard. (D) is wrong because it states conduct outside the scope of the M'Naghten test. Also, the facts do not show that the teachers mental illness had deprived him of his volitional controls.

Question 10- Mistake - Intro A woman purchased a wardrobe closet at an antique auction. Three days later, while cleaning the inside of the closet, she discovered a small quantity of a white powder inside a box. She showed the box to her boyfriend, a paralegal. He identified the powder as driscamine, a controlled substance. He told her that it was illegal to buy driscamine, but because she did not know that it was in the closet when she purchased it, it was okay to keep it, which she did. A state statute prohibits "Willful and unlawful possession of a controlled substance." If the woman is charged with violating this statute, will she be found guilty? A Yes, because she knowingly possessed the driscamine. B Yes, because she acquired the driscamine when she intentionally purchased the wardrobe closet and, in doing so, committed the requisite unlawful act. C No, because she thought she was acting lawfully. D No, because she did not willfully acquire the driscamine and, hence, committed no unlawful act.

(A) She should be found guilty. As soon as her boyfriend informed her that driscamine was a controlled substance and she decided to keep it, she violated the statute, because she willfully possessed a controlled substance. (B) is incorrect because the statute does not punish mere possession; the possession must be "willful." (C) is incorrect because ignorance of the law is generally no excuse. It clearly does not negate the mental state required for this statutory crime. (D) is incorrect because the statute does not punish willful acquisition but willful possession.

Question 4-Cross Claim-Intermediate A three-car accident occurred in which the drivers were a citizen of State A, a citizen of State B, and a citizen of State C. The State A citizen filed a negligence action against the other in federal district court and lost his case. After judgment, may the State C citizen assert and maintain a negligence action against the State B citizen seeking damages for the injuries the State C citizen sustained in the same accident? A Yes, because, while the State C citizen could have asserted the claim as a cross-claim in the prior action, he may wait and assert it as an independent action. B Yes, because the State C citizen could not have asserted the claim in the prior action and thus may assert it independently. C No, because the State C citizen's claim was a compulsory cross-claim in the prior action and, since it was not asserted as a cross-claim in that action, it is now barred. D No, because the State C citizen's claim is barred by claim preclusion.

(A) The State C citizen may assert and maintain a negligence action against the State B citizen. The claim could have been asserted as a cross-claim in the prior action because it arose from the same transaction or occurrence, but cross-claims are never compulsory. (B) and (C) are therefore incorrect. (D) is incorrect because claim preclusion applies to cases brought by the same claimant against the same defendant. The State C citizen was a defendant in the first case and would be a claimant in the second. Therefore, claim preclusion would not bar the State C citizen's claim.

Question 5-Default Judgment—Intermediate A consumer purchased a luxury automobile from a dealer on credit. After the consumer failed to make a number of the required payments, the dealer filed a civil action against the consumer in federal district court to recover the balance due on the account. The dealer properly served process on the consumer. Several months passed, and the consumer did not file any response to the complaint. The dealer then filed a motion asking the clerk of court to make an entry of default, and the clerk did so. What procedure should the dealer follow to obtain a default judgment against the consumer? A File a motion to have the clerk of court enter the default judgment, and the clerk may do so without the consumer receiving any further notice of the motion. B File a motion to have the clerk of court enter the default judgment, and the clerk may do so, provided the consumer receives additional notice of the motion for default judgment. C File a motion to have the judge enter the default judgment, and the judge may do so without the consumer receiving any further notice of the motion for default judgment. D File a motion to have the judge enter the default judgment, and the judge may do so, provided the consumer receives additional notice of the motion for default judgment.

(A) The dealer should file a motion with the clerk of court. On request of the plaintiff, supported by an affidavit as to the amount due, the clerk may sign and enter judgment for that amount and costs against the defendant if: (i) the plaintiff's claim against the defaulted defendant is for a sum certain; (ii) the default was entered because the defendant failed to appear; (iii) the defaulted defendant is not an infant or incompetent person; and (iv) the damages amount requested is not greater than the amount requested in the complaint. The dealer's claim here meets these requirements, so the clerk may enter the default judgment, and no notice is required. (B) is incorrect because no further notice is required under these facts. (C) is incorrect because the clerk may enter the default judgment when the amount is for a sum certain. (D) is incorrect because no further notice is required under these facts and because the clerk may enter the default judgment.

Question 22-Mistake of Fact- State of Mind - Intermediate State law required all businesses in the state to report any deposits of toxic waste discovered on property owned by the reporting firm. Another provision of the law required that the toxic waste reports be made on a specific state form and that duplicate copies of the form be filed with both the state department of labor and the state environmental protection agency. The statute provided that after each report was filed, the relevant state agencies would ensure that any dangerous area was cordoned off and work out a plan with the reporting firm for the clean-up of the waste deposit. The effective date of the statute was June 15, but the state office did not produce any of the special forms until July 6. The defendant, a safety inspector employed by a chemical company, was making her first visit to one of the company's plants on June 17 when she noticed a strange substance on the premises. The defendant filed no report to the state agencies regarding the substance because she was not aware of the requirement, nor did she know that the substance was toxic. On June 20, another employee of the chemical company was walking in an area near the substance when he slipped and fell face first into it. The substance turned out to be very toxic. The employee's skin contact with the substance was sufficient to kill him in a matter of minutes. A later investigation revealed that it would have been impossible to clean up the substance to make the area safe before the employee encountered it. If the defendant is charged with murder, which of the following represents her best defense? A She did not know that the substance was toxic. B She was unaware of the state reporting law. C The state printing office did not produce any copies of the special forms until July 6. D It would have been impossible to clean up the toxic waste deposit before the time that the employee encountered it.

(A) The defendant's best defense is that she did not know that the substance was toxic and therefore did not have the state of mind necessary for murder. Ignorance or mistake as to a matter of fact will serve as a defense to a crime if it shows that the defendant did not have the state of mind required for the crime. If the mistake is offered as a defense to a malice crime, the mistake must be reasonable; i.e., it must be the type of mistake that a reasonable person might make under the circumstances. Although the statute itself may create a strict liability offense, for which the defendant's mistake as to the identity of the substance would not be a defense, the defendant here is being charged with murder, which is a malice crime. For the defendant to be convicted of murder, it must be shown that she had: (i) intent to kill; (ii) intent to inflict great bodily injury; (iii) reckless indifference to an unjustifiably high risk to human life; or (iv) intent to commit a felon. If she knew that the substance was toxic, the prosecution would have an argument that she acted with reckless indifference to an unjustifiably high risk to human life by not taking any action. However, if she did not know that the substance was toxic, her failure to act is far less likely to establish reckless indifference. In other words, the defendant was operating under such ignorance as to a matter of fact that she did not have the requisite state of mind for murder. This would provide the defendant with a defense. (B) is incorrect because it is not a defense to crime that a defendant was unaware that her acts were prohibited by the criminal law, or that the law compelled her to do something. Thus, even though the defendant was unaware of the reporting law, such ignorance of the law is no defense. Furthermore, regardless of her awareness of the reporting law, the defendant, as a safety inspector for the chemical company, may have owed a duty to the employee to act to clean up toxic waste. For this reason as well, the defendant's ignorance of the reporting law would not provide her with a strong argument. (C) does not provide a strong defense argument because the requirement of filing the reports on a specified form is a mere administrative detail. The fact that the forms were not yet available would not relieve the defendant of her duty to report any known deposits of toxic waste or to otherwise act to safeguard any employees from such waste. (D) is incorrect because, even if it would have been impossible to clean up the waste deposit by the time the employee encountered it, an immediate report by the defendant on the existence of the deposit would at least have resulted in warning the employees to stay away from the area, perhaps leading also to the cordoning off of the area.

Question 3- Conspiracy to commit larceny- Intro In a property settlement after a divorce, the wife was awarded all personal property that had been accumulated during the marriage, including the husband's classic l9-inch black-and-white TV set. In order to get his prized TV set back, the husband lied to his friend, telling him that the wife took the TV set in violation of the property settlement. The friend remembered that the wife gave the friend's wife a key to her new home, and he volunteered to go with the husband to get the TV back while the wife was at work. The husband and the friend went to the wife's house, but, unbeknownst to them, the wife had taken the day off work. After the friend noisily opened the back door with his wife's key, the wife called the police, who quickly arrived and arrested the husband and the friend. As to a charge of common law conspiracy to commit larceny, how should the friend be found? A Not guilty, because he did not intend to steal. B Not guilty, because he did not have a corrupt motive. C Guilty, because there was an agreement, and the opening of the locked door was sufficient for the overt act. D Guilty, because good motives are not a defense to criminal liability.

(A) The friend should be found not guilty because he did not intend to steal. At common law, conspiracy consists of (i) an agreement between two or more persons, (ii) an intent to enter into an agreement, and (iii) an intent to achieve the objective of the agreement. The object of the agreement must be something unlawful. Here, the friend did not intend to achieve the objective of the conspiracy--to permanently deprive the owner of her property--because, the friend thought the husband was the true owner of the TV. (C) is incorrect because there must be an agreement to reach an unlawful objective. Because the friend thought he was achieving a lawful objective, he did not have the intent required for conspiracy. (B) is incorrect because a "corrupt motive" is not an element of a crime. A person could be found guilty of a crime even if he did not have a corrupt motive, assuming all required elements for a crime are present. (D) is incorrect for a similar reason "Good motive" largely irrelevant; the intent, or lack thereof, what is important.

Question 4 -Statute of Limitations and Tolling Intermediate A patient complaining of pain in his side underwent a physical exam and x-rays, which showed that the patient had a small surgeon's clamp lodged in his left side. It had been left in the patient during an operation nine years previously. The patient brought an action against the hospital and the surgeon who operated on him, alleging negligence. The hospital and the surgeon filed a general denial and a special affirmative defense alleging that the statute of limitations is a bar to the action. Recovery for negligence in the jurisdiction is limited to two years. The patient's attorney makes a motion to strike the special affirmative defense. Should the motion be granted? A Yes, because the statute of limitations is tolled under the circumstances. B Yes, because of the common law rule that one must know of a wrong in order to have a remedy. C No, because the facts do not present an exception to the statute of limitations. D No as to the surgeon, but yes as to the hospital.

(A) The motion by the patient's attorney should be granted, because the statute of limitations is tolled under the circumstances. The general rule in malpractice actions is that the patient must know of the injury before the statute of limitations begins to run. Here, the subsequent physical examination is the first indication of the negligence. The statute would begin to run only from the time of that examination, and would have been tolled until that point in time. Thus, <(A) is correct and (C) is incorrect. (B) is incorrect because there is no established rule to that effect. (D) is incorrect because the tolling of the statute of limitations would apply to both the surgeon and the hospital. Quiz Three Torts Causation Supplemental Practice Questions with Answers

Question 3-Merger & Claim Preclusion-Intermediate A pedestrian was injured in an auto accident caused by a driver. The pedestrian's injuries included a broken nose and a broken toe. Not sure of the strength of her case, she sued the driver only for the injuries to her nose. She was awarded $15,000 in damages. Encouraged by this success, she now' wishes to sue the driver for the injuries to her toe. May she sue the driver again? A No, because all related claims "merged" with the final decision in the first case. B No, because she will be collaterally estopped from pursuing the claim. C Yes, and she may use the prior decision against the driver. D Yes, but she will have to relitigate the driver's liability.

(A) The pedestrian may not sue the driver again because of merger. Merger occurs when the plaintiff wins; her cause of action is said to "merge" into the judgment such that she cannot relitigate the cause of action later. The court would hold that all of the personal injuries received by the pedestrian in one accident constitute a single cause of action, and that claim preclusion principles forbid relitigation. (B) is incorrect. Issue preclusion, also called "collateral estoppel," applies to issues, not entire cases. (C) and (D) are incorrect for the reasons stated above.

Question 8-Incident Reports Discoverable-Intermediate A customer choked on a bone in her fish fillet while eating at a restaurant. The restaurant's manager, who was not in the restaurant when the incident occurred, met with the customer and the restaurant employees who witnessed the incident. As required by state law, the manager prepared a report regarding the incident and filed it with the state board of health. The manager then accepted a job in another country and moved there. The customer subsequently commenced a civil action against the restaurant in a federal court, alleging negligence and a breach of the warranty of merchantability. The customer's attorney served a request for all documents and reports prepared by the restaurant relating to the incident. Must the restaurant produce the manager's report? A Yes, because the manager prepared the report to comply with state law rather than to prepare for trial. B Yes, because the customer can establish that she has a substantial need for the report to prepare her case. C No, because the manager's report is hearsay and therefore not discoverable. D No, because the report constitutes work product and is therefore not discoverable.

(A) The restaurant must produce the report because it is relevant, proportional to the needs of the case, and it was not prepared in anticipation of litigation. Generally, a party may obtain discovery regarding any matter that is proportional to the needs of the case and that is relevant to any party's claim or defense, provided the matter is not subject to a privilege or to the exception for trial preparation materials, also called work product. Trial preparation materials are documents or other tangible things prepared in anticipation of litigation or for trial. Here, the manager's report is relevant because it contains facts relating to the incident that is the basis of plaintiff's claim. Moreover, it is not subject to the exception for trial preparation materials because it was prepared in the ordinary course of business to comply with state law, not in anticipation for litigation. Finally, the costs of producing the report likely would not be prohibitive. (B) is incorrect because it invokes the exception to the rule against discovery of work product, namely that work product is discoverable if the party seeking it "has substantial need for the materials to prepare its case and cannot, without undue hardship, obtain their substantial equivalent by other means." This exception is not pertinent because the manager's report is not work product. (C) is incorrect because material need not be admissible as evidence to be discoverable. The standard is relevancy, not admissibility. (D) is incorrect because, as discussed above, the report is not work product.

Question 3 Defamation-Advanced Two law students ranked high in their class were competing for one opening at a prestigious law firm. During the interview with the hiring partner, one student was asked what he thought of the other's work as an editor of the law review. The student responded that there was a rumor around the school that the editor got outside help on her law review comment. Based in large part on his statement, that student was chosen over the law review editor, who later accepted a less lucrative position with another firm. If the law review editor brings a slander action against the other student and establishes the above facts, will she prevail? A Yes, because the student's statement to the hiring partner was defamatory. B Yes, because the law review editor suffered special damages. C No, because the hiring partner asked the student for his opinion. D No, because the law review editor did not establish that the student made the statement with at least negligence.

(A) The student's statement constitutes slander per se and therefore the student will be liable. In order to establish a prima facie case for defamation, the following elements must be proved: (i) defamatory language on the part of the defendant; (ii) the defamatory language must be "of or concerning" the plaintiff (i.e., it must identify the plaintiff to a reasonable reader, listener, or viewer); (iii) publication of the defamatory language by the defendant to a third person; and (iv) damages to the reputation of the plaintiff. Here, the student's suggestion that the law review editor received outside help on an article she authored impeaches her integrity and legal skills. The defamatory language directly related to the editor. The publication requirement is satisfied because the student made the statement to the hiring partner. To recover damages for slander, special damages must be pleaded and proved unless the spoken defamation falls within one of four categories, characterized as slander per se. Hence, a defamatory statement adversely reflecting on the plaintiff's abilities in his business, trade, or profession is actionable without pleading or proof of special damages. The student's statement adversely reflected on the law review editor's honesty and capability in her profession, and as such is slander per se. (B) is incorrect because, as noted above, the student's defamatory statement adversely reflecting on the editor's abilities in her profession is actionable without proof of special damages. (C) is incorrect because the mere fact that the interviewer asked the student his opinion does not justify a defamatory response. The student did not have a common law qualified privilege to make the statements because he was not a former employer of the law review editor and was not yet a member of the hiring partner's firm (negating any common interest privilege). Furthermore, once publication is established, it is no defense that the defendant had no idea that the publication was defamatory. It is the intent to publish, not the intent to defame, that is the requisite intent. Thus, even if the student thought that his comments did not constitute defamation because they were in response to the interviewer's question, he could still be found liable for defamation assuming all other elements of the tort were satisfied. (D) is incorrect because it states a standard for private persons suing on matters of public concern Private plaintiffs must show that the defendant was at least negligent as to truth or falsity in making the statement, but here no matter of public concern is involved, so fault need not be shown. Also, since a matter of public concern is not involved, the plaintiff does not need to establish that the statement was false; a defamatory statement is presumed false at common law.

Question 10- Interrogatories- Intermediate A car collector bought a car with gold leaf paint from a manufacturer. During the first month, all the gold leaf paint peeled off. The collector sued the manufacturer, and during discovery served an interrogatory asking the manufacturer to identify all other purchasers of the gold leaf paint over the previous 10 years. The manufacturer was aware that only about 25 of the 2 million buyers of its cars have ordered the gold leaf option. The manufacturer has retained copies of all sales forms, but has not maintained separate files of the buyers of each particular option. In a court using the federal rules, what are the manufacturer's obligations with respect to the collector's interrogatory? A It must search its files and then disclose the information. B It may allow the collector to search the records himself. C It may respond by stating that only about 25 of the 2 million buyers ordered the option. D It may state that searching the records is too great a burden and so it is excused from answering the interrogatory.

(B) Assuming the interrogatory is otherwise proper, the manufacturer may search the 2 million order forms itself or it may allow the collector access to the files. In a situation where desired information may be ascertained from the business records of the party on whom the interrogatory was served, and where the burden of finding the information is substantially the same for the party serving the interrogatory as for the party served, it is a sufficient answer to provide the serving party reasonable opportunity to examine the records. (Fed. R. Civ. P. 33) (A) is incorrect because instead of searching the files itself, the manufacturer can allow the collector access to do it. (C) is incorrect because it is not responsive to the interrogatory. (D) is incorrect because a party is not relieved of its duty to answer just because the search is burdensome

Question 5- Conspiracy - Intro A wife wanted to have her husband murdered in order to collect the proceeds from a life insurance policy. The wife solicited the services of a hitman, agreeing to split the insurance proceeds with him. The plan was for the hitman to shoot the husband at home and make it look like a botched burglary attempt, while the wife would be at her church hall playing bingo that night. However, when the hitman broke into the apartment, the husband was able to subdue him and call the police. The hitman was arrested and implicated the wife, who was arrested in the parish priest's office just after she had confessed the plan to the priest. The wife is charged with attempted murder and conspiracy to commit murder in a state that retains the common law definitions of these crimes. A psychiatric evaluation of the hitman after he was arrested revealed that he was legally insane, and he later committed suicide while awaiting trial. As to the conspiracy charge against the wife, which of the following would be her most promising defense? A She cannot be tried for both offenses; the prosecutor must choose between them. B The hitman was insane when the conspiracy was formed. C The hitman died in jail before trial. D She had second thoughts about the plan while playing bingo and had revealed it to her priest.

(B) If the hitman was insane, the wife cannot be said to have conspired with him to commit the murder. A conspiracy consists of: (i) an agreement between two or more persons; (ii) an intent to enter into an agreement; and (iii) an intent to achieve the objective of the agreement. The parties must agree to accomplish the same objective by mutual action. This agreement requires a meeting of at least two "guilty minds"; i.e., two persons who are actually committing themselves to the scheme. If the hitman was insane, then he lacked the capacity to intentionally enter into an agreement to kill the husband and to intend to achieve the murder under this circumstance, the wife was the only one who was actually committed to the crime and who intended to bring it about. Because there was no meeting of the wife's "guilty mind" with at least one other, the wife cannot be convicted of conspiracy under the common law rule. (A) is incorrect because a defendant can be tried and convicted of both conspiracy and the substantive offense (here, attempted murder). (C) is incorrect because a conviction of conspiracy does not require that all parties be tried and convicted. While an acquittal of all persons with whom a person is alleged to have conspired precludes conviction of the remaining defendant, that is not the case here. The hitman was not acquitted. Also, note that a state's decision to discontinue prosecution would not be deemed to be an acquittal, and thus most likely a state's inability to prosecute because of a co-conspirators death would not be an acquittal. (D) is incorrect for two reasons: (i) withdrawal from a conspiracy is not a defense to a charge of conspiracy, and (ii) withdrawal is a defense to subsequent crimes committed by other members of the conspiracy only if the withdrawing party performs an affirmative act notifying all members of the conspiracy in time for them to have the opportunity to abandon their plans. Here, the wife notified the priest, but took no further action to thwart the conspiracy. Thus, she has no defense to the conspiracy, and unlikely that merely telling a priest would be a sufficient withdrawal as to subsequent crimes.

Question 4-Motion for New Trial & Jury Selection- Intermediate A plaintiff sued a defendant in federal court for medical malpractice. During jury selection, the defendant's attorney asked the potential jurors whether they had ever been involved in a medical malpractice lawsuit. One potential juror stated that he had been a plaintiff in a medical malpractice case and was distrustful of doctors because of his experience. The defendant's attorney used a peremptory strike against this potential juror. Another potential juror, a doctor, had been sued for medical malpractice twice in the past, but she denied ever being involved in a medical malpractice suit. She was selected for the jury along with 11 others. During the course of the trial, one juror was excused because he became ill. The case went to the jury, and the 11 jurors returned with a unanimous verdict for the defendant. Shortly thereafter, the plaintiff discovered through independent investigation that the juror who was a doctor had lied. The plaintiff filed a motion for a new trial on the grounds that the defendant's attorney had exercised his peremptory strike for an improper reason, that a juror had lied during the jury selection process, and that the verdict was rendered by less than the required 12 jurors. Is the court likely to grant the plaintiff's motion? A Yes, because the defendant's attorney exercised his peremptory strike for an improper reason. B Yes, because the juror who is a doctor lied during the jury selection process. C Yes, because juries in federal civil cases must have 12 jurors. D No, because there was no court error or juror misconduct in this case.

(B) The court is likely to grant the plaintiffs motion because the juror lied during the jury selection process. A new trial is appropriate if a juror gave false testimony on voir dire or concealed material facts relating to her qualifications to serve. (D) is therefore incorrect. (A) is incorrect because a party may exercise a peremptory strike for any reason as long as it is race- and gender-neutral. (C) is incorrect because a jury must have at least six and no more than 12 jurors. A juror may be excused for good reason without causing a mistrial so long as at least six jurors participate in reaching the verdict.

Question 8 - Murder with Alcohol- Intermediate After drinking at his favorite bar all day long and becoming very intoxicated, the defendant pulled out a gun to reenact a scene from a movie he had seen the other day. Pointing his gun at a bystander, he slurred a line from the movie and pulled the trigger. He was shocked to see the bystander fall down dead with a bullet in him. Due to his intoxicated state, the defendant cannot remember pulling out or firing the gun. The crimes below are listed in descending order of seriousness. A state statute defines all murders as second degree murders unless deliberation and premeditation can be shown, in which case the crime is elevated to first degree murder. Manslaughter is defined as at common law. What is the most serious crime for which the defendant may be convicted? A First degree murder B Second degree murder C Manslaughter D No crime

(B) The defendant may be convicted of second degree murder. In states that divide murder into degrees, evidence that the defendant was intoxicated may be used to show that the defendant was unable to premeditate and deliberate on the idea of killing, making (A) incorrect. In such jurisdictions, however, the fact that the defendant was intoxicated may not further reduce the killing from second degree murder to manslaughter, given that second degree murder encompasses common law murder and thus includes depraved heart murder. Carrying and firing a gun while intoxicated certainly shows a reckless indifference to an unjustifiably high risk to human life, thus meeting the definition of second degree murder. This makes (B) correct and (C) and (D) incorrect

Question 8- Personal Jurisdiction- Intermediate A college student from State A hit another car driven by a resident from State B when the college student was traveling through State B. The State B driver brought an action in State B state court against the State A college student, who has limited financial resources. The college student filed a motion to dismiss, claiming the State B court lacks personal jurisdiction. What is the best argument to support the college student's motion to dismiss? A The college student did not take actions to purposely avail herself of State B simply by driving through State B. B It is unfair, inconvenient, and highly unreasonable to require a college student with limited financial resources to defend the action in State B for financial reasons. C The interstate judicial system's interest in obtaining the most efficient resolution of the controversy is violated by requiring the college student to defend the action in the forum state. D State B is not the proper venue to file this claim.

(B)The best argument the college student has to defend a motion to dismiss for lack of personal jurisdiction is to argue that the fairness prong of the constitutional minimum contacts test is not met. In addition to sufficient minimum contacts with the forum state, personal jurisdiction must not offend "traditional notions of fair play and substantial justice." The Supreme Court in International Shoe listed several factors relevant to assess the fairness factor, including the burden on the defendant in terms of convenience in defending the action. Here, the best argument out of the four choices listed is that it would be unfair, inconvenient, and highly unreasonable for a college student from State A with limited financial resources to defend the action in State B. Note that this still may be a losing argument, as this argument will not prevail unless the burden to the defendant is "so gravely difficult and inconvenient that a party is unfairly put at a severe disadvantage in comparison to his opponent." However, this is still the best argument to support the motion to dismiss for lack of personal jurisdiction. (A) is wrong because, to establish the contacts prong of the minimum contacts test, the defendant's contact with the forum state must result in purposeful availment of the state, meaning the defendant took actions that were purposely directed toward the forum state and from which the defendant derived the benefits and protections of the state's laws, such as using the roads in the state. Here, driving through State B was sufficient because it was a purposeful activity and she was protected by State B's police and traffic laws while doing so. (C) is wrong because, although considering the interest in obtaining the most efficient resolution of the controversy is one of the factors the Supreme Court listed in assessing the fairness prong, it is inapplicable to the facts. Here, litigation in either State B or State A would likely be equally efficient. (D) is wrong because State B clearly is a proper venue, as it is the judicial district where the claim arose. Moreover, the question stated that the motion to dismiss was based on lack of personal jurisdiction, not lack of venue.

Question 21 -State of Mind- Intermediate Acting pursuant to a valid search warrant, the police entered and searched the defendant's garage and discovered a cardboard box containing cocaine in the rafter's storage area. The box was securely taped and bore a freight label addressed to the defendant's friend. At his trial for violation of the jurisdiction's statute making it a felony to knowingly possess cocaine, the defendant testified that his friend had brought him the package a week before it was seized by the police, telling him that he needed to store it in the defendant's garage. The defendant also testified that he had not asked the friend what it contained. What additional facts must the prosecution prove to establish the defendant's liability for the charged felony? A That he knew or believed that the box contained cocaine and had moved or handled the box. B That he knew or believed that the box contained cocaine. C That he should have known that the box contained cocaine and had moved or handled the box. D No additional facts.

(B) The defendant should be found guilty of the charged felony if he knew or believed that the box contained cocaine. The defendant is being tried for "knowingly" possessing cocaine. A person does not act knowingly unless he is aware that his conduct is of the proscribed nature or that the proscribed circumstances exist. Thus, the defendant could not have acted knowingly unless he knew or believed that the box contained cocaine. (A) is incorrect because criminal statutes that penalize the possession of contraband generally require only that the defendant have control of the item for a long enough period to have had an opportunity to terminate the possession. Thus, the defendant need not have moved or handled the box. (C) is incorrect for the same reason as (A), and also because the defendant's failure to know when he should have known would constitute negligence - failure to be aware of a substantial risk that prohibited results will follow or that circumstances exist - and negligence is not sufficient to establish knowledge. Note, however, that a defendant may not consciously avoid learning the true nature of the item possessed; knowledge may be inferred when the defendant is aware of a high probability of the true nature of the item and deliberately avoids learning the truth. (D) is incorrect because, as discussed above, the statute requires that the defendant knew or believed that the box contained cocaine.

Question 10 -Felony Murder- Defenses- Intermediate The victim owned a cottage in an ocean resort area. He stayed there only during the summer months, and left the cottage unoccupied during the balance of the year. The defendant, a resident of a neighboring cottage, was aware of this practice. For a change in his routine, however, the victim decided to spend a week at the cabin in the off-season, unaware that the victim was occupying the cottage, the defendant decided to borrow a portable television set that he knew the victim kept in the cottage. To avoid being seen, he entered the cottage late at night, using a key under the front doormat. He found the television set, disconnected it, and headed for the rear of the house to leave. He opened the kitchen door and found the victim seated there in the dark, having a late-night snack. Both men were startled and neither man recognized the other in the dark. The defendant assumed that the victim was a burglar, and was afraid that he might be armed. Trying to flee the kitchen as quickly as possible, the defendant dropped the television set in the middle of the kitchen floor. As the set hit the floor, the picture tube exploded with a loud noise. The noise so frightened the victim that he had an immediate heart attack and died. If the defendant is charged with felony murder as the result of the victim's death, what is his best defense? A He did not intend to kill the victim. B His only intent was to borrow the television set for a few days. C Larceny is not an inherently dangerous crime, and it was not being committed in an inherently dangerous manner. D The victim's heart attack was an unforeseeable consequence of the defendant's acts.

(B) The defendant's best defense to felony murder is that he only intended to borrow the television set for a few days. By establishing this intent, the defendant will show that he did not have the intent to commit a felony and therefore cannot be guilty of felony murder. A killing (even if accidental) committed during the course of a felony is murder. Malice is implied from the intent to commit the underlying felony. To convict a defendant of felony murder, the prosecution must prove that he committed or attempted to commit the underlying felony. Here, the possible felonies being committed by the defendant, during which the victim's death occurred, would be larceny and burglary. Burglary requires the intent to commit a felony within the dwelling, and larceny requires the intent to permanently deprive a person of his interest in property. If the defendant's only intent was to borrow the victim's television set for a few days, then the defendant lacked the intent to permanently deprive the victim of his interest in the set; e.g., the requisite intent for larceny is missing. Likewise, the absence of intent to steal the set would mean that, at the time of breaking and entering the cottage, the defendant did not intend to commit a felony therein. Consequently, the defendant is not guilty of burglary. Because under these circumstances no felony would have been attempted or committed, it cannot be shown that the death of the victim occurred during the commission of a felony. Therefore, the defendant would not be guilty of felony murder. (D) is tempting, because generally a conviction of felony murder requires that the death must have been a foreseeable result of commission of the felony. However, some courts do not apply a foreseeability requirement and require only that the underlying felony be malum in se. Furthermore, even those courts applying a foreseeability requirement have been willing to find most deaths occurring during the commission of a felony to be foreseeable. Here, the defendant believed that the cottage was unoccupied for the winter. Thus, it was arguably unforeseeable that the defendant's entering the cottage and taking a television set would result in the death of an occupant, but it is by no means certain that a court would agree. Furthermore, in those jurisdictions that do not require foreseeability of death, the defendant could be convicted of felony murder if the death occurred during the commission of a burglary, because burglary is always classified as a malum in se felony. Because the circumstances in choice (B) would assure the defendant of avoiding conviction in all jurisdictions, (B) is a better answer than (D). (A) incorrectly focuses on intent to kill. Intent to kill is one of the states of mind by which a defendant is deemed to have malice aforethought, which is necessary for a killing to constitute murder. However, this question refers to felony murder, wherein malice aforethought exists in the form of intent to commit a felony. Thus, it is irrelevant whether the defendant intended to kill the victim. Regarding (C), it is true that most courts limit the felony murder doctrine to felonies that are inherently dangerous, and that larceny generally is not considered to be inherently dangerous. However, assuming the existence of the requisite intent, the defendant may have committed burglary, which is deemed to be inherently dangerous. Thus, (C) might provide no defense at all to a charge of felony murder.

Question 8- Third Party Claim/Indemnity-Intermediate A college filed a breach of contract action against a general contractor in federal district court. If the contract was breached, the general contractor believes that the fault was that of the framing subcontractor. The general contractor thus files a third-party claim against the subcontractor seeking indemnification for any sums the college recovers from the general contractor. The general contractor also wants to assert a tort claim against the subcontractor for $80,000 worth of equipment belonging to the general contractor that the subcontractor took from the construction site. All parties are citizens of different states. May the general contractor assert that claim in the pending action? A Yes, because the tort claim is a proper third-party claim. B Yes, because, while the tort claim is not a proper third-party claim, the general contractor may join the tort claim with the indemnification claim. C No, because the tort claim is not derivative of the college's original claim and does not arise from the same transaction or occurrence as the original claim. D No, because the breach of contract claim and the tort claim do not share common questions of law or fact.

(B) The general contractor may assert the tort claim in the pending action. Once the general contractor asserts a proper third-party claim against the subcontractor (the indemnity claim), the general contractor may join any other claims it has against the subcontractor. (A) is incorrect because the tort claim is not a proper third-party claim; it does not involve the general contractor seeking indemnification or contribution for the college's claims against the general contractor. (C) and (D) are incorrect because they refer to tests that are not applicable here.

Question 6- Intro - Accomplice- when accomplice is part of protected class A 15-year-old girl who worked as a prostitute in a city approached a man and offered her services. The man agreed, and the two engaged in sexual intercourse in the back seat of the man's car. These events were noticed by an undercover police officer, who arrested the girl and the man. The man admitted to having sex with the girl, and he was charged with statutory rape. The girl was charged with being an accomplice to statutory rape At the girl's trial, what is her best defense? A Because the man has not yet been convicted, the girl may not be convicted as an accomplice. B The statutory rape law is designed to protect minors and therefore the girl cannot be convicted as an accomplice. C As a minor, the girl does not have the capacity to be an accomplice. D Because the girl is a prostitute and consented to the sexual acts, the man cannot be convicted of statutory rape; therefore, the girl cannot be an accomplice.

(B) The girl's best defense is that she cannot be convicted under the statute because it was designed to protect minors. Statutory rape is the crime of carnal knowledge of a minor under a certain age. In theory, a minor female could be convicted as an accomplice to statutory rape (such as by being an accomplice to the statutory rape of another minor by an adult perpetrator. If, however, the legislative intent is to protect members of a class from exploitation or overbearing, members of that class are presumed to be immune from liability, even if they participate in the crime. Here, the girl can argue that the crime of statutory rape was intended to protect minors from exploitation and therefore, the legislative intent is that she, as the minor victim, be immune from prosecution. (A) is wrong because an accomplice may be convicted before the principal. Although the common law rule required that the principal's guilt be determined first, or at least at a joint trial, this rule has been abandoned by most jurisdictions. (C) is wrong because it is too broad; as noted above, a minor may be an accomplice to a crime. To be convicted as an accomplice, a person must have acted with the intent to aid or encourage the principal in the commission of the crime. As long as a child is old enough to be able to form this intent, she could be convicted as an accomplice. A 15-year-old is certainly capable of forming that intent. (Even under the common law presumptions, children over age 14 are treated as adults) (D) is wrong because the man could be convicted despite the girl's consent. Consent is not a defense to statutory rape; even if the female willingly participates in the sexual acts, the male may be convicted. Thus, the girl's consent, or her status as a prostitute, is not the reason she may not be convicted as an accomplice.

Question 6 - Intermediate - Indemnity and Contribution: A company let an employee borrow one of its company cars for a cross-country vacation trip which the employee had planned for his family. While driving through a remote stretch of farmland, the employee decided to see how much power the car really had, and was driving in excess of 90 mph when he came to a curve. He applied the brakes and attempted to slow down, but the car went across the double line and struck head-on a minivan coming in the opposite direction. The driver of the minivan was killed in the accident and the minivan was destroyed. A "permissive use" statute is in effect making the bailor of an automobile liable for personal injury, death, or property damage caused by any person operating the automobile with his consent, up to a maximum of $25,000. The jurisdiction follows traditional contribution rules. If the driver's estate files suit against the company pursuant to the "permissive use" statute, and recovers the full 525,000, what rights, if any, would the company have against the employee? A None, unless there was a written agreement with the employee that obligated him to assume any such liability imposed on the company. B The company may obtain contribution from the employee to the extent of $12,500, but not indemnity. C The company may obtain indemnity against the employee for the full $25,000. D The company may obtain contribution from the employee to the extent of the employee's relative fault, but not indemnity

(C) The company may obtain indemnity from the employee. One who is held liable for damages caused by another simply because of his relationship to that person may seek indemnification from the person whose conduct actually caused the damage. The company has been held liable for the damages caused by the employee solely because loaned a car to the employee. Such vicarious liability (imposed pursuant to the permissive use statute) being imposed on the company will entitle it to be indemnified by the employee, whose conduct actually caused the damage. Thus, the entire loss will be shifted from the company to the employee. (A) is incorrect because a written agreement to indemnify is not the only means by which a right of indemnification can come into existence. For example, a right of indemnity exists as a matter of law in the circumstances here, as detailed above. Thus, the absence of any obligation of the employee contained in a written agreement will not mean that the company has no rights against the employee. (B) is incorrect because contribution is a device whereby responsibility is apportioned among those who are at fault. Contribution allows any defendant required to pay more than his share of damages to have a claim against any other jointly liable parties for the excess. The company is not really at fault in bringing about the harm. Rather, the company can only be held liable by virtue of the permissive use statute. There is no responsibility to be apportioned here; instead, the entire loss should be shifted from the company to the person who actually caused the damage. Thus, indemnity is available, and contribution is not the appropriate remedy. (D) is incorrect for the same reasons that (B) is incorrect. In addition, (D) incorrectly provides for contribution to the extent of the employee's relative fault (comparative contribution). Traditional contribution rules require all defendants to pay equal shares regardless of their respective degrees of fault.

Question 3 - Murder of co-felon-Intro Two robbers planned to rob a local convenience store, with one using a gun to force the clerk to turn over all of the money in the cash register while the other stood lookout near the door. The robbery did not go as planned Instead of turning over any cash, the store clerk tried to disarm the gunman. During their struggle for the gun, the lookout decided that her best course of action was to grab what she could and flee the scene. The lookout took a newspaper and a bag of potato chips and ran out of the store. On her way out, she heard a gunshot. Later that day, she learned from news accounts that the gun accidentally discharged, killing the gunman. After an investigation, the lookout was arrested. If the lookout is charged with felony murder, what would be her most promising defense? A She did not intend for the gunman to get killed. B The only person killed was the gunman. C The killing occurred after the robbery was over. D The robbery was not a felony because the items that the lookout took had only minimal value.

(B) The lookout's best defense is that the gunman was the only person killed under the felony murder doctrine, a killing committed during the course of a felony is murder, malice being implied from the intent to commit the underlying felony. However, under the majority view, criminal liability for murder cannot be based on the death of a co-felon from resistance by the victim or police pursuit. Thus, given that the gunman's death resulted from an act by the clerk, the victim of the robbery, the lookout cannot be found guilty of the felony murder of the gunman, a co-felon. (A) is incorrect because any desire or lack of desire by the lookout to see her co-felon harmed is irrelevant to liability for felony murder. The only mens rea required is the intent to commit the underlying felony. Here, the lookout had the intent to commit robbery, the underlying felony. From this intent, the malice required for murder is implied. (C) is incorrect because the fact that the felony was technically completed before the gunman's death does not prevent the killing from being felony murder. A death caused while fleeing from the crime is considered to have been caused during the commission of the felony. (D) incorrect because robbery a felony regardless of the value of the property that taken.

Question 1- Introductory - Comparative negligence A motorcyclist was injured in a collision and suffered $100,000 of injuries, including $20,000 in hospital and physician's bills. The motorcyclist's medical insurance company paid her $20,000 to cover hospital and medical expenses. Later, she filed suit against the driver of the car that struck her motorcycle. When the case came to trial, the jury agreed with the motorcyclist's contention that her injuries were $100,000. The jury also determined that the motorcyclist was 30% negligent and that the driver was 70% negligent. How much should the motorcyclist recover from the driver? A $100,000 B $70,000 C $56,000 D $50,000

(B) The motorcyclist should recover $70,000 from the driver under a pure comparative negligence system, a contributorily negligent plaintiff is allowed to recover a percentage of her damages. The plaintiff's damages are reduced according to her proportionate share of the fault. Thus, the motorcyclist can recover 70% of her total of $100,000 in damages, because she was 30% at fault, leaving her with a recovery of $70,000. (A) is incorrect because it fails to reflect the reduction in damages required under comparative negligence. Because the motorcyclist was 30% negligent, she cannot recover the entire $100,000. (C) is incorrect because it is derived from an initial reduction of damages by the amount of the insurance payments ($100,000 minus $20,000, leaving $80,000). This $80,000 figure is then reduced by the 30% negligence of the motorcyclist, leaving an amount of $56,000. As a general rule, damages are not reduced or mitigated by reason of benefits received by the plaintiff from other sources, such as health insurance. Thus, the 30% reduction made from the figure of $100,000, not from $80,000. Similarly, (D) is incorrect because it is derived from a reduction of the $70,000 proportionate recovery by the $20,000 insurance payment. As noted above, the insurance payments are not allowed to reduce damages. Therefore, the $20,000 paid by the motorcyclist's insurance company will not reduce the $70,000 in damages to which she is entitled.

Question 6 Removal Intermediate A plaintiff sued a defendant in state court in State A, alleging that the defendant violated the plaintiff's patent on a new type of air conditioner. The defendant removed the case to federal court in State A and quickly lost on the plaintiff's motion for summary judgment. The defendant appeals, claiming that because the case was improperly brought in state court, which does not have jurisdiction over patent cases, it was not removable to federal court, and the federal court thus lacked jurisdiction. The plaintiff argues that no rule requires that the state court have had jurisdiction over a removed case. Who is correct? A The plaintiff, because the defendant waived the argument by seeking removal. B The plaintiff, because the federal court may hear a removed case even though the state court from which the case was removed did not have jurisdiction. C The defendant, because if the state court did not have subject matter jurisdiction, then it could not have ordered removal. D The defendant, because the state court was required to dismiss the action upon finding that it lacked subject matter jurisdiction.

(B) The plaintiff is correct. An action originally filed in a state court may be removed to federal court if: (i) the case could have originally been filed in a federal court; and (ii) for cases removed on the basis of diversity, no defendant is a citizen of the state where the action is filed. The rule is that the federal court may hear a removed case even though the state court from which the case was removed did not have jurisdiction. CMR (28 U.S.C. S1441(f)) (C) and (D) are therefore incorrect. (A) is incorrect because the defendant is not precluded from making this argument even though he sought removal.

Question 5-Judgment/Motion for New Trial - Intermediate The plaintiff sued the defendant in federal court for breach of contract. The case went to trial, and the jury found in favor of the plaintiff and awarded her $125,000. Judgment was entered on June 1. On June 10, the defendant filed a motion fora new trial. On June 18, the plaintiff files to enforce the judgment. The court has not issued any orders since the final judgment on June 1. May the plaintiff enforce the judgment? A Yes, because judgments are enforceable as soon as they have been entered. B Yes, because judgments are enforceable during pendency of post-trial motions unless the court otherwise orders. C No, because judgments are not enforceable until 28 days after entry. D No, because judgments cannot be enforced while a post-trial motion is pending.

(B) The plaintiff may enforce the judgment because judgments are enforceable during pendency of post-trial motions unless the court otherwise orders and on such conditions for the security of the adverse party as are proper. Here, the court has not ordered a stay on enforcement; therefore, the plaintiff can enforce the judgment. (D) is therefore incorrect. (A) and (C) are incorrect because execution on judgments is not allowed for 14 days after entry except in the case of injunctions or receiverships, which are immediately enforceable unless otherwise ordered by the court.

Question 8 - Difficult- Vicarious Liability A facility for mentally ill patients had a high security area for patients deemed to be dangerous. The doors in that section had sophisticated double locks, and the staff was regularly trained in security procedures. However, one night a newly hired employee, who had not yet been instructed in all of the security procedures and had not finished reading the training manual, was left in charge of the high security area. A dangerous patient escaped from the hospital out of a rear door which the employee had not locked. Without any provocation, the patient brutally beat a resident of the town in which the hospital was located. If the resident sues the hospital for his injuries, what is the resident's best argument for a recovery? A The hospital is strictly liable for the actions of its abnormally dangerous patients. B Negligence is inferred from the fact of the patient's escape. C The hospital is vicariously liable for the intentional torts of its patients. D The employee acted negligently when he failed to lock the door.

(B) The resident's best argument is that negligence is inferred from the fact that the patient escaped This choice raises the doctrine of res ipsa loquitur, which allows the trier of fact to infer negligence simply from the fact that a particular injury occurred. Res ipsa loquitur requires the plaintiff to establish that: the event causing the injury is of a type that would not normally occur unless someone was negligent; the negligence was attributable to the defendant; and plaintiff was not at fault regarding his injury. Here, the event causing injury is due to the patient's escape, and it seems that the patient would not have escaped unless someone was negligent (e. g., the employee in not locking the door or the hospital in not making sure that the employee knew of his responsibility to lock the door). The fact that the hospital had sole control over the patient can be used to show that the negligence is attributable to the hospital (through its employees). Finally, the resident was not at fault regarding his injury. Thus, res ipsa could be used to find negligence and allow the resident to recover. (A) is wrong because the hospital is not strictly liable. Strict liability occurs only where there is an absolute duty on the defendant's part to make safe (e.g., in abnormally dangerous activity situations). Hospitals do not have an absolute duty to protect against dangerous acts of their patients. (C) is wrong because there is no basis for holding the hospital vicariously liable. A person or entity can be vicariously liable for the acts of others only if there a relationship between them that gives rise to such liability (e.g., employer-employee). There is generally no such basis for holding a hospital liable for its patients' actions, nor is there any specific reason to do so here. (D) is wrong because it is not clear that the employee acted negligently even if he did fail to lock the door. The employee was under a duty to act reasonably under the circumstances. If he was unaware of the duty to lock the door, he was not negligent in failing to do so. Arguably the employee was negligent in not reading the entire manual, but if the hospital had not told him that it was important for him to read it immediately, he would not be negligent in not finishing the manual.

Question 7 Third Party Claim -Intermediate A homeowner entered into a contract with a builder to construct an addition to the homeowner's house. The builder engaged a subcontractor to perform the carpentry and framing. The homeowner filed a breach of contract action against the builder for defective work on the house. The builder in turn impleaded the subcontractor, claiming the subcontractor was responsible for any defects and seeking indemnity for any sum to which the homeowner is entitled. The subcontractor some months before had done some carpentry work for the homeowner on an apartment building that the homeowner owned, and the subcontractor was never paid for that work. May the subcontractor file a breach of contract claim in the pending action to recover from the homeowner the money due for the apartment building work? A No, because the subcontractor can assert a claim against the plaintiff only if the claim arises under federal law. B No, because the claim does not arise from the same transaction or occurrence as the homeowner's initial claim. C Yes, because the subcontractor must assert the claim in the pending action and will be barred from later asserting the claim as an independent action, since the subcontractor and the homeowner are both parties to the pending action. D Yes, because the subcontractor may at his election assert his claim against the homeowner in the pending action or he may assert it as an independent action.

(B) The subcontractor cannot assert a breach of contract claim in the pending action. A third-party defendant may assert a claim against the original plaintiff only if the claim arises from the same transaction or occurrence that is the subject of the plaintiff's original claims. Here, the subcontractor's claim arises out of the work he did on the homeowner's apartment building, but the homeowner's claim arises out of the work the subcontractor did on the addition to the homeowner's house. Therefore, the subcontractor cannot assert his claim in the pending action, and (C) and (D) are incorrect. (A) is incorrect because whether the subcontractor's claim arises under federal law is not the test for determining whether the subcontractor can assert a claim.

Question 4- Trespass, Defense of Necessity Late one night, a man was walking up a very narrow city street when he noticed that a large garbage truck parked near the top of the hill had begun to roll, driverless, down the street toward him. Seeing that he could not proceed forward and escape the truck, and if he turned and ran downhill, he would soon be overtaken and crushed, he pushed open a window in the apartment building he was passing and climbed inside. The truck narrowly missed him as it careened down the street. The elderly tenant in the apartment, who had a serious heart condition and lived in fear of intruders, thought that the man was a local gang member trying to burglarize his apartment and suffered a major heart attack. If the elderly tenant sues the man, is the elderly tenant likely to recover damages? A Yes, because of the man's negligent infliction of his emotional distress. B Yes, because of the man's trespass on his property. C No, because the man had no alternative to avoid the garbage truck. D No, if a reasonable person would not have been frightened.

(B) The tenant can recover damages because of the man's trespass under the defense of necessity, a person may interfere with the real or personal property of another when the interference is reasonably and apparently necessary to avoid threatened injury from another force and the threatened injury is substantially more serious than the invasion that is undertaken. However, when the act is done not for the general public good but for the benefit of a limited number of people, the defense is qualified; i e , the actor must pay for any injury he causes. Here, although the man was privileged to enter the tenant's apartment to prevent his injury or death (private necessity), the privilege does not extend to the infliction of damages. Thus, he must pay for any injury he caused, including the tenant's damages from his heart attack. (A) is wrong because negligent infliction of emotional distress requires that the distress be caused by negligent conduct by the defendant. These facts give no indication that the man acted negligently. (C) is wrong because, as stated above, the man's privilege is limited. While he cannot be forced off the premises as a trespasser, he is liable for the injury he caused. (D) is wrong because it is irrelevant what a reasonable person would have felt. If the man's actions caused the tenant's injury, he is liable because the defense of private necessity provides only an incomplete privilege.

Question 16 - Conspiracy (Arson) - Intermediate A man and woman agreed to burn down a neighbor's house in retribution for some wrong the neighbor allegedly committed against them. Both the man and woman were arrested shortly after they poured gasoline on the neighbor's front porch. The man revealed to the police that he participated in the plan to ensure that nothing bad would happen to the neighbor, and that he had made an anonymous telephone call to the police alerting them to the crime, which enabled the police to arrest him and the woman "in the act." The woman stated that she would not have participated if not for the man's encouragement. If the woman is charged with a conspiracy at common law to commit arson, how should she be found? A Not guilty, because she was not predisposed to commit the crime but for the man's encouragement. B Not guilty, because the man did not intend to commit arson. C Guilty, because there was an agreement, and pouring gasoline on the front porch was sufficient for the overt act. D Guilty, because arson is not a specific intent crime.

(B) The woman should be found not guilty of a conspiracy to commit arson. To be convicted of a conspiracy at common law, it must be shown that at least two persons agreed to achieve an unlawful objective. Having two or more persons is a necessary element of conspiracy under the traditional bilateral approach. Here, the facts indicate that the man did not intend to achieve the objective of the conspiracy--to burn the dwelling house of another. Thus, the woman cannot be guilty of conspiracy to commit arson. (C) is incorrect. The man feigned his agreement, making the answer factually inaccurate. (D) is also incorrect. Although it is true that arson is not a specific intent crime, conspiracy is a specific intent crime, in that the prosecution must show that the defendant intended to agree and intended to achieve the unlawful objective. Thus, the fact that the underlying crime is not a specific intent crime is irrelevant. (A) is incorrect. Even if the woman would not have committed the crime without the man's inducement, that is not a defense for the woman. A person cannot be entrapped by a private citizen.

Question 24- Intoxication - Advanced A former construction worker became intoxicated one night and decided to move some heavy construction equipment that was parked at a construction site. Ignoring "no trespassing" signs, the worker jumped the fence and climbed into a large dump truck and stalled it. However, even though he knew how to operate the truck, he quickly lost control of it due to his intoxication. It rumbled a short distance and crashed into a trailer housing the main office of the construction site. The worker is prosecuted for recklessly damaging property. A separate statute in the jurisdiction prohibited the unauthorized operation of construction equipment. Should the worker be found guilty of recklessly damaging property? A Yes, because his actions constituted an unlawful operation of the construction equipment. B Yes, because he was intoxicated while attempting to move the construction equipment. C No, because at most he could be found guilty of criminal negligence. D No, because he must have been aware that his conduct would cause the damage to the trailer in order to be found guilty of reckless damage.

(B) The worker should be convicted because he was intoxicated when he damaged the trailer. The worker being charged with reckless damage to property. A person acts recklessly when he consciously disregards a substantial and unjustifiable risk that a prohibited result will follow, and this disregard constitutes a gross deviation from the standard of reasonable care. Attempting to move a large piece of construction equipment while intoxicated should be considered to be reckless conduct because of the great potential for destruction arising from the size and destructive power of the construction equipment. Therefore, (B) is correct (A) is incorrect because merely operating the equipment in violation of the law would not necessarily be reckless. For instance, here, the statute prohibiting unauthorized operation of the equipment likely was enacted to prevent untrained persons from driving dangerous equipment, but the worker was trained to operate the truck in question; thus, if not for the fact that he was drunk, his action would not necessarily have been reckless. Violating the statute may be evidence of negligence, but negligence is insufficient to establish recklessness. (C) is incorrect for the same reason that (B) is correct--driving the equipment while intoxicated constitutes reckless conduct. Although voluntary intoxication is a defense to a crime that requires purpose or knowledge, it is no defense to crimes involving recklessness. Even though the workers condition may in fact have precluded him from being consciously aware of the risk, one who is not consciously aware of a risk only because he was voluntarily intoxicated will be deemed to have acted recklessly with regard to the risk. (D) is incorrect because it states the mental state for knowing conduct--if the defendant is aware that his conduct will necessarily or very likely cause a certain result, he acts knowingly with respect to that result. Recklessness is a lesser standard of fault.

Question 18- Conspiracy-withdrawal - Intermediate The police of a resort town discovered that a well-known cat burglar was currently living in town under an assumed name. To try to catch her in the act of burglary, an undercover officer approached the burglar with a plan for a burglary. The undercover officer told the burglar that he knew who she was and that he had a plan to steal from someone staying in one of the town's resorts. The burglar initially refused the offer; however, after lengthy cajoling, she finally agreed to the plan. As the time for the burglary drew near, the burglar had second thoughts. Three hours before the theft was scheduled to take place, the burglar called the police and told them of the plan. She told them that she was not going to show up, but that her cohort (the undercover officer) would be there, and told them how to recognize the undercover officer. Is the burglar guilty of conspiracy at common law? A Yes, because the burglar made an agreement with the undercover officer to commit the theft. B No, because there was no agreement. C No, because the intended crime was never completed. D No, because the burglar effectively withdrew.

(B) There was an insufficient agreement for conspiracy liability at common law. Conspiracy consists of: (i) an agreement between two or more persons; (ii) an intent to enter into an agreement; and (iii) an intent to achieve the objective of the agreement. In addition, most states require an act in furtherance of the conspiracy, although an act of mere preparation will usually suffice. The agreement requirement means that the parties must agree to accomplish the same objective by mutual action. There must be a meeting of at least two "guilty minds"; i.e., between two or more persons who are actually committing themselves to the scheme. If one person in a two-party conspiracy is only feigning agreement, the other person cannot be convicted of conspiracy under the common law bilateral approach. Here, the officer, in his undercover capacity, was simply trying to set up a situation in which the burglar would be caught. Thus, the undercover officer merely pretended to reach an agreement with the burglar to commit a burglary. At no time did the undercover officer actually commit himself to the burglary. Therefore, there could have been no agreement of two "guilty minds." Absent the requisite agreement, the burglar cannot be guilty of conspiracy. (A) is incorrect because, as explained above, there was no agreement sufficient for a conspiracy conviction, since the undercover officer never intended to commit the burglary. (C) is incorrect because completion of the substantive crime is not necessary for a conviction of conspiracy. Consequently, although the actual burglary was not consummated, this would not preclude a conviction of conspiracy to commit burglary. (D) is incorrect because withdrawal is not a defense to a charge of conspiracy. Note that, by withdrawing, a person may limit her liability for subsequent acts of the other members of the conspiracy. However, this question pertains to the burglar's potential guilt for conspiracy. As applied to the conspiracy charge, withdrawal will not afford a defense to the burglar.

Question 9- Venue- Intermediate A resident of State A (which has only one judicial district) properly brought a diversity action in federal court against a resident of the Western District of State B and a resident of the Eastern District of State C for a cause of action that arose from events that occurred in the Northern District of State D. In which judicial districts is venue proper? A Only in the Western District of State B and in the Eastern District of State C. B Only in the Northern District of State D. C Only in the District of State A. D In the Western District of State B, the Eastern District of State C, and the Northern District of State D.

(B) Venue is proper only in the Northern District of State D, where the events giving rise to the claim occurred. (A) and (D) are incorrect because venue is not proper in the judicial district of either defendant's residence; the rule states that venue is proper in "a judicial district where any defendant resides, if all defendants reside in the same state." Since the defendants reside in different states, neither of their judicial districts of residence is proper in a single action. (C) is incorrect because State A is not a proper venue because a plaintiff's residence is not a valid basis for fixing venue.

Question 9- Burglary - Intent- Intro Even though the gambling laws of the state prohibit gambling on professional sports games, the defendant placed a bet with a bookie on the outcome of a football game. There was a disputed call near the end of the game that resulted in the defendant losing the bet. However, later films showed that in fact the call should have been for the defendant's team, which would have changed the outcome. The bookie refused to pay the bet to the defendant. Later that night, the defendant broke into the bookie's home and took the amount he would have won. What is the defendant's best defense to a charge of common law burglary? A He was so enraged that he had an irresistible impulse to take the money. B Since the original gambling agreement was illegal the two are in pari delicto and the court should not interfere. C He lacked the specific intent necessary for burglary because he believed that the bookie owed him the money. D He had a mistaken belief that the definition of burglary would not apply to a dispute over gambling winnings.

(C) A belief that the money was rightfully his is the defendant's best defense, because the definition of common law burglary requires that the defendant break into the dwelling place with the intent to commit a felony. If the defendant actually believed that the bookie owed him the money, he could not have been committing larceny because there was no intent to deprive another of his property. Absent the intent to commit a felony, the defendant would not have had the specific intent necessary to be found guilty of burglary. (A) is incorrect because the "irresistible impulse" must be a product of some mental disease as opposed to anger. (B) is incorrect because the theory of pari delicto is a tort theory, not a criminal defense. (D) is incorrect because a mistake as to the law is not an excuse for an act otherwise criminal.

Question 5 -Warrant Requirements - Intro A plainclothes police officer who frequently ate lunch at a certain deli heard rumors that the deli's owner often placed illegal bets on sporting events. Based on the rumors, the officer peeked into an envelope next to the register and saw betting slips. The officer asked his waitress about the envelope, and she told him that the owner had given it to her and that a man in a brown cap was to pick it up at 2 pm. The officer stayed at the deli until 2 pm and watched the waitress hand the envelope to a man in a brown cap. The officer passed the above information on to a friend in the F.B.I. Several weeks later, based on that information, F.B.I. agents obtained a search warrant for the owner's home, described in the warrant as a condominium in a large multi-unit complex identified by the street address only. The agents went to the home in the early evening, while the owner was at the deli. After announcing their purpose to the owner's wife, they searched the home and found betting slips and other materials related to illegal gambling. The owner was indicted for conspiracy to violate a federal statute prohibiting the use of interstate phone lines to conduct gambling, and for the possession of betting slips. Which of the following would be the best reason for excluding the evidence found at the owner's home? A The owner was not home when the warrant was executed. B The search was conducted in the evening when it easily could have been conducted during daylight hours. C The warrant failed to specify which condominium unit was to be searched. D The waitress had never been used before as an informant.

(C) If the warrant was invalid, any evidence obtained thereunder will be excluded. A warrant must be based upon a showing of probable cause. When requesting a warrant, officers must submit to a magistrate an affidavit setting forth sufficient underlying circumstances to enable the magistrate to make a determination of probable cause, independent of the officers' conclusions, that evidence of a crime will be found at the premises. Also, a warrant must describe with reasonable precision the place to be searched and any items to be seized. A finding that a warrant was invalid because it was not supported by probable cause will not entitle the defendant to exclude evidence obtained under the warrant if the police have acted in good faith and reasonable reliance on a facially valid warrant. However, a police officer cannot in good faith rely on a defective search warrant if: (i) the affidavit underlying the warrant is so lacking in probable cause that no reasonable police officer would have relied on it; (ii) the warrant is defective on its face (e g, it fails to state with particularity the place to be searched or the things to be seized); (iii) the affiant lied to or misled the magistrate; or (iv) the magistrate has wholly abandoned his judicial role. Because the deli owner lives in a condominium, the warrant should have specified which unit in the multi-unit dwelling was to be searched. Because the warrant did not so specify, it failed to describe with sufficient particularity the place to be searched. Such an absence of precision renders the warrant defective on its face, so that the FBI agents cannot be said to have relied in good faith on a facially valid warrant. Thus, the evidence obtained pursuant to this facially defective warrant will be excluded. (A) and (B) each incorrectly imply that there was something wrong with the execution of the warrant. A warrant must be executed by the police without unreasonable delay, with the police knocking and announcing their purpose (unless they reasonably believe that such notice will endanger them or lead to the destruction of evidence). The FBI agents conducted themselves in accordance with these standards. The fact that a warrant was executed in the evening or at a time when a particular person was not on the premises will not invalidate a search conducted pursuant to the warrant. Here, the agents announced themselves to the deli owner's wife, and they were not required to wait until the owner returned in order to conduct their search. (D) is incorrect because it states a fact that will not, in and of itself, invalidate a warrant and a search pursuant thereto. The sufficiency of a search warrant affidavit based on an informer's hearsay is evaluated under the totality of the circumstances. The informer's reliability and credibility, as well as her basis of knowledge, are all elements that may illuminate the issue of probable cause, but they are not strictly separate requirements. Had the waitress been used before as an informant, and been previously found to be reliable, this would have been one factor in determining her present reliability. However, the fact that the waitress had not previously served as an informer does not invalidate the warrant. Given the specificity and accuracy of the waitress's information (a man in a brown cap would pick it up at 2 pnm), there was enough of a basis for her information to establish sufficient credibility to permit a magistrate to make a determination of probable cause.

Question 11- Strict Liability - Intro A liquor store clerk was working the counter when a young girl came up with a bottle of wine to purchase. The clerk thought that she looked too young to buy wine, so he asked her for her driver's License. The young girl reached into her purse as if to get her license. As the clerk reached under the counter to get a bag, the girl grabbed the bottle of wine, threw $10 on the counter, and ran out. In this state, it is a misdemeanor to sell wine or liquor to a person under the age of 21. If the clerk is prosecuted for violation of this statute, what is his best defense? A He asked for a driver's license. B He did not know that the girl was actually a minor. C He did not sell her the wine. D He was only a salaried employee rather than the owner and benefits in no way from the transaction.

(C) In most states, the sale of liquor to someone underage is a strict liability crime. Therefore, the clerk's best defense is that he did not sell the young girl the wine. (A) is not a valid defense, because under a strict liability statute it is illegal to sell to a minor even if she presented a false identification. Thus, if the clerk had actually made a sale to the young girl, he would have violated the statute even if he had asked for her driver's license. Similarly, (B) is a wrong answer because under a strict liability statute it is illegal to sell to a minor even if the defendant did not know that the purchaser was a minor. Even if the statute were not a strict liability crime, (C) would be his best defense. (B) would only be a defense if the statute required knowledge of age. (D) ignores the stated facts that the seller of the liquor is the guilty party, not the seller's employer.

Question 17- Assault- Intermediate A blacksmith ran a small forge in a tourist attraction depicting village life in the 1800s, and produced small metal trinkets for sale as souvenirs. A tourist came into the forge and stalled ridiculing the blacksmith, telling him that he was foolish for practicing such an out-of-date trade when modern equipment could produce the same trinkets faster and far more cheaply. Although he maintained a calm demeanor, the blacksmith was enraged by the time the customer finished and headed back out the door. He picked up an anvil and hurled it in the general direction of the Customer. The anvil fell harmlessly to the ground after traveling maybe a foot. If the blacksmith is charged with assault, which of the following statements would be most helpful for his defense? A The blacksmith did not succeed in hitting the customer with the anvil, and he knew that it was impossible to do so. B The blacksmith knew that it was impossible to hit the customer with the anvil. C The customer did not see the blacksmith throw the anvil, and the blacksmith knew that it was impossible to hit the customer with the anvil. D The customer did not see the blacksmith throw the anvil.

(C) That the customer did not see the blacksmith throw the anvil, and that the blacksmith knew it was impossible to hit the customer with the anvil, would be most helpful to the blacksmith's defense. For purposes of the MBE, an assault is either (i) an attempt to commit a battery, or (ii) the intentional creation, other than by mere words, of a reasonable apprehension in the mind of the victim of imminent bodily harm. The fact that the blacksmith knew that it was impossible to hit the customer with the anvil negates the specific intent to commit a battery that is required for the first type of assault (If the blacksmith knew that, when he threw the anvil, it was impossible to hit the customer, the blacksmith's conduct was not motivated by the intent to commit a battery against the customer). The fact that the customer did not see the blacksmith throw the anvil negates the second type of assault because no apprehension of harm would have been created in the customer if he did not see the blacksmith throw the weight. Because the type of assault is not specified here, (C) is a better choice than (B) or (D) because both types of assaults are negated. Choice (A) is not correct because the fact that the blacksmith failed in his attempt to hit the customer with the anvil establishes only that there was a failure to commit a battery. It does nothing to negate the blacksmith's potential liability for assault/attempted battery.

Question 7 - Larceny-Intro After a business lunch at a restaurant, a patron went to the restaurant's cloakroom to get his coat. He removed a coat he believed to be his own from the coat rack. The coat was of similar color and of the same brand as the patron's own coat, but it actually belonged to another customer. The patron walked back to his office and removed the coat. As he was hanging it up, he noticed that another person's name was written on the inside of the collar. He immediately left the office and sprinted back in the direction of the restaurant, intending to return the coat and pick up his own. As he crossed the street, the patron was struck by a car. Although his injuries were minor, the coat was destroyed. If the patron is acquitted of larceny, what is the best reason for his acquittal? A The coat was destroyed through no fault of his own. B The coat looked so much like the patron's coat that his mistake was reasonable. C The patron thought the coat was his when he took it. D When he realized the coat was not his, the patron tried to return it.

(C) The best reason for the patron's acquittal is that he thought the coat was his when he took. Because the patron believed the coat to be his own, he lacked the intent to commit larceny. Larceny consists of a taking and carrying away of the tangible property of another by trespass, with the intent to permanently deprive the person of his interest in the property. The intent to deprive the owner of his property generally must exist at the time of the taking (except for the continuing trespass situation; see below). Here, the patron took and carried away a coat belonging to another. But when he took it, the patron believed that the coat was actually his own coat. Thus, the patron did not take the coat with the intent to permanently deprive the owner of his property. Absent this intent, the patron cannot be guilty of larceny. (A) is incorrect because although the patron did not actually destroy the coat himself, he could still be guilty of larceny. As mentioned, larceny requires the intent to permanently deprive the owner of the property. This intent includes intentionally dealing with property in such a way as to create a substantial risk of loss. Thus, even if the patron did not destroy the property himself, if he dealt with it in a way as to risk its destruction, he could still be guilty of larceny, and so this is not the best reason for his acquittal. (B) is incorrect because the reasonableness of the mistake is not relevant. The test for whether a defendant has the intent for larceny is a subjective one - what this defendant intended. Thus, even if the mistake were unreasonable, the patron would not be guilty of larceny if he believed the coat was his. (D) is incorrect because it attempts to negate the continuing trespass situation, which is not applicable here. Under the continuing trespass theory, if a defendant wrongfully (i.e., with a "bad" mental state) takes the personal property of another without the intent to permanently deprive the owner, but later, while still in possession of the property, decides to keep it, he is guilty of larceny. This theory does not apply here because the patron's taking of the coat was not wrongful. Thus, it is irrelevant that he was trying to return the coat; even if he had decided to keep it, he would not be guilty of larceny, although he would be guilty of some other crime. Therefore, (D) is not the best reason for the patron's acquittal because it raises irrelevant issues.

Question 1- Intermediate -Joint and Several with divisible Injury A bicyclist was riding his bicycle in the street when a negligently driven car struck the bike, knocking the bicyclist off the bike and breaking his right ankle. The driver of the car immediately stopped and went to his assistance. She got him to his feet and was slowly moving him toward the curb when a negligently driven taxicab struck him in the left leg. The bicyclist required surgery on both his right ankle and his left leg. If the bicyclist sues the driver and the cabbie, which of the following statements is most accurate? A The bicyclist can recover from either the driver or the cabbie for all of his injuries because the driver and the cabbie are jointly and severally liable. B The bicyclist can recover from the driver only for the injury to his right ankle and recover from the cabbie only for the injury to his left leg. C The bicyclist can recover from either the driver or the cabbie for the injury to his left leg and recover from the driver only for the injury to his right ankle. D The bicyclist cannot recover against the driver for the injury to his left leg unless the jury determines that the driver acted negligently when she came to his aid.

(C) The bicyclist can recover from either party for the left leg injury but only from the driver for the right ankle injury. When two or more tortious acts combine to proximately cause an indivisible injury to a plaintiff, each tortfeasor is jointly and severally liable to the plaintiff for the entire damage incurred. Joint and several liability applies even though each tortfeasor acted entirely independently. However, if the actions are independent, plaintiff's injury is divisible, and it is possible to identify the portion of injuries caused by each defendant, then each will be liable only for the identifiable portion. Here, the cabbie would not be liable for the injury to the right ankle, because the cabbie did not cause the injury. (A) therefore incorrect. With regard to the left leg, the cabbie was not the only cause of that injury. The original tortfeasor is liable for harm caused by the negligence of third persons when such negligence was a foreseeable risk created by the original tortfeasors conduct. Here, as a result of the driver's original negligence, the bicyclist was in a position of danger while he was still in the street. The negligence of the cabbie in striking the bicyclist was a foreseeable risk while the bicyclist was in the street; it is therefore a foreseeable intervening force that will not cut off the driver's liability. Hence, both the driver and the cabbie will be jointly and severally liable for that injury. (B) is therefore incorrect. (D) is incorrect because the driver remains responsible for the foreseeable consequences of her original negligence in striking the bicyclist, regardless of whether she acted with due care when she came to his aid

Question 5 -Vicarious Liability -Intermediate A professional painter and his apprentice, in business as a partnership, were hired to paint a store. Midway through the job they ran out of paint, so the painter lent his truck to the apprentice to pick up more. On his way to pick up the paint, the apprentice stopped at a post office along the way to mail a personal letter. On pulling out of the post office parking lot, he negligently ran into a parked car, causing extensive damage. If the car owner brings a negligence action against the painter, will she prevail? A No, because the apprentice is an independent contractor. B No, because a bailor is not vicariously liable for the tons of his bailee. C Yes, because the apprentice's stop at the post office was not a frolic. D Yes, because the apprentice was using the painter's truck.

(C) The car owner can recover because the apprentice was acting within the scope of the partnership business when he drove negligently. Vicarious liability for the conduct of another can arise in partnership and joint venture situations. Each member of the partnership is vicariously liable for the tortious conduct of another partner committed in the scope of the partnership's affairs. As with respondeat superior situations, if the tortfeasor has gone off on a frolic of his own, he is no longer acting within the scope of the partnership and the other partners will not be liable. On the other hand, a minor deviation from the partnership activity will not take it outside of the scope of the partnership's affairs. Here, the painter and the apprentice are partners in their painting business. The apprentice's detour to a post office along the way simply to mail a letter did not take his activity outside the scope of the partnership's affairs; hence the painter is vicariously liable simply because of his status as a partner. (A) is incorrect because the apprentice's status as an independent contractor is irrelevant to this question. While the apprentice is, with respect to the person who hired him, an independent contractor, the apprentice is a partner rather than an independent contractor as to the painter. Because this question concerns the car owner and the painter, the apprentice's status as to the person who hired him is irrelevant and would not save the painter from liability. (B) is a true statement (a bailor is not vicariously liable for the torts of his bailee). However, (B) is incorrect because it does not take into account the status of the apprentice and the painter as partners. It is because of their partnership status rather than their bailor- bailee status that the car owner might be able to recover. (D) is incorrect because the painter's ownership of the truck is not enough to make him liable for the apprentice's torts. Absent a statute to the contrary, a vehicle owner not vicariously liable for the apprentice's tortious conduct of another driving his vehicle.

Question 17- Conspiracy Unilateral Approach - Intermediate A minor entered a liquor store and asked the cashier to let him purchase a case of beer for a party even though he was underage. The cashier agreed as long as he paid double the retail price for the beer as compensation for the risks of the sale. As the cashier stalled to ring up the transaction, an undercover officer who overheard them intervened and arrested them. The state criminal code provides that it is a felony to knowingly provide alcohol to any person under the age of 21. If the state follows the unilateral theory of conspiracy, of what crimes can the minor and cashier be convicted? A The minor can be convicted of solicitation and the cashier can be convicted of conspiracy to violate the statute. B Both the minor and the cashier can be convicted of conspiracy to violate the statute. C The minor cannot be convicted of either solicitation or conspiracy, but the cashier can be convicted of conspiracy to violate the statute. D The minor cannot be convicted of either solicitation or conspiracy, and the cashier cannot be convicted of conspiracy.

(C) The cashier can be convicted of conspiracy in a unilateral jurisdiction even though the minor can be convicted of neither solicitation nor conspiracy. Under the unilateral approach adopted by the Model Penal Code, the crime of conspiracy requires that only one party have genuine criminal intent, and may be shown by proof that the defendant agreed with another to commit a crime, even if the other person does not share the commitment. Thus, the fact that no other party to the conspiracy could be found guilty does not prevent the defendant from being convicted of conspiracy. Here, the fact that the minor is a member of the class that the statute was designed to protect prevents him from being found guilty of conspiracy. (B) is therefore incorrect. However, this fact has no bearing on the cashier's liability for conspiracy under the unilateral approach. Thus, (C) is correct and (D) is incorrect (A) is incorrect because the crime of solicitation is treated the same as conspiracy. If the solicitor could not be guilty of the completed crime because of legislative intent to exempt him, he cannot be guilty of solicitation of the crime. Because the minor is a member of the class intended to be protected by the statute, he cannot be found guilty of soliciting the cashier to complete it.

Question 5- Venue - Intermediate A homeowner from State A hired a contractor from State B to build a vacation home for her in State C. The parties signed the contract in State A. The contractor breached the contract, and the homeowner sued the contractor in a court of State A, seeking damages of $100,000. The contractor removed the case to the federal court for State A. The homeowner promptly moved to remand the case to state court, arguing that venue was improper. Which of the following facts is most relevant to the court's decision on the homeowner's motion? A The contract was signed in State A. B The contractor resides in State B. C The homeowner commenced the action in a State A court. D The vacation home was to be built in State C.

(C) The commencement of the action in State A is most relevant. When a case is removed from state court to federal court, venue is set in the federal district court that embraces the state court in which the action was pending, making the federal district court of State A the only appropriate venue choice. Thus, (C) is the correct answer because the most (indeed, only) relevant fact is the fact that the action was commenced in a court of State A. Unlike cases commenced in federal court, in cases removed to federal court, the residence of the defendant contractor (answer choice (B)) and the place where a substantial part of events relating to the claim occurred (answer choices (A) and (D)) are irrelevant.

Question 5 - Arson and Attempted Larceny-Intro A student started a small fire in a trash can in the men's room at his university. His plan was to set off the school's fire alarms so that he could break into the computer lab and steal a laptop computer while the building was being evacuated. The student was stopped after he had set the fire and was attempting to smash in the glass on the computer lab door. The fire was quickly extinguished and no serious damage was done to the building by the fire beyond some charring on the walls in one stall of the men's room. A statute in the jurisdiction extends the crime of arson to buildings other than dwellings. Which of the following best describes the crimes of which the student could be properly convicted? A Larceny only. B Larceny and attempted arson. C Attempted larceny and arson. D Arson only.

(C) The student is guilty of the crimes of arson and attempted larceny. At common law, arson was defined as the malicious burning of the dwelling of another. The mens rea required for arson is malice, which is broader than the intent required for specific intent crimes. All that malice requires is that the defendant have acted with the intent or knowledge that the structure would burn, or with reckless disregard of an obvious risk that the structure would burn is sufficient if he intended a burning that creates an obvious fire hazard to the building. Here, the student's intent was to set off the fire alarm so that the building would be evacuated. Nevertheless, he intended to start a fire with reckless disregard of a high risk that it would cause damage to the building. The risk or hazard is not that the building will burn down, merely that damage to the structure from a burning will occur. The "burning" required for arson does not require significant damage to the building; a charring of the combustible material is sufficient. Here, the wall of a stall in the men's room was charred. This satisfies the "burning" requirement. The common law requirement that the structure be a dwelling has been broadened by the statute in this question to include other buildings. Thus, the student's conduct satisfies all of the elements of the crime of arson. The student is also guilty of attempted larceny. Common law larceny requires a taking and carrying away of the personal property of another by trespass with intent to permanently (or for an unreasonable time) deprive the other of his interest in the property. Here, the student planned to take and carry away a laptop computer by trespass (i.e., without permission) and with the intent to permanently deprive the university of it. However, he cannot be convicted of a completed larceny because he never actually took the laptop. (A) and (B) are therefore incorrect. At common law, the crime of attempted larceny required both a specific intent by the actor to commit a larceny and an act that put the defendant in close proximity to completing the crime. The Model Penal Code and most state criminal codes modify the "proximity" test for the act requirement, instead requiring an act that constitutes a "substantial step" towards commission of the crime. Here, the student intended to do what he did in order to commit a larceny for which he had the requisite intent. Also, he was apprehended after he set a fire to cause the building's evacuation and while he was smashing the glass on the computer lab door. Together, these acts satisfy the act requirement for attempt liability, regardless of which test is employed. Thus, the student is liable for attempted larceny and arson, making (C) correct and (D) incorrect.

Question 7-Renewed Judgment as a Matter of Law-Intermediate A homeowner is suing a contractor in federal court for fraud and misrepresentation. Subject matter jurisdiction is based on diversity, and is clearly appropriate. The homeowner presented three witnesses, while the contractor presented one witness. The contractor's witness is impressive on the stand, and all observers believe that the contractor will prevail. The case then goes to the jury without any motions being made by either party, and the jury returns a verdict for the homeowner. If the contractor moves for a renewed motion for judgment as a matter of law, will the contractor be successful? A Yes, if the court also believes the contractor's witness was the more credible witness. B Yes, if the homeowner cannot reopen her case to present more evidence. C No, because the contractor did not meet the procedural requirements. D No, because a jury verdict based on witness testimony cannot be overturned.

(C) The contractor will not be successful in moving for a renewed motion for judgment as a matter of law because he did not meet the procedural requirements for doing so. In most courts, a party may not move for judgment notwithstanding the verdict (JNOV) unless he has previously moved for a directed verdict at some time during the trial. In federal court terminology, a party may not make a renewed motion for judgment as a matter of law unless he has previously moved for judgment as a matter of law at some point during the trial on the issue. (Fed. R. Civ. P. 50) Hence, (A) and (B) are incorrect. (D) is incorrect because there is no such rule.

Question 12 - Arson & Burglary - Intermediate The owner of a furnished cottage leased it to another for one year. While this lease was in effect, the cottage owner found herself in immediate need of cash, and decided to burn down the cottage to collect the insurance on it. She waited until one evening when the tenant was away. The cottage owner then used her own key to gain access to it. To make it appear that the fire was caused accidentally by the tenant, she soaked one end of the mattress on the bed in the bedroom with gasoline and then left a lighted cigarette burning at the other end of the mattress. She planned that the cigarette would ignite the mattress and that when the fire smoldered to the area soaked in gasoline, the entire bed would burst into flames, and the resulting fire would destroy the house. However, the tenant returned home earlier than expected and discovered the fire just as the mattress burst into flames. He immediately put it out with a fire extinguisher. A police investigation revealed the cottage owner's activities. What crime(s), if any, has the cottage owner committed? A Burglary as to the house and arson as to the mattress. B Neither burglary nor arson because she owned the structure and its contents. C Burglary and attempted arson. D Attempted arson but not burglary because she entered with her own key.

(C) The cottage owner is guilty of burglary because the right of occupancy belonged to the tenant. However, the fact that there was no burning of the structure means that the cottage owner is guilty of attempted arson, rather than arson. Burglary at common law is a breaking and entering of the dwelling of another at nighttime, with the intent of committing a felony therein. A breaking requires some use of force to gain entry, but minimal force is sufficient. In determining whether the dwelling is that of another, occupancy rather than ownership is material. Thus, an owner can commit burglary of her own structure if it is rented and used as a dwelling by someone else. Here, although the cottage owner owned the cottage, the tenant had the right to occupy it pursuant to a lease. Thus, for purposes of the crime of burglary, the cottage owner is deemed to have entered the dwelling of another. Although the cottage owner used her own key to gain access to the cottage, this was still an unconsented use of force to effectuate entry, thereby constituting a breaking. This breaking and entering of the tenant's dwelling occurred in the evening. At the time of the entry, the cottage owner intended to commit the felony of arson. Consequently, all the elements of burglary are in place, making her guilty of this crime. Arson consists of the malicious burning of the dwelling of another. There is a requirement of some damage to the fiber of the wood or other combustible material. As with burglary, ownership of the structure not material for determining whether the dwelling is that of another; rather, the right to occupancy is material. The cottage owner left a lighted cigarette on the mattress, intending to burn down the entire cottage. However, the tenant extinguished the fire before any damage was done to the structure of the cottage, even mere charring. Absent such damage, arson cannot have been committed. The cottage owner did commit attempted arson. A criminal attempt is an act which, although done with the intention of committing a crime, falls short of completing the crime. The defendant must intend to perform an act and obtain a result that, if achieved, would constitute a crime. Also, the defendant must have committed an act beyond mere preparation for the offense. The cottage owner intended to perform an act that would have culminated in the crime of arson. By soaking the mattress with gasoline and leaving a lighted cigarette on it, the cottage owner committed an act that came dangerously close to successfully burning the cottage. This act, in combination with the intent to commit arson, means that the cottage owner is guilty of attempted arson. (A) is incorrect because there can be no arson as to the mattress. Arson requires a burning of a dwelling. Because the cottage was not burned, the cottage owner is not guilty of arson. (B) is incorrect because the key element in determining whether a dwelling is that of another, for both arson and burglary, is the right of occupancy under the terms of his lease, the tenant had the right to occupy the cottage for one year. Therefore, the cottage owner's ownership of the cottage will not be a defense to either arson or burglary. (D) is incorrect because, as explained above, the cottage owner's use of a key to gain access to the cottage without the consent of the person who had the right of occupancy is deemed to be a use of force to gain entry, in the same way as if a person who did not own the cottage were to gain entry by means of a key.

Question 1 -Erie-Intermediate A restaurant owner in State A bought large freezers from a manufacturer of commercial refrigeration equipment with its principal place of business in State B. Within one week and after being fully stocked with meat, one of the freezers broke down. The restaurant owner filed a state-based products liability action against the manufacturer in federal court in State A, and included a demand for a jury trial. Under the law in State A, jury verdicts do not need to be unanimous, but the Federal Rules of Civil Procedure require jury verdicts to be unanimous. At trial, the restaurant owner makes a motion asking the court to apply the State A law. How should the court rule on the motion? A Grant the motion, because applying the federal rule may change the outcome of the case. B Grant the motion, because, when a federal court has diversity jurisdiction, it is required to apply the substantive law of the state in which it is sitting. C Deny the motion, because the Federal Rules of Civil Procedure apply in federal court as long as they are consistent with the Rules Enabling Act and not unconstitutional. D Deny the motion, because the Supreme Court's balancing factors indicate that federal law should apply.

(C) The court should deny the motion. Under the Erie doctrine, when a state law-based claim is brought in federal court based on diversity of citizenship, the federal court generally applies the substantive law of the state in which it is sitting. However, where a specific federal statute or the Federal Rules of Civil Procedure are on point, the federal court must apply federal procedural law as long as the federal rule is valid. Under the Rules Enabling Act, a Federal Rule is valid if it deals with "practice or procedure" and does not "abridge, enlarge, or modify" a substantive right. Here, there is a specific federal procedural rule that is on point (Fed. R. Civ. P. 481, which requires jury verdicts to be unanimous, unless the parties agree otherwise. Since there is no evidence of agreement, the federal procedural rule will apply, and the motion should be denied. (A) is wrong because it states the wrong conclusion, and it incorrectly cites the "outcome determinative" test. This is the test that may be applied when there is no federal procedure law on point, and it is unclear whether the state matter is substantive or procedural. As stated above, there is a federal procedural rule on point. Therefore, this test does not apply. Similarly, (D) is wrong because it is referring to the balance of interests test, which is another test that may be applied when there is no federal procedural law on point, and it is unclear whether the state matter is substantive or procedural. This test is not applicable here, however, because there is a federal procedural rule on point. (B) is wrong because, although it is a true statement of law that a federal court with diversity jurisdiction is required to apply the substantive law of the state in which it is sitting, the requirements for a jury verdict is a procedural rule, not a substantive rule. Therefore, as set forth above, the federal procedural rule governs.

Question 12 - Insanity- Intro The defendant was charged with murder in a jurisdiction that has modeled its criminal code after the Model Penal Code, and is claiming insanity as a defense. In these cases, the state takes a "bifurcated trial" approach. The first part of the trial determines the defendant's guilt or innocence, and the second part determines whether the defendant was legally sane at the time the crime was committed. At the guilt phase of the trial, the prosecution produced three witnesses who testified that they had seen the defendant shoot the victim. The defense put the defendant on the stand, and the defense attorney asked him if he shot the victim. The defendant that he did not shoot a man, and that he had shot a tiger instead. The prosecution immediately objected, and moved that the defendant's testimony be stricken. How should the court rule on that motion? A Granted, because the defendant's purported delusions are only relevant to the second part of the trial. B Granted, if the judge is convinced that the defendant is lying. C Not granted, because the testimony is relevant to the defendant's mens rea. D Not granted, because the testimony is relevant to the defendant's conduct.

(C) The court should not grant the motion because the testimony is relevant to the defendant's mens rea. The defendant's testimony makes it more likely than not that he did not intend to kill or inflict great bodily injury to a human being, thus negating the malice aforethought required for murder. The events here take place during the first stage of the bifurcated trial process. During this stage, there is a determination of the perpetrator's guilt; [e.g., did the perpetrator perform a criminal act with the requisite mental state? Here, the defendant is on trial for murder. Murder is the unlawful killing of a human being with malice aforethought. Malice aforethought exists if the defendant has any of the following states of mind: (i) intent to kill; (ii) intent to inflict great bodily injury; (iii) awareness of an unjustifiably high risk to human life; or (iv) intent to commit a felony. Thus, at this first stage of the bifurcated process, it must be determined whether the defendant shot the victim with malice aforethought, thus causing the death. Although the bifurcated trial process is designed to separate, as much as possible, the issues of his guilt and his sanity, the defendant's testimony is being offered not as evidence of his insanity, but rather as evidence that, because he believed he was shooting at a tiger, he did not have the intent to kill or inflict great bodily injury to a human being. The distinction may be somewhat fine, but is in accord with the modern approach followed by the Model Penal Code. Because the defendant's testimony tends to show that, at the time of the shooting, he did not have the requisite mens rea for murder, such testimony is admissible in the "guilt" phase of the bifurcated trial process, and the prosecution's motion to strike the testimony is denied. (A) is incorrect because, as has been explained, the defendant's purported delusions are relevant to the first phase of the trial, as well as to the second, because they relate to his malice aforethought, or lack thereof. (B) is incorrect because the judge's opinion as to whether a witness is lying does not determine the admissibility of the witness's testimony (but will affect the weight to be given that testimony if the judge is the trier of fact). The defendant's testimony is relevant to his mens rea and is thus admissible even if the judge believes that the defendant is lying. (D) incorrect because the testimony not relevant to his conduct, in that it does not make more or less probable anything concerning the conduct itself. It is apparently undisputed that the defendant shot the victim. At issue is the existence of intent on the part of the defendant at the time of the shooting. It is to this matter that the defendant's testimony is relevant.

Question 11 -Embezzlement - Intermediate The defendant was fired from his sales job while calling on customers in another city. He failed to return the company car that he was using for his sales visits; instead, he sold the car to a "chop shop" for cash. As to the car, what crime has the defendant committed? A Larceny B Larceny by trick C Embezzlement D Theft by false pretenses

(C) The defendant has committed embezzlement because a court will probably find that he had "possession" of the car at the time he appropriated it. Embezzlement is the fraudulent conversion of the property of another by a person in lawful possession of it. In the instant case, the car belonged to the company for which the defendant worked, but the company probably gave the defendant lawful possession of it so that he could make sales calls. When he sold the car rather than return it, he wrongfully converted the car. This is embezzlement. (A) is incorrect. Larceny is the taking and carrying away of the property of another by trespass with the intent to permanently deprive the person of the property. The intent to permanently deprive must be concurrent with the taking and carrying away. In the instant case, the defendant did not have the intent to permanently deprive the company of the car when he was given the car. Thus, the intent element is missing. (B) is also incorrect. Larceny by trick is a specialized form of larceny. For larceny by trick, the defendant must acquire possession of the property by some misrepresentation concerning a present or past fact. The defendant did not come into possession of the car by misrepresentation, making (B) incorrect. For similar reasons, (D) is also incorrect. The crime of false pretenses is the obtaining of title to the property of another by an intentional (or knowing) false statement of pastor existing fact with the intent to defraud. As with larceny by trick, the defendant here came into possession of the vehicle without use of a misrepresentation. Furthermore, he also never obtained title to the car. Thus, D is an incorrect answer.

Question 9-Mandatory Physical Examination of Witness/Discovery- Intermediate The plaintiff filed a negligence action against the defendant to recover compensatory damages for injuries sustained in a car accident. The defendant believes that the plaintiff's key witness has such poor eyesight that she could not possibly have seen the things that she claims to have seen. May the defendant require the witness to undergo an eye exam as part of discovery? A Yes, because the witness is a pivotal eyewitness and her eyesight is relevant to a claim or defense of a party. B Yes, but only if the defendant obtains a court order requiring the eye exam, but the court is likely to issue the order because of the witness's importance. C No, because the witness is not a party to the action. D No, because the discovery rules do not provide for obtaining any sort of physical exam against the wishes of the person examined.

(C) The defendant may not require the witness to undergo an eye exam because the witness is not a party to the action. A party may request a physical or mental exam of another party when that party's physical or mental condition is in controversy. The exam must be ordered by the court on a showing of good cause. (A) and (B) are therefore incorrect because only a party may be required to undergo an exam. (As a side note, remember that a nonparty may be deposed. During the deposition, the witness may be asked about her medical condition. Furthermore, it also might be possible to obtain copies of the witness's medical records by means of a subpoena or subpoena duces tecum.) (D) is incorrect because the court may order a party to undergo a physical exam against his wishes if there has been a showing of good cause.

Question 6-Concurrent Jurisdiction - Intermediate A plaintiff brought an action in a state court against a defendant, a city police chief in the state, alleging civil rights violations under 42 U.S.C. section 1983. The defendant moves to dismiss the state suit on the ground that the action must be brought in federal court because a federal question is involved. Should the court grant the defendant's motion to dismiss? A Yes, because the federal court has exclusive jurisdiction over the action. B Yes, but the plaintiff may defeat dismissal by adding a state law claim. C No, because the federal courts and state courts have concurrent jurisdiction over the action. D No, but the defendant may remove to federal court if he has a defense based on federal law.

(C) The defendant's motion will be denied. The federal courts and the state courts have concurrent jurisdiction over most types of actions. The few areas over which the federal courts have exclusive jurisdiction include bankruptcy cases, patent and copyright cases, antitrust cases, and a few other less common types of actions. (A) is incorrect for the reasons discussed above. (B) is incorrect because the appropriate remedy for a defendant would be to have the case removed to federal court. Furthermore, a defendant would be able to remove notwithstanding the state law claim; a court could invoke its supplemental jurisdiction to have a pendent state claim heard with claims based on federal law. (D) is incorrect. A defense based on federal law would not enable a defendant to remove the case to federal court.

Question 2- Privacy Torts - Intermediate A recently retired detective who had been instrumental in solving many important crimes in his community and had received many commendations from his police superiors was the subject of a docudrama aired on network television. The credits for the show indicated that it was based on the life of the detective but stated that not every event depicted in the show actually happened. The detective saw the show and was furious because, while most pans of the show dealt fairly accurately with some of the crimes he had helped solve, other pans portrayed his character as being involved in James Bond-type sexual escapades. If the detective sues the network for invasion of privacy, who likely will prevail? A. The detective, because his name was appropriated by the network fora commercial purpose. B. The detective, because the seclusion of his retirement has been upset. C. The detective, because he has been portrayed in a false light. D. The network, because the show as a whole was complimentary to the detective.

(C) The detective probably will prevail in a suit for invasion of privacy because the network has published facts about the detective that place him in a false light in the public eye by attributing to him actions that he did not take. To establish a prima facie case for invasion of privacy based on a publication by defendant of facts placing plaintiff in a false light, the following elements must be proved: (i) publication of facts about plaintiff by defendant placing plaintiff in a false light in the public eye; (ii) the "false light" must be something that would be highly offensive to a reasonable person under the circumstances; and (iii) malice on the part of defendant where the published matter is in the public interest. A fact will be deemed to present plaintiff in a false light if it attributes to him: (i) views that he does not hold, or (ii) actions that he did not take. Several parts of the network's docudrama portray the detective's character as being involved in James Bond-type sexual escapades. Thus, they attribute to the detective actions he did not take and were highly offensive to him. The general disclaimer that not every event in the show actually happened is not sufficient to dispel the false light liability. Furthermore, this false light would likely be found to be highly offensive to a reasonable person under the circumstances. Finally, the detective does not need to show malice because the episodes he is objecting to are not in the public interest. (A) is incorrect. To establish a prima facie case for invasion of privacy based on an appropriation of plaintiff's name, only one element need be proved: unauthorized use by defendant of plaintiff's picture or name for defendant's commercial advantage. Liability is generally limited to the use or plaintiff's name in connection with the promotion or advertisement of a product or service. The mere use of a personality's name in a television show or magazine story, even though motivated by profit, does not suffice for liability. While the detective's name was listed in the credits, there is no evidence that his name was used in connection with the promotion or advertisement of the program and therefore this particular type of invasion of right to privacy is not as applicable as a false light action. (B) is incorrect. To establish a prima facie case for invasion of privacy based on an intrusion upon the plaintiff's affairs or seclusion, the following elements must be proved: (i) act of prying or intruding upon the affairs or seclusion of the plaintiff by the defendant; (ii) the intrusion must be something that would be highly offensive to a reasonable person; and (iii) the thing to which there is an intrusion or prying must be "private." This tort does not provide special protection for the seclusion of a retirement. There is nothing in the facts to suggest that the network invaded plaintiff's private affairs in creating the scenes to which he objects. The accurate material in the show was drawn from the detective's crimefighting activities, which are not in his private domain. Thus, this branch of the privacy tort is inapplicable. (D) is incorrect because if the detective's privacy has been invaded, it is no defense that the program as a whole was not offensive. The invasion of privacy torts do not involve a balancing of complimentary and offensive statements to determine overall whether plaintiff's privacy was invaded.

Question 1- Introductory Negligence Slip and Fall After enjoying a wonderful meal in a restaurant, a diner went into the kitchen through a door marked "employees only' to personally compliment the chef. However, before he could get the attention of the chef, he slipped on a puddle of bright yellow grease that had congealed on the floor by the stove. He fell hitting his head and sustaining a severe head injury. If the diner sues the restaurant for damages in a jurisdiction following the traditional rules for landowners and possessors of land, is he likely to recover? A Yes, because the restaurant is a place of public accommodation and breached its duty of care owed to its patrons. B Yes, because a restaurant employee could have discovered the dangerous condition of the floor by making reasonable inspections. C No, because patrons were not allowed in the kitchen. D No, because the puddle of grease was visible on the floor.

(C) The diner is not likely to recover because the restaurant did not permit patrons in the kitchen. In a jurisdiction following the traditional rules for landowners and possessors of land, the duty owed by an owner or occupier of land to those entering the land depends on whether the entrant is characterized as a trespasser, licensee, or invitee. Here, the diner was an invitee of the restaurant while he was dining at the restaurant. However, a person loses his status as an invitee if he exceeds the scope of the invitation--if he goes into a portion of the premises where his invitation cannot reasonably be said to extend. Here, the diner lost the status of an invitee when he entered the kitchen; he became, at best, a licensee, perhaps even a trespasser, because patrons were not permitted in the kitchen. While a landowner owes no duty to an undiscovered trespasser, he owes a discovered trespasser the duty to warn of or make safe artificial conditions known to the landowner that involve a risk of death or serious bodily harm and that the trespasser is unlikely to discover. For a licensee, the duty extends to all dangerous conditions that create an unreasonable risk of any harm to the licensee. Had the diner been an invitee, he could have argued that under the duty owed to invitees to make reasonable inspections, a puddle of grease that had time to congeal should have been discovered and cleaned up, or at least been the subject of a warning. Here, however, there are no facts to suggest that anyone in the kitchen knew of the diner's presence or the puddle of grease, even though had congealed; hence, the facts do not establish a breach of the duty to warn discovered trespassers or licensees of dangerous conditions. (A) is incorrect. While places of public accommodation have an affirmative duty to use reasonable care to aid or assist their patrons, that duty rule does not alter the duty rules pertaining to the condition of the land, which are based on the status of the person on the premises (as discussed above). (B) is incorrect because the failure to exercise reasonable care to discover a dangerous condition breaches a duty owed only to invitees, as discussed above. However, the diner was no longer an invitee when he entered the kitchen area. He was at most a licensee, for whom the duty to make reasonable inspections does not apply. (D) is incorrect. While a duty to warn does not exist where the dangerous condition is so obvious that the invitee should reasonably have been aware of it, just the fact that the grease was visible on the floor does not establish this "Obviousness" is determined by all of the surrounding circumstances; e.g., one whose attention may have been directed elsewhere may recover even though the condition was visible. In any case, even if the puddle were not visible, the diner would not be able to recover because he did not have invitee status when he entered the kitchen.

Question 3- Introductory - Contribution The owner and manager of a large office building contracted its elevator maintenance to an elevator repair company that did not have a good reputation for safety. One of its employees incorrectly set a switch while repairing an elevator. As a result, the elevator dropped suddenly when an office worker in the building used it the next day. The worker sued the elevator company in a jurisdiction that has adopted comparative contribution rules but has retained joint and several liability. The trier of fact determined that the elevator company was 70% at fault and the building owner was 30% at fault in causing the worker's damages. After the worker had obtained a full recovery of his damages from the elevator company, the company sued the building owner to obtain reimbursement for the damages it paid to the worker. How much should the company recover from the building owner? A All of the damages through indemnity because the building owner owed a nondelegable duty to occupants of its building B None of the damages because the company was more at fault than the building owner for the worker's injury C 30% of the total damages because the building owner is jointly liable for the injury to the worker. D 50% of the total damages because joint tortfeasors are liable for contribution in equal shares.

(C) The elevator company can recover 30% of its damages from the building owner under a comparative contribution system. Contribution rules allow any defendant required to pay more than his share of damages under joint and several liability to have a claim against the other jointly liable parties for the excess. In states with a comparative contribution system, the traditional method of equal apportionment of damages is rejected; nonpaying tortfeasors are required to contribute only in proportion to their relative fault. Thus, the company should be able to recover 30% of the total damages from the building owner because the building owner was 30% at fault. (A) is incorrect for several reasons. The fact that the building owner may have had a nondelegable duty to occupants of its building would only establish that the building owner would be vicariously liable to the plaintiff (the worker) for the company's conduct even if the building owner were not at fault; that rule does not allow a defendant to recover indemnity from another defendant. Furthermore, if the building owner were only vicariously liable here, the building owner would have had a right of indemnity against the company if the building owner had had to pay damages to the worker, but the company has no such right against the building owner. (B) is incorrect because the fact that the company was more at fault would not preclude it from recovering contribution from the building owner under a comparative contribution system, which replaces indemnification rules based on identifiable differences in degree of fault. (D) is incorrect because, as discussed above, traditional contribution rules based on equal shares are not applied in a comparative contribution system, which apportions contribution based on relative fault.

Question 19-Beyond scope of Conspiracy-Intermediate A former employee and his friend agreed to rob a grocery store. They entered the store just before closing. The former employee held a gun on the store manager and made him empty out the safe, and the friend directed a clerk who was the only other person in the store to a storeroom in the back. Before tying her up, however, the friend raped her at gunpoint. The friend returned to the front of the store, where the former employee had just finished putting the store's money in a large Sack. To prevent the store manager from pursuing the pair or from quickly notifying the police, the friend shot the store manager in both knees. Neither the rape nor the shooting were part of the plan between the former employee and the friend. In a common law jurisdiction, of which of the following crimes is the former employee guilty? A Robbery only. B Robbery and rape only. C Robbery and aggravated battery only. D Robbery, rape, and aggravated battery.

(C) The former employee is guilty of robbery and aggravated battery. Robbery is (i) the taking (ii) of the personal property (iii) of another (iv) from the other person's presence (v) by force or intimidation (vi) with the intent to permanently deprive the person of the property. The former employee clearly committed a robbery by forcing the store manager, at gunpoint, to empty the contents of the safe into a sack. Given that the former employee did not physically commit either the rape or the aggravated battery, criminal liability would hinge on whether he could be held accountable based on a conspiracy theory. Provided that a conspiracy exists, a person will be criminally liable for the acts committed by a co-conspirator that were committed in furtherance of the objectives of the conspiracy and were a natural and probable consequence of the conspiracy (i.e., foreseeable). At common law, a conspiracy exists when there is (i) an agreement between two or more persons; (ii) the intent to enter into the agreement; and (iii) the intent to achieve the objective of the agreement. Here, the former employee and his friend intentionally entered into an agreement to rob the grocery store. Furthermore, the fact that they armed themselves shows that they considered the possibility that they might need to use force during the crime; thus, the fact that the friend actually used force by shooting the store manager was foreseeable. The shooting was also done in furtherance of the conspiracy, as it was intended to aid in the pair's getaway. Thus, the former employee may be found guilty of the aggravated assault on the store manager, and (A) and (B) are incorrect. However, the rape was neither committed in furtherance of the conspiracy (it did nothing to further the robbery or aid in the pair's getaway), nor do the facts indicate that it was foreseeable that the friend would commit such a crime during a robbery under these circumstances, criminal liability probably would not attach to the former employee for the acts of his friend in raping the produce clerk, making (B) and (D) incorrect.

Question 1 - Accomplice Liability- Introductory A friend knew that a man and a woman cared deeply for each other and encouraged the man to ask the woman to marry. Unbeknownst to the friend, the man was already married. Finally, after continued encouragement by the friend, the man went through a marriage ceremony with the woman, and the friend was the man's best man at the ceremony. If the friend is charged with being an accomplice to bigamy, should he be found guilty? A Yes, because he encouraged the man, and his mistake as to the existence of a prior marriage is no defense to a charge of bigamy. B Yes, because he was present when the crime occurred and is thus a principal in the second degree. C No, because he did not have the mental state required for accomplice liability. D No, because his encouragement and assistance was not the legal cause of the crime.

(C) The friend should not be found guilty as an accomplice to bigamy because he did not have the requisite mental state. To hold a person liable as an accomplice to a crime, the state must show that his assistance or encouragement was given with the intent to assist the principal and the intent that the principal commit the crime. The facts indicate that the friend did not have this state of mind. The friend did not know that the man was already married, and thus the friend is not guilty, because he did not have the mental state required for accomplice liability. (A) is wrong because it involves a mistaken legal premise insofar as liability as an accomplice is concerned. (B) is accurate in part, but ignores the requirement of a mental state. (D) is wrong because the accomplice can be liable even if he did not cause the crime.

Question 15-Conspiracy and Intent (Burglary) Intermediate A man believed that a woman who was a former roommate had taken a book of his, so he persuaded a mutual friend to help him get it back surreptitiously. He had the friend take the woman out to dinner, leaving her apartment empty so that the man could pick the lock on the woman's front door and take the book back. On seeing the man trying to pick the lock, a neighbor called the police, who found the man still trying to pick the lock when they arrived. The man and the friend were charged with burglary and conspiracy to commit burglary. If the man and the friend are acquitted of the conspiracy charge, what would be the most likely reason? A There was no oven act. B There was no agreement. C There was no intent to commit burglary. D The man did not actually commit a burglary.

(C) The man and the friend will be acquitted if there was no intent to commit a burglary. At common law, a conspiracy required: (i) an agreement between two or more persons; (ii) the intent to enter into an agreement; and (iii) the intent to achieve the objective of the agreement. Although an overt act was not required at common law, the majority rule today is that an overt act committed in furtherance of the conspiracy is required, but "mere preparation" usually suffices as an overt act. Thus, conspiracy to commit burglary requires the intent and agreement to commit a burglary. Burglary is the breaking and entry of the dwelling of another at nighttime with the intent of committing a felony therein. If the man and the friend's intent was only to retrieve the man's property and not to commit a felony, they did not have the necessary intent for burglary. (A) is wrong; to the extent one is required, there was an overt act here. (In fact, but for the lack of intent to commit a felony, sufficient steps were probably taken to constitute an attempt). (B) is also factually wrong; there was an agreement. (D) is wrong; even though the actual burglary was not committed, they could be found guilty of conspiracy if they had the required intent.

Question 1 - Parole- Intro A man on parole after being convicted of possession of cocaine was suspected of selling cocaine out of his home. His parole officer came to his house and rang the bell. As soon as the man opened the door to see who was there, the officer entered the home, despite the man's protests. After searching the home, the parole officer discovered several bags of marijuana in a drawer. The man was arrested and charged with possession of marijuana with intent to sell. A statute in the jurisdiction in which the search took place provides that, as a condition of parole, a parolee is on notice that his parole officer may conduct a search of the parolee's person or home, without probable cause, at any time of the day or night. The man moved to have evidence of the marijuana suppressed by the court, claiming that the state statute that authorized the search was unconstitutional under the Fourth Amendment prohibition of unreasonable searches and seizures. Will he prevail? A Yes, unless probable cause was established by the officer's tip in conjunction with other circumstances. B Yes, because a search warrant was not obtained and no exception to the warrant requirement applies. C No, because the man had a diminished expectation of privacy and the government has a heightened need to search parolees' homes. D No, because the search was incident to a lawful arrest.

(C) The man will not prevail in his motion to suppress. To be reasonable under the Fourth Amendment, most searches must be pursuant to a warrant. However, several types of inspections and searches do not require a warrant or even probable cause. The Supreme Court has held that the Fourth Amendment is not violated by a statute authorizing warrantless searches of a parolee's home - even absent probable cause - if a statute provides for such searches. The Court reasoned that in such circumstances, the parolee has a diminished expectation of privacy and the government has a heightened need for searching parolees; thus the search is reasonable in a constitutional sense. (A) is incorrect because while probable cause may be based on this type of tip under the "totality of the circumstances" test, probable cause is not necessary to establish the validity of the search based on the above discussion. (B) is incorrect because a search warrant is not required for a search of a parolee's home that otherwise complies with procedures. (D) is incorrect because this was not a search incident to a constitutionally valid arrest. There was no basis for an arrest until after the search occurred; if the search were not otherwise independently valid, the fact that the man was arrested after the search revealed the drugs would not make the search valid.

Question 3- Parent consent to search- Intro The police received information linking a man to drug trafficking and went to the man's residence, where he lived with his mother. The police found the mother at home, and she told them that her son was not expected back until later. The police informed the mother that they suspected the man of selling drugs and asked if they could search his room. She replied, "I'm finished with that no-good bum; not only is he into drugs, but he has been stealing my money to pay for them, and all the time I'm making his bed and fixing his food. You can search his room. He likes to keep his private stuff under his pillow. I hope he goes to jail." The police searched the man's room and discovered a quantity of marijuana under the pillow of his bed. If before trial the man's attorney moves to suppress the marijuana on grounds that the search was invalid, should the court grant the motion? A Yes, because the man had a legitimate expectation of privacy in the area searched, and the police did not have a warrant. B Yes, because the man's mother's consent was given at a time when police knew her interests were in conflict with the man's. C No, because the man's mother had the authority to consent to the search of his room. D No, because with the mother's statement the police had probable cause to search the room.

(C) The man's motion to suppress should be denied because his mother had authority to consent to the search of his room. A search of a residence can be based on the voluntary consent of the occupant. Where a parent has general access to a room occupied by a son or daughter, the parent can give a valid consent to a general search of the room even if the son or daughter is an adult. The facts in the question indicate that the man's mother had general access to his room ("and all the time I'm making his bed"). Therefore, her consent is valid and eliminates the need for probable cause and a warrant. (A) is wrong. The man had a legitimate expectation of privacy in the area searched, but the consent of his mother eliminated the need for a warrant. (B) is wrong. At one time, some courts required an "amicable relationship" between the parties before the police could rely on a third party's consent. The "amicable relationship requirement" is no longer recognized by the courts. (D) is not a good answer. It is true that with the mother's statement the police had probable cause to search the man's room. However, probable cause alone would not validate the search. The police would need probable cause plus a warrant or a valid consent. In this question the search would have to be based on consent

Question 14 -Homicide -Intermediate Two teenagers who had recently gotten their driver's licenses agreed to a game of "chicken,' with the one who turned his car away first having to pay the other $25. The two cars raced toward each other, and when they were about 40 feet apart, the victim suddenly turned his car to the right, but the defendant was going too fast to avoid the victim's car. He crashed into the side of the victim's car, killing the victim instantly. The homicide statute in this jurisdiction reads in pan as follows: "Murder is the unlawful killing of a human being with malice aforethought. Such malice may be express or implied. It is express when there is manifested a deliberate intention to unlawfully take away the life of a fellow creature. It is implied when no considerable provocation appears or when the circumstances attending the killing show an abandoned and malignant heart. All murder that is perpetrated by willful, deliberate, or premeditated killing or committed in the perpetration of or attempt to perpetrate arson, rape, robbery, or burglary is murder of the first degree. All other kinds of murders are of the second degree. The defendant is charged with first degree murder. What is his most effective defense? A An automobile is not a dangerous weapon, and therefore there can be no deliberate killing. B The defendant, because of his youth, could not have formed the necessary malice aforethought to support a conviction for murder. C The defendant is guilty of murder, but not first degree murder. D The defendant cannot be guilty of any degree of murder because he did not intend to kill the victim.

(C) The most effective argument is that the defendant is not guilty of first degree murder. The only theory of first degree murder applicable here is that the murder was "willful, deliberate, or [and] premeditated." This type of murder requires an actual intent to kill, which the defendant did not have. (A) is wrong because an automobile can be a dangerous weapon when used as one, and the intentional driving of an automobile at another can support and permit, though not require, an inference that defendant intended to kill. (B) is wrong because, at common law, 14 is the cutoff age for youthfulness to be a factor. The facts state that defendant was licensed and thus must be over the age of 14. (D) is clearly wrong. Although the defendant might be able to avoid liability for first degree murder because of his lack of intent to kill, he could be liable for second degree murder.

Question 4 -Probable Cause for Search Warrant- Intro The president of a private college received a report that there was a great deal of cocaine use occurring on the second floor of the dormitory. The president persuaded the school security officers to place several concealed microphones in the second-floor student lounge. Conversations occurring in the lounge were monitored by the security officers and they recorded a conversation in which a sophomore at the college offered to sell cocaine to a freshman. A tape of the conversation was taken to the local police, who played it for a local judge. The judge issued a warrant to search the sophomore's room. While searching the room the police discovered a large amount of cocaine and the sophomore was arrested and charged with unlawful possession of narcotics. His attorney moved to prevent the introduction of the cocaine into evidence. Will the motion will most likely be granted? A Yes, because the sophomore's privacy was unreasonably invaded. B Yes, because the electronic surveillance was "fundamentally unfair." C No, because the police properly obtained a search warrant. D No, because the college president was acting on behalf of the college population in general.

(C) The motion to suppress should be denied because a valid search warrant was obtained. A search warrant must be based on probable cause. Here, there was sufficient information for a judge to conclude that there was probable cause to believe that evidence of a crime would be found in the sophomore's room. Thus, the warrant was properly obtained. (A) is wrong because the sophomore had only a limited expectation of privacy in a dormitory's lounge. Also, even if his privacy had been invaded, any invasion here was done by private persons, not the state, and thus would not prevent introduction of the evidence. (B) is wrong because it is untrue as a matter of law. (D) is wrong because it is not a sufficient basis to deny the sophomore's motion. It is irrelevant that the college president acted on behalf of the college population.

Question 1 Battery (Introductory) A patient sought psychiatric treatment from a psychiatrist. During the treatment, the psychiatrist, unbeknownst to the patient, videotaped her. No sound recording was made of the sessions, but the psychiatrist was conducting a study on "body language" and planned to use the videotapes in those experiments. The patient learned that the psychiatrist had been videotaping their analysis sessions and brought an action against him for battery. Which of these statements would best explain why the patient is unlikely to prevail on this theory? A She did not suffer any injury as a result of the Psychiatrist's actions. B The psychiatrist had an implied consent to take the actions he did as pan of the patient-physician privilege. C She did not suffer an offensive touching. D The psychiatrist intended that his actions would foster medical research.

(C) The patient cannot make out a case for battery because she did not experience an offensive touching. To make out a prima facie case for battery, the plaintiff must show an intentional act by the defendant that caused harmful or offensive contact to the plaintiff's person. Here, nothing indicates that the patient was touched in any way; thus, (C) is correct. (A) is incorrect because injury is not an element of battery; battery can be established even absent a showing of injury or damages. (B) is incorrect because if the elements of battery were present, the physician-patient privilege would not protect the psychiatrist here because the patient did not impliedly consent to the taping. When a patient consents to a doctor's treatment, she impliedly consents to all necessary touching that goes along with the treatment. However, when the doctor goes beyond the scope or the acts consented to and does something substantially different, the defense of consent is no longer available. Here, the patient consented to talk with the psychiatrist, but that is substantially different from agreeing to be the subject of an experiment. Thus, if taping the patient were a battery, the psychiatrist could not rely on implied consent to relieve him of liability. (D) is incorrect because a benevolent motive is not a defense to a battery. If the other elements of battery were present, the fact that the battery occurred to foster medical research would not be a defense--that would allow psychiatrists to physically strike their patients merely to study their reactions!

Question 16 - Larceny of own property- Intermediate A student and a few of his friends were making their way to spring break. Along the way, the old van that they were driving broke down. Not wanting to miss any part of spring break festivities, the student asked the mechanic on duty at the repair shop for a rush job. The mechanic provided the student with a repair estimate, and the student, on the basis of the estimate, authorized the repair and promised to pay when he came back to pick up the van. When the mechanic called the student to tell him that the van was repaired, the student, rather than paying for the repair, told one of his friends that the mechanic had agreed to finance the repair charges and that the only thing left to do was pick up the van in the garage's parking lot. The student handed the friend a key to the van and told him to go pick the van up so that they could continue their trip to spring break. The friend did so. The mechanic makes a criminal complaint against the student for larceny of the van. If the case is prosecuted, will the student likely be found guilty? A No, because it was the student's van to begin with. B No, because the friend took the van. C Yes, because the friend took the van from the mechanic without the mechanic's knowledge or permission. D Yes, because the student promised to pay the mechanic for his work when he came to get the van.

(C) The student will most likely be found guilty. Larceny is the taking and carrying away of the tangible personal property of "another' by trespass with the intent to permanently deprive the other person of the property. It is possible to commit larceny of your own property if another person, such as a bailee, has a superior right to possession of the property at that time. Because the mechanic had a right to possession of the van until he was paid, the student committed larceny when he had his friend take the van without the mechanic's consent. (B) is wrong because a person can be guilty even though he did not personally engage in the behavior if he acts through an innocent agent. (D) is also incorrect. The student is guilty, but not for the reason stated in (D). He would be guilty even he had not made the promise to pay for the van; he incurred an obligation to pay by having the repairs done.

Question 19 - Burglary & Larceny-Advanced A neighbor of a teenager purchased an expensive sports car and parked it in his attached garage. To get into the garage, the teenager manufactured a radio device that could be programmed to activate any automatic garage door opener, and the teenager used the device to open the neighbor's garage one night. The teenager hotwired the car and drove it away, intending to cruise by a few of his friends' favorite hangouts and then return it to its owner with no one the wiser. After driving the car around the town for a while, the teenager decided to take his friends for a ride to the nearby seacoast. When the party arrived at the bluffs above the beach, the teenager impulsively decided to push the car over the cliff. The jurisdiction's penal statutes have codified the common law without alteration of any kind. Of what is the teenager guilty? A No crime B Burglary C Larceny D Both burglary and larceny

(C) The teenager is guilty of larceny only. Larceny is the taking and carrying away of the tangible personal property of another by trespass, with the intent to permanently deprive the person of his interest in the property. Under the continuing trespass doctrine, if a defendant takes property with a wrongful state of mind but without the intent to steal, and later, while still in possession of it, forms the intent to steal it, the trespass involved in the initial wrongful taking is regarded as "continuing" and the defendant is guilty of larceny. Here, the teenagers initial wrongful taking continued to the time the teenager intentionally destroyed the car, thereby permanently depriving the owner of possession. Therefore, the teenager committed larceny and (A) is incorrect. Burglary requires the breaking and entering of the dwelling of another at nighttime with the intent to commit a felony therein. The teenager is not guilty of burglary because, when he entered the attached garage, he did not intend to commit a felony therein; he merely intended to borrow the car. Thus, (B) and (D) are incorrect.

Question 13- Deadly Force-Fleeing Felon - Intro A police officer saw an unarmed thief grab a coat off the rack of a store and run. As the police officer gave chase, the thief ducked into an alley. When the police officer came to the alley, she saw movement behind some boxes. She yelled for the thief to come out or she would shoot. When the thief did not, the police officer shot at the boxes. A bag lady had been asleep behind them, and she was killed. The police officer is charged with homicide. What should the jury's verdict be? A Not guilty of homicide, because she was attempting to catch a thief who was fleeing from a crime, and the police officer had the right to use deadly force. B Not guilty of homicide, because she saw the thief commit a misdemeanor, and therefore she had the right to use deadly force if necessary to arrest him. C Guilty of homicide, unless she had reason to believe that the thief was a felon. D Guilty of homicide, because she was not entitled to use deadly force.

(D) A jury should find the police officer guilty of homicide because she was not entitled to use deadly force. The Supreme Court has held that a police officer may use deadly force to apprehend a fleeing felon when the felon threatens death or serious bodily harm and deadly force is necessary to prevent his escape. For purposes of state criminal law, under this rule a police officer cannot use deadly force to apprehend an unarmed, nondangerous felon. An officer may use deadly force to prevent a felon from escaping if the police officer has probable cause to believe that the felon poses a threat of serious bodily harm to the officer or others. Here it does not appear that shooting at the thief was necessary to prevent his escape or that the thief threatened serious bodily injury. There is no indication that the thief was armed or a dangerous threat to others. Therefore, the officer was not entitled to use deadly force and should be found guilty of homicide. Thus, (A), (B), and (C) are incorrect.

Question 7- Criminal Statutory Interpretation- Intro A state statute reads in pertinent part: "Any licensed medical doctor who willfully neglects to assist anyone with a life-threatening injury shall be guilty of a crime, subject to punishment of up to two years' imprisonment in the state penitentiary or a fine of $15,000, or both." A licensed physician, while jogging in a park, heard the sounds of a man moaning in the bushes next to the jogging path. The physician stopped to investigate and found the man bleeding in the bushes. The physician asked the man if he needed help, and the man responded that he would be fine. Not having seen any indications of a serious condition, the physician returned to her morning jog. The man subsequently bled to death an hour later. The coroner's report made clear that the man could have been saved if he had received prompt medical attention, even if only of a "first aid" variety until an ambulance arrived. The physician was charged with violating the statute. At trial, the physician testified that she was unaware of the statute, which had been in effect for one week when the incident took place. Should the physician be convicted of violating the statute? A Yes, because she neglected someone whose life was in danger. B No, because the statute is too vague and raises mere negligence to criminal status. C No, because the physician was unaware of the new statute. D No, because the physician believed that the man's condition was not life-threatening.

(D) Because the physician believed that the man's condition was not life-threatening, she should not be convicted because she lacked the requisite intent for the crime. The statute here makes it a crime for a licensed physician to willfully neglect someone with a life-threatening injury. A requirement of willful action means that the person must have acted knowing that her conduct would necessarily cause such a result. Thus, she must be aware of the circumstances and act with the awareness of what the results of her conduct will be. To be convicted under this statute, the physician must have been aware that the man's condition was life-threatening and that leaving him there without any assistance could cost him his life. Because the facts indicate that the physician believed that the man was not in a life-threatening situation, she did not violate the statute by leaving the man unattended. She did not "willfully" neglect him, because she was not aware of the status of his situation. (A) is wrong because it fails to account for the required mental state (i.e., willfulness). Choice (A) would impose a strict liability; a doctor would be liable whether or not she knew of the life-threatening situation. The statute here does not impose strict liability; only willful neglect is Punishable. (B) is wrong because the statute is not too vague. The statute gives a person of ordinary intelligence fair notice of what sort of action or inaction is punishable (i.e., the willful failure of a licensed medical physician to assist someone with a life-threatening injury). (C) is wrong because ignorance of the law is no excuse. It not a defense to a crime that the defendant did not know that her acts were prohibited by criminal law This is true regardless of whether her ignorance was reasonable. Thus, despite the fact that the statute was in effect for only one week, the physician may be convicted whether she was aware of it or not.

Question 7- Venue- Intermediate A three-car accident occurred in a city in the Northern District of State A. The cars were driven by a citizen of State B who resides in the Southern District of State B, a citizen of State A who resides in the Southern District of State A, and a citizen of State C who resides in the Northern District of State C. The State B citizen filed a negligence action against the other two drivers in the United States District Court for the Southern District of State A. Although the two defendants believed that venue was improper, neither filed a pre-answer motion objecting to venue. They instead proceeded to file their answers, responding to the merits of the claim. The State C defendant, however, included in her answer a motion to dismiss the action for improper venue. How should the court rule on the State C defendant's motion to dismiss for improper venue? A Deny the motion, because the defendants waived the defense of venue by not asserting it in a separate, pre-answer motion prior to filing their answers addressing the merits of the action. B Deny the motion, because all defendants must object to venue before the court may dismiss on venue grounds. C Deny the motion, because venue is proper. D Grant the motion, because the motion is timely and venue is improper.

(D) The court should grant the motion. Objections to venue may be raised in a defendant's answer if the defendant did not assert a Rule 12(b) pre-answer motion, as is the case here. Venue here is improper because (i) none of the events giving rise to the claim occurred in the Southern District of State A, and (ii) all defendants do not reside in State A. (A) is incorrect because a defendant can object to venue in a pre-answer motion or in the answer. (B) is incorrect because all defendants do not have to object to venue before the court may dismiss the case on venue grounds. (C) is incorrect because venue is improper, as explained above.

Question 23-Deadly Force for Fleeing Felon- Intermediate A robber approached a newsstand, with her knife drawn, in an attempt to rob the attendant. The attendant gave the robber the small sum of money in his possession. As the robber turned away, the attendant grabbed a gun and fired several times. He intended only to wound the robber but instead killed her. If the attendant cannot be prosecuted for manslaughter, what is the likely reason? A He only intended to wound the robber. B The robbery constituted a provocation. C The attendant was trying to get his money back. D He was apprehending a fleeing wrongdoer.

(D) If the attendant cannot be prosecuted for manslaughter, will be because he was privileged to use deadly force to apprehend a fleeing felon. A private person may use deadly force to apprehend a fleeing felon if the felon threatens death or serious bodily harm and deadly force is necessary to prevent her escape. Also, for a private person to use force to effect an arrest, the person harmed must actually be guilty of the felony for which the arrest was made; i.e., not sufficient that it reasonably appeared that the person was guilty. Here, the robber actually committed the felony of robbery. Because she was armed with a knife, the robber threatened death or serious bodily harm, and deadly force was necessary to prevent her escape. Thus, the newsstand attendant's best argument is that he had the right to use deadly force to apprehend her, and this serves as a defense to a charge of manslaughter. (A) is incorrect because acting with the intent to inflict serious bodily injury is a sufficient mens rea for voluntary manslaughter. Thus, the fact that the attendant intended only to inflict great bodily injury is no defense to manslaughter. (B) is incorrect because the existence of provocation is not a defense to manslaughter. Rather, provocation must be found to reduce a killing from murder to manslaughter. Thus, the fact that the attendant may have been provoked by the robbery would, if anything, provide a basis for a manslaughter prosecution, rather than a defense thereto. (C) is incorrect because deadly force may not lawfully be used merely to regain possession of property. Deadly force could only be justified by another basis of privilege, such as to effectuate an arrest.

Question 10- Venue- Intermediate A resident of City A (located in the Northern District of State A) brought a diversity action against a resident of City B (located in the Eastern District of State B). The cause of action arose in City. The defendant commutes daily from his home in City B to his office in City A and would find it much more convenient to litigate the action there than in City C, where the appropriate court for the Eastern District of State B is located. The plaintiff would also find it more convenient to litigate in City A, but will go to City C if necessary. May the action be litigated in City A? A No, because venue is not proper there. B No, if process is served in State A. C Yes, because a plaintiff is entitled to choose venue. D Yes, if the defendant waives proper venue.

(D) The action may be litigated in City A because improper venue may be waived. The defendant may inform the plaintiff that he will waive any objection to venue in City A. Given that the plaintiff has indicated a willingness to bring the action in City C if necessary, the defendant would not seem to gain any advantage by insisting on proper venue at great inconvenience to him, unless other factors make City C a preferable venue. (A) is incorrect because although City A is not a proper venue under 28 U.S .C. section 1391, improper venue may be waived. (B) is incorrect because the place where process is served has no effect on venue. (C) is incorrect because a defendant may object to improper venue.

Question 4 - Notice In Rem Action - Intermediate A town planned to build a new street through a parcel of vacant land on the edge of town. The land records show a deed dated October 5, 1947, to the current property owner, for whom an address is given in the capital city of the state. The town instituted an in rem condemnation action in state court, with notice to all interested parties published in the town newspaper. The property owner, who still resides at the address in the capital city, does not see the notice. The court allows the condemnation to proceed as requested by the town. Is the condemnation valid as to the property owner? A Yes, because a court has jurisdiction over the subject matter. B Yes, because notice by publication was sufficient. C No, because the property owner was required to be served by summons. D No, because the property should have been mailed notice of the proceeding.

(D) The condemnation is not valid because the property owner should have been mailed notice of the proceeding. In an in rem action, persons whose interests are known to be affected and whose addresses are known must be notified at least by ordinary mail. The town should at least have sent notice to the property owner at the address it had for him, even if it seemed unlikely that he would still be there. (B) is thus incorrect. (A) is incorrect because, although the statement is true, it does not address the notice requirement. (C) is incorrect. It is well established that service by mail meets the due process requirement of notice in certain cases (i.e., a summons is not always required).

Question 3-Supplemental Jurisdiction - Intermediate A corporation manufactured a patented automatic potato peeler. Subsequently, that corporation's biggest rival manufactured a similar potato peeler. Both are corporations of the same state. The state has a unique statute authorizing quadruple damages for unfair business competition relating to potatoes; however, there is considerable debate whether the law is constitutional under the state constitution. The corporation holding the patent sued its rival in federal court, alleging that the rival corporation violated its patent and the unfair business law. Shortly after the pretrial meeting of the parties required under the Federal Rules, the two companies reached a settlement of the patent infringement claim. May the court dismiss the remaining claim? A No, because the federal court still has supplemental jurisdiction over the case. B No, because the federal court already exercised jurisdiction over the case and thus cannot now dismiss it. C Yes, because the federal court never had jurisdiction over the claim. D Yes, because the federal court may decline to exercise its supplemental jurisdiction.

(D) The court may dismiss the remaining claim. The court may decline to exercise its supplemental jurisdiction on one of four bases: (i) the case presents a novel or complex issue of state law; (ii) in a federal question case, the nonfederal claim substantially predominates the case; (iii) the claims over which the federal court had original jurisdiction have been dismissed (taking into consideration the amount of time the court has spent on the case); or (iv) there are extraordinary circumstances for declining jurisdiction. Here, bases (i) and (ii) above would apply. (A) is incorrect, because the court may dismiss the case even if it still has subject matter jurisdiction, as discussed above. (B) is an incorrect statement of law. (C) is incorrect. The patent claim raised a federal question and provided the court with federal question jurisdiction, and the claim based on the unfair business competition statute arose from a common nucleus of operative fact as the federal claim.

Question 2- Erie- Intermediate A patient filed a malpractice claim against a doctor in a federal court in State A. A State A state statute bars introduction of evidence concerning plaintiff behaviors in medical malpractice cases. The statute was enacted with the policy goal of ensuring that recoveries against negligent doctors would not be reduced due to plaintiff negligence. While there is no federal statute or rule that specifically addresses this situation, federal judges generally admit relevant evidence. At trial, the doctor wants to present evidence that the patient's obsessive exercise routine contributed to stitches popping, which would be relevant to negate his claim. How will the court likely rule as to the admissibility of the evidence? A Admit the evidence, because the admission of evidence is a procedural issue rather than a substantive issue. B Admit the evidence, because the admission of evidence is arguably procedural. C Not admit the evidence, because this is a substantive issue rather than a procedural issue. D Not admit the evidence, because not following the state law would likely lead to forum shopping.

(D) The court should not admit the evidence. This situation highlights the Erie doctrine, which requires federal courts sitting in diversity jurisdiction to apply the substantive law of the state, and federal procedural law. The initial issue is whether there is a federal law on point, because, if so, that law should govern if it is "arguably procedural." The facts state that there is no federal statute, rule, or law on point; accordingly, (B) is incorrect because the "arguably procedural test" does not apply. Thereafter, simply considering whether the state evidentiary statute is procedural or substantive is not adequate because that is not the complete test. Accordingly, choices (A) and (C) are both incorrect in focusing only on whether the evidence is substantive or procedural. When there is no federal statute or rule on point, the Court has used various tests at different times. Here, the outcome determination test would likely mean that the federal court would apply the state standard and not admit the evidence.

Question 20- Larceny, False pretense - Advanced The defendant happened to have the same name as a famous attorney in the city. Coincidentally, they also shared the same birthday, and their Social Security numbers nearly matched except for one number. The defendant went to an electronics store to buy a new portable television set, and decided to finance the purchase because he had little money. The defendant truthfully filled out the store's credit application. As had happened previously, the defendant was mistaken for the famous attorney. As a result, he was approved for a credit line well beyond his means. Instead of explaining the situation, the defendant took advantage by purchasing a state-of-the-art high-definition television set with a surround sound system. The defendant knew that he would be unable to make payments on the equipment when due, but he decided to deal with that when the time came. After several months of nonpayments, the defendant received notice that the store was going to repossess the television and sound system. Before it could do so, a fire burned the defendant's home to the ground, destroying all of his property, including the electronics. The electronics store filed a criminal complaint against the defendant, outlining all of the above facts. What will the authorities likely determine about the defendant's conduct? A The defendant has committed false pretenses, because he improperly gained title to the equipment. B The defendant has committed larceny by trick, because he improperly gained possession of the equipment. C The defendant has committed neither false pretenses nor larceny by trick, because he may have initially intended to pay for the electronics on receipt of the bill. D The defendant has committed neither false pretenses nor larceny by trick, because he was under no duty to disabuse store employees of their mistake as to his true identity.

(D) The defendant did not make a false representation for purposes of either false pretenses or larceny by trick. Both false pretenses and larceny by trick require a misrepresentation on the part of the defendant. In the case of false pretenses, the victim intends to convey title to the property, while in the case of larceny by trick, the victim intends to convey only possession of the property. For both crimes, the misrepresentation required is an intentional (or knowing) false statement of past or existing fact. The defendant must have created a false impression as to a matter of fact. There is no misrepresentation if the defendant merely fails to correct what is known to be a mistaken impression held by the victim, as long as the defendant is not responsible for creating the mistake or has no fiduciary duty to the victim. Here, although this defendant did not obtain unconditional title to the stereo because the seller could repossess it if payments were not made, he probably obtained enough of the title to qualify for the crime of false pretenses. However, he did not create a false impression as to a matter of fact. Although this defendant was aware of the clerk's mistake regarding his identity, this defendant did not create this mistake, nor did he have a fiduciary duty to the clerk. Thus, he had no duty to correct the false impression, and it cannot be said that there was a misrepresentation on the part of this defendant. Absent a misrepresentation, by means of which this defendant obtained either possession of or title to the stereo, he cannot be convicted of either larceny by trick or false pretenses. (C) is incorrect because, in the absence of a misrepresentation on the part of this defendant, his intent to pay for the stereo is not determinative of his guilt of either crime mentioned. Even if this defendant intended to pay at a later point, he would be guilty if he had obtained title to or possession of the stereo by a misrepresentation, with intent to defraud the store. Subjecting the store to a greater risk of loss as a result of the misrepresentation (because the store would otherwise have obtained a down payment) satisfies the intent to defraud element. (A) and (B) are incorrect because, as explained above, this defendant is guilty of neither crime due to the absence of misrepresentation. In addition, (B) is incorrect because most courts would find that title has been transferred under these circumstances, rather than merely possession, making the potential crime false pretense rather than larceny by trick

Question 4- Larceny - Intro The defendant, while visiting the victim, asked for permission to borrow the victim's car so he could drive to a convenience store to buy cigarettes. In fact, he intended to keep the car and sell it for cash. The victim agreed, and the defendant took the car and drove off. After thinking about it further, the defendant decided that it would be wrong to sell the victim's car, and headed back to the victim's house. On the way back, the car was destroyed in a collision through no fault of the defendant. May the defendant be convicted of larceny? A No, because he intended to return the car and therefore lacked the requisite mens rea for the crime. B No, because he abandoned his plan of selling the vehicle. C Yes, because withdrawal is not a defense to the crime. D Yes, because he intended to permanently deprive the victim of the car when he drove off in it.

(D) The defendant has committed larceny. The defendant's change of heart after taking the car will not provide him with a defense because it is irrelevant. Larceny requires the taking and carrying away of the tangible personal property of another by trespass. In the instant case, the larceny was committed at the time he took the victim's car with the intent to permanently deprive him of possession. (Note that the taking was trespassory because the defendant obtained possession by misrepresentation - larceny by trick). Given that the crime was complete once the defendant drove off, he is guilty of larceny, which makes (D) the correct answer. (A) is incorrect because the defendant did not intend to return the car at the time of the taking and thus had completed the crime at that time. He had the requisite mens rea at the time of the taking and carrying away, and this is sufficient to convict. (B) is incorrect because abandonment may be a defense in some states to an attempt of a crime. In the instant case, the defendant had already completed the crime, as described above. Thus, abandonment cannot act as a good defense. Choice (C) is incorrect because withdrawal is not relevant to the crime of larceny.

Question 13 -Larceny By trick - Intermediate The defendant's neighbor owned an authentic major league baseball signed by Babe Ruth. The defendant asked if he could show it to some friends who were visiting. The neighbor agreed as long as he kept it in the display case, which the defendant promised to do. In fact, the defendant intended to use the ball in a pickup game. During the game, the ball was hit over the fence and into a yard with a guard dog, which had chewed up several other balls that had previously landed in the yard. The dog did the same to that ball. When the neighbor learned what happened to the ball, he pressed charges against the defendant. If the defendant is convicted, he will most likely be found guilty of what crime? A Common law larceny B Embezzlement. C False pretenses D Larceny by trick

(D) The defendant is guilty of larceny by trick because he obtained possession of the baseball by means of a misrepresentation. Larceny is the taking and carrying away of tangible personal property of another by trespass, with intent to permanently (or for an unreasonable time) deprive the person of her interest in the property. The taking must be without the consent of the person in possession of the property. If such consent is induced by a misrepresentation of a pastor existing fact, the consent is not valid. The resulting larceny is called larceny by trick. Here, the defendant obtained possession of the baseball with the owner's consent. However, this consent was obtained by means of the defendant's misrepresentation about friends visiting. This was a false statement of an existing fact, made with the intent that his neighbor rely on the statement, and the misrepresentation induced his neighbors consent. At the time of this taking, the defendant intended to deal with the baseball in a manner that involved a substantial risk of damage or loss. This suffices as intent to permanently deprive. Therefore, all the elements are in place for larceny by trick. (A) is not as good a choice as (D) because the taking in this case is better characterized as larceny by trick rather than larceny, given that the defendant induced his neighbor to consent to his taking possession of the baseball. (C) is incorrect because the defendant obtained only possession of the baseball, not title. False pretenses differs from larceny by trick in what is obtained If the defendant obtains only possession of the property, the offense is larceny by trick, whereas obtaining of title means that false pretenses has been committed. What the victim intended to convey to the defendant is determinative. The neighbor intended only to let the defendant borrow the baseball for a short time, not to convey title to him. Consequently, the only thing the defendant obtained was possession of the baseball. Because title to the baseball was not obtained, there can be no conviction of false pretenses. Regarding (B), embezzlement is the fraudulent conversion of property of another by a person in lawful possession of that property. In embezzlement, misappropriation occurs while the defendant has lawful possession of the property, while in larceny, it occurs generally at the time the defendant obtains wrongful possession of the property. Here, as detailed above, the defendant's taking of possession of the baseball was trespassory due to the manner in which he obtained consent to such possession. The crime of larceny was complete on the defendant's taking possession with the requisite intent to permanently deprive. Thus, at the time the baseball was destroyed, the defendant had already misappropriated it and was not in lawful possession of it. As a result, there can be no conviction for embezzlement.

Question 2- First Degree Murder - Intro The defendant, angered because a rival gang member had twice beaten him up after school, obtained a heavy lead pipe and waited in a deserted alleyway which he knew the rival took as a route home every day after school. When his enemy came walking down the alley, the defendant leapt out behind him and smashed the pipe into the victim's head, knocking him to the ground. The defendant then rolled the victim over and pounded his face with 15 to 20 heavy blows with the lead pipe, killing him. The jurisdiction defines first degree murder as murder committed with premeditation and deliberation. All other murders are defined as second degree murders. Why will the defendant be convicted of first degree murder (as opposed to second degree murder)? A The relationship between the defendant and the victim requires that a finding of first degree murder be made. B The degree of causative relationship between the defendant's acts and the death of the victim renders it murder in the first degree. C The nature of the acts causing death distinguishes the defendant's action as first-degree murder. D The defendant's mental state up to and including the moment of the attack determines that the act is first degree murder.

(D) The defendant will be convicted of first degree murder because of his mental state up to and including the moment of the attack. The degree of murder under the statute is determined by the defendant's mental state--whether the killing was intentional and accomplished after premeditation and deliberation. Here, the defendant obtained a lead pipe and waited in a deserted alleyway for the victim, which shows premeditation and deliberation. The defendant's relationship with the victim and the manner of killing may have evidentiary significance with regard to the defendant's mental state, but do not themselves distinguish first from second degree murder. Thus, (A) and (C) are incorrect. The causal relationship between the defendant's act and the death of the victim may determine whether the act murder, but once that analytical hurdle has been passed, has no further significance as to the degree of murder Therefore, (B) is incorrect.

Question 8 - Self-Incrimination - Confession for Impeachment- Intro A defendant was arrested on suspicion of running an illegal "moonshine" operation. After taking the defendant back to the police station, an officer began questioning the suspect, thinking that his partner had already given the defendant Miranda warnings. The defendant voluntarily confessed to each and every element of the crime. At trial, the defendant took the witness stand and testified on his own behalf, declaring that he was innocent and that a distillery that the officers found at his home belonged to someone else. The prosecution, on cross-examination, produced the confession that the defendant gave concerning his illegal activities. The defense counsel objected to the admission of the confession. How should the court rule on the defendant's objection? A Sustained, because all evidence obtained in violation of Miranda rights is inadmissible. B Sustained, because the prosecution did not get permission from the court in advance to use the confession for any purpose. C Overruled, because the prosecution may question the defendant on cross-examination concerning any issue that was brought out in his defense. D Overruled, but the confession should be admitted only for the limited purpose of impeachment.

(D) The defendant's objection should be overruled. A confession obtained in violation of Miranda, but otherwise voluntary, can be used for the limited purpose of impeaching a defendant who testifies at trial. In contrast, an involuntary confession cannot be used to impeach. Here, there are no facts to indicate that the defendant's statement was involuntary. Thus, it can be used to impeach the defendant. (A) is too broad a statement. Although a confession obtained in violation of Miranda is inadmissible in the state's case-in-chief as evidence of guilt, as discussed above, such evidence is admissible for limited purposes. (B) is wrong. Advance permission from the court is not a requirement if the confession is used to impeach. (C) is a correct statement but it does not speak directly to the issue of whether the confession is admissible and to what extent.

Question 2- Introductory - Joint and Several Liability for damages The owner of a speedboat let his friend operate it in a busy channel. While operating the boat, the friend collided with a canoe that had the right of way, injuring its occupant. The canoeist filed suit against both the owner and the friend, alleging that the friend was negligent in operating the boat and that the owner was negligent in letting him operate it, having reason to know that the friend was not qualified to operate a boat. The jurisdiction follows traditional rules for joint and several liability and contribution. There is no other applicable statute. If the jury finds both defendants liable and assesses the plaintiff's damages at $100,000, how should the judgment be entered? A $50,000 against each defendant. B $25,000 against the owner and $75,000 against the friend. C $100,000 against the owner only. D $100,000 against both defendants.

(D) The judgment should be entered in the full amount of $100,000 against both defendants Where two or more tortious acts combine to proximately cause an indivisible injury to the plaintiff, each tortfeasor will be jointly and severally liable for that injury. Joint and several liability of such parties means that each is liable to the plaintiff for the entire damage incurred, so that the plaintiff may recover the entire judgment amount from any defendant (with the plaintiff of course being limited to one total recovery). The owner and the friend have been found jointly and severally liable for the harm caused to the canoeist by their tortious acts. Thus, the entire damage amount of $100,000 is recoverable from either of these defendants. As a result, judgment in the amount of $100,000 should be entered against both the owner and the friend. (A) and (B) incorrectly provide for each defendant to be liable for less than the entire damage incurred. It is only where the actions of each defendant are independent, the plaintiff's injury is divisible, and it is possible to identify the portion of injuries caused by each defendant that each will only be liable for an identifiable portion. Here, there is an indivisible injury, and the defendants have been found to be jointly and severally liable. Thus, there is no basis for entering judgment against either defendant for less than the entire amount of damages. (C) is incorrect because both defendants have been found liable, so there no basis for entering judgment against the owner only.

Question 2 - Conspiracy- Intro A man was approached by his cousin, who planned to rob a bank, to act as the escape driver. The man pretended to agree, and he planned to report his cousin to the police before the crime was committed. On the day appointed for the crime, the man picked up his cousin and drove him to the bank, and they stopped at a convenience store on the way. He left the car running with his cousin inside and from the store he called the police to warn that a bank robbery was about to occur. The police officer answering the phone thought it was a prank call and hung up. After another unsuccessful attempt the man went back to the car to talk his cousin out of the bank robbery, but he found the car missing. The cousin had grown nervous after a few minutes, driven to the bank, and robbed it at gunpoint. He fled the scene, but was apprehended soon thereafter. If the man is charged with conspiracy, is the trial court likely to find him guilty? A Yes, because his withdrawal was ineffective because it was not communicated to his cousin in time for him to change his plans. B Yes, because he provided his cousin with an instrumentality of the crime. C No, because the man withdrew from any conspiracy. D No, because the man lacked criminal intent.

(D) The man is not guilty of conspiracy because he never had the required mental state for conspiracy. Conspiracy requires: (i) an agreement between two or more persons; (ii) the intent to enter into an agreement; and (iii) the intent to achieve the objective of the agreement. The two intent elements indicate that conspiracy is a specific intent crime. Here, because the man planned to contact the police before his cousin could commit the crime, he never intended to achieve the criminal objective of robbing the bank. Thus, he cannot be found guilty of conspiracy. (A) is incorrect because the man never entered into the conspiracy to begin with since he lacked the requisite intent; thus, he did not need to withdraw. (B) is incorrect because merely providing the instrumentality to commit the crime is insufficient by itself to establish liability for conspiracy. There also must be some intent to facilitate the crime, which is absent here. (C) is incorrect because, as stated above, the man never entered into the conspiracy. If he had, (C) would still be incorrect because, at common law and in most states, withdrawal is not a defense to the conspiracy charge itself, merely a limitation on liability for the subsequent acts of the co-conspirators.

Question 8- Self-Defense- Introductory While in a department store, a man picked up a sweater and slipped it under his shirt. The man then stalled for the door. A woman, who also was shopping in the store, saw the man take the sweater. The woman grabbed a baseball bat from the sporting goods aisle and chased the man into the parking lot. The woman began swinging the bat at the man's head, hoping to knock him out and thus prevent the theft. The man pulled a knife from his pocket and stabbed the woman, killing her. The man was arrested and charged with murder. At trial, will the man most likely be found guilty? A Yes, because the evidence shows that he provoked the assault on himself by his criminal misconduct. B Yes, because the evidence shows that the man intended to kill or cause serious bodily harm. C No, because the jury could find that the man acted recklessly and not with the intent to cause death or serious bodily harm. D No, because the man was acting in self-defense.

(D) The man will be found not guilty because he was acting in self-defense. A person is privileged to use deadly force to prevent a crime only if it is an inherently dangerous felony. Shoplifting is not an inherently dangerous felony; thus, the woman's use of deadly force was not privileged. A person may used deadly force in self-defense if he (i) is without fault, (ii) is confronted with unlawful force, and (iii) reasonably believes that he is threatened with imminent death or great bodily harm. The man was without fault because, although he shoplifted the sweater, he did not initiate any violence or provoke the woman. He was confronted with the woman's unlawful force, and it was reasonable for him to believe that a baseball bat swung at his head threatened him with death or great bodily harm. Therefore, the man was entitled to defend himself against the woman's improper use of deadly force by using deadly force himself. (A) is wrong because the man did not initiate an assault. (B) is wrong because even if the man intended to kill the woman, his action was justified. (C) is a misstatement of the law and of the facts.

Question 6-Informants - Intro The police suspected that a man was dealing in illegal drugs. They gathered a sufficient amount of evidence and obtained a search warrant. They went to the man's home and conducted a thorough search. They found and confiscated large amounts of cocaine, and then placed the man under arrest. A ta suppression hearing, the man sought to suppress the evidence against him. The prosecution informed the court that the warrant had been issued on the basis of information provided by an informant. The man's attorney requested the prosecution to produce the informant for questioning. When the prosecution refused to do so, the attorney made a further motion to suppress all of the evidence against his client obtained from the search on the basis that his client was denied the right of confrontation by the prosecution's refusal to produce the informant. Should the attorney's motion to suppress be granted? A Yes, because the denial of the right of cross- examination effectively prevents a fair trial. B Yes, because the rights of a defendant override any right of the police to keep an informant anonymous. C No, because defendants have no right to know the identity of informants. D No, because the prosecution is not required to either name or produce the informant.

(D) The man's motion should be denied. The United States Supreme Court has held that when the only issue is that of probable cause for issuance of a warrant, the name of the informer is a type of privileged information. (Only if the informer were a material witness to the crime might his identity have to be revealed at or before trial). The validity of a warrant based on information obtained from informers is based on the totality of the circumstances. (A) is incorrect because although the right of cross-examination of one's accusers has been deemed to be a fundamental right, it does not apply when the issue is only that of probable cause for issuance of a warrant, as compared to guilt or innocence. (B) is incorrect because, as noted above, unless the informer were also a material witness to the crime, his identity need not be revealed. Here, the facts do not indicate that the informer was a material witness to the crime. (C) is incorrect because it is overbroad. As indicated above, there a right to know the identity of an informant the informant is also a material witness to the crime.

Question 2-Warrantless Search- Intro A detective obtained a valid search warrant for the home of a man suspected of manufacturing methamphetamine and counterfeiting. The man was not at home, but his wife told the detective that he was at cabin. After announcing his purpose, the detective searched the home and found a large sum of cash, along with a printing plate for $20 bills and several large stacks of fresh $20 bills. Thereafter, the detective went to the man's cabin with an arrest warrant in hand. After knocking on the door and announcing his presence, the detective found that the door was unlocked and entered the cabin. The man was not in the cabin but the detective found several boxes of the over-the-counter drugs and other ingredients for making methamphetamine in a small footlocker. The man was indicted for various criminal offenses, including the illegal manufacture of methamphetamine and counterfeiting. Was the search at the man's cabin following the search of his home valid? A Yes, based on the wife's consent. B Yes, as a search incident to arrest. C No, because the detective lacked probable cause. D No, because the detective did not have a warrant.

(D) The search that resulted in the seizure of the methamphetamine supplies at the cabin is invalid because the search was executed without a warrant, and no exception to the warrant requirement is applicable. All warrantless searches are unconstitutional unless they fit into one of the six recognized exceptions to the warrant requirement. To be valid, a warrantless search must meet all the requirements of at least one exception. These exceptions include search incident to a lawful arrest and search based on consent. When the officer conducted the search of the cabin, he had no warrant for the search; he only had an arrest warrant. Thus, the validity of the search and seizure depends on whether an exception to the warrant requirement applicable. (A) is incorrect because the wife's statement that the man was at the cabin did not constitute consent to search the cabin. Even though she had authority to give consent to the search, the search and seizure was not justified under the consent exception. (B) is incorrect because the detective was not engaged in arresting someone, which would have permitted him to search the arrestee and areas into which he might reach to obtain weapons or destroy evidence, given that the man was not on the premises. (C) is incorrect because it is irrelevant It implies that if the detective had probable cause, he could have executed the warrantless search. Probable cause alone is not grounds to conduct a warrantless search.

Question 2- Amend Complaint- Intermediate A consumer was injured when his recently purchased gas oven exploded. The consumer timely filed a products liability action against the manufacturer in federal district court on a theory of strict products liability. Shortly after the complaint was filed and sewed on the manufacturer, the statute of limitations for any tort claim arising from the explosion expired. The manufacturer then timely filed an answer denying liability and denying the material allegations weeks after the manufacturer filed and served its answer, the consumer filed and served an amended complaint that added a negligence claim arising from the same oven explosion. The manufacturer moves to strike the amended complaint, claiming that the amendment was improper and that the negligence claim is barred by the statute of limitations. How should the court rule on those claims? A The court should strike the negligence claim, because it was filed after the statute of limitations expired. B The court should strike the amendment, because a plaintiff can never amend her complaint after the defendant has served his answer. C The court should strike the amendment, because the consumer did not obtain leave to amend the complaint. D The court should allow the amendment and rule that the negligence claim is not barred by the statute of limitations.

(D)The court should allow the amendment and rule that the negligence claim is not barred by the statute of limitations. A plaintiff may amend a complaint as a matter of right (without leave or consent) as long as the plaintiff does so within 21 days after the defendant's response. Thereafter, leave of court is required. Therefore, (B) and (C) are incorrect. The amended pleading relates back to the time the original pleading was filed (i.e., it is treated as if it were filed at that time) if the amendment and the original pleading arise from the same transaction or occurrence. The negligence and strict products liability claims do arise from the same transaction or occurrence, so the amendment relates back. As a result, (A) is incorrect and (D) is correct.

Question 10 Indispensable Party-Intermediate A husband and wife were traveling in a car with the wife driving when they were in an accident with a truck. The accident occurred in a jurisdiction that followed the traditional rule as to joint and several liability among tortfeasors. The husband sued the truck driver in federal district court. The truck driver, contending that the wife was an indispensable party, filed a motion to dismiss the action because the husband did not join his wife as a party to the action. How should the court rule on the truck driver's motion? A Grant the motion, because the wife is an indispensable party. B Deny the motion and order service of process on the wife to make her a party, provided she is subject to the court's personal jurisdiction. C Deny the motion, because the wife and the truck driver cannot be joined as co-defendants in the action. D Deny the motion, because the wife is not "needed for just adjudication.

(D)The court should deny the truck driver's motion because the wife is not "needed for just adjudication." When deciding whether an absentee party is indispensable, the court considers a number of factors. However, the Supreme Court has held that a joint tortfeasor subject to joint and several liability is not a person needed for just adjudication. (Temple v. Synthes Corp., 498 U.S. 5 (1990)). Because the wife here is a joint tortfeasor subject to joint and several liability, she is not an indispensable party in this case. Therefore, (D) is correct, and (A) and (B) are incorrect. (C) is incorrect because the wife could be joined as a co-defendant if the husband chose to do so; a spouse's immunity from being sued by the other spouse has been abolished in almost all jurisdictions.

Question 9 Cross Claim-Intermediate While driving in a city in State A, a citizen of State B struck a pedestrian who was a citizen of State C. The pedestrian sued both the driver and the State A city in federal district court, seeking $100,000. The pedestrian alleged that the driver was liable for negligently operating his car and that the State A city was liable for negligently maintaining a traffic signal. The driver, who owns an office supply wholesaler, also has a contract claim against the State A city for $80,000 worth of furniture that the city allegedly purchased and received but never paid for. Assume that State A has waived any applicable governmental immunity. If the driver files a cross-claim for negligence against the State A city to recover damages for his injuries in the accident with the pedestrian, can the driver join with that cross-claim his contract claim for the purchase price of the furniture and maintain the claim in the same federal action? A No, because defendants cannot add claims against co-defendants that are not related to the original claims asserted by the plaintiff. B No, because the court does not have supplemental jurisdiction over the driver's contract claim against the State A city. C Yes, the driver must assert the contract claim or he will be foreclosed from asserting it later. D Yes, the driver may join his contract claim with his negligence cross-claim against the State A city, but is not required to do so.

(D)The driver can join his contract claim with his negligence cross-claim, but he is not required to do so. As a general rule, a party may assert a cross-claim against a co-party only if the cross-claim arises from the same transaction or occurrence as that of the original action or of a counterclaim. However, once the party has filed such a cross-claim, he also may join with it any other claim that he has against the same party. Here, the driver's contract claim is unrelated to the pedestrian's negligence claim; however, the driver's negligence claim is related to the pedestrian's negligence claim. Thus, the contract claim also can be asserted in this case. (A) is therefore incorrect. (B) is incorrect because the court would have diversity jurisdiction over the contract case. (C) is incorrect because the contract claim is not a compulsory counterclaim.

Question 1-Murder - Intro A motorist drove home from work late one night, and fell asleep behind the wheel of his car. His car drifted across the middle of the road and struck another car. The other driver was killed instantly in the collision. Angered by the noise of the collision, a homeowner fired a gun out the window of his house at the car. The bullet struck and killed a bystander. Both the motorist and the homeowner were arrested and charged with common law murder. Which of the defendants likely would be found guilty? A Both the motorist and the homeowner. B The motorist. C Neither the motorist nor the homeowner. D The homeowner.

(D)The homeowner likely would be found guilty. At common law, murder was the unlawful killing of a human being with malice aforethought. Malice aforethought could be established with any one of the following states of mind: intent to kill; intent to cause serious bodily harm; the depraved heart killing (a reckless indifference to an unjustifiably high risk to human life); or the commission of a felony. The homeowner would be guilty of murder. Firing a gun out of his window at a car would demonstrate a reckless indifference to a high risk to human life. Thus, (B) and (C) are incorrect. It is unlikely that the motorist would be guilty of murder. While his action might be classified as negligent or even reckless, would not represent a depraved heart (reckless indifference to life) state of mind. Thus, (A) and (B) are incorrect.

Question 2- Introductory - Emergency Situations A pilot discovered that her twin engine plane was losing fuel at an alarming rate, caused by a latent defect in the fuel system. Realizing that she could not make it to an airport, she saw a highway off to her left and a lake farther off to her right, and decided to try an emergency landing on the highway. As she maneuvered over the highway, the plane's engine quit for good. The plane's wing clipped a car as it landed, causing the car to veer off the road and crash, injuring the driver. If the driver of the car brings an action for personal injuries against the pilot, who is likely to prevail? A The driver, if the jury finds that the pilots selection of the highway rather than the lake was not a reasonable choice under the circumstances. B The driver, because his injuries were the result of the pilots flying a plane with a dangerously defective fuel system. C The pilot, if the jury finds that she was not negligent in failing to discover the defect in her fuel system. D The pilot, because she made the decision to land on the highway rather than the lake under emergency conditions.

A) The driver will recover from the pilot if she acted unreasonably under the circumstances A prima facie case of negligence requires proof of: (i) the existence of a duty on the part of the defendant to conform to a specific standard conduct for the protection of the plaintiff against unreasonable risk of injury, (ii) breach of that duty by the defendant, (iii) that the breach of duty was the actual and proximate cause of the plaintiff's injury, and (iv) damage to the plaintiffs person or property. If the pilot's selection of the landing site was unreasonable, then she has breached her duty of care to motorists on the highway, including the driver, and was the actual and proximate cause of his injuries. (B) would be correct if this were a products liability action based on strict liability. However, the pilot is not a commercial supplier of a product and would not be liable without some evidence of negligence. (C) is incorrect because even if the pilot was not negligent in failing to discover the defect in the fuel system, she could have been negligent in selecting the landing site, as (A) states. (D) is also incorrect. In selection of the landing site, the pilot is held to the standard of care of a reasonable person in an emergency. If she acted unreasonably in selecting the landing site, she will be liable in negligence.


Conjuntos de estudio relacionados

CHS 111 Medical Terminology Chapter 1

View Set

EMTB CH 21 HW and Quiz Questions

View Set

social studies praxisWhich of the following ancient civilizations used hieroglyphics as a way to communicate?

View Set

Pre-intermediate: Grammar test analysis: Correct the mistakes

View Set